You are on page 1of 576

This page

intentionally left
blank
Copyright 2005, New Age International (P) Ltd., Publishers
Published by New Age International (P) Ltd., Publishers

All rights reserved.


No part of this ebook may be reproduced in any form, by photostat, microfilm,
xerography, or any other means, or incorporated into any information retrieval
system, electronic or mechanical, without the written permission of the publisher.
All inquiries should be emailed to rights@newagepublishers.com

ISBN (13) : 978-81-224-2535-2

PUBLISHING FOR ONE WORLD


NEW AGE INTERNATIONAL (P) LIMITED, PUBLISHERS
4835/24, Ansari Road, Daryaganj, New Delhi - 110002
Visit us at www.newagepublishers.com
Dedicated
to
all my teachers
This page
intentionally left
blank
Foreword
Mathematics is considered as the mother of Science as well as the mother of Philosophy. In fact, it is
the basic knowledge on which any human endeavour in the field of intellectual pursuit is based. But
somehow, in our academic field Mathematics is considered as a subject tough to comprehend and to
master. The otherwise enjoyable subject becomes sometimes to the students a frightening nightmare
and in the course of my long career in the field of education, I have found many students even becoming
nervous at the mention of Mathematics. That is, in my experience, mostly because of the way it is
taught by some teachers and the way some of the books are written.
My esteemed colleague Smt. Veena, G.R. is not only a good teacher who would make the students
to enjoy Mathematics, but also a good Narrator of Mathematics. Extremely resourceful and com-
pletely committed, Veena has brought into this book her direct experience in the class room as a teacher.
In lucid style, she has written this book making it students-friendly, teacher-friendly and teaching-
learning focused.
While congratulating her for this good work, I wish that both the students and the teachers would
welcome this book and make use of it.

PROF. K.E. RADHAKRISHN


ADHAKRISHNAA
Principal, Surana College,
Bangalore-560004.
This page
intentionally left
blank
Preface
This book is written as per the Syllabus of Basic Mathematics for II Year Pre-University Course of
Karnataka. Each topic has been discussed exhaustively as per the requirements of the latest syllabus.
The book has been written in a very simple style, to enable students to understand the subject effec-
tively. More focus is given for a systematic approach to enhance the grasping of the subject so that all
types of questions could be answered well in each chapter. I hope this book will satisfy all the require-
ments of the students for learning the subject successfully and getting through the examination with
flying colours.
My sincere thanks to Prof. K.E. Radhakrishna, Principal, Surana College, an eminent academician
and educationist for his foreword. I also thank Ms. Sudha S., Department of Mathematics, Surana
College for reading the manuscript and identifying the unforeseen computational errors.
I am also thankful to Mr. R.K. Gupta, Chairman, Mr. Saumya Gupta, Managing Director, New Age
International Pvt. Ltd., New Delhi and Mr. Vincent Dsouza, Branch Manager, Mr. Babu V.R., Marketing
Manager, Bangalore branch for accepting to publish the book.
I sincerely welcome criticism, views and suggestions from readers.

G.R. VEEN
EENAA
veena.gr@gmail.com
This page
intentionally left
blank
Contents
Foreword vii
Preface ix

1. Mathematical Logic 1
1.1 Introduction 1
1.2 Propositions 1
1.3 Logical Connectives and Compound Propositions
Conjunction, Disjunction, Conditional, Biconditional, Negation 2
1.4 Tautology and Contradiction 9
1.5 Logical Equivalence 12
1.6 Converse, inverse and Contrapositive of a Conditional 14

2. Permuta
erm tion and Combina
utation tion
Combination 21
2.1 Introduction 21
2.2 Fundamental Principle 21
2.3 Permutation and Combination 22
2.4 Factorial of a Positive Integer 22
2.5 Permutation 22
2.5.1 Linear Permutation 23
2.5.2 Value of npr 23
2.5.3 Value of npn 23
2.5.4 Value of 0 24
2.6 Permutation of Things of which Some are Alike 28
2.7 Circular Permutation 31
2.8 Combination 33
2.8.1 Value of ncr 33
2.8.2 Complementary Combinations 34

3. Proba
Proba bility
obability 46
3.1 Introduction 46
3.2 Terminology 46
3.3 Definition of Probability 47
3.4 Addition Rule of Probability 52
3.5 Conditional Probability 56
xii Contents

3.6 Multiplication Rule 56

4. Binomial Theorem
heorem 65
4.1 Introduction 65
4.2 Statement of Binomial Theorem 65

5. Par tial Fractions


artial 88
5.1 Definitions 88
5.2 Partial Fractions 89

6. Matrices & Determinants 105


6.1 Introduction 105
6.2 Matrix 105
6.3 Types of Matrices 105
6.4 Algebra of Matrices 107
6.5 Transpose of a Matrix 108
6.6 Determinants 117
6.7 Properties of Determinants 120
6.8 Minor, Co-factor, Adjoint and Inverse of a Square Matrix 136
6.9 Characteristic Equation of a Square Matrix 144
6.10 Cayley Hamilton Theorem 146
6.11 Solution of Linear System of Equations 153
6.12 Application of Matrices in Business Problems 164

7. Ra tio and Pr
Ratio opor
Propor tions, Var
oportions, ia
aria tions
iations 182
7.1 Introduction 182
7.2 Ratio 182
7.3 Proportion 190
7.4 Direct Proportion or Direct Variation 192
7.5 Problems on Time and Work 202
7.6 Problem on Time and Distance 209
7.7 Problems on Mixture 213

8. Averages
era 221
8.1 Introduction 221
8.2 Arithmetic Average or Mean 221
8.3 Combined Average 222

9. Bill Discounting 234


9.1 Introduction 234
9.2 Terminology 234
9.3 Formulae 235
Contents xiii

10. Stoc ks and Shar


Stocks es
Shares 246
10.1 Stock 246
10.2 Shares 246
10.3 Distinction between Stock and Shares 246
10.4 Terminology 247

11. Learning Cur


Learning Curvve 255
11.1 Introduction 255
11.2 Learning Curve 255
11.3 The Learning Curve Ratio 255
11.4 Graphical Representation of Learning Curve 255
11.5 Learning Curve Equation 257

12. Linear Programming


Pro 263
12.1 Introduction 263
12.2 Linear Programming 263
12.3 Solution to Linear Programming Problem 264

13. Cir
Circcles 278
13.1 Definitions 278
13.2 Equation of a Circle 278
13.3 Point of Intersection of a Line and a Circle 292
13.4 Equation of Tangent to the Circle
x2 + y2 + 2gx + 2fy + c = 0 at the Point (x1, y1) on it 298
13.5 Length of the Tangent from the Point (x1, y1) to the
Circle x2 + y2 + 2gx + 2fy + c = 0. 299
13.6 Condition for the Line y = mx + c to be a Tangent to
the Circle x2 + y2 = a2 and point of contact 300
13.7 Condition for the Line lx + my + n = 0 to be a Tangent
to the Circle x2 + y2 + 2gx + 2fy + c = 0. 301

14. Parabola
ara 317
14.1 Introduction 317
14.2 Parabola 317
14.3 Equation of the Parabola in the Standard Form 318
14.4 Different Forms of Parabola with Vertex (0, 0) 320
14.5 Different Forms of Parabola with Vertex (h, k) 321

15. Limits and Continuity 343


15.1 Introduction 343
15.2 Constants and Variables 343
15.3 Function 343
xiv Contents

15.4 Limits 344


15.5 Standard Limits 345
15.6 Continuous Functions 356

16. Dif
Diffferential Calculus
erential 366
16.1 Introduction 366
16.2 Derivative of a Function 366
16.3 Derivative of Some Standard Functions from First Principles 367
16.4 Rules of Differentiation 371
16.5 Differentiation of Composite Functions 386
16.6 Differentiation of Implicit Functions 390
16.7 Differentiation of Parametric Functions 393
16.8 Logarithmic Differentiation 397
16.9 Successive Differentiation 401

17. Application of Der


pplication Deriivatives
tiv 429
17.1 Derivative as a Rate Measure 429
17.2 Maxima and Minima 438

18. Integration 455


18.1 Standard Integrals 455
18.2 Algebra of Integrals 456
18.3 Substitution Method 461
18.4 Integration by Partial Fraction Method 466

z
18.5 Integration by Parts 474
18.6 af af
Integrals of the Type e x f x + f x dx 477

19. Definite Inte


Definite grals
nteg 484
19.1 Introduction 484
19.2 Properties of Definite Integrals 490
19.3 Application of Definite Integrals to Find Area 494

20. Application of Calculus in Business


pplication 504
20.1 Terminology 504

Examination Cor
Examination ner
Corner 515
Blue Print 516
Model Question Paper 1 517
Model Question Paper 2 520
Model Question Paper 3 522
Chapterwise Arranged Question Bank 526
Gist and Formulae 546
1
Mathematical Logic
1.1 INTRODUCTION:
Logic is the science dealing with principles of reasoning. We can find all the different ways of solving
a problem by logical reasoning. The English Mathematician George Boole is the founder of mathemati-
cal logic. To express the principles of reasoning, a symbolic language has been developed. This sym-
bolic language is called mathematical logic or symbolic logic.
Mathematical logic finds application in switching circuits, digital computers and other digital devices.

1.2 PROPOSITIONS:
A proposition is a statement which in the given context is either true or false but not both. The propo-
sitions are denoted by small letters p, q, r...
Examples:
1. Sum of two even integers is even integer.
2. 3 is a rational number.
3. Earth is flat.
4. Delhi is the capital of Karnataka.
5. 7 is a prime number.
6. 5 7 = 2.
Note: The statements involving opinions, question marks, exclamatory mark, command, wish are not
propositions.
Examples:
1. Logic is interesting.
2. What a beautiful weather!
3. Where are you going?
4. Please sit down.
5. May God bless you.
2 Basic Mathematics

TR UTH VALUE: The truthness or falsity of a proposition is called its truth value. If a proposition is
TRUTH
true it is denoted by T and if it is false it is denoted by F.
Example: The truth value of
1. 5 + 6 = 11 is T.
2. Asia is in India is F.
3. Today is Sunday is either T or F in the given context i.e., on a particular day it is only one
of T or F.

1.3 LOGICAL CONNECTIVES AND COMPOUND PROPOSITIONS


Two or more simple propositions are connected by using the words and, or, if ... then, if and
only if. These words or phrases are called logical connectives. Any proposition containing one or more
connectives is called a compound proposition. The simple propositions occurring in a compound propo-
sition are called its components.
TRUTH
TR ABLE:: The truth values of the compound proposition for all possible truth values of its
UTH TABLE
components is expressed in the form of a table called truth table.
For a compound proposition with only one proposition, Truth table consists of 2 possibilities (either
T or F).
For a compound proposition with two propositions truth table consists of 22 = 4 possibilities. For a
compound proposition with 3 propositions truth table consists of 23 = 8 possibilities.
CONJUNCTION:: If p and q are 2 simple propositions. Then the proposition p and q is called the
CONJUNCTION
conjunction of p and q. It is denoted by p q.
Example:
If p : 7 is a prime number.
q : 6 is an even number, then
pq : 7 is a prime number and 6 is an even number.
The truth value of the compound proposition p q depends on the truth values of p and q. Note that
the conjunction of p and q is true only when both p and q are true, otherwise it is false.
Truth Table

p q pq
T T T
T F F
F T F
F F F

DISJUNCTION
DISJUNCTION:: If p and q are 2 simple propositions, then proposition p or q is called the disjunc-
tion of p and q. It is denoted by p q.
Example:
If p : 2 is rational number.
Mathematical Logic 3

q : 2 is odd number, then


p q = 2 is rational or 2 is odd number.
The truth value of p q depends on the truth values of p and q. Note that the disjunction of p and
q is false only when both p and q are false. Otherwise it is true.

Truth Table

p q pq
T T T
T F T
F T T
F F F

CONDITION
CONDITIONAL AL (IMPLICA TION)
TION):: If p and q are two simple proposition, then the proposition if p
(IMPLICATION)
... then q is known as conditional or implication. It is denoted by p q or p q.
Example:
If p : 6 is an even number.
q : 6 is divisible by 2, then
p q : If 6 is an even number then 6 is divisible by 2.
The truth value of p q depends on the truth values of p and q. Note that p q is false only when
p is true and q is false.
Truth Table

p q pq
T T T
T F F
F T T
F F T

BICONDITION
BICONDITIONAL AL (DOUBLE IMPLICA
IMPLICATIONTION OR EQ UIV
EQUIV ALENCE): If p and q are simple propo-
UIVALENCE):
sitions, then the proposition p if and only if q is called biconditional or double implication. It is
denoted by p q.
Example:
If p : k is odd number.
q : k2 is odd number, then
p q : k is odd number if and only if (iff) k2 is odd number.
Note that p q involves both the conditionals p q and q p.
4 Basic Mathematics

a
p q is p q q p f a f
The biconditional p q is true if p and q are both true or both false i.e., if p and q have same truth
values. Otherwise it is false.
Truth table
tab

p q pq q p a f a
p q i. e., p q q p f
T T T T T
T F F T F
F T T F F
F F T T T

NEGA TION: If p is a proposition then the proposition not p is called negation of p. It is denoted
NEGATION:
by ~p.
Example:
If p : 6 is odd number then
~p : 6 is not an odd number.
If p is true then ~p is false and if p is false then ~p is true.
Truth table
tab

p ~p
T F
F T

WORKED EXAMPLES
I. Write the following propositions in symbols:
1. An integer is even if and only if it is divisible by 2.
Solution: Let p : An integer is even.
q : It is divisible by 2.
The given proposition is p q.
2. If 6 + 3 = 7, then 7 3 = 6
Solution: Let p : 6 + 3 = 7
q:73=6
Then the given proposition is p q
3. I play chess or I study at home.
Solution: Let p : I play chess
q : I study at home.
Mathematical Logic 5

The given proposition: p q.


4. It is raining and the ground is wet.
Solution: Let p : It is raining.
q : The ground is wet.
The given proposition : p q.
5. If it rains today then government declares a holiday and we are happy.
Solution: Let p : It rains today
q : Government declares a holiday
r : we are happy.
Given proposition: p (q r).
6. If a number is not real then it is complex.
Solution: Let p : A number is real.
q : It is complex.
Given proposition in symbols: ~p q.
7. If Rama is intelligent or hardworking then logic is easy.
Solution: Let p : Rama is intelligent
q : Rama is hardworking
r : Logic is easy.
Given proposition: (p q) r.
8. If 3 is not odd and 2 is not even then 7 is not odd or 8 is not even.
Solution: Let p : 3 is odd
q : 2 is even
r : 7 is odd
s : 8 is even.
Given proposition: (~p ~q) (~r ~s)
II. Express each of the following compound propositions in sentences if
p : The question paper is difficult.
q : I get good marks.
r : I can go abroad.
1. p ~q
Solution: The question paper is difficult and I do not get good marks.
2. q r
If I get good marks, then I can go abroad.
3. (~p q) r
Solution: If the question paper is not difficult and I get good marks then I can go abroad.
4. r q
Solution: I can go abroad iff I get good marks.
5. (p ~q) ~r
6 Basic Mathematics

Solution: If the question paper is difficult and I do not get good marks then I can not go abroad.
III. If p, q, r are propositions with truth values T, F, T respectively then find the truth
value of the following propositions:
1. p ~q

p q ~q p ~ q
T F T T
Given Given

So Truth value of p ~q is T when p is T and q is F.


2. p (q r)
p q r qr p qr a f
T F T F F

3. (~p q) r

p q ~p p q r a ~ p qf r
T F F F T F

4. (p ~q) (q ~r)
(1) (2) (1) (2)

p q ~q p ~ q r ~r q ~ r a p ~ qf aq ~ r f
T F T T T F F F
Given Given Given

5. p (q ~r)
p q r ~r q ~ r a
p q ~ r f
T F T F T T

IV. Write the truth values of the following propositions if


p : 2 is even prime number.
q : 2 is rational number.
r : 7 is a composite number.
1. p (q r)
Given p : 2 is even number True proposition.
So Truth value of p : T.
Similarly Truth value of q : F
Truth value of r : F
Mathematical Logic 7

p q r qr a f
p qr
T F F F F

Truth value of p (q r) is F.
2. p (q ~ r)

p q r ~r q ~ r a
p q ~ r f
T F F T F F

Truth value of p (q ~ r) is F.
3. p (~ q ~ r)

p q ~q r ~r ~ q ~ r a
p ~ q ~ r f
T F T F T T T

Truth value of p (~ q ~ r) is T.
V. 1. A certain compound proposition (p q) r is known to be false. Find the truth values of p, q
and r.
Given: a p qf r is F
This implies p q is T and r is F [3 T F is F]
p is T, q is T and r is F 3 T T is T
2. A certain compound proposition (p q) (r ~s) is known to be false. Find the truth values of
p, q, r and s.
Given: a p qf ar ~ sf is F
p q is T and r ~ s is F 3 T F is F

p is T , q is T and r is F, ~ s is F 3 T T is T and F F is F

p is T , q is T , r is F and s is T . 3 ~ F is T .

a f a f
3. A certain compound proposition ~ p ~ q r s is given to be True. Find the truth values of
p, q, r and s.
Given: a~ p ~ qf ar sf is T
~ p ~ q is T and r s is T

~ p is T , ~ q is T and r is T , s is T 3 T T is T

p is F, q is F; r is T ; s is T . 3 ~ T is F .
8 Basic Mathematics

VI. Construct the truth table for the following compound propositions:
1. p ~p

p ~p p ~ p
T F F
F T T

Explana tion: A compound proposition with one component will have 2 possibilities either T or F.
Explanation:
Write ~p finally p ~p.
2. ~p ~q.

p q ~p ~q ~ p ~ q
T T F F T
T F F T T
F T T F F
F F T T T

Explana tion: A compound proposition with 2 components will have 4 possibilities. Write 2 T and 2F
Explanation:
under p, alternatively T, F under q to get all possible combinations. Now
~p is T when p is F and vice versa.
~q is T when q is F and vice versa.
~p ~q is F only when ~p is T and ~q is F. Otherwise it is T.
3. p (p q)

p q pq a
p pq f
T T T T
T F F F
F T F T
F F F T

4. p ~(~p q)

p ~p q ~ pq a
~ ~ pq f a
p ~ ~ p q f
T F T T F T
T F F F T T
F T T T F F
F T F T F F
Mathematical Logic 9

5. p (q r)

p q r qr a f
p qr
T T T T T
T T F T T
T F T T T
T F F F F
F T T T F
F T F T F
F F T T F
F F F F F

Explana tion: A compound proposition having 3 components will have eight possibilities. Write 4T and
Explanation:
4F under p, 2T, 2F, 2T, 2F under q and alternately T and F under r to get all the possible combinations
of truth values.
Now q r is F only when both q and r are F. Otherwise it is T.
p (q r) is T only when p is T and (q r) is T otherwise it is F.
6. (p ~q) (q ~ r)

p q r ~q p ~ q ~r q ~ r a p ~ q f aq ~ r f
T T T F F F F T
T T F F F T T F
T F T T T F F F
T F F T T T F F
F T T F T F F F
F T F F T T T T
F F T T T F F F
F F F T T T F F

1.4 TAUTOLOGY AND CONTRADICTION


A tautology is a compound proposition which is always true irrespective of the truth values of its
components.
A contradiction is a compound proposition which is always false irrespective of the truth values of
its components.
Note: To determine whether a given proposition is a tautology or a contradiction, construct its truth
table. If its truth value for all possibility is True, then it is tautology. If its truth value for all
possibility is False then it is contradiction. Otherwise it is neither tautology nor contradiction.
10 Basic Mathematics

WORKED EXAMPLES
1. Prove that p ~p is a contradiction.

p ~p p ~ p
T F F
F T F

From the last column p ~p is a contradiction.


2. Prove that p ~p is a tautology.

p ~p p ~ p
T F T
F T T

From the last column p ~p is a tautology.


3. Find whether p ~p is tautology or contradiction or neither

p ~p p ~ p
T F F
F T T

From the last column, clearly p ~p is neither tautology nor contradiction.


4. Prove that (p ~q) q is neither tautology nor contradiction.

p q ~q p ~ q a p ~ qf q
T T F F F
T F T T F
F T F F F
F F T F T

From the last column it is clear that (p ~q) q is neither tautology nor contradiction.
5. Prove that (p q) (~q ~p) is a tautology.

a xf a yf
p q ~q ~p pq ~ q ~ p XY
T T F F T T T
T F T F F F T
F T F T T T T
F F T T T T T

From the last column it is clear that (p q) (~q ~p) is a tautology.


Mathematical Logic 11

6. Prove that (p q) (~p ~q) is a contradiction.

p q ~p ~q pq ~ p ~ q a p q f a ~ p ~ qf
T T F F T F F
T F F T T F F
F T T F T F F
F F T T F T F

From the last column, the given proposition is contradiction.


7. Examine whether (p q) (q r) is a tautology or contradiction or neither

p q r p q qr a p qf a q r f
T T T T T T
T T F T F F
T F T F T F
T F F F T F
F T T T T T
F T F T F F
F F T T T T
F F F T T T

From the last column it is clear that the given proposition is neither tautology nor contradiction.
8. Prove that a p qf a q r f p r is a tautology

p q r p q qr a p qf a q r f p r a p qf a q r f p+r
T T T T T T T T
T T F T F F F T
T F T F T F T T
T F F F T F F T
F T T T T T T T
F T F T F F T T
F F T T T T T T
F F F T T T T T

From the last column clearly the given proposition is tautology.


12 Basic Mathematics

1.5 LOGICAL EQUIVALENCE:


Two propositions X and Y are said to be logically equivalent if and only if they have same truth values
and we write X Y. Note that X Y if and only if X Y is always true or X Y is tautology.

SOME STANDARD RESULTS ON LOGICAL EQUIVALENCE:


1. Prove that p ~(~p).

1 2 3
p ~p a f
~ ~p
T F T
F T F

from column (1) and (3) p ~(~p).


2. Prove that ~(p q) ~p ~ q

1 2 3 4 5 6 7
p q pq a
~ pq f ~p ~q ~ p ~ q
T T T F F F F
T F F T F T T
F T F T T F T
F F F T T T T

From columns 4 and 7, ~ (p q) ~p ~q.


3. Prove that ~(p q) ~p ~ q

1 2 3 4 5 6 7

p q ~p ~q pq a
~ pq f ~ p ~ q
T T F F T F F

T F F T T F F
F T T F T F F

F F T T F T T

From column 6 and 7, ~ (p q) ~p ~ q.


Mathematical Logic 13

4. Prove that ~ (p q) p ~ q

1 2 3 4 5 6
p q pq a
~ p q f ~q p ~ q
T T T F F F
T F F T T T
F T T F F F
F F T F T F

From columns 4 and 6, the given propositions are logically equivalent.


5. Prove that ~ (p q) (p ~q) (q ~p)

1 2 3 4 5 6 7 8 9
p q pq ~ pqa f ~p ~q p ~ q q ~ p a p ~ qf aq ~ pf
T T T F F F F F F
T F F T F T T F T
F T F T T F F T T
F F T T T T F T T

a f a
From columns 4 and 9, ~ p q p ~ q q ~ p f a f
Note: The results
a f
~ p q ~ p ~ q

~ a p q f ~ p ~ q

~ a p qf p ~ q

~ a p qf a p ~ qf a~ p qf
are used to find the negation of compound propositions.

WORKED EXAMPLES:
I. Negate the following:
1. 6 is an odd number or 3 is an even number.
Let p : 6 is an odd number.
q : 3 is an even number.
Given proposition in symbol is
pq
14 Basic Mathematics

We know a f
~ p q ~ p ~ q
Negation: 6 is not an odd number and 3 is not an even number.
2. He is rich and He is not happy
Let p : He is rich.
q : He is happy.
Given: p~q
We know a f
~ p ~ q ~ p ~ ~ q a f
~ p q
Negation: He is not rich or He is happy.
3. If the cow is big, then it is healthy.
Let p : Cow is big.
q : It is healthy.
Given proposition in symbols: p q
We know a f
~ p q p ~ q
Negation: Cow is big and it is not healthy.
4. If the triangles are not equiangular then the sides are not proportional.
Let p : The triangles are not equiangular.
q : The sides are not proportional.
Given proposition: p q
Its negation: ~ ( p q)
But a f
~ p q p ~ q
Negation: The triangles are not equiangular and the sides are proportional.
5. 6 is even if and only if it is divisible by 2.
Let p : 6 is even.
q : 6 is divisible by 2.
Given: p q.
Its negation is ~(p q)
But a f a
~ p q p ~ q ~ p q f a f
Negation: 6 is even and it is not divisible by 2 or 6 is not even and it is divisible by 2.

1.6 CONVERSE, INVERSE AND CONTRAPOSITIVE OF A


CONDITIONAL
Let p q be the given conditional then the conditional q p is called converse of p q. The
conditional ~p ~q is called inverse of p q. The conditional ~q ~p is called contrapositive of
p q.
Note that contrapositive is converse of inverse.
Mathematical Logic 15

WORKED EXAMPLES:
Wr ite the con
Write convverse
se,, in
erse ver
inv se and contr
erse contraapositive of the ffollo
positiv ollowing conditionals:
ollowing
2
1. If x = 3, then x = 9.
Solution: Let p : x = 3, q : x2 = 9.
Given: p q
Converse: q p
If x2 = 9, Then x = 3
Inverse: ~p ~q
If x 3. Then x2 9.
Converse: ~q ~p
If x2 9. Then x 3.
2. If two triangles are congruent then they are similar.
Let p : 2 triangles are congruent.
q : 2 triangles are similar.
Given: pq
Converse: qp
If 2 triangles are similar then they are congruent.
Inverse: ~p ~q
If 2 triangles are not congruent then they are not similar.
Converse: ~q ~p
If 2 triangles are not similar then they are not congruent.
3. If cows can fly then birds cannot fly.
Let p : cows can fly.
q : Birds can fly.
Given: p ~q
Converse of p q is q p
Converse of p ~q is ~q p
i.e. If birds cannot fly then cows can fly.
Inverse of p q is ~p ~q
So inverse of p ~q is ~p ~(~q)
~p q
i.e. If cows cannot fly then birds can fly.
Contrapositive of p q is ~q ~p
Contrapositive of p ~q is ~(~q) ~p
i.e., q ~p.
If birds can fly then cows cannot fly.
16 Basic Mathematics

4. If I work hard then I can score 90% and I can go for engineering.
Solution: Let p : I work hard.
q : I can score 90%
r : I can go for engineering.
Given p (q r)
Converse: qrp
i.e. If I can score 90% and I can go for engineering then I work hard.
Inverse is ~p ~(q r) i.e., ~p ~ q ~r
If I do not work hard then I can not score 90% or I cannot go for engineering.
Contrapositive is ~(q r) ~p i.e., (~q ~r) ~p
If I cannot score 90% or I cannot go for engineering then I do not work hard.
5. If e is not irrational and is rational then 6 is not even or 2 is odd.
Let X : e is not irrational and is rational
Y : 6 is not even or 2 is odd.
Given XY
Converse: YX
If 6 is not even or 2 is odd then e is not irrational and is rational.
Inverse: ~X ~Y
If e is irrational or is not rational
Then 6 is even and 2 is not odd.
Contrapositive: ~Y ~X
If 6 is even and 2 is not odd then e is irrational or is not rational.

REMEMBER
p q is T only when both p and q are true otherwise it is false i.e., T T is T otherwise it is F.
p q is F only when both p and q are false. Otherwise it is True i.e., F F is F otherwise it is
T.
p q is F only when p is true and q is false i.e., T F is F otherwise it is T.
p q is T only when both p and q are together True or False, i.e., T T and F F is T
otherwise it is F.
~p is T when p is F and vice-versa.
Tautology is a compound proposition which is always true for all possible combinations of the
truth values of its components.
Contradiction is a compound proposition which is always False for all possible combinations of
the truth values of its components.
2 propositions X and Y are logically equivalent if and only if they have identical truth values. It
is denoted by X Y.
~ (~p) p
Mathematical Logic 17

~ (p q ) ~ p ~ q
~ (p q) ~p ~q
~ (p q ) p ~ q
~ (p q) (p ~ q) (~p q)
Converse of the conditional p q is q p
Inverse of the conditional p q is ~p ~q
Contrapositive of conditional p q is ~q ~p.

EXERCISE
I. Wr ite the ffollo
Write ollowing compound pr
ollowing opositions in symbols.
propositions
1. If a triangle is equilateral then all the sides of the triangle are equal.
2. If I dont go to picnic then I will study at home.
3. Sun rises in the east and earth is not flat.
4. 2 is irrational or 5 is real.
5. A number is prime if and only if it is not composite.
6. If 2 + 2 4 and 6 + 6 12, then 4 + 7 = 6 or 5 + 3 = 9.
7. ABC is a right angled triangle if and only if one of the angle = 90 and square on the hypotenuse
= sum of the squares on other 2 sides.
8. a + ib = x + iy iff a = x and b = y.
II. If p, q and r ar
aree 3 pr
propositions truth
opositions with tr uth vvalues espectiv
alues T, F and T respecti ely
vel y then ffind truth
ind the tr uth
values of the ffollo
ollo wing:
ollowing:
1. a p qf ~ r a f
2. ~ p q r 3. a p qf ~ a q r f 4. a p qf r p
5. p a p qf 6. a p rf ~ q
III. Constr uct the tr
Construct uth ta
truth tabble ffor
or the ffollo
ollowing pr
ollowing opositions:
propositions:
1. ~ p ~ q 2. p ~ q 3. ~ p ~ q a
4. ~ p ~ q f
5. a p qf a q ~ r f a
6. p ~ p q f a f
7. ~ p q r
IV. Ver ify w
erify hether the ffollo
whether ollowing compound pr
ollowing opositions ar
propositions aree tautologies or contr
tautolog adictions or
contradictions
neither:
a
1. p p q f 2. p ~ p 3. p ~ p a
4. p ~ p q f
a f
5. ~ ~ p p 6. a ~ p q f aq p f 7. a p qf p
8. a p qf a q ~ r f a p r f 9. ~ p a p ~ q f q

10. p a q r f a p qf a p r f
18 Basic Mathematics

V. Find w hether the ffollo


whether ollowing compound pr
ollowing opositions ar
propositions aree logicall
logicallyy equivalent:
equiv
1. a p qf a p qf ; a p qf a p qf 2. a p q f r ; p aq r f
3. p aq rf ; a p q f a p rf 4. p q ; ~ p ~ q

5. p a q r f ; a p qf a p r f a f
6. ~ p q ~ r ; p ~ q r a f
VI. Ne
Neggate the ffollo
ollowing compound pr
ollowing opositions:
propositions:
1. 5 is odd and 6 is even.
2. Cow is not big or it is black.
3. If 2 lines are parallel then they do not intersect.
4. I will pass the examination iff the questions are easy.
5. p ~ q
6. p ~ q

a
7. p q r f
8. p ~ q
VII. Find the in ver
inv se con
erse ver
conv se and contr
erse apositi
contrapositiv ve of the ffollo
ollowing:
ollowing:
2
1. If x is even then x is even.
2. If a2 + b2 = c2 and a2 = c2 then b2 = 0.
3. If a number is real then it is rational or it is irrational.
4. If Smitha gets a first class then she is either intelligent or hard working.
5. If 3 is not prime and 7 is not an odd number then 37 is not an even number or 73 is an odd number.
6. ~p ~q
7. a p qf a p qf
8. p aq r f

9. p a~ q ~ rf

10. a p qf r.
ANSWERS

I. 1. p q 2. ~ p q 3. p ~ q

4. p q 5. p ~ q 6. a ~ p ~ qf a r sf
7. p q r a f 8. p q r a f
II.
1. F 2. T 3. F 4. F 5. F 6. T
Mathematical Logic 19

III.

~ p ~ q p ~ q ~ p ~ q a
~ p ~ q f
F F T T
F T T F
1. F 2. F 3. F 4. T
T F T F

a p qf a q ~ r f pqr
F T

a f
T T
T p ~ p q T
T T T
T T T
5. 6. T 7.
F F
T T F
T F

IV.
1. Tautology 2. Tautology 3. Contradiction 4. Neither
5. Tautology 6. Contradiction 7. Neither 8. Neither
9. Contradiction 10. Tautology
V.
1. No, not logically equivalent. 2. Logically equivalent.
3. Logically equivalent. 4. No, not logically equivalent.
5. Logically equivalent. 6. Logically equivalent.
VI.
1. It is not odd or 6 is not even
2. Cow is big and it is not black.
3. 2 lines are parallel and they intersect
4. I will pass the examination and the questions are not easy or I will not pass the examination and
the questions are easy.
5. ~p q
6. pq
7. ~p (q ~r)
8. (p q) (~p ~q)
20 Basic Mathematics

VII.
1. Inverse: If x is not even then x2 is not even
Converse: If x2 is even then x is even.
Contrapositive: If x2 is not even then x is not even.
2. Inverse: If a2 + b2 c2 or a2 c2 Then b2 0.
Converse: If b2 = 0 then a2 + b2 = c2 and a2 = c2.
Contrapositive: If b2 0 then a2 + b2 c2 or a2 c2.
3. Inverse: If a number is not real then it is not rational and it is not irrational.
Converse: If a number is rational or it is not irrational then it is real.
Contrapositive: If a number is not rational and it is not irrational then it is not real.
4. Contrapositive: If Smitha is neither intelligent nor hardworking then she doesnt get a first class.
Converse: If Smitha is either intelligent or hardworking then she gets a first class.
Inverse: If Smitha does not get first class then she is neither intelligent nor hardworking.
5. Contrapositive: If 37 is even and 73 is not odd then 3 is prime or 7 is odd.
Converse: If 37 is not even or 73 is odd then 3 is not prime and 7 is not odd.
Inverse: If 3 is prime or 7 is odd then 37 is even and 73 is not odd.
6. Inverse: p q
Converse: ~q ~p
Contrapositive: q p
a f a
7. Inverse: ~ p ~ q ~ p ~ q f
Converse: a p qf a p qf

Contrapositive: a ~ p ~ qf a ~ p ~ qf

8. Inverse: ~ p a q ~ r f

Converse: a q r f p

Contrapositive: a q ~ r f ~ p

9. Inverse: ~ p a ~ q r f

Converse: a ~ q ~ r f p

Contrapositive: a ~ q r f ~ p
10. ~ p ~ q ~ r
r pq
~ r ~ p ~ q
2
Permutation and Combination
2.1 INTRODUCTION:
In our daily life we come across situations where we have to select or arrange certain things out of a
given number of things. This selection or arrangement involves a principle known as fundamental
principle which is illustrated by the following example.
Suppose that in a auditorium there are 4 different
entrance doors (say I1, I2, I3 and I4) and there are 5
different exit doors (say O1, O2, O3, O4 and O5). In
how many ways can a person enter and leave the au-
ditorium?
If a person enters the auditorium through the door
I1, he can go out by any one of the exit doors O1 O2
O3 O4 O5. So there are 5 ways of leaving the auditorium
if the person enters it through door I1.
Similarly corresponding to the entrance door I2 there
are 5 ways of leaving the auditorium. Altogether 5 +
5 + 5 + 5 = 20 different ways. In general if there are
m different entrance doors and n different exit doors,
a person can enter and leave the auditorium in mn
ways. This is fundamental principle.
Fig. 2.1
2.2 FUNDAMENTAL PRINCIPLE:
If one event can be done in m different ways and after it has been done in one of these ways, a second
event (which is independent of the first) can be done in n different ways then the two events together
can occur in mn ways.
The extension of this principle (also called the mnp ... principle) to the case of more than 2 events
is obvious.
Example:
1. A boy and a girl have to be selected from a group of 5 boys and 6 girls. In how may ways can
the selection be made?
22 Basic Mathematics

Solution: Here First operation is selecting a boy from a group of 5 boys. This can be done in 5 ways.
After this is done, the second operation is selecting a girl from 6 girls. This can be done in 6 ways.
By fundamental principle, the total number of selections = 5 6 = 30 ways.
2. There are 4 candidates for the post of manager. 3 candidates for the post of officer and 5 for the
post of clerk. In how many ways can these posts be filled?
Solution: A manager may be selected in 4 ways. An officer may be selected in 3 ways and A clerk
may be selected in 5 ways.
By fundamental principle, the 3 posts together can be filled in 4 3 5 = 60 ways.

2.3 PERMUTATION AND COMBINATION:


Suppose 3 members (say a, b, c) went to a cinema where they get only 2 tickets. So it is required to
select 2 members out of 3 members (a, b, c). The following selections are possible: ab, bc, ca. So there
are 3 ways of selecting 2 members out of 3 members. In symbols this can be written as 3c2. It is called
number of combination of 3 members taken 2 at a time.
Now consider 3 symbols , and . If we wish to arrange these letters taken 2 at a time, we get
the following arrangements , , , , , . These arrangements are called number of
permutations of 3 symbols taken 2 at a time. In symbols this can be written as 3p2.
Note that in arrangement order is important. Therefore is different from , whereas in selection
order is not important. Hence ab is same as ba and so it is regarded as only one selection.

2.4 FACTORIAL OF A POSITIVE INTEGER:


The product of first n natural numbers i.e. 1. 2. 3. ... (n 1). n is called factorial n. It is represented
by the symbol n or n!.

a f a f
n ! = 1 2 3 ... n 1 n = n n 1 ... 3 2 1

So 1=1
2 = 2 1

3 = 3 2 1
4 = 4 3 2 1 = 4 3 = 4 3 2 and so on.
In general
a f
n = n n 1 = n n 1 n 2 and so on.

2.5 PERMUTATION:
An arrangement of all or part of a set of objects in some order is called permutation.
If the objects are arranged along a straight line it is called a linear permutation. If the objects are
arranged around a circle then it is called circular permutation.
Permutation and Combination 23

2.5.1 Linear Permutation:


Each of the different arrangements in a straight line that can be made by taking some or all of a number
of things at a time is called linear permutation.
The number of permutation of n distinct things taken r at a time is denoted by npr.
2.5.2 Value of npr:
The number of permutation of n distinct things taken r at a time i.e. npr will be same as the number
of ways in which r blank places can be filled up with n given objects.
As the first place can be filled in by any one of the n objects, there are n ways of filling the first
place.
After having filled in the first place by any one of the n things, there are (n 1) objects left. Hence
2nd place can be filled in (n 1) ways. Similarly 3rd place can be filled in (n 2) ways and so on.
Hence rth place can be filled in (n (r 1)) ways i.e., (n r + 1) ways.

Position of the object 1st 2 nd 3 rd ...... r th


Number of ways n a f a
n 1 n2 f ...... an r + 1f
By fundamental principle, r places can be filled by any r of n objects in n (n1) (n2) ... (nr +1)
ways.
n
a fa fa f a
pr = n n 1 n 2 n 3 ... n r + 1 f
Multiplying and Dividing by (n r) (n r 1) ... 2 1 in RHS we get

a fa fa f a fa fa
n n 1 n 2 n 3 ... n r + 1 n r n r 1 ... 2 1 f
a fa f
n
pr =
n r n r 1 ... 2 1

n n
pr =
nr

n n
Hence value of pr = .
nr

2.5.3 Value of npn:


The number of permutation of n distinct objects taken all at a time is npn. It is same as the number of
ways in which n blank spaces can be filled up with n given objects.

Position of the object 1st 2 nd 3 rd ...... n 1th n th


Number of ways n n 1 n2 ...... 2 1

As the first place can be filled in by any one of the n objects there are n ways of filling the first
place. After having filled in the first place, 2nd place can be filled in (n 1) ways and so on.
By fundamental principle,
n
a fa f
pn = n n 1 n 2 ... 2 1
24 Basic Mathematics

n
pn = n

n
Value of pn = n .

2.5.4 Value of 0 :

n n
We have pr =
nr
Put r = n.

n n
pn =
nn
n
But pn = n

n
n=
0
Cross-multiplying,
n
0=
n

0 =1

WORKED EXAMPLES:
1. Evaluate:
5 6 7
( a) p2 (b) p3 (c) p7

n n
We have pr =
nr

5 5
p2 = = 5 4 = 20.
52
6
p3 = 6 5 4 = 120.
7
p7 = 7 = 5040.
2. If npn = 720 find n.

We have n
pn = n = 720 givena f
Permutation and Combination 25

n = 6 5 4 3 2 1 2 720
3 360
n= 6 4 120
5 30
n = 6. 6 6
1
3. If np2 = 72, find n.
We have np = n (n 1) = 72.
2
n (n 1) = 9 8 [By inspection: 72 = 9 8].
n = 9.
4. In how many of the permutations of 7 things taken 4 at a time will (a) One thing always occur
(b) One thing never occur?
Solution: (a) Keeping aside the particular thing which will occur, the number of permutation of 6
things taken 3 at a time
= 6p3 = 6 5 4 = 120.
Now this particular thing can take up any one of the four places and so the total number of ways
= 120 4 = 480 ways.
(b) Leaving aside the particular thing which has never to occur, the number of permutation of 6 things
taken 4 at a time
= 6p4 = 6 5 4 3 = 360 ways.
5. How many 4 digit numbers can be formed with the digits 2, 4, 5, 7, 9. (Repetitions not being
allowed). How many of these are even?
Number of 4 digit numbers that can be formed with the digits 2, 4, 5, 7, 9 (without repetitions)
= 5p4 = 5 4 3 2 = 120.
Th H T U
Since we require an even number, we must
have 2 or 4 in the units place. After filling the
units place by 2 or 4, the remaining 3 places
(Tens, Hundreds and Thousands) can be
filled by remaining digits 5, 7, 9, 2 or 4 in 4p3
ways 4 3 2 = 24 ways.
Number of even numbers that can be formed 4
2 or 4
p3
= 2 4 p3 = 2 24 = 48
6. How many numbers can be formed by using any number of digits 3, 1, 0, 5; no digit being
repeated in any number:
Solution: The number of single digit numbers = 3p1 (excluding zero) = 3.
The permutation of 4 digits taking 2 at a time are 4p2 but 3p1 of these have zero in tens place so
reduce to single digit number.
Number of 2 digit numbers = 4p2 3p1.
Similarly number of 3 digit numbers = 4p3 3p2. Number of 4 digit numbers = 4p4 3p3.
26 Basic Mathematics

Total number of numbers

= 3+ d p p i+d p p i+d p p i
4
2
3
1
4
3
3
2
4
4
3
3

= 3 + a 4 3 3f + a 4 3 2 3 2 f + a 4 3 2 1 3 2 1f

= 3 + a12 3f + a24 6f + a24 6f


= 3 + 9 + 18 + 18 = 48.
7. There are 4 Kannada books, 3 Hindi books and 5 English books. In how many ways can these
be placed on a shelf if the books of the same language are to be together?
Solution: Since the books of the same language are to be together. Let us consider the 4 Kannada books
as 1 unit, 3 Hindi books as another unit and 5 English books as a different unit. Then we have to arrange
3
3 different units. This can be done in p3 = 3 = 6 ways.
4
But the 4 Kannada books remaining together can be arranged in p4 = 4 ways. Similarly, 3 Hindi
books can be arranged in 3 ways and 5 English books can be arranged among themselves in 5 ways.

Number of permutations = 3 4 3 5
=3243213254321
= 6 24 6 120
= 103680 ways.
8. In how many different ways can 5 examination papers be arranged in a row so that the best and
the worst papers may never come together.
Solution: Without any restrictions the 5 exam papers can be arranged among themselves in 5p5 =
5 = 120 ways.
Considering now, the best and the worst papers as a single paper, we have only 3 + 1 = 4 papers.
Now these 4 papers can be arranged taking all at a time in 4 p3 = 4 3 2 = 24 ways.

But in each of the 24 ways, the best and worst papers can be arranged among themselves in 2 =
2 ways.
Total number of ways, where the best and the worst paper always come together
= 24 2 = 48.
Hence required number of arrangements where best and the worst paper never come together =
120 48 = 72 ways.
9. In how many ways can 5 boys and 3 girls be seated in a row so that (a) each girl in between 2
boys (b) no two girls sit together (c) all the girls are together.
Solution: (a) First we arrange 5 boys in a row. This can be done in 5p5 = 5 = 120 ways. .
Consider one such arrangement:
B1 B2 B3 B4 B5
Permutation and Combination 27

There are 4 places available for 3 girls, so that each girl is between 2 boys. The 4 places can be filled
by 3 girls in 4p3 ways = 4 3 2 = 24 ways.
For one arrangement of boys, there are 24 ways of arranging girls.
For 120 arrangement of boys
= 120 24 = 2880 ways.
(b) Again we arrange 5 boys in a row. This can be done in 5 = 120 ways. Considering 1 such
arrangement
*B 1 *B 2 * B 3 *B 4 * B 5 *
There are 6 places available for the girls so that no two girls are together. The 6 places can be
filled by 3 girls in 6p3 ways = 6 5 4 = 120 ways.
Required number permutations = 120 120 = 14400 ways.
(c) We require all the 3 girls to be together. So we consider 3 girls as one unit.
Now 5 boys and 1 unit (of 3 girls) can be arranged in 5 + 1 = 6 = 720 ways.

Again 3 girls, wherever they are together can be arranged in 3 = 6 ways.


Number of permutations = 720 6 = 4320 ways.
10. How many different words can be formed with the letters of the word ORDINATE.
(a) Beginning with O (b) beginning with O and ending with E. (C) So that vowels occupy odd
places.
(a) Keeping O in the first place. We can arrange the remaining 7 letters in 7 places in
7
p7 = 7 ways = 5040 ways.

O 7
p7

Number of words that can be formed with the letters of word ORDINATE that begins with
0 = 5040.
(b) Keeping O in the first place and E in the last place, the remaining 6 letters can be arranged
in 6 place in 6p6 = 6 = 720 ways.

O E

6
p6

(c) In the word ORDINATE O, I, A, E are vowels.


There are 4 odd places and 4 vowels. The 4 vowels can be arranged in 4 odd places in 4p4
= 4 ways = 4 3 2 1 = 24 ways.
28 Basic Mathematics

st rd th th
1 3 5 7

Remaining 4 even places can be occupied by remaining 4 letters in 4p4 ways


= 4 ways = 24 ways.
Total number of permutations = 24 24 = 576.
11. If the letters of the word MAKE be permuted and the words so formed be arranged as in
dictionary what is the rank of the word?
Solution: The alphabetical order of the letters is AEKM.
If A is fixed in 1st place, the other 3 letters can be permuted in 3 = 3 2 = 6 ways. .
The number of words begin with A = 6.
Similarly the number of words which begin with E and K = 3 = 6 each.
The words beginning with M are
MAEK and MAKE.
Rank of the word = 6 + 6 + 6 + 1 + 1
= 20.
12. Prove that the number of ways in which n books can be placed on a shelf when 2 particular books
a
are never together is n 2 n 1. f
Solution: Regarding the 2 particular books as one book, there are (n 1) books now, which can be
n 1
arranged in pn 1 = n 1 ways. Now these two books can be arranged in 2 ways. Therefore the
number of permutations in which 2 particular books are always together = 2 . n 1 .

The number of permutations of n books without any restriction = n .


The number of permutations in which 2 particular books never occur together
= n 2. n 1
= n n 1 2. n 1
a
= n 1 n 2 f
= an 2 f n 1
Hence proved.

2.6 PERMUTATION OF THINGS OF WHICH SOME ARE ALIKE


Theorem: If x is the number of permutations of n things taken all at a time of which one set p are alike,
Theorem
another set q are alike and so on. Then prove that
n
x=
p q ...
Permutation and Combination 29

Proof: Replacing p like objects by p unlike objects, q like objects by q unlike objects and so on,
we arrive at a stage where all n objects are distinct. By permuting these unlike objects amongst
themselves each of the x permutations would give rise to p q ... permutations. Hence x permutations
give rise to x p q ... . But the number of permutations of n distinct objects taken all at a time = n .

x p q ... = n

n
x= .
p q ...

WORKED EXAMPLES:
1. In how many ways can letters of the word INDIA be arranged?
Solution: There are 5 letters in the word INDIA of which I occur 2 times.
5
Number of words possible =
2
= 5 4 3 = 60.
2. In how many ways can the letters of the word PERMANENT be arranged so that 2Es are always
together.
Solution: There are 9 letters in the word PERMANENT of which E occurs 2 times, N occurs 2 times.
If 2 Es are together, taking them as one letter we have to arrange 8 letters in which N occurs two times.
8
Number of arrangements =
2
= 8 7 6 5 4 3 = 20,160
3. In how many ways can be letters of the word HOLLOW be arranged so that the 2Ls do not
come together.
Solution: There are 6 letters in the word HOLLOW in which L occurs 2 times, O occurs 2 times.
6
Number of arrangement =
2 2

6 5 432
=
2 1 2 1
= 180.
If 2 Ls are together, taking them as one letter, we have to arrange 5 letters in which O occurs
2 times.
5
Number of arrangements in which 2 Ls are together = = 5 43
2
= 60.
Number of arrangements in which 2Ls do not come together = 180 60 = 120.
30 Basic Mathematics

4. Find the number of permutations of the word EXCELLENCE. How many of these permuta-
tions (i) begin with E (ii) begin with E and end with C (iii) begin with E and end with E (iv)
do not begin with E.
Solution: The word EXCELLENCE contains 10 letters of which E occurs 4 times C occurs 2 times,
L occurs 2 times.

10
Number of permutations =
4 2 2

10 9 8 7 6 5
= = 37800.
2 1 2 1
( i) Put E in the first place and arrange the rest. Now there are 9 letters of which E occurs 3 times,
L occurs twice and C occurs twice.

9
Number of permutations =
3 2 2

98765 4
=
2 1 2 1
= 15,120.
(ii) Put E in the first place and C in the last place, and arrange the rest. There are 8 letters of which
E occurs 3 times, L occurs twice.

8
Number of permutations =
3 2

8 7654
= = 3360.
2
( iii) Put one E in the first place and another E in the last place and arrange the rest. There are 8 letters
of which E occurs 2 times, L occurs twice and C occurs twice.

8
Number of permutations =
2 2 2

8765 43
=
2 1 2 1
= 5040.
(iv) Number of permutations that do not begin with E = Total number of permutations.
Number of permutations that begin with E.
= 37800 15120
= 22680.
5. How many numbers less than 3 millions can be formed using the digits of the number 2123343?
Permutation and Combination 31

Solution:

Million HTh TTh Th H T U

1 or 2

Since the number should be less than 3 millions, we can have either 1 or 2 in the millions place.
Keeping 1 in the millions place and arranging the rest. There are 6 numbers 2, 2, 3, 3, 3, 4 of which
2 repeats twice, 3 occurs thrice.
6 65 4
Number of numbers = =
2 3 2
= 60.
Now keeping 2 in the millions place and arranging the rest. There are 6 numbers 1, 2, 3, 3, 3, 4
in which 3 occurs thrice.
6
Number of numbers = =654
3
= 120.
Total number of numbers less than 3 millions.
= 60 + 120 = 180.

2.7 CIRCULAR PERMUTATION:


If we have to arrange 4 letters A, B, C, D in a row. Then 2 of the arrangements would be ABCD, BCDA
and we treat these two arrangements as different. But if we arrange along the circumference of the

circle, the two arrangements and are one and the same.

So we conclude that circular permutations are different only when the relative order of the objects
is changed; otherwise they are same.
In circular permutation of n different things one thing is kept fixed and the balance (n 1) things
are arranged relative to it in n 1 ways.
If the clockwise and anticlockwise orders are distinguished, the required number of permutations =
n 1 .

Number of ways in which n persons can occupy the chairs in a round table = n 1 .
If the clockwise and anticlockwise orders are not distinguished then required number of permuta-
n 1
tions = .
2
32 Basic Mathematics

n 1
Number of ways in which n flowers or n beads are strung to form garland or necklace = .
2

WORKED EXAMPLES:
1. In how many ways 5 people sit around a table?
Solution: Fixing the position of one person, the remaining 4 persons can sit around a table in 4 ways
= 4 3 2 1 = 24.
5 people can sit round a table in 24 ways.
2. In how many ways can 7 different jewels be strung into a necklace?
Solution: Keeping one jewel fixed, remain 6 jewels can be arranged in 6 = 720 ways.
Since clockwise and anticlockwise arrangements are same, Required number of permutations.
1
720 = 360 ways.
2
3. In how many ways can 7 people be arranged at a round table so that 2 particular persons always
sit together.
Solution: First, the two particular persons can be arranged in 2 = 2 ways. .

Considering them as one fixed person, the remaining 5 persons can be arranged in 5 = 120 ways.
The required number of permutation = 120 2 = 240.
4. A round table conference is to be held between delegates of 9 countries. In how many ways can
they be seated if 2 particular delegates must not sit next to each other?
Solution: The number of ways in which 2 particular delegates must not sit next to each other = Total
number of permutations Number of permutations in which 2 particular delegates sit next to each
other.
Now Total number of circular permutations of 9 delegates = 9 1 = 8

Considering 2 delegates as one fixed person, the remaining 7 delegates can be arranged in 7 ways.
2 delegates again can be arranged in 2 ways.
Number of permutations in which 2 particular delegates sit next to each other = 7 2
Required number of permutation
a f
= 8 7 2 = 8 7 7 2 = 7 8 2 = 6 7 = 30240
5. In how many ways can 6 persons sit around a table so that all shall not have the same neighbours
in any 2 arrangements?
Solution: 6 persons can sit round a table in 5 = 120 ways. But each person will have the same
neighbours in clockwise and anticlockwise arrangements.
Permutation and Combination 33

5 120
Required number of ways = = = 60.
2 2
6. In how many ways can 4 gentlemen and 4 ladies sit down together at a round table so that no
two ladies may come together.
Solution: Let the gentlemen first take up their seats. They can sit in 3 = 6 ways. When they have
been seated, there remain 4 places for the ladies each between 2 gentlemen. Therefore the 4 ladies can
sit in 4 places in 4 = 24 ways.
Required number of ways = 3 4
= 6 24 = 144.

2.8 COMBINATION
Each of the different groups or selection which can be made by taking some or all of a number of things
at a time (irrespective of the order) is called a combination.
The number of ways of selection of n different things taken r at a time is called the number of
combination of n different things taken r at a time. It is written as ncr.
2.8.1 Value of ncr:
The number of combinations of n different things taken r at a time can be arranged in r ways.

nc
r combinations will produce n cr r permutations.

Now n cr r = Number of permutations of n different things taken r at a time = npr.


n
cr r = n pr
n
n pr
cr =
r

n n
But pr =
nr

n n
cr = .
nr r

Note: (1) When r = n.


1. When r = n.

n n n
cn = = =1 3 0 =1
n n n 0 n
n
cn = 1
34 Basic Mathematics

2. When r = 0

n n
cr = n c0 = =1
n0 0
n
c0 = 1
3. When r = 1.

n n n n 1
c1 = = =n
n 1 1 n 1
nc = n.
1

2.8.2 Complementary Combinations:


1. Prove that ncr = ncnr
Proof: 1. By using formula for ncr:

n n
LHS: cr = ...(1)
nr r

n n n
RHS: cn r = =
nn+r nr r nr

n n
cn r = ...(2)
nr r
From (1) and (2)
n
cr = n cn r
Proof: By analytic method:
nc is selecting r things from n things. If we select r things from n things then ( n r) things are left.
r
For every combination of (n r) things, there corresponds a combination of r things.
nc = nc
r n r. Hence proved.
n n
2. Prove that cr + cr 1 = n + 1 cr
Proof: By using formula for ncr
LHS: nc + nc
r nr

n n
=
nr r
+
n r 1 r 1 a f
n n
= +
n r r r 1 n r + 1 r 1

n n
=
n r r r 1
+
an r + 1f n r r 1
Permutation and Combination 35

n 1LM 1 OP
=
N
+
n r r 1 r n r +1 Q
n LM n r + 1 + r OP
n r r 1 N r an r + 1f Q
=

n an + 1f n +1
= =
n r + 1 r n +1 r r
= n + 1c = RHS.
r
Proof by analytic method: n + 1cis the total number of combination of n + 1 things taken r at a time
r
which is nothing but the combination that contain a particular thing (ncr) plus the combination that do
not contain a particular thing (ncr 1).
n +1
i.e., cr = n cr + n cr 1

WORKED EXAMPLES:
1. Find the value of (i) 7c3 (ii) 5c2 + 5c1
( i) We have

n n
cr =
nr r

7 7
c3 =
7 3 3

765 4
= = 35.
4 3 2 1

(ii) 5
c2 + 5c1 = 5+1c2 = 6 c2 3 n cr + n cr 1 = n +1cr

6 6 65 4
Now c2 = = = 15.
6 2 2 42

OR

5 5 5
c2 + 5 c1 = +
5 2 2 5 1 1

5 4 3 5 4
= +
32 4 1
= 10 + 5 = 15.
36 Basic Mathematics

2. Find n if nc20 = nc6


Solution: In the formula ncr = ncn r
We observe r + n r = n
n
c20 = n c6
20 + 6 = n
n = 26.
3. If nc7 = nc23, find nc29.
n
c7 = n c23 d3 n
cr = n cn r r + n r = n i
n = 7 + 23
n = 30

n 30
Now, c29 = 30 c29 =
30 29 29
= 30.
4. If 20c = 20c then find r.
r+2 2r 6,
20
Given cr + 2 = 20 c2 r 6
r + 2 = 2r 6
2 + 6 = 2r r
r=8

OR

20
cr + 2 = 20 c2 r 6
n
r + 2 + 2 r 6 = 20 cr = n cn r r + n r = n

3r 4 = 20
3r = 24
r = 8.
5. If nc2 = 36, find n.

n n
Solution: c2 = = 36
n2 2

a f
n n 1 n 2
= 36
n 22


a f
n n 1
= 36
2
Permutation and Combination 37

n (n 1) = 72
n (n 1) = 9 8 (By inspection)
n = 9.
6. If nc5 = 24 nc4, find n.
n
c5 = 24 n c4

n n
= 24
n 5 5 n 4 4

n 4 4
= 24
n 5 5

an 4f n 5 4 = 24
n 55 4
n 4 = 120
n = 120 + 4
n = 124.
n n
7. If pr = 60 and cr = 10, then find n and r.
n
n pr
Solution: We have cr =
r

60
10 =
r

60
r= =6
10
r= 3
r = 3.
Given: np = 60
r
np = 60
3

n
= 60
n3 60
2
a fa
n n 1 n 2 n 3 f
= 60
2
3
30
15
n3 5 5
n (n 1) (n 2) = 60 1
n (n 1) (n 2) = 5 4 3. (By inspection)
n = 5.
38 Basic Mathematics

8. If 18c
r =18cr + 2, then find rc5.
Given 18c = 18c
r r+2

3
LM c = c OP
n
r
n
n r
r r+2
N r + n r = nQ
r + r + 2 = 18
2 r = 16
r = 8.
r 8
c5 = 8 c5 =
8 5 5
876 5
= = 56.
3 2 1 5
9. In how many ways can 4 persons be selected from amongst 9 persons? How many times will a
particular person be always selected?
Solution: The number of ways in which 4 persons can be selected from amongst 9 persons
9 98 76
= 9c4 = = = 126.
9 4 4 4 3 2 1
Let a particular person is selected always. Then we have to select 3 persons from the remaining 8
8
persons. This can be done in 8c3 ways = = 56.
8 3 3
10. A student has to answer 7 out of 10 questions in an examination. How many choices has he, if
he must answer the first three questions.
Solution: There are 10 questions of which a student must answer first 3 questions. Remaining 4
questions (3 he has to answer 7 questions) can be selected among 10 3 = 7 questions in 7c4 ways.
7
Number of combinations = 7 c 4 =
7 4 4

765 4
=
3 2 1 4
= 35 ways.
11. In how many ways 5 red and 4 green balls can be drawn from a bag containing 7 red and 8 green
balls.
7 7
Solution: Number of ways of drawing 5 red balls from 7 red balls = c5 =
7 5 5
76 5
= = 21.
2 5
Permutation and Combination 39

8 8
Number of ways of drawing 4 green balls from 8 green balls = c4 =
8 4 4

8 765
= = 70.
4 3 2 1
Total number of ways (By fundamental principle)
= 70 21
= 1470.
12. Find the number of (a) Straight lines (b) triangles that can be drawn from 20 points of which
4 are collinear.
Solution: Two points are needed for a straight line. If none of the 20 points are collinear then we would
get 20c2 straight lines. But 4 points are given to be collinear. So we would not get 4c2 lines, instead we
get only one straight line containing all the 4 points.
20
Number of straight lines = c2 4 c2 + 1

20 4
= +1
20 2 2 4 2 2

20 19 4 3
= +1
2 2
= 190 6 + 1 = 185.
(b) We need 3 non-collinear points for a straight line. If none of the 20 points are collinear then we
would get 20c3 triangles. Since 4 points are given to be collinear, we would not get 4c3 triangles
from these points.
Number of triangles = 20c3 4c3
20 4
=
20 3 3 4 3 3
= 1140 4 = 1136.
13. A committee of 10 members is to be chosen from 9 teachers and 6 students. In how many ways
this can be done if
( i) The committee contains exactly 4 students.
(ii) There is to be a majority of teachers.
( iii) There are atleast 4 students.
( iv) There are at most 7 teachers.
Solution:
( i) The committee contains exactly 4 students and 10 4 = 6 teachers.
4 students can be selected out of 6 students in 6c4 ways and 6 teachers out of 9 teachers can be
selected in 9c6 ways.
40 Basic Mathematics

The number of selections =6c4 9c6 (By fundamental principle)


6 9
=
6 4 4 9 6 6

65 4 987 6
=
2 1 4 6 3 2 1
= 15 84 = 1260.
(ii) As there is to be majority of teachers, the committee may consist of
( a) 6 teachers 4 students
( b) 7 teachers 3 students
(c ) 8 teachers 2 students
( d) 9 teachers 1 student.
Number of selection =
9
( a) c6 6 c 4 = 1260
9
( b) c7 6 c3 = 720
9
(c ) c8 6 c2 = 135
9
( d) c9 6 c1 = 6
Total number of selections = 1260 + 720 + 135 + 6
= 2121.
( iii) As there is to be at least 4 students, the committee may consist of
( a) 4 students 6 teachers
( b) 5 students 5 teachers
(c ) 6 students 4 teachers.
Number of selections =
6
( a) c4 9 c6 = 1260
6
( b) c5 9 c5 = 756
6
(c ) c6 9 c 4 = 126
So Total number of selections =
1260 + 756 + 126 = 2142.
(iv) As there is to be atmost 7 teachers, the committee may consist of
( a) 7 teachers and 3 students
( b) 6 teachers and 4 students
(c ) 5 teachers and 5 students
( d) 4 teachers and 6 students.
Permutation and Combination 41

Number of selections =
9
(a ) c7 6 c3 = 720
9
(b ) c6 6 c 4 = 1260
9
(c ) c5 6 c5 = 756
9
(d ) c4 6 c6 = 126

Total number of selections


= 720 + 1260 + 756 + 126 = 2862.
14. A team of eleven is to be chosen out of 16 cricketers of whom 4 are bowlers and 2 others are
wicket keepers. In how many ways can the team be chosen so that there are at least 3 bowlers
and at least one wicket keeper.
Solution:

4 Bowlers 2 Wicket keepers 10 others

(a) 3 1 7
(b) 4 1 6
(c ) 3 2 6
(d) 4 2 5

Number of ways
4
(a) c3 2 c1 10 c7 = 960
4
(b) c4 2 c1 10 c6 = 420
4
(c ) c3 2 c2 10 c6 = 840
4
(d) c4 2 c2 10 c5 = 252
Total number of ways = 960 + 420 + 840 + 252
= 2472.
15. Arun has 7 friends, 4 of them are boys and 3 are girls. His sister, Aalekya has 7 friends, 4 of them
are girls and 3 of them are boys. In how many ways can they invite for a party of 3 girls and
3 boys. So that there are 3 of Aruns friends and 3 of Aalekyas friends.
42 Basic Mathematics

Solution:

Arun's friends Aalekya's friends

4 boys 3 girls 3 boys 4 girls

(a) 3 3
(b) 2 1 1 2
(c ) 1 2 2 1
(d) 3 3

Number of selections:
4
( a) c3 3c0 3 c0 4 c3 = 16
4
( b) c2 3 c1 3 c1 4 c2 = 324
4
(c ) c1 3 c2 3c2 4 c1 = 144
4
( d) c0 3c3 3 c3 4 c0 = 1
Total number of selections = 16 + 324 + 144 + 1
= 485.
16. How many diagonals are there in a octagon?
Solution: Number of diagonals in octagon = 8 c2 8 [3 octagon has 8 sides]
= 20.

REMEMBER:

n n
pr =
nr
n n n
p0 = 1, pn = n p1 = n
Permutation of n objects of which p objects are of one kind, q are of another kind and so on is
n
.
p q ...

Number of circular arrangement of n persons round a table = n 1

n 1
Number of circular arrangement of n beads or n flowers to form a necklace or garland = .
2

n n
cr =
nr r
Permutation and Combination 43

n
cn = 1
n
c1 = n
n
c0 = 1
n
cr = n cn r

n
cr + n cr 1 = n +1cr
Number of straight lines that can be drawn from n points of which p points are collinear =
nc pc + 1.
2 2
Number of triangles that can be drawn from n points of which p points are collinear = nc3 pc3.
Number of diagonals in a polygon of n sides = nc2 n.

EXERCISE
I. Find the value of:
1. 10p3 2. 12p3 3. 15c8 4. 8c5 5. 14c10
II. Find n if
1. np2 = 90 2. nc3 = 20
III. Find r if
1. 6pr = 360 2. 13pr = 156
IV. Find n and r if
1. npr = 240 and ncr = 120 2. npr = 336 and ncr = 56
V.
1. If np4 = 12, np2 = 120, find n.
2. If np4 = 56, np2 = 120, find n.
VI.
1. How many 3 digit numbers can be formed by using the digits 9, 7, 6, 5, 3, 2 (repetitions not
allowed)?
(a) How many of these are less than 400?
(b) How many of these are multiples of 5?
(c) How many of these are multiples of 2?
2. In how many ways can the letters of the word STRANGE be arranged so that
(a) The vowels never come together.
(b) The vowels are never separated.
3. A shelf contains 6 Hindi books, 5 Kannada books and 8 English books. In how many ways can
they be arranged so that
(a) Hindi books are together?
(b) Hindi books are together and Kannada books are together.
44 Basic Mathematics

(c) Books of the same languages are together.


( d) No two English books are together.
4. How many arrangements of the letters of the word SUNDAY can be made if the vowels are to
appear only in the odd places.
5. How many numbers of four different digits can be formed using the digits 0, 1, 2, 3, 4, 5? How
many of them are even?
6. If the letters of the word GATE be permuted and the words so formed be arranged as in a
dictionary what will be the rank of the word?
7. Find the number of permutations of the letters of the word INSTITUTION, when all the letters
are taken at a time. How many of them (i) Have 3Ts together (ii) begin with 2Ns.
8. Find the number of permutations of the letters of the word ASSASSINATION. How many of
them (i) have 3 As together (ii) begin with 2 Ns.
9. Find the number of permutation of letters of the word TOMORROW. How many of them have
(i) 3Os together. (ii) End with 2Rs.
10. Find the number of ways in which 6 different beads can be arranged to form a necklace.
11. In how many ways can 5 persons sit around a table.
12. In how many ways can 4 boys and 4 girls be seated round a table so that no two boys are adjacent.
13. In how many ways can 7 persons sit around a table so that all shall not have the same neighbours
in any two arrangements?
14. A round table conference is to be held between delegates of 20 countries. In how many ways can
they be seated if 2 participants may wish to sit together always.
15. (i) If nc10 = nc6, find n. (ii) If 43cr 6 = 43c3r + 1, then find r.
16. From 8 lecturers and 4 students a committee of 6 is to be formed. In how many ways can this
be done so that the committee contains (i) exactly 2 students (ii) atleast 2 students.
17. How many (i) straight lines (ii) triangles are determined by 12 points, no three of which lie on
the same straight line.
18. How many (i) straight lines (ii) triangles are determined by joining 20 points in a plane of which
6 are collinear.
19. In how many ways a student can choose 8 questions from a set of 12 questions if the questions
1 and 10 are compulsory.
20. Find the total number of diagonals of a hexagon.
21. Out of 3 books on maths, 4 books on Physics and 5 books on Chemistry, how many collections
can be made, if each collection consists of
( i) exactly one book on each subject
(ii) at least one book on each subject.

ANSWERS
I. 1. 720 2. 132 3. 6435 4. 56 5. 1001
II. 1. n = 10 2. n = 6.
Permutation and Combination 45

III. 1. r=4 r=2


IV. 1. n =16, r = 2 2. n = 8, r = 3
V. 1. n=4 2. n = 8
VI. 1. 120 (a) 40 (b) 20 (c) 40
2. (a) 3600 (b) 1440
12
3. (a) 13 6 ( b) 10 6 5 (c) 3 6 5 8 (d) 11 p8
4. 144
5. 300, 156
6. 14
11
7. ( i) 30240 (ii) 10080
3 3 2

12 10 10
8. ( i) (ii)
144 24 72
9. 3360 ( i) 360 (ii) 120
10. 60
11. 24
12. 144
13. 360
14. 2 18
15. (i) 16 (ii) 12
16. (i) 420 (ii) 672
17. (i) 12c2 (ii) 12c3
18. (i) 176 (ii) 1120
19. 210
20. 9
21. (i) 60 (ii) 3255.
46 Basic Mathematics

3
Probability
3.1 INTRODUCTION:
The term probability refers to the chance of happening or not happening of an event. The theory of
probability provides a numerical measure of the elements of uncertainity. It enables us to take decision
under conditions of uncertainity with a calculated risk. The theory of probability has its origin in the
games of chance, related to gambling for instance throwing a dice or tossing a coin.
Generally speaking, the probability of an event denotes the likelihood of its happening. The value of
probability ranges between zero and one. If an event is certain to happen its probability would be 1 and
if it is certain that the event wouldnt take place, then the probability of its happening is zero. Ordinarily
in social sciences probability of the happening of an event is rarely 1 or 0. The reason is that in social
sciences we deal with situation where there is always an element of uncertainity about the happening
or not happening of an event.

3.2 TERMINOLOGY:
Before we give definition of probability, it is necessary that we familiarise ourselves with certain terms
that are used in this context.
(i) Random eexper
xperiment:
xper iment: It is an experiment which if conducted repeatedly under homogeneous
conditions doesnt give the same result. The result may be any one of the various possible out-
comes.
For example: If a die is thrown it wouldnt always fall with number 3 up. It would fall in any one
of six ways which are possible.
(ii) Trial and eev
vent: The performance of a random experiment is called a trial and the outcome an
event.
Event could be either simple or compound (or composite). An event is called simple if it corre-
sponds to a single possible outcome. Thus in tossing a die, the chance of getting 3 is a simple
event (Q 3 occurs in a die only once). However the chance of getting an odd number is compound
(Q odd numbers are more than one 1, 3 and 5).
(iii) Exhausti
Exhaustiv ve cases: All possible outcomes of an event are known as exhaustive cases. In the throw
of a single die the exhaustive cases are six, as the die has only 6 faces each marked with different
Probability 47

numbers. Similarly the number of exhaustive cases in tossing 2 coins would be 4: HH, HT ,TH
and TT (H-Head, T-tail).
(iv) oura
Favour able cases: The number of outcomes which result in the happening of a desired event are
called favourable cases. Thus in a single throw of a die the number of favourable cases of getting
an odd number are 3 (i.e. 1, 3 and 5).
(v) Mutually
Mutuall y eexc
xclusi
xc lusiv
lusive cases: Two or more cases are said to be mutually exclusive if the happening
of any one of them excludes the happening of all others in a single experiment. Thus in a throw
of a single die, the events 5, 4 and 3 are mutually exclusive.
(vi) Equally
Equall likel
y lik ely
el y cases: Two or more events are said to be equally likely if the chance of their
happening is equal, i.e., there is no preference of any one event over the other. Thus in the throw
of a die, the coming up of 1, 2, 3, 4, 5 or 6 is equally likely.
(vii) Independent
Inde pendent and de dependent
pendent eev
vents: An event is said to be independent if its happening is not
affected by the happening of the other events. So in the throw of a die repeatedly coming up of
5 on the first-throw is independent of coming up of 5 again in the second throw. However we are
successively drawing cards from a pack without replacement, the event would be dependent.

3.3 DEFINITION OF PROBABILITY:


We can define probability in 3 ways.
(i) Mathematical or classical definition.
(ii) Statistical or Empirical definition.
(iii) Subjective approach or set theoretic approach definition.

i Mathematical or classical definition:


If there are n mutually exclusive, exhaustive and equally likely simple events in a trial, m of them
are favourable to the occurrence of an event A. Then probability or chance of occurrence of A equal to

af
P A =
Number of favourable cases
Total number of all possible
equally likely cases

af
P A =
m
n
.

Note:
m
1. Since 0 m n we have 0 1.
n
or af
0 P A 1.
2. When m = n, P (A) = 1 and when m = 0, P (A) = 0, i.e. when m = n, the event A is certain and
when m = 0, the occurrence of event A is an absolute impossibility.
3. The probability of non-occurrence of A is denoted by
48 Basic Mathematics

di
P A =
Number of unfavourable cases
Total number of all possible cases
.

di
P A =
nm
n
n m
=
n n

=1
m
n
=1 P A af
P d A i = 1 P a Af

P a Af + P d A i = 1 .

4. The main disadvantage of mathematical method is that it fails when there are infinite number of
possible outcomes and it cannot be applied to trials where the outcomes are not equally likely.

ii Statistical or Empirical definition of probability:


If a random experiment is repeated for an indefinitely large number of times under identical conditions,
then the limiting value of the ratio of the number of times an event occur to the total number of trials
is said to be the probability of occurrence of the event, provided the limit is a definite finite number.
If T is the number of trials and event A occurs f times in T trials, then the probability of occurrence

af F f I.
of event A is given by probability = P A = lim
T HTK
We use this method when the elementary events are not equally likely and the exhaustive number of
cases in a trial is infinite. The limitation of this method is that in practice an identical experimental
condition doesnt exist while repeating a random experiment for a large number of times. Moreover the
f
relative frequency i.e. may not attain a unique limiting value when T .
T

iii Subjective probability or set theoretic approach:


A set of points representing all possible elementary outcomes of a random experiment is called the
sample space (S). The number of all possible sample points in the sample space S is represented by n
(S).
The definition of probability is based on the following assumptions. (i) Total number of elementary
events in the sample space (S) is finite say N (ii) N elementary events of the experiment are equally
likely.

af
P A =
Number of elementary events favourable to event A
Total number of equally elementary events in S.
Probability 49

i.e., a f nnaaASff = Total


P A =
Number of sample points in A.
number of sample points in S.
Note: A pack of cards contain 52 cards, 26 red cards and 26 black cards.
Among 26 black cards, 13 are calavar and 13 are spade. Among 26 red cards, 13 are Diamond and
13 are hearts.
Each symbol contain A, Q, K, J, 2, 3, 4, 5, 6, 7, 8, 9, 10. (13 cards)
So there are 4 As, 4 Qs, ... 4-10s.
A is also known as Ace.
Face cards are A, Q, K and J.

WORKED EXAMPLES:
1. If one card is drawn at random from a well shuffled pack of 52 cards. Then find the probability
of each of the following.
(a) Drawing an ace card,
(b) Drawing a face card,
(c) Drawing a diamond card,
(d) Drawing either spade or hearts,
(e) Not drawing an ace of hearts.
Solution: (a) One card can be drawn out of 52 cards in 52c1 = 52 ways = n (S).
One ace card can be drawn out of 4 ace cards in 4c1 = 4 ways = n (A).

Number of favourable cases af


n A
Probability of drawing an ace card = =
af
Total number of all possible equally likely cases n S

4 1
= = .
52 13
(b) A face card can be drawn out of 12 face cards in 12c1 = 12 ways.
Number of favourable cases = 12
Total number of all possible equally likely cases = 52c1 = 52.
12 3
Probability of drawing a face card = = .
52 13
(c) A diamond card can be drawn out of 13 diamond cards in 13c = 13 ways.
1
Probability of drawing a diamond card
13 1
= = .
52 4
(d) There are 13 spade and 13 hearts cards in a pack of cards. Either a spade or a heart can be drawn
in 26c1 = 26 ways.
50 Basic Mathematics

26 1
Probability of drawing either a spade or a hearts card = = .
52 2
(e) There is one ace of hearts.
1
Probability of drawing an ace of hearts = .
52
1 51
Probability of not drawing an ace of hearts = 1 = .
52 52
2. Three balls are drawn at random from a bag containing 6 blue and 4 red balls. What is the
probability that two balls are blue and one is red?
Solution: The bag contains (6 + 4) = 10 balls.
10
3 balls can be drawn out of 10 balls in 10c3 = = 120 ways.
10 3 3
Total number of cases = 120.
6
Now 2 blue balls can be drawn out of 6 in 6c2 = = 15 ways.
62 2
1 red ball can be drawn out of 4 in 4c1 = 4 ways.
2 blue balls and 1 red ball can be drawn in 15 4 = 60 ways.
Number of favourable cases for the event = 60.
60 1
Probability of drawing 2 blue balls and 1 red ball = = ..
120 2
3. Three unbiased coins are tossed. What is the probability of obtaining (a) all heads (b) two heads
(c) one head (d) atleast one head (e) atleast two heads (f) All tails.
Solution: There are 23 = 8 mutually exclusive exhaustive and equally likely cases
HHH, HHT, HTH, THH, HTT, THT, TTH, TTT.
1
(a) Probability of all heads = [There is only one HHH amongst 8 possiblities]
8

3
HHT LM OP
(b) Probability of 2 heads =
8
HTH
THH MN PQ
3
HTTLM OP
(c) Probability of 1 head =
8
THT
TTH MN PQ
7
(d) Probability of atleast one head =
8
Probability 51

HHH LM OP
4 1
(e) Probability of atleast 2 heads = =
8 2
HHT
HTH MM PP
THH N Q
1
(f) Probability of all tails = .
8
4. The marks obtained by 100 students are given below.

Marks 0 10 11 20 21 30 31 40 41 50 51 60 61 70 71 80 81 90 91 100
No. of students 5 10 13 14 17 7 11 8 9 6

If a student is selected at random from the entire group of 100 students, find the probability that his
marks (i) is under 40. (ii) above 50 (iii) either between 31 to 40 or 41-50.
(i) Total number of students = 100.
Number of students obtaining marks less than 40 = 5 + 10 + 13 + 14 = 42.
42 21
Required probability = = .
100 50
(ii) Number of students scoring above 50
= 7 + 11 + 8 + 9 + 6 = 41
41
Required probability =
100
(iii) Number of students obtaining marks between 31 to 40 = 14 and number of students obtaining
marks between 41 to 50 = 17.
Total number of students whose score is either between 31 to 40 or 41 to 50 = 14 + 17 = 31.
31
Required probability =
100
5. If a pair of dice is thrown, find the probability that the sum of digits is neither 7 nor 11.
Solution: A pair of dice is thrown.
n (S) = 6 6 = 36.
Let A be an event of getting the sum 7 and B be an event of getting the sum 11.
Then A = {(1, 6), (2, 5), (3, 4), (4, 3), (5, 2), (6, 1)}
n (A) = 6
B = {(5, 6) (6, 5)}
n (B) = 2.

af
n A 6
Probability of getting 7 =
af
n S
=
36
.
52 Basic Mathematics

af
n B 2
Probability of getting 11 =
af
n S
=
36
.

6 2 8
Probability of getting either 7 or 11 = + = .
36 36 36
8 28 7
Probability of getting neither 7 nor 11 = 1 = = .
36 36 9

3.4 ADDITION RULE OF PROBABILITY:


Sta tement: If A and B are 2 events, then probability that at least one of them occurs is given by
Statement:
a f af af a
P A B = P A + P B P A B f
Pr oof: Consider the Venn diagram. The shaded portion denotes AB, i.e., set of all outcomes where
Proof:
some of the outcomes are common to both A as well as B. So P (AB) is the probability of happening
of atleast one of the events A and B.

A B
A

B

Fig. 3.1

P (A) + P (B) is the sum of all the probabilities in A and all the probabilities in B.
So the probability in A B has been added twice in P (A) as well as in P (B). So we must subtract
P (A B) once from P (A) + P (B) to obtain probabilities in AB.
a f af af a
P A B = P A + P B P A B f
Corollar
Corollary: If A and B are mutually exclusive, then P (AB) = P (A) + P (B)
ollary:
Proof:
Proof: From addition rule we have
a f af af a
P A B = P A + P B P A B f
Since A and B are mutually exclusive, A and B are disjoint sets.
AB =

a f
P A B = 0

P a A Bf = P a Af + P a Bf 0

P a A Bf = P a Af + P a Bf
Probability 53

Note: 1. P (AB) means P (A or B) i.e., probability of happening of atleast one of the events A and B.
P (A B) means P (A and B) i.e., probability of happening of both the events A and B.
2. P (ABC) is probability of happening of atleast one of the events A, B and C. It is given by
addition rule as
a f af af af a f
P A BC = P A + P B + P C P A B

P a B C f P aC Af + P a A B C f
This can be proved by writing the Venn diagram.

A B

Fig. 3.2

WORKED EXAMPLES:
1. A ticket is drawn from a bag containing 25 tickets bearing number 1, 2, 3, ..., 24, 25. Find the
probability of its bearing a number which is either even or a multiple of 3.
Solution: The events even number and a multiple of 3 are not mutually exclusive as there are some
numbers which are even as well as multiples of 3. Ex: 6, 12, 24.
P (an even number or a multiple of 3)
= P (an even number) + P (a multiple of 3) P (an even number and a multiple of 3)

ck ph ma fr ma
P 2, 4, 6, 8, ..., 24 + P 3, 6, 9, ..., 24 P 6, 12, 24 fr
12 8 3 17
= + = .
25 25 25 25
[Since there are 12 even numbers from 1 to 25, 8 multiples of 3 and 3 numbers which are even as
well as multiples of 3 from 1 to 25].
2. What is the probability of getting either total of 7 or 11 when a pair of dice is tossed?
Solution: Total outcomes when a pair of dice is tossed = 6 6 =36.
The events a total of 7 and a total of 11 are mutually exclusive events.
P (a total of 7 or 11)
= P (a total of 7) + P (a total of 11).
54 Basic Mathematics

= P {(6, 1), (5, 2), (4, 3) (3, 4) (2, 5) (1, 6)} + P {(6, 5) (5, 6)}
6 2 8 2
= + = = .
36 36 36 9
3. The probability that a contractor will get a plumbing contract is 2/3 and the probability that he
will not get an electric contract is 5/9. If the probability of getting atleast one contract is 4/5, what
is the probability that he will get both the contracts.
Solution: Let A be an event that a contractor gets plumbing contract.
B be an event that a contractor gets electrical contract.
Then Given:

af
P A =
2
3

di 5
P B = .
9

af di
P B = 1 P B = 1
5 4
= .
9 9

a f 45
P A B =

P a A Bf = ?
From addition rule,
a f af af a
P A B = P A + P B P A B f
= + P a A Bf
4 2 4
5 3 9

a
P A B = f 2 4 4
+
3 9 5
30 + 20 36
=
45

a
P A B = f 14
45
.

14
Probability that a contractor will get both the contracts = .
45
4. One card is drawn from a pack of 52 cards, what is the probability that the card drawn is neither
red nor king.
Solution: The event card drawn is red and card drawn is king is not mutually exclusive because
there are two cards in the pack which are red as well as king.
P (card drawn is red or king)
Probability 55

= P (card drawn is red) + P (card drawn is king) P (card drawn is red and king)
Since there are 26 red cards, 4 king cards and 2 cards which are red as well as king,
P (Cards drawn is red or king),
26 4 2 28
= + = .
52 52 52 52
P (card drawn is neither red not king)
= 1 P (card drawn is red or king)
28 52 28 24
=1 = = .
52 52 52
6
= .
13
5. A card is drawn at random from a well shuffled pack of 52 cards. What is the probability that it
is a heart or a queen or black card.
Solution: Let A be an event that the card drawn is heart.
B be an event that the card drawn is queen and C be an event that the card drawn is black.
A, B and C are not mutually exclusive events. So
a f af af af a f
P A BC = P A + P B + P C P A B

P a B C f P a C Af + P a A B C f .
There are 13 heart cards,
4 queen cards,
26 black cards,
1 heart queen card
2 queen black cards and
No heart black card.
No card which is heart, queen and black.

a
P A BC = f 13 4 26 1
+ +
2
52 52 52 52 52
0+0

a
P A BC = f 13 + 4 + 26 1 2
52
40 10
= = .
52 13
P (Card drawn is heart or queen or black)
10
= .
13
56 Basic Mathematics

This can be illustrated by the following Venn diagram.

12 other
12 Queen cards
Hearts 1 1
2
24
Black

Fig. 3.3

3.5 CONDITIONAL PROBABILITY:


Let us consider the following example. A fair dice is thrown and the number that appeared is even.
What is the probability that the number 4 has appeared? Since it is given that the number appeared is
even. Possible outcomes are no longer {1, 2, 3, 4, 5, 6} but only {2, 4, 6}. Out of these 3 possible
outcomes there is 1 outcome in favour of appearance of 4. Probability of appearance of 4, given that
1
an even number has appeared = .
3
Formally, if E is the event The number that appeared is 4 and F the event Number that appeared
is even then P (E/F) denotes the probability of E given that F has happened.

a f
P E F = .
1
3
The probability that an event B occurs subject to the condition that A has already occurred is called
the conditional probability of occurrence of the event B. It is denoted by P (B/A).

3.6 MULTIPLICATION RULE:


Sta tement: The probability of the simultaneous occurrence of 2 events is the product of the probability
Statement:
that one of the events will occur and the conditional probability that the other event will occur given that
the first event has occurred.
If A and B are two events then P (A B) is the probability of their simultaneous occurrence and it
is given by
a f af a f
P A B = P A P B A

or P a A Bf = P a Bf P a A Bf
Proof: Let A be any event with sample points n (A), i.e., P (A) > 0. If n (S) is the total number of sample
Proof:
points in S and B = another event such that A and B are not disjoint sets. (i.e., AB ). Let the sample

points in AB be n (AB). Then from definition P A = a f nnaaASff


Probability 57

a
P A B = f n anAaSfBf

A A B

B

Fig. 3.4

Now Let P (B/A) = Conditional probability of event B given A has occurred =

a
n A B f a
n A B n S f af a f
P A B
=
n A af
=
n A n Saf af
=
P A af
P a A Bf
P a B Af =
P a Af
i.e.,

P a A Bf = P a Af P a B Af
Similarly we can establish
a f af a f
P A B = P B P A B

Alternative Proof:
If A and B are 2 mutually dependent events then conditional probability of B when A has occurred is
proportional to P (AB).
i.e, a f a
P B A P A B f
a f a f
P B A = K P A B where K is the constant of proportionality
To find K,
Conditional probability of A when A has occurred is 1
a f
P A A = KP A A a f
af
1 = KP A
1
K=
af
P A
58 Basic Mathematics

Substituting K=
1
af
P A
a f
in P B A = KP A B a f
We get

a f P a1Af P a A Bf
P BA =

P a A Bf = P a Af P a B Af
Corollar
Corollar y: If A and B are independent events then P (A B) = P (A) P (B).
ollary:
oof: If A and B are independent events then P (B/A) = P (B) and P (A/B) = P (A).
Proof:
Pr
By multiplication theorem,
a f af a f
P A B = P A P B A
Substituting P (B/A) = P (B) we get
a f af af
P A B = P A P B
Hence proved.
Note: 1. The multiplication theorem can be extended. For 3 dependent events A, B and C
a f af a f a
P A BC = P A P B A P C A B f
2. If there are n independent events A1, A2, .... An,
b g b g b g b g b g
P A1 A2 ... An = P A1 P A2 P A3 ... P An .

WORKED EXAMPLES:
1. A pair of dice is thrown and sum of the numbers on the two dice comes to be 7. What is the
probability that the number 4 has come on one of the dice?
Solution: Let the events A and B be such that
Event B: Sum of numbers on the two dice is 7.
Event A: The number 4 has come.
Total outcomes when a pair of dice is thrown = 36 = n (S)

a f ma f a f a f a f a f a fr
P B = P 6, 1 , 5, 2 , 4, 3 , 3, 4 , 2, 5 , 1, 6

P a Bf = .
6
36
To get P (AB) select the outcomes favourable to A from the outcomes that are favourable to B.

a f ma f a fr
P A B = 4, 3 , 3, 4 =
2
36
.

From Multiplication theorem,


a f af a f
P A B = P A P A B
Probability 59

a f P aPAaAfBf
P A B =

2
a f 2 1
P A B = 36 = = .
6 6 3
36

1
Probability that the number 4 has come on one dice given that sum of numbers on 2 dice is 7 = .
3
2. Two cards are drawn from a pack of 52 cards with replacement (i.e., the second card is drawn
after replacing the first card in the pack). Find the probability that (a) Both are ace, (b) First card
is jack and second card is king, (c) One is king and other is queen.
Solution: (a) In a pack of 52 cards, there are 4 ace cards.
P (both cards are ace)
4 4 1
= = .
52 52 169
(b) P (First card is jack and second card is king)
4 4 1
= = .
52 52 169
[Q There are 4 jack and 4 king cards in a pack of 52 cards]
(c) P [One card is king and other is queen]
= P [First is king and 2nd is queen] P [First is queen and 2nd is king]
4 4 4 4
= + [Using addition theorem P (AB) = P (A) + P (B)]
52 52 52 52
1 1 2
= + = .
169 169 169
3. Two cards are drawn without replacement from a pack of 52 cards. What is the probability that
(i) both are queen.
(ii) both are diamond cards.
(iii) one is king and the other is ace.
Solution: We are taking 2 cards from 52 cards. This can be done in 52c2 ways. There are 4 queen cards
from which we require 2 queen cards. This can be done in 4c2 ways.
4
c2 43
Required Probability = =
52
c2 52 51
1
= .
221
60 Basic Mathematics

OR
There are 4 queen cards in a pack of 52 cards.
4
Probability of drawing first queen card =
52
Since the card drawn is not replaced, we are left with 51 cards and 3 queen cards.
3
Probability of drawing 2nd queen card =
51
4 3 1
Probability of both queen cards = = .
52 51 221
13
c2
(ii) Required probability = 52
c2

13 12
= 2 1 = .
1
52 51 17 (Since there are 13 diamond cards).
2 1
(iii) There are 4 favourable choices to take out a king and 4 favourable choices to take out an ace.
Number of favourable cases = 4c1 4c1
In total, there are 52 cards out of which any 2 cards can be taken.
4
c1 4 c1
Required probability = 52
c2

442 8
= = .
52 51 663
4. A lot contains 10 items of which 3 are defective. 3 items are chosen from the lot at random one
after another without replacement. Find the probability that all the 3 are defective.
Solution: Let A, B and C be the events of drawing defective items in the first, second and third drawing
respectively. Hence Probability of all the three items being defective is given by
a f af a f a
P A BC = P A P B A P C A B f
3 2 1
=
10 9 8
1
= .
120

OR
3 defective items can be picked from 3 defective items in 3c3 ways.
Probability 61

3 items can be picked from 10 items in 10c ways.


3
3
c3
Required probability = 10
c3

1 6
=
10 9 8 10 9 8
3 2 1

1
= .
120
5. Anil and Bharath appear in an interview for 2 vacancies. The probability of their selection being
1 1
and respectively.
7 5
Find the probability that (i) both will be selected (ii) only one is selected (iii) none will be selected
(iv) atleast one of them will be selected.
Solution: Let A: Anil be selected.
B: Bharath be selected.

Given af
P A =
1
7
and P B =
1
5
af
P (both will be selected)
a f a
= P A and B = P A B f
= P a Af P a Bf [Q A and B are independent events]

1 1 1
= = .
7 5 35
(ii) P (Only one will be selected)

= P A and B or A and B

d i d i
= P A B A B

= P a Af P d B i + P d A i P a Bf

= F 1 I + F1 I
1 1 1 1
7 H 5K H 7K 5
1 4 6 1
= +
7 5 7 5
4 6 10 2
= + = = .
35 35 35 7
62 Basic Mathematics

(iii) P (none will be selected)

d
= P A and B i
= P d A B i = P dA i PdB i

= F1 I F1 I
1 1
H 7 K H 5K
6 4 24
= = .
7 5 35
(iv) P (at least one of them will be selected)
= 1 P (none of them will be selected)
24 11
1 = .
35 35

REMEMBER:

P A =a f nnaaASff af
0 P A 1.

P d A i = 1 P a Af

P (AB) means P (A or B) and P (AB) means P (A and B)


Addition rule: P (AB) = P (A) + P (B) P (AB)
For mutually exclusive cases
P (AB) = P(A) + P(B)
a f af af af a f a f a f a
P A BC = P A + P B + P C P A B P BC P C A + P A BC f
P a A Bf = P a Af P a B Af

P a A Bf = P a Bf P a A Bf

P a A Bf = P a Af P a Bf if A and B are independent events.

P a A B C f = P a Af P a B Af P aC A Bf

If A , A , A , ... A are independent events P b A A ... A g = P b A g P b A g ... P b A g


1 2 3 n 1 2 n 1 2 n

EXERCISE
1. A bag contains 100 tickets each bearing a distinct number from 1 to 100. A ticket is drawn from
the bag. Find the probability that
Probability 63

(a) the ticket bears an odd number.


(b) the ticket bears a number divisible by 3.
(c) the ticket bears a number divisible by 3 or 5.
2. A pair of dice is thrown. Find the probability that the sum will be (a) equal to 4 (b) less than 4
(c) greater than 4.
3. A bag contains 4 white, 5 red and 6 green balls. 3 balls are drawn at random. What is the
probability that (a) All are green (b) All are white.
4. A card is drawn from a pack of playing cards. What is the probability of drawing
(a) black card (b) king card (c) diamond card.
5. A box has 3 silver and 2 gold coins. 2 coins are drawn at random. Find the probability that (a)
both the coins are silver ones. (b) both are gold coins (c) one of them is a silver coin.
6. A problem in statistics is given to 3 students A, B and C. Their probabilities in solving it are
1 1 1
, and respectively. What is the probability that the problem would be solved?
8 6 4
7. The following table gives a distribution of wages of 1000 workers.

a f
Wages Rs. 120 140 141 160 161 180 181 200 201 220 221 240 241 260
No. of workers 9 118 478 200 142 35 18

An individual is selected at random from the above group. What is the probability that his wages
are
(a) under Rs. 160 (b) above Rs. 200 (c) between Rs. 160 and 200.
8. One card is drawn from a pack of 52 cards. What is the probability that the card drawn is
(a) either red or king
(b) either king or queen
9. A ticket is drawn from a bag containing tickets bearing numbers 1 to 25. Find the probability that
the number is either even or a multiple of 3.
10. A coin and a die are thrown. What is the probability of getting a head or an even number.
11. A box contains 50 bolts and 150 nuts. Half of the bolts and half of the nuts are rusted. If one item
is chosen at random, what is the probability that it is rusted or a bolt?
12. A die is rolled. If the outcome is an odd number, what is the probability that it is prime.
13. A pair of dice is rolled. If the sum on the 2 dice is 9. Find the probability that one of the dice
showed 3.

14. If A and B are 2 events in a sample space S such that P A = af 1


2
d i5
8
a 3
, P B = , P A B = .
4
f
d
Find (a) P (AB) (b) P A B . i
15. 3 children are randomly selected from a class. What is the probability that (a) all 3 were born on
Monday (b) 2 were born or Friday and the other on Tuesday (c) none were born on Wednesday.
64 Basic Mathematics

ANSWERS

1 33 47
1. (a) (b) (c)
2 100 100
3 3 30
2. (a) (b) (c)
36 36 36
4 4
3. (a) (b)
91 455
1 1 1
4. (a) (b) (c)
2 13 4
3 1 3
5. (a) (b) (c)
10 10 5
290
6.
64
127 195 678
7. (a) (b) (c)
1000 1000 1000
28 2
8. (a) (b)
52 13
17 3 5
9. 10. 11.
25 4 8
2 1 1 1
12. 13. 14. (a) (b)
3 2 8 4
4
Binomial Theorem
4.1 INTRODUCTION:
We know

a x + af1
= x+a

a x + af 2
= x 2 + 2ax + a 2

a x + af 3
a
= x 3 + a 3 + 3ax x + a f
= x 3 + 3 x 2 a + 3xa 2 + a 3 .
The general expansion for (x + a)n where n is positive integer is given by Sir Isaac Newton and is
known as Binomial Theorem.

4.2 STATEMENT OF BINOMIAL THEOREM:


If x and a are real numbers and n is any positive integer then

a x + af n
= x n + n c1 x n 1a + n c2 x n 2 a 2 + n c3 x n 3 a 3 + ... n cn x n n a n .

= x n + nx n 1 a +
a f
x a +
a fa
n n 1 n 2 2 n n 1 n 2 n 3 3
x a + ... + a n .
f
2! 3!

n
Since,
n
cr =
n r r

n n n 1
n
c1 = = = n.
n 1 1 n 1

n
c2 =
a f
n n 1 n
, c3 =
n n 1 n 2 a fa f
2! 3!
66 Basic Mathematics

and so on ... ncn = 1.

By taking 1 = C0 , nc1 = C1, n c2 = C2 and so on ... n cn = 1 = Cn .


We can write,

a x + af n
= C0 x n + C1 x n 1a 2 + C2 x n 3 a 3 + ... Cn a n .

Note:

(1) a x + af
1
= x + a : Power : 1, Number of terms : 2

a x + af 2
= x 2 + 2ax + a 2 : Power : 2, Number of terms 3.

a x + af 3
= x 3 + 3 x 2 a + 3 xa 2 + a 3 . Power : 3, No. of terms : 4.
The number of terms in the expansion of (x + a)n is n + 1.
(2) In the expansion of (x + a)n, from left to right the power of x decreases by 1 from one term to the
next term and the power of a increases by 1.
(3) In any term, the power index of x plus the power index of a = n.
(4) The co-efficients C0, C1, C2 ... Cn are called binomial co-efficients.

(5) The general term in the expansion is n
cr x n r a r which is (r + 1)th term. Hence Tr +1 = n cr x n r a r .

(6) Since cr = n cn r , the binomial co-efficients of first and last term are equal: Second and penul-
n

timate term are equal and so on.


(7) If n is even then the number of terms in (x + a)n is (n + 1) which is odd. So there will be only one
middle term and it is TF n I . If n is odd then the number of terms in (x + a)
n is (n + 1) which is
H2 K +1

even. So there will be two middle terms. They are TF n +1 I and TF n +1 I


H2K H 2
+1
K

WORKED EXAMPLES:

F 2I
1. Expand: G x + J
5

H yK 2

FG x + 2 IJ 5
F2I
G J
1
F 2I
+ c x G J
2

H yK = x + c1 x
Hy K Hy K
5 5 4 5 3
2 2 2 2

F 2I
+ c x G J +
3
FG 2 IJ 4
FG 2 IJ 5

Hy K Hy K + 5 c5 x 0
Hy K
5 2 5
3 2 c4 x 1 2 2
Binomial Theorem 67

2 5 4 3 4 5 43 2 8 5 432 16 32 FG IJ
= x 5 + 5x 4
y2
+
2
x 4 +
y 3!
x 6 +
y 4!
x 8 + 10
y y H K
10 x 4 x 3 80 x 2 80 x 32
= x5 + + 40 + 6 + 8 + 10 .
y2 y4 y y y

F a bI
2. Expand H + K .
4

b a

F a + bI = F aI 4
F aI F bI + 4 3 F aI F bI + 4 3 2 F aI F bI + F bI
4 3 2 2 3 4

H b aK H bK H b K H a K 2 H b K H a K 3! H b K H a K H a K
+4

= F I + 4F I + 6 + 4 F I + F I .
4 2 2 4
a a b b
H bK H bK H aK H aK
F 1I F F 1II 4
3. Expand G x J = G x + G J J
4

H yK H H yKK
2 2

F 1 I 4 3 d x i FG 1 IJ + 4 3 2 d x i FG 1 IJ + FG 1 IJ
= dx i + 4 d x i G J +
2 4 2 3 2 2
2 3 4

H y K 2 H y K 3! H y K H y K
2

4 x6 6x 4 4 x2 1
= x8 + 2 3 + 4.
y y y y

4. Simplify: 3 + 2d i + d3 2 i
5 5

Solution: From Binomial theorem we have

a x + af 5
= x 5 + 5x 4 a +
54 3 2 5 4 3 2 3 5 4 3 2 4
2
x a +
3!
x a +
4!
xa + a 5 .

a x + af 5
= x 5 + 5 x 4 a + 10 x 3 a 2 + 10 x 2 a 3 + 5 xa 4 + a 5 . ...(1)
Changing a to a we get

a x af 5
= x 5 5 x 4 a + 10 x 3 a 2 10 x 2 a 3 + 5xa 4 a 5 . ...(2)
Adding (1) and (2) we get

a x + af + a x af
5 5
= 2 x 5 + 20 x 3 a 2 + 10 xa 4 .

= 2 x x 4 + 10 x 2 a 2 + 5a 4
68 Basic Mathematics

Putting x = 3 and a = 2 we get

d3 + 2 i + d3 2 i
5 5
LM d 2i 2
d 2 i OPQ
4

N
= 2 3 3 4 + 10 32 +5

= 6 81 + 180 + 5 a 4f

= 6 261 + 20 = 1566 + 120

= 1686.

5. Simplify: 1 + 2 d i d1 2 i
4 4

We have from Binomial theorem

a x + af 4
= x 4 + 4 x 3a +
43 2 2 432 3
2
x a +
3!
x a + a4

a x + af 4
= x 4 + 4 x 3a + 6 x 2 a 2 + 4 x a3 + a 4 ...(1)
Replacing a by a.

a x af 4
= x 4 4 x 3a + 6 x 2 a2 4 x a3 + a 4 ...(2)
Subtracting (2) from (1)

a x + af a x af
4 4
= 8 x 3 a + 8 xa 3

Putting x = 1 and a = 2 we get

d1 + 2 i d1 2 i
4 4
= 8 13 2 + 8 1 d 2i 3

= 8 2 + 82 2

= 8 2 + 16 2 = 24 2.

Fa I
6. Find the middle term in the expansion of H + bxK .
8

x
Solution: The expansion has 8 + 1 = 9 terms.
So T8 is the middle terms i.e., T5 is the middle term.
+1
2

We have

Tr +1 = n cr x n r a r .

F a + bxI 8
Comparing
Hx K with (x + a)n,
Binomial Theorem 69

a
We get x=
x
a = bx
n=8
r +1= 5
r=4
Substituting in

Tr +1 = n cr x n r a r

F a I abx f
8 4
T5 = 8 c4
H xK
5

c F I a bx f
4
a
=8
H xK
5
4

a4 5 5
= 8 c4 b x
x4

a4b5 x5
= 8 c4
x4

8
= a 4b5 x
84 4

8 7 6 5 4/ 4 5
= a b x
4 4/

8765
= a4b5 x
4 3 2 1

= 2 35a 4 b 5 x

= 70a 4 b 5 x.

F y I 11
7. Find the middle term in the expansion of 2 x + H x K .

Solution. The expansion has 11 + 1 = 12 terms.


So there are 2 middle terms.
Tn +1 and Tn +1 are middle terms i.e., T12 and T12 = T6 and T7 are middle terms.
+1 +1
2 2 2 2
70 Basic Mathematics

F y I 11
a f y
H
Comparing 2 x +
K with x + a we get x = 2x, a =
n
and n = 11.
x x

For T6 , r + 1 = 6 r = 5.

Tn +1 = n cr x n r a r formula

c a2 x f F I
5
y
H xK
11 5
T6 = 11
5

c a2 x f F I
5
y
= 11
H xK
6
5

11 y5
= 26 x6 5
11 5 5 x

11 10 9 8 7 6
T6 = 2 xy 5
5 4 3 2 1

T6 = 29568 xy 5 .

For, T7 , r + 1 = 7 r = 6

Tr +1 = n cr x n r a r .

a f F yI 6

H xK
11 6
T7 = 11c6 2 x

11 y6
T7 = 25 x 5 6
11 6 6 x

11 10 9 8 7 5 y 6
T7 = 2 .
5 4 3 2 1 x

y6
T7 = 14784 .
x

Fx 1 I . 9
8. Find the middle term in the expansion of
H2 x K 2

The expansion has 9 + 1 = 10 term.


So there are 2 middle terms Tn +1 and Tn +1 are middle terms.
+1
2 2
Binomial Theorem 71

T10 and T10 are middle terms.


+1
2 2

T5 and T6 are middle terms.

Fx 1 I 9
Comparing
H2 x K 2 with (x + a)n we get

x 1
x= , a = 2 , n = 9.
2 x
For T5, r + 1 = 5 r = 4.
Tr +1 = n cr x n r a r .

F xI F 1 I
9 4 4
T5 = 9 c 4
H 2K H x K 2
.

9 8 7 6 x5 1
T5 =
4 3 2 1 25 x8

126 x 5
T5 =
32 x 8
63
T5 = .
16 x 3
For T6, r + 1 = 6 r = 5.

F xI F 1 I
9 5 5
T6 = 9 c5
H 2K H x K 2

x4 1
= 126
2 4 x 10
126 63
= = .
x 6 8x 6

F y I 10
9. Find the middle term in the expansion of 3 x H K
2
.
3
The expansion has 10 + 1 = 11 terms.
So T10 = T6 is the middle term.
+1
2

F y I 10

H
Comparing 3 x 2
3 K with (x + a)n we get
72 Basic Mathematics

x = 3x 2
y
a=
3
n = 10
r +1= 6
r=5

Tr +1 = n cr x n r a r .

d i 10 5 F yI 5
T6 = 10 c5 3x 2
H 3K

10 5 10 y 5
T6 = 3 x 5
5 5 3

10 9 8 7 6 10 5
T6 = x y
5 4 3 2 1

T6 = 252 x 10 y 5 .

F 2 I 10
10. Find the middle term in the expansion of H x
x K .

The expansion has 10 + 1 = 11 terms.


So T10 is the middle term
+1
2

T6 is the middle term.

LM 2 OP 10
Comparing
N x
x Q with (x + a)n we get

n = 10,
x= x
2
a=
x
r+1=6
r=5

aformulaf
Tr +1 = n cr x n r a r

c d xi F I
5
2 10 5
T6 = 10 5 H xK
Binomial Theorem 73

T6 = c510
5
x2
a2f 5

x5
5
10 5
= x2 25
10 5 5

10 9 8 7 6 5 2 5
= x 2
5 4 3 2 1

= 252 x 5 2 2 5
= 8064 x 5 2

F 1 I.8
11. Find the co-efficient of x2 in the expansion of 3 x + H 2x K
F 1 I 8
1
H
Comparing 3 x +
2x K with (x + a)n we get x = 3x, a =
2x
and n = 8.

Tr +1 = n cr x n r a r formula

a f FH 21x IK
r
8 r
Tr +1 = 8 cr 3 x

c a3 x f F I
r
1
H 2x K
8r
Tr +1 = 8 r

1
= 8 cr 38 r x 8 r
2 xr
r

= 8 cr 38 r 2 r x 8 r r

Tr +1 = 8cr 38 r 2 r x 8 2 r .
To get the co-efficient of x2, equating power index of x to 2.
8 2r = 2
8 2 = 2r
6 = 2r

r = 3.

Substituting r = 3 in (1),

T3+1 = 8 c3 383 2 3 x 8 2 a 3f
74 Basic Mathematics

8
T4 = 35 2 3 x 2
8 3 3

8 7 6 35 2
T4 = x
3 2 23

35
T4 = 56 3
x2.
2
Co-efficient of x2 is 35 7.

Fx 1 I 15

H 3
K
4
12. Find the co-efficient of x25 in the expansion of .
x

F 1 I 15

H K
1
Solution. Comparing x with (x + a)n we get x = x4, a =
4
and n = 15.
x3 x3

Tr +1 = n cr x n r a r formula

d i 15 r F 1 I r
Tr +1 = 15 cr x 4
H

x3 K
a f
Tr +1 = 15 cr x 60 4 r 1 2 x 3r

= 15cr x 60 4 r 3r a 1f r

Tr +1 = 15 cr x 60 7r a 1f r
...(1)
To get the coefficient of x25, equating power index of x to 25.
60 7r = 25
60 25 = 7r
35 = 7r
r = 5.
Substituting r = 5 in (1)

T5+1 = 15 c5 x 25 1 5 a f
T6 = 15 c5 x 25 3 1 5 = 1 a f
Co-efficient of x25 = 15c5.

F 3 I 25
13. Find the co-efficient of x60 in 2 x 2
H x3 K .
Binomial Theorem 75

F 3 I 25
3
H
Comparing 2 x 2
x3 K with (x + a)n we get x = 2x2, a =
x3
and n = 25.

Tr +1 = n cr x n r a r formula

d i 25 r F 3 I r
Tr +1 = 25 cr 2 x 2
H x3 K
= 25 cr 2 25 r x 50 2 r a 3f x
r 3 r

= 25 cr 2 25 r x 50 2 r 3r a 3f r

= 25 cr 2 25 r x 50 5r a 3f r

To get the co-efficient of x60, equating power index of x to 60.


i.e. 50 5r = 60
5r = 50 60
5r = 10
r = 2.

F 3 I 25
Since r is negative, there is no x60 term in the expansion of 2 x 2
H x3 K .

F 3x
14. Find the constant term in the expansion of G
I. 9

JK
2
1
H2
3x

Comparing G
F 3x 1 I with a x + af .
9

H 2 3x JK
2
n

3x 2 1
We get x = , a = , n = 9.
2 3x

Tr +1 = n cr x n r a r formula

F 3x I F 1 I
9 r r
GH 2 JK H 3x K
2
Tr +1 = 9 cr

= 9 cr
9r
3 x

a1f
18 2 r r

29r 3r x r

a f
= 9 cr 39 r r x 18 2 r r 1 r 2 9 + r ...(1)
76 Basic Mathematics

To get the constant term, equating power index of x to zero


18 2 r r = 0
18 3r = 0
3r = 18
r = 6.
Substituting r = 6 in (1)

a f
T6 +1 = 9 c6 39 6 6 x 0 1 6 2 9 + 6

a 1f
9
= 3 3 6
2 3
96 6

987 1 1 7
= .
3 2 1 27 8 18

F
15. Find the constant term in the expansion of G 2
1 IJ 23

H x
x xK
.

F
Comparing G 2 x
1 I
J with a x + af .
23

H x xK
n

1
We get x = 2 x, a = and n = 23.
x x

Tr +1 = n cr x n r a r

c d2 x i
F 1 IJ
G
23 r
r
Tr +1 =
H x xK
23
r

23 r
a1f
r

= 23 cr 2 23 r x 2 x r x 2 r

23 r
a f
r
r
= 23 cr 2 23 r x 2 2 1 r
23 r 2 r r
= cr 2
23 23 r
x 2 a f
1 r
23 4 r
= 23 cr 2 23 r x 2 a f
1 r .
To get the constant term equating power index of x to zero.
23 4 r = 0
Binomial Theorem 77

4r = 23
23
r= .
4
Since r is a fraction there is no term independent of x or there is no constant term.

F x 2 IJ
16. Prove that the constant term in the expansion of G
10
45
H2 xK 2 is
64
.

FG x 2 IJ with a x + af
10

oof: Comparing H
x K
n
2
2
Pr
Proof:

x 2
We get x= , a = 2 and n = 10.
2 x

Tr +1 = n cr x n r a r formula

F x IJ
c G
10 r
F 2 I r
Tr +1 =
H2K
Hx K
10
r 2

a f
r
5
= cr x
10 2 2 10 + r 2 r x 2 r

a f
r
5 2 r
Tr +1 = 10 cr x 2 2 10 + r 2 r ...(1)

To get the constant term equating power index of x to zero.


r
5 2r = 0
2
10 r 4r
=0
2
10 5r = 0
5r = 10 r = 2.
Substituting r = 2 in (1) we get

T2 +1 = 10 c2 x 0 2 10 + 2 1 2 2 2a f
10 9 8
= 2 1 4
2 1
10 9 4 45
= = .
2 28 64
78 Basic Mathematics

F x 2 IJ
Hence the constant term in the expansion of G
10
45
H2 xK 2 is
64
.

17. Find the value of (0.99)5 correct to 4 decimal places.


Solution: We know
0.99 = 1 0.01

a0.99f = a1 0.01f
5 5

We have from Binomial theorem,

a x + af 5
= x 5 + 5x 4 a +
54 3 2 5 43 2 3 5 432
2
x a +
3!
x a +
4!
xa 4 + a 5

Replacing a by a.

a x af 5
= x 5 5x 4 a + 10 x 3 a 2 10 x 2 a 3 + 5 xa 4 a 5 .
Taking x = 1 and a = 0.01, we get

a1 0.01f = 1 5a1f a0.01f + 10 a1f a0.01f


5 5 4 3 2

10 a1f a0.01f + 5 a1fa 0.01f d0.01 i.


2 3 4 5

= 1 0.05 + 0.001 0.00001 + 5 10 8 1 10 10


0.9414801.
a
0.9415 Correct to 4 decimal places . f
18. Prove that sum of Binomial co-efficients of order n = 2n. Also prove the sum of odd binomial co-
efficients = sum of even Binomial co-efficients = 2n 1.
Proof:
Proof: We have from Binomial theorem.

a x + af n
= C0 x n + C1 x n 1 a + C2 x n 2 a 2 + ... + Cn a n ...(1)
To get the sum of binomial co-efficient,
Taking x = a = 1 we get

a1 + 1f n
= C0 1n + C11n 1 1 + C2 1n 2 1 + ... + Cn .

2 n = C0 + C1 + C2 + ... + Cn .

i.e., C0 + C1 + C2 + ... + Cn = 2 n . ...(2)


Now,
Taking x = 1 and a = 1 in (1)

a1 1f n
a f a f
= C0 1n + C11n 1 . 1 + C2 1n 2 1 2 + ... + C n
Binomial Theorem 79

0 = C0 C1 + C2 C3 ... + Cn ...(3)

3 a1f 1
a f a f
= 1, 1 2 = 1, 1 3 = 1 & so on.
Adding (2) and (3) we get

2 n = 2C0 + 2C2 + 2C4 +...

2 n = 2 C0 + C2 +...

2n
= C0 + C2 + C4 +...
2

C0 + C2 + C4 ... = 2 n 1
Hence sum of even binomial co-efficients = 2n 1.
Subtracting (3) from (2) we get

2 n = 2C1 + 2C3 + 2C5 + ...

2n
= C1 + C3 + C5 +...
2

C1 + C3 + C5 + ... = 2 n 1 .
Hence sum of odd binomial co-efficients = 2n 1.
19. The 1st, 3rd and 5th term in the expansion of (a + b)n are respectively 32, 240 and 90. Find a, b and
n.
Given: In the expansion of (a + b)n,
T1 = 32 , T3 = 240 and T5 = 90.
We know

Tr +1 = n cr x n r a r formula

a
Comparing a + b f n
a
with x + a f n
we get x = a, a = b and n = n.
For 1st term r + 1 = 1 r = 0.
For 3rd term r + 1 = 3 r = 2.
For 5th term r + 1 = 5 r = 4.

T1 = n c0 a n 0 b 0 = 32.

1 a n = 32

a n = 32 ...(1)

T3 = n c2 a n 2 b 2 = 240
80 Basic Mathematics

a f
n n 1 n 2 2
a b = 240
2

a f
n n 1 a n 2 b 2 = 480 ...(2)

T5 = n c4 a n 4 b 4 = 90

a fa fa f
n n 1 n 2 n 3
a n 4 b 4 = 90
4!

a fa fa f
n n 1 n 2 n 3 a n 4 b 4 = 2160 ...(3)

From (1) a n = 32.


Since n is a +ve integer.
By inspection, 25 = 32.
a = 2 and n = 5.
Substituting a = 2 and n = 5 in (2).
a f
5 5 1 2 5 2 b 2 = 480

5 4 2 3 b 2 = 480
480
b2 =
20 8

b2 = 3
b = 3.

Verification:

By a fa fa f
n n 1 n 2 n 3 a n 4 b 4 = 2160

5 4 3 2 2 5 4 d 3i 4
= 2160

120 2 9 = 2160
2160 = 2160.
Hence a = 2, n = 5 and b = 3.
20. Using Binomial theorem prove that 6n 5n always leaves the remainder 1 when divided by 25.
We know,
6n = (1 + 5)n
We have from Binomial theorem.
Binomial Theorem 81

a x + af n
= x n + nx n 1 a +
a f
n n 1
x n 2 a 2 + ... + a n .
2!

a1 + 5f n
=1+ n5 +
a f
n n 1 2 n n 1 n 2 3
5 +
a fa
5 + ... + 5 n .
f
2 3!

6 n = 1 + 5n +
a f
n n 1 2
5 + ... + 5n .
2

6 n 5n = 1 +
a f
n n 1 2 n n 1 n 2
5 +
a fa
5 3 + ... + 5 n .
f
2 3!
Now right hand side contains all terms containing 5n except 1st term 1.
when RHS is divided by 25, it leaves the remainder 1.
21. Find the greatest term in the expansion of (x y)20 when x = 12 and y = 4

a x yf = LMNx FH1 xy IK OPQ LM OP


20 20
y
N Q
20
= x 20 1
x
Consider
Tr +1 20 r + 1 y F I
Tr
=
r x H K
Tr +1 21 r F y I
r H xK
=
Tr
When y = 4 and x = 12
Tr +1 21 r 4 F I
21 r
Tr
=
r 12
=
H K
3r

If Tr +1 Tr . Then 21 r 3r
21 3r + r
21 4r
4r 21
Greatest value of r is 5
r +1= 5
Greatest term in the expansion is Tr +1 = n cr x n r a r with
r=4

n = 20, x = 12 and a = 4.

a f
T5 = 20 c 4 12 20 4 4 4
82 Basic Mathematics

T5 = 20 c4 1216 4 4 .
22. Find the greatest term in the expansion of (2p + 2q)17 when p = 12 and q = 14.

a2 p + 3qf = a2 pf LMN1 + 23qp OPQ


17
17 17

17 r + 1 3 F q I
G J
T
2 H pK
r +1
=
T r r

18 r 3 F 14 I
r 2 H 12 K
=

18 r L 7 O
r MN 4 PQ
T r +1
=
T r

If Tr +1 Tr

a18 rf 7 4r
126 7r 4 r
126 11r
11r 126.
The greatest value of r is 11.
Greatest term is 11th term.

Tr +1 = n cr x n r a r

T11 = 17 c10 2 p a f a3qf


17 10 10

= 17 c10 a24f a 42f .


7 10

23. In the expansion of (1 + x)43, the co-efficients of (2m + 1)th and (m + 2)nd terms are equal,
find m.
Solution. Given T2m + 1 = Tm + 2
We have Tr +1 = n cr x n r a r . formula

Here n = 43, x = 1 and a = x.

T2 m +1 = 43c2 m 143 2 m x 2 m .

T2 m +1 = 43 c2 m x 2 m . ...(1) 3 143 2 m = 1
Now

Tm + 2 = 43 cm +1 143 m +1 x m +1
Binomial Theorem 83

Tm +2 = 43 cm +1 x m +1 ...(2)
Given (1) = (2)
43
c2 m x 2 m = 43 c m +1 x m +1
43
c2 m x 2 m
= 1.
43
c m +1 x m +1

43
x 2m
43 2 m 2 m
=1
43
x m +1
43 m + 1 m + 1

43 m + 1 m + 1 2 m a m +1f
x = 1.
43 2 m 2 m

43 m 1 m + 1 2 m m 1
x = 1.
43 2 m 2 m

42 m m + 1 m 1
x = 1 = x0.
43 2 m 2 m
m1=0 [By equation power index of x]
m =1 .

Verification:
By Substituting m = 1 we get
42 1 1 + 1
x11 = 1
af
43 2 1 2 1

41 2 0
x =1
41 2
1 = 1.
24. The 21st and 22nd terms in the expansion of (1 + x)44 are equal. Find x.
Solution: Given T21 = T22
We know Tr +1 = n cr x n r a r . Formula
Comparing (1 + x)44 with (x + a)n we get
x = 1, a = x and n = 44.

T21 = 44 c20 144 20 x 20


84 Basic Mathematics

44
= x 20
44 20 20

T22 = 44 c21 144 21 x 21

44
= x 21
44 21 21
Given T21 = T22
44 44
x 20 = x 21
44 20 20 44 21 21

44 21 21 x 21
=
44 20 20 x 20

23 21
=x
24 20

23 21 20
=x
24 23 20

21
x=
24
7
x= .
8

REMEMBER:

a x + af n
= x n + nx n 1 a +
a f x a +
a fa
n n 1 n 2 2 n n 1 n 2 n3 3
x
f
a + ... + a n .
2! 3!
There are (n + 1) terms in the expansion of (x + a)n. The power indices of x go on decreasing
by 1 and those of a go on increasing by 1 at each stage so that the sum of power indices is n.

The general term or (r + 1)th term is given by Tr +1 = n cr x n r a r .
If n is even the number of terms in the expansion of (x + a)n is (n + 1) which is odd.
There will be only one middle term i.e. Tn .
+1
2

If n is odd, the number of terms in the expansion of (x + a)n is (n + 1) which is even. there will
be 2 middle terms: Tn +1 and Tn +1 .
+1
2 2
Binomial Theorem 85

To find the term containing xm in the expansion of (x + a)n i.e., to find the co-efficient of xm, write
Tr + 1. Simplify and equate power index of x to m. Get r and substitute the value of r in Tr + 1.
For getting the term independent of x or constant term, equate the power index of x to zero after
writing Tr + 1 simplify then get the value of r. If r is a positive integer greater than 0, substitute in
Tr + 1.
If r is negative or a fraction then conclude that there is no term independent of x in the expansion.

EXERCISES

I. Expand b
byy using Binomial theorem:
theorem:

F x + 1I 4
a f
1. H xK 2. 1 + xy 7

FG 2 p q IJ 6
F 2a b I 6
3.
H q 2pK 4. H 3K
F y+ 1 I 5

5. GH y JK

II. Find the indicated term in the expansions:

F 1I
2. 12th term in G y + J
13
1. 4th term in (2 + a)7
H yK
F 1 IJ
3. 3rd term in G x +
4
F x 3I
4. 8th term in G J
10

H xK H 2 yK
2

F bI
5. 10th term in H 2 a + K
12
.
a
III. Find the mid dle ter
middle m(s) in the ffollo
term(s) ollowing:
ollowing:

FG x + 1 IJ 20
Fx 1 I 10
1.
H 2 xK 2. H2 x K 2

F a bI 14
F a abI 12
3. H b aK 4. Hb K
Fa + 3 I F 3x x I 9

GH 2 JK
19 2
5. H2 a K 2 6.
86 Basic Mathematics

FG 2 x 1 IJ 11
Fx + 3 I 19

H
K H2 x K
2
7. 8. 2
x
IV
IV.. Find the ter
termm independent of x in the ffollo
independent ollowing eexpansions:
ollowing xpansions:

F 4a 3 I 9
Fx 3 I
GH 3 2a JK
2 10
1. 2.
H xK 4

FG x 2 IJ 10
FG 2 x 1 IJ 15
3.
H2 xK 2 4.
H xK

F 2x + 1 I 10
Fx 1 I 21
5.
H xK 2 6.
H xK 2

V. Find the coef


coeffficient of:

d i F 1 I 15

H K
20
2. x in x +
4 4
1. x 23 in x 2 x
x3

1 1 F
I 15
F b I 9
3. 17 in x 3
H
K 4. a b in 2 a H K
4 6 3
x x 3

in F 2 x + I
1 5
1 F 1 I 8

H xK H
in 2
K
5 2
5. x 3 6. 4
x x
VI. Find the vvalue
alue of:

1. d2 + 3 i + d2 3 i 5 5
2. d 2 +1 i d 6
2 1 i 6

3. d1 + 5 i + d1 5 i
5 5

VII.

F
1. Prove that in the expansion of G ay
b IJ 25

H +
K
2
. There is no term independent of y.
y

2. The second, 3rd and 4th term of expansion of (x + y)n are 108, 54 and 12 respectively. Find x, y
and n.
3. Find the value of (1.01)5 correct to four decimal places.
4. Prove that the sum of odd binomial co-efficients of order n = 2n 1.
5. Prove that the sum of binomial co-efficient of order n = 2n.
Binomial Theorem 87

ANSWERS
4 1
I. 1. x + 4 x + 6 + + 4
4 2
2
x x

2. 1 + 7 xy + 21x y + 35 x y + 35x y + 21x y + 7 x y + x y


2 2 3 3 4 4 5 5 6 6 7 7

FG p IJ 6
FG p IJ 4
FG p IJ 2
15 q FG IJ 2
FG IJ
3 q
4
FG IJ
3 q
4
q6
3. 64
H qK 96
H qK + 60
H qK 20 +
4 p H K
H K
4 p

H K
4 p
+
64 p 6
.

80 4 2 160 3 3 20 4 b6
4. 64a 64a b + a b a b + + a 2 b 2 ab 5 +
6 6
.
3 27 27 81 729

1 LM 5 1
y 3 + 5 y 2 + 10 y + 10 + + 2
OP
5.
y N y y Q
II. 1. 560 a 3 2. 13
c2 y 9 3. 6 x 3

3 3 7 b9
4. 10
c7 2 3 x y
7
5. 1760 .
a6
63
III. 1. 20
c10 2 10 2.
8x5

3. 14 c7 4. 12
c6 312 b 6

15309 13 5103 14
5. 19
c9 39 2 10 x 8 and 19 c10 310 2 9 x 10 6. x and x .
8 16
11
7. c5 64 x 19 2 and 11c5 32 x 7 . 8. 19
c9 39 2 10 x 9 and 19 c9 310 2 9 x 11 .
IV. 1. 2268 2. 405
45
3. 4. 15
c10 2 5
64

5. No term independent of x. 6. 21 c7
V. 1. 1140 2. 15c
8 3. 1365

26
4.
9
c3 . 5. 80 6. 2 4 8c 4 .
27

VI. 1. 724 2. 140 2 3. 352


VII. 2. x = 3, y = 1 and n = 4. 3. 1.0510.
5
Partial Fractions
5.1 DEFINITIONS:
1. Polynomial: An expression of the form
olynomial:

a0 x n + a1 x n 1 + a2 x n 2 + ... + an where a0 , a1 , a2 ... an


are constants, x is a variable is called a polynomial in x of degree n (provided a0 0).

af
f x
2. Proper and impr
Proper oper fr
improper actions: If f (x) and g (x) are two polynomials in x, then
fractions:
af
g x
is called

a rational fraction. If the degree of f (x) is less than degree of g (x) then it is called proper fraction.
Otherwise it is called an improper fraction. By division, an improper fraction can always be
reduced to the sum of a polynomial and a proper fraction.

Examples:

4x 1
1. is a proper fraction since degree of numerator (=1) is less than degree of denomi-
2 x 2 + 8x 1
nator (=2).

x2 1
2.
a f
x +1 2
is an improper fraction since degree of numerator = Degree of denominator = 2.

x2 1 x2 1
Note that
a f =
x + 1 2 x2 + 2x + 1
By division

1
x + 2x + 1 x 1
2 2

x 2 + 2x + 1
( ) ( ) ( )
2x 2
x2 1 2 x 2 2x + 2
We get
a fx +1 2
=1+ 2
x + 2x + 1
=1 2
x + 2x + 1
which is polynomial and a proper fraction.

5.2 PARTIAL FRACTIONS:

1 2 x 4 + 2 x +1 a f
Consider +
x +1 x 4
=
x +1 x 4 a fa f
x 4 + 2x + 2 3x 2
=
a fa
x +1 x 4
= 2
f
x + x 4x 4

3x 2
= .
x 3x 4
2

Here we have expressed the sum of 2 proper fractions as a single proper fraction. The reverse process
of expressing a single proper fraction as the sum of the two or more proper fractions is called as
Resolving into partial fractions. The following rules are used to resolve a proper fraction into partial
fractions:
1. To each linear factor (ax + b) which occurs only once as a factor of the denominator, there
A
corresponds a partial fraction of the form where A is constant.
ax + b

x2 1 A B
Example:
a fa
x + 1 2x + 3
=
x +1
+
f a f a
2x + 3
.
f
2. To each linear factor (ax + b) which occurs r times as a factor of the denominator, there corre-
sponds r partial fractions of the form,
A1 A2 A3 Ar
ax + b
+
aax + b f aax + bf
2
+ 3
+ ... +
aax + bf r
where A1 , A2 , Ar are constants.

x2 A B C D
Example:
a
x 2x + 3 f 3
=
x
+
a
2x + 3
+
2x + 3f a f a2 x + 3f
2
+ 4
.

Note, here corresponding to linear factor x which occur only once in denominator, we have taken
only one constant A and corresponding to linear factor 2x + 3 which occur 3 times, we have taken 3
constants B, C and D.
3. To each non factorisable quadratic factor ax2 + bx + c which occur only once as a factor of
Ax + B
denominator, there corresponds a partial fraction of the form where A and B are
ax + bx + c
2

constants.
3x 7 Ax + B C
= +
Example: d x + 1ia x 1f
2
x +1 x 1
2
90 Basic Mathematics

Observe here, corresponding to x2 + 1, which is quadratic and non-factorisable we have taken Ax + B


and corresponding to x 1 which is linear we have taken one constant C.
4. To each repeated non-factorisable quadratic factor (ax2 + bx + c) which occurs r times as a factor
of the denominator, there corresponds r partial fractions of the form.
A1 x + B1 A2 x + B2 Ar x + Br
+ + ...
ax + bx + c
2
d
ax 2 + bx + c i 2
dax 2
+ bx + c i r

where A1, A2, ..., Ar and B1 B2, ... Br are constants.

x2 1 Ax + B Cx + D Ex + F
= + + .
Example:
d2 x + 1i
2 3 2 2
d
2 x + 1 2x + 1 2
i d
2x 2 + 1
3
i
WORKED EXAMPLES:

I. Resolve into partial fractions:

x 1
1.
a fa
x+3 x4 f
This is a proper fraction. Resolve into partial fractions:
x 1 A B
a x + 3fa x 4f = x + 3 + x 4 ...(1)

Multiplying both sides by (x + 3) (x 4)


a f a
x 1 = A x 4 + B x + 3 f ...(2)
Put x4=0
x = 4 in (2)
af a f
4 1 = A 0 + B 4 + 3

3 = B a 7f B = .
3
7
Put x+3=0
x = 3 in (2)
a
3 1 = A 3 4 + B 0 f af
4 4
4 = 7 A A = A=
7 7

Substituting A=
4
7
3
and B = in 1
7
af
Partial Fractions 91

4 3
x 1
a fa
x+3 x4
= 7 + 7
f
x+3 x4

4 3
=
a f a+
7 x +3 7 x 4 f
4x 1
2.
x2 1

4x 1 4x 1
x 1
2
=
a fa f
x 1 x +1
This is a proper fraction. Resolve into partial fractions.
4x 1 A B
a fa f
x 1 x +1
= +
x 1 x +1 ...(1)

Multiplying by (x 1) (x + 1)
a f a f
4x 1 = A x + 1 + B x 1 ...(2)
Put x+1=0
x = 1 in (2)
a f af a f
4 1 1 = A 0 + B 1 1

5 = B a 2f B =
5
2
Put x1=0
i.e. x = 1 in (2)
af a f af
4 1 1 = A 1+1 + B 0

3 = A a2 f A =
3
2

3 5
Substituting A = and B = in (1)
2 2

3 5
4x 1 4x 1
x2 1
=
a fa f
x 1 x +1
= 2 + 2
x 1 x +1

3 5
=
a f a f
+
2 x 1 2 x +1
.
92 Basic Mathematics

x
3.
x 5x + 6
2

x x x
= 2 =
x 5x + 6 x 3x 2 x + 6 x x 3 2 x 3
2
a f a f
x
=
a x 3fa x 2f .
This is a proper fraction. Resolve into partial fractions.
x A B
a x 3fa x 2f = +
x3 x2 ...(1)

Multiplying by (x 3) (x 2)
a
x= A x2 +B x3 f a f ...(2)
Put x2=0
x = 2 in (2)
af a f
2= A 0 +B 23

2 = B a 1f B = 2 .
Put x3=0
x = 3 in (2)
a f af
3= A 32 + B 0

3= A A=3 .
Substituting A = 3 and B = 2 in (1)
x 3 2 3 2
a x 3fa x 2f = + =
x 3 x 2 x 3 x 2
.

2x 1
4.
a fa fa
x + 1 x + 2 2x + 1 f
This is a proper fraction. Resolve into partial fractions.
2x 1 A B C
a fa
x + 1 x + 2 2x + 1 fa
= +
f
+
x + 1 x + 2 2x + 1 ...(1)

Multiplying by (x + 1) (x + 2) (2x + 1)
a fa f a fa
2x 1 = A x + 2 2x + 1 + B x + 1 2x + 1 + C x + 1 x + 2 f a fa f ...(2)
Put x + 1 = 0 x = 1 in (2)

a f a fb a f g a f a f
2 1 1 = A 1 + 2 2 1 + 1 + B 0 + C 0
Partial Fractions 93

a fa f
3 = A +1 1 A = +3 .
Put x + 2 = 0 x = 2 in (2)

a f af a f ba f g af
2 2 1 = A 0 + B 2 + 1 + 2 2 + 1 + C 0

a fa f
5 = B 1 3 B =
5
3
.

Put 2x + 1 = 0 x =
1
2
af
in 2

2 F I 1 = A a 0f + B a0f + C F + 1I F + 2I
1 1 1
H 2K H 2 KH 2 K
1 1 = C F I F I
1 3
H 2 K H 2K
2 = C F I C =
3 8
H 4K 3

in a1f
5 8
Substituting A = 3, B = and C =
3 3
2x 1 3 5 3 8 3
a fa fa
x + 1 x + 2 2x + 1
=
f
+ +
x + 1 x + 2 2x + 1
.

2x 1
5.
a fa f
x+2 x3 2

This is a proper fraction. Resolve into partial fraction.


2x 1 A B C
a fa f
x+2 x3 2 = + +
x +2 x 3 x 3a f 2 ...(1)

Multiplying by (x + 2) (x 3)2

a f
2x 1 = A x 3 2
a fa f a
+B x+2 x3 +C x+2 f ...(2)

x3=0
x = 3 (Put x = 3 in Eq. (2))
af af af a
2 3 1 = A 0 + B 0 + C 3 + 2 f
5 = C a 5f C = 1

x+2=0
94 Basic Mathematics

x = 2 (Put x = 2 in Eq. (2))

a f a f af af
2 2 1 = A 2 3 2 + B 0 + C 0

5 = A a 5f A =
5 1
=2
25 5
1
Put x = 0, A = and C = 1 in (2)
5

af
2 0 1=
1
5
3 a f 2
a fa f a f
+ B 2 3 + 1 2

9
1 = 6B + 2
5
9
1 2 + = 6B
5
9 6 1
3 +
5
= 6 B B =
5 6
=
5 a f
1 1
Substituting A = , B = and C = 1 in 1
5 5
af
2x 1 1 5 15 1
a x+2 x3fa f 2 = + +
x +2 x 3 x 3 a f 2

1 1 1
=
a +
5 x +2 5 x 3
+
f a f a f
x3 2
.

x2 +1
6.
a x 2f d x 2
4 i
x2 + 1 x2 + 1 x2 + 1
i a fa fa f a fa f
= =
a x 2f d x 2
22 x2 x2 x+2 x+2 x2 2

This is a proper fraction. Resolve into partial fractions.

x2 + 1 A B C
a x+2 x2 fa f 2
=
x+2
+
x2
+
a
x2 f a f 2 ...(1)

Multiplying with (x + 2) (x 2)2

x2 + 1 = A x 2 a f 2
a fa
+ B x+2 x2 +C x+2 f a f ...(2)
Partial Fractions 95

Put x 2 = 0 in (2)
x=2

af af a f
22 + 1 = A 0 + B 0 + C 2 + 2

5 = C a 4f C =
5
.
4
Put x +2 = 0 in (2)
x = 2

a2f 2
a
+ 1 = A 2 2 f 2
af af
+B 0 +C 0

a f
5 = A 16 A =
5
16
.

Put x = 0, A =
5
16
5
and C = in 2
4
af
1=
5
16
a f
2 2
a fa f 54 a2f
+ B 2 2 +

1=
5
16
a f
4/ + 4 B +
5
2
4

5 5
1 = 4 B
4 2
4 5 10 11 11
4
= 4 B B = =
4 4 16 a f
11
B= .
16

Substituting, A=
5
16
, B=
11
16
5
and C = in 1
4
af
5 11 5
x2 + 1 16 16 4
a x + 2fax 2f 2
= +
x+2 x2
+
x2 a f 2

5 11 5
=
a +
f
16 x + 2 16 x 2
+
a
4 x2 f a f 2 .
96 Basic Mathematics

2x + 3
7.
dx 2
ia
+9 x3 f
This is a proper fraction. Resolve into partial fractions.
2x + 3 Ax + B C
= +
dx 2
+9 i a x 3f x +9 x3
2 ...(1)

Multiplying by (x2 + 9) (x 3)

a fa f d
2 x + 3 = Ax + B x 3 + C x 2 + 9 i
2 x + 3 = Ax 2 + Bx 3 Ax 3 B + Cx 2 + 9C.

2 x + 3 = Ax 2 + Cx 2 + Bx 3 Ax 3B + 9C.
Equating the co-efficients of like powers.

x 2 : 0 = A + C A = C ...(2)

x : 2 = B 3A ...(3)

1 : 3 = 3B + 9C
1 = B + 3C ...(4)

Substituting Eq. (2) i.e., A = C in (3)

a f
B 3 C = 2
B + 3C = 2
B + 3C = 1
Equation (4):
1
6C = 3 C =
2

1
A = C A=
2

B + 3C = 1

F 1I = 1
B + 3
H 2K
3 1
B= 1 =
2 2

1
B=
2
Partial Fractions 97

Substituting A=
1
2
1 1
, B = and C = in 1
2 2
af
1 1 1
x+
2x + 3 2 2
= + 2
d ia f
x +9 x 3
2 x +9
2
x3

x + 1 1
i a f
= +
d
.
2 x +9
2 2 x3

8x 2 + 1
8.
x3 1

8x 2 + 1 8x 2 + 1 8x 2 + 1
= =
x 3 1 x 3 13 a fd
x 1 x2 + x + 1 i
Resolve into partial fractions:

8x 2 + 1 A Bx + C
= + 2
a x 1fd x 2
+ x +1i x 1 x + x +1 ...(1)

a fd
Multiplying by x 1 x 2 + x + 1 i
8x 2 + 1 = A d x + x + 1i + a Bx + C fa x 1f
2

8 x 2 + 1 = Ax 2 + Ax + A + Bx 2 Bx + Cx C
Equating the co-efficients of like powers:

x2 : 8 = A + B ...(2)
x : 0= A B+C ...(3)
1 : 1= AC ...(4)
Solving (3) and (4)

A B+C = 0
AC =1 ...(5)
2A B = 1
Solving (2) and (5)

A+ B=8
2A B = 1
3A = 9 A=3
98 Basic Mathematics

Substituting A = 3 in (2)
3+ B = 8
B=83

B=5

Substituting A = 3 in (4)
AC =1
3C =1

3 1 = C C = 2

Substituting A = 3, B = 5 and C = 2 in (1)

8x 2 + 1 3 5x + 2
= +
a x 1fd x 2
+ x +1 i x 1 x2 + x + 1

2x + 1 2x + 1
=
9.
d
x + x x x2 + 1
3
i
This is a proper fraction. Resolve into partial fractions.
2x + 1 A Bx + C
= + 2
d
x x +1 2
i x x +1 ...(1)

Multiplying by x (x2 + 1)

d
2 x + 1 = A x 2 + 1 + Bx + C x i a fa f
2 x + 1 = Ax 2 + A + Bx 2 + Cx
Equating the co-efficients of like powers:

x2 : 0 = A + B

x : 2=C

1 : 1= A

Now A + B = 0 1 + B = 0 B = 1

Substituting A = 1, B = 1 and C = 2 in (1)

2x + 1 1 x + 2
= +
d
x x +1 2
i x x2 + 1
.
Partial Fractions 99

x2 2
10.
x 2 + x 12
This is an improper fraction. Since degree of numerator = degree of denominator = 2.

1
x + x 12 x 2
2 2

x 2 + x 12
( ) ( ) ( + )
x + 10

x2 2 x + 10
=1+ 2 ...(1)
x + x 12
2
x + x 12
Consider
x + 10 x + 10 x + 10
= 2 =
x + x 12 x + 4 x 3 x 12 x x + 4 3 x + 4
2
a f a f
x + 10
=
ax + 4f a x 3f .
This is a proper fraction. Resolve into partial fractions.
x + 10 A B
a fa f
x +4 x 3
= +
x+4 x3 ...(2)

Multiplying by (x + 4) (x 3) we get
a
x + 10 = A x 3 + B x + 4 f a f
Put x3=0 x=3
af a f
3 + 10 = A 0 + B 3 + 4

7 = B a7f B = 1.

Put x+4=0
x = 4
a f
4 + 10 = A 4 3 + B 0a f af
a f
14 = A 7 A = 2.

Substituting A = 2 and B = 1 in (2)


x + 10 2 1
a fa f
x +4 x 3
= +
x +4 x 3
100 Basic Mathematics

Substituting this in equation (1)

x2 2 2 1
= 1+ +
x + x 12
2
x +4 x 3
2 1
=1 + .
x+4 x3

8x 2 + x 2
11.
4 x 2 + 5x + 1
This is an improper fraction. Since degree of numerator = Degree of denominator = 2.

2
4 x + 5x + 1
2
8x +
2
x2
8 x 2 + 10 x + 2
( ) ( ) ( )
9x 4

8x 2 + x 2 9 x 4
=2+ 2
4 x + 5x + 1
2
4 x + 5x + 1

8x 2 + x 2 9x + 4
=2 2 ...(1)
4 x 2 + 5x + 1 4 x + 5x + 1
9x + 4 9x + 4
Consider =
4 x 2 + 5 x + 1 4 x 2 + 4 x + 1x + 1
9x + 4 9x + 4
=
a f a f a fa
4x x + 1 + 1 x + 1
=
x + 1 4x + 1 f
This is a proper fraction. Resolve into partial fractions.
9x + 4 A B
a fa
x + 1 4x + 1
=
f +
x + 1 4x + 1 ...(2)

Multiplying by (x + 1) (4x + 1)
a f a f
9x + 4 = A 4x + 1 + B x + 1
Put 4x + 1 = 0
1
4 x = 1 x =
4

F 1 I + 4 = A a0f + B F 1 + 1I
H 4K
9
H 4 K
Partial Fractions 101

9 3 F I

4
+4= B +
4 H K
7 3 F I 7
4
=B
4 H K
B=
3
.

Put x + 1 = 0 x = 1

a f ba f g af
9 1 + 4 = A 4 1 + 1 + B 0

5 = A a 3f A =
5
.
3

5 7
Substituting A = and B = in (2)
3 3

5 7
9x + 4
a fa f
= 3 + 3
x + 1 4x + 1 x + 1 4x + 1
Substituting this in equation (1)

8x 2 + x 1 53 73 RS UV
4 x + 5x + 1
2
=2 +
T
x + 1 4x + 1 W
5 7
=2
a f a
3 x + 1 3 4x +1
.
f
x3 1
12. .
x3 + x
This is an improper fraction since degree of numerator = Degree of denominator = 3.

1
x + x x 1
3 3

x3 + x
( ) ( )
x 1

x3 1 x 1 x +1
= 1+ 3 =1 3 ...(1)
x +x
3
x +x x +x
Consider
x +1 x +1
=
d i
This is a proper fraction.
x + x x x2 + 1
3
102 Basic Mathematics

Resolve into partial fractions


x +1 A Bx + C
= + 2
d
x x +12
i x x +1 ...(2)

Multiplying by x (x2 + 1)

d i a
x + 1 = A x 2 + 1 + Bx + C x fa f
x + 1 = A x 2 + A + Bx 2 + Cx
Equating the co-efficients of like powers

x2 : 0 = A + B

x : 1= C

1 :1= A
A + B = 0 1 + B = 0 B = 1
Substituting A = 1, B = 1 and C = 1 in (2)
x +1 1 x +1
= +
d
x x +1 2
i x x2 + 1

Substituting this in equation (1)

x3 1 1x + 1 RS UV
x +x
3
=1
x
+ 2
x +1 T W
1 x + 1
=1 .
x x2 + 1

REMEMBER:
Proper fraction Partial fraction
Nr A B C
ba x + b gb
1 1 gb
a2 x + b2 a3 x + b3 ... g + +
a1 x + b1 a2 x + b2 a3 x + b3
+ ...

Nr A B C

aax + bf acx + d f
2 +
ax + b ax + ba f a
2 +
cx + d f
Nr Ax + B C
+
dax 2
ia
+ bx + c dx + e f dax 2
+ bx + c i dx + e

Provided ax2 + bx + c is non-factorisable. If ax2 + bx + c is factorisable as (a1x + b1) (a2x + b2), then
replace ax2 + bx + c by (a1x + b1) (a2x + b2). Then resolve,
Partial Fractions 103

Nr A B C
b gb
a1 x + b1 a2 x + b2 dx + e
as
ga
a1 x + b1
+
f b
a2 x + b2
+
dx + e g b g a f
If an improper fraction is given to resolve, first by dividing the numerator by denominator write the
given fraction as the sum of the polynomial and the proper fraction. Then the proper fraction is resolved
into partial fractions.

EXERCISE
Resolve into partial fractions:
8x 1 4x + 6 2x + 3
1.
a x 2fa x 3f 2.
x2 1
3.
x 3x + 2
2

1 x 2 10 x + 13 3x + 5
4. a fax +1 x + 2 x + 3fa f 5. a x 1fd x 2
5x + 6 i 6. a fa f
x+2 2 x3

9 4 x 2 + 3x 1 x2 + 1
7.
a fax +1 x + 2 f 2 8.
dx 2
ia f
+ 2x + 1 x 1 9. a x 2f d x 2
4 i
1 3x + 2 5x 2 + 1
10.
a fa f
x 1 1+ x 2 11.
x3 + x
12.
x3 1

3x 1 x2 + 2x + 4 x 1
13. a x + 2fd1 x + x i 2 14. a x 2f d2 x + 3i2 15. a x + 1fd x + 1i
2

x2 2 2 x 2 + 3x + 2 x 3 + 7 x 2 + 17 x + 11
16.
a fa f
x 4 x +3
17.
x2 x 2
18.
x 2 + 5x + 6

2 x 2 3x 4 x 4 3 x 3 3x 2 + 10
19.
x2 x 6
20.
a fa f
x +1 2 x 3

ANSWERS
3 5 5 1 7 5
1. + 2. 3.
x 2 x +3 x 1 x +1 x 2 x 1
1 1 1 2 3 4
a f
4. 2 x + 1 x + 2 + 2 x + 3 a f 5. +
x 1 x 2 x 3

14 1 14
a f a
6. 25 x + 2 + 5 x + 2 f 2 +
a f
25 x 3
104 Basic Mathematics

9 9 9 5 0 3
7. x + 1 x + 2 x + 2
a f 2
a f a f
8. 2 x + 1 + x + 1 2 + 2 x 1 a f
5 11 5
a f a f a
9. 16 x + 2 + 16 x 2 + 4 x 2
f 2

1 1 1
a f a f a f
10. 4 x 1 + 4 1 + x + 2 1 + x 2

2 2 x + 3 2 3x + 1 1 x
11. + 2 12. + 13. +
x x +1 x 1 x2 + x + 1 x + 2 1 x + x2

12 13x + 4 1
a
f
x 1 2
d
14. 11 x 2 11 2 x 2 + 3 i 15. +
x + 1 x2 + 1
16. 1 +
x+3 x4

17. 2 +
16
a f a f
1
3 x 2 3 x 1
18. a x + 2f + x 1+ 2 + x 2+ 3 19. 2
2
+
1
x+2 x3

20. a x 2f + 16 a17x + 1f 4 a x11+ 1f 2


17
a f
16 x 3
.
6
Matrices & Determinants
6.1 INTRODUCTION:
The theory of matrices was developed in 1857 by the French mathematician Cayley. It was not well
advanced till 20th century. But now a days matrices are powerful tool in modern mathematics having
wide applications.

6.2 MATRIX:
A matrix is an arrangement of numbers in rows (horizontal lines) and columns (vertical lines). The
arrangement is usually enclosed between square brackets [ ] or curved brackets ( ) or pairs of vertical
lines || ||. The matrix is usually denoted by a capital letter.
Order of a matrix = Number of rows Number of columns.
If a matrix has m rows and n columns, then
Order = m n (read as m by n)

LM1 2 3OP
Example: Matrix A =
N4 5 6 Q
has the order 2 3.

6.3 TYPES OF MATRICES:


1. Rectangular mamatrtrix: If the number of rows is not equal to number of columns in a matrix, then
trix:
that matrix is called rectangular matrix.

LMa 1 a2 a3 a4 a5 OP
Example: MNc
A= b 1
1
b2
c2
b3
c3
b4
c4
b5
c5 PQ 3 5

(a) Row ma
Row tr
matr ix or Ro
trix Roww vvector:
ector: If a matrix has only one row, then it is called row matrix.

Example: X= 1 2 3 13
(b) Column ma tr
matrix or column vvector:
trix ector: If a matrix has only one column then it is called column
matrix.
106 Basic Mathematics

LM1OP
Example: MN3PQ
Y= 2
3 1

2. Squar
Squaree matr
matr ix: If the number of rows is equal to number of columns in a matrix, then it is called
trix:
square matrix.

LM
a1 a2 a3 OP
Example: A = b1
c1MN b2
c2
b3
c3 PQ 3 3

In a square matrix, the entries from the left top corner to the right bottom corner are called principal
diagonal elements. So in the above example a1 b2 c3 are Principal diagonal elements.
(a) Diagonal matrix: If in a square matrix all the non-diagonal elements are zero, then it is called
diagonal matrix.

LM1 0OP , B = LM20 0 0 OP


Example: A=
N0 2Q MN0 1
0
0
3 PQ
(b) Scalar matrix: If in a diagonal matrix, all the diagonal elements are equal, then it is called scalar
matrix.

LM5 0OP , B = LM06 0 0 OP


Example: A=
N0 5Q MN0 6
0
0
6 PQ
(c) Identity matrix or unit matrix: If in a scalar matrix all the diagonal elements are equal to 1, then
it is called unit matrix or identity matrix.

LM1 0OP , I LM
1 0 0 OP
Example: I2 =
N0 1 Q 3 = 0
0MN 1
0
0
1 PQ
(d) Upper tr iangular ma
triangular matrtrix: A square matrix is called upper triangular if all the non-diagonal
trix:
elements below the principal diagonal are zeroes.

LMa 1 a2 a3 OP
Example: MN 00 b1
0
b2
c1 PQ
(e) Lo
Lowwer triangular ma
triangular tr
matr ix: A square matrix is called lower triangular if all the non-diagonal
trix:
elements above the principal diagonal are zeroes.

LMa 1 0 0 OP
Example: MN bc 1
1
b2
c2
0
c3 PQ
Matrices & Determinants 107

Null matrix: If each element of a matrix is zero then it is called null matrix or zero matrix.

LM0 0 0OP 0 LM 0 OP
Example: A=
N0 0 0 Q
, B=
0 N 0 Q
6.4 ALGEBRA OF MATRICES:
1. Equality of ma tr
matr ices: Two matrices of same order are said to be equal iff the corresponding
trices:
elements are equal,

LM
a1 a2 3OP LM 4 OP
N
i.e., b
1 b2
=
1 Q N 0 Q
iff a1 = 3, b1 = 1, a2 = 4, b2 = 0.

2. Ad dition and subtr


Addition action of ma
subtraction tr
matr ices: Two or more matrices of the same order can be added or
trices:
subtracted. It is the matrix obtained by adding or subtracting the corresponding elements i.e.,

LM a a2 a3 OPb LM b2 b3 OP
A=
Na Q
and B = 1
N Q
1
If a5 a6 b4 b5 b6
4

LM a + b a + b a + b OP
A+ B=
Na + b a + b a + b Q
1 1 2 2 3 3
Then
4 4 5 5 6 6

A B = LM
a b a b a b O
N b PQ
1 1 2 2 3 3
a b 4a b 4 a 5 5 6 6

A=L
MN0 1OPQ and B = LMN4 6OPQ , then
1 2 3 1
Illustration: If

A+ B= L
MN0 + 4 1 + 6OPQ = LMN4 5OPQ
1+ 3 2 +1 4 3

A B= L
MN0 4 1 6OPQ = LMN4 7OPQ
1 3 2 1 2 1

Scalar m ultiplica
multiplica tion: If A is any matrix and k is any scalar or constant, then kA is a matrix obtained
ultiplication:
by multiplying every element of A by k.

LM a a a OP , then
Na a a Q
if A = 1 2 3
i.e.,
4 5 6

kA = LM
ka O
PQ
ka ka
N
1 2 3
ka ka ka 4 5 6

Illustration: If A = LM
1 2O L 1 2O L 3 6O
N3 4QP . Then 3A = 3 MN3 4QP = MN9 12QP .
Ma tr
Matr ix m
trix ultiplica
multiplica tion: The product of 2 matrices exists only when the number of columns in 1st
ultiplication:
matrix is equal to the number of rows in the 2nd matrix.
108 Basic Mathematics

If A is a matrix of order m n and B is a matrix of order n p. Then AB exists and is of order


m p. To get the elements of AB, the elements of 1st row of A multiplied by the corresponding elements
of the first column of B and the products are added. The sum is the element in the first row, first columns
of AB. Similarly other elements are obtained.

b1 LM OP
i.e., If A = a1 a2 a3 1 3
& B = b2
b3 MN PQ 31
, then AB exists and is of order 1 1.

AB = a1b1 + a2 b2 + a3 b3

LM a a12 a13 OP b11 LM b12 OP


A=
Na Q
& B = b21
MN PQ
11
2. If b22
21 a22 a23
b31 b33

d
Then AB exists 3 2 3 i
3 2 and is of order 2 2.

LM a b + a12 b21 + a13 b31 a11b12 + a12 b22 + a13 b33 OP


AB =
Na b Q
11 11
.
21 11 a22 b21 + a23 b31
+ a21b12 + a22 b22 + a23 b33
Illustration:

LM1 2 3 OP 1 LM 4 OP
If A=
N4 5 6 Q 23
& B= 2
3 MN 5
6 PQ 3 2
, then

AB exists and is of order 2 2.

LM 1 + 4 + 9 4 + 10 + 18 OP
AB =
N4 + 10 + 18 16 + 25 + 36 Q
=L
MN32 77OPQ .
14 32

Note:
Note:(1) A + B = B + A i.e., commutative law with respect to addition holds good.
(2) AB BA in general i.e., commutative law with respect to multiplication doesnt hold good in
general.

6.5 TRANSPOSE OF A MATRIX:


If A is any matrix then the matrix obtained by interchanging rows and columns of A is called transpose
of A. It is denoted by A or AT.

LM1 2OP LM 1 3OP


For example: If A=
N3 4Q 2 2
then A T =
N2 4 Q 2 2
Matrices & Determinants 109

LM 1 OP
A = LM OP 4
1 2 3
N Q MN 3
= 2
PQ
T
If then A 5
4 5 6 2 3 6 3 2

If A is of order m n then AT is of order n m.

WORKED EXAMPLES:

LM 1 0OP and B = LM 2 3OP Then find (i) A + B (ii) 2A 3B (iii) A + 2A


1. If A =
N4 3Q N1 6Q
A+ B= L
MN4 3OPQ + LMN1 6OPQ = LMN4 + a1f 3 + a6fOPQ
1 0 2 3 1+ 2 0+3
(i)

=L
MN5 3OPQ
3 3

2 A 3B = 2 LM
1 0O
3L
3O
N4 3Q N1 6PQ
P M
2
(ii)

=L
MN8 6OPQ LMN3 18OPQ = LMN8 a3f 6 a18fOPQ
2 0 6 9 26 09

=L
MN5 24OPQ
4 9

A + 2 A = L
MN4 3OPQ + 2 LMN0 3OPQ = LMN4 3OPQ + LMN0 6OPQ
1 0 1 4 1 0 2 8
(iii)

=M
L 1 + 2 0 + a8fOP = L 3 8O .
N4 + 0 3 + 6Q MN4 9PQ
L 1 2 1OP L 1 3 2O
2. If A = M 4 0 3 and B = M 1 1 5P
MN 1 1 5PQ MN6 2 0PQ
Then find (i) (A + B) (ii) 2A 3B.

1 LM 2 1OP 1 LM 3 2 OP
(i) A+ B= 4
1 MN 0
1 5 PQ
3 + 1
6 MN 1
2
5
0 PQ
LM 1 + 1 2+3 1 + 2 OP LM OP
a f
2 5 1
MN1 + 6
A + B = 4 +1 0 + 1
1 + 2
3 + 5 = 5
5+0 7 PQ MN 1
1
2
5 PQ
110 Basic Mathematics

L OP
a f MM
2 5 7

A+ B = 5
1 N
1
2
1
5 PQ
LM 1 4 1 1 OP LM 3 OP
2
(ii) 2 A 3B = 2
MN21 0
3
1 3 1
5 6 PQ MN 1
2
5
0PQ
LM 2 8 2 OP
3 LM
OP 9 6
MN42 06
= 2 3
10 18PQ MN
PQ 3
6
15
0

LM 2 3 89 2 6O L 1 1 4 OP
0 a 3f 2 15P = M 1
2 A 3B =
MN24183 6 6 10 0PQ MN 20
3
12
17
10 PQ
LM 4OP
3. If A = M 0P and B = 1 2 3 , then find AB and BA. Is AB = BA?
NM1PQ
LM 4 OP

MM1PP
AB = 0 1 2 3 B
1 3

N Q 31

LM 4 1 42 fOP L
4 3 a OP
12
fP MM a
4 8
AB =
MN1 0 a11f 02
1 2
0 3 = 0
fQ N
1 3 1 a
0
2
0
+3PQ
LM 4OP
MM01PP B


BA = 1 2 3
N Q
BA = 1 4 + 2 0 + a 3fa 1f

BA = 4 + 0 + 3 = 7 .
AB BA.

1 LM 1 1 OP
4. If A= 1
1 MN 1
1 1 PQ
1 , then find A2 .
Matrices & Determinants 111

A2 = A A

LM1 1 1OP LM1 1 OP


1
MN1 1 1PQ MN1 1
A = 1 1 1 1 1
PQ
2
1
1

L1 + 1 + 1 1 + 1 + 1 1+1+1 OP
= M1 + 1 + 1 1 + 1 + 1
MN1 + 1 + 1 1 + 1 + 1 1+1+1
1+1+1 PQ
L3 3 3O L1 1 OP
= M3 3 3P = 3 M1 1
1
A2
MN3 3 3PQ MN1 1 PQ
1 = 3 A.
1
Hence A2 = 3A.

LM x 3OP + LM2 yOP LM


5 7 OP
5. Find x and y if
N4 5Q N3 1 Q N
=
7 4 Q
Given

LM x 3OP + LM2 yOP = LM5 7 OP


N4 5Q N3 1Q N7 4 Q
= LM
x+2 3 + yO L 5 7O
N 4 + 3 5 + a1fPQ = MN7 4PQ

x + 2 = 5, 3 + y = 7

x = 5 2, y = 7 3

x = 3 and y = 4.

LM 2 1OP LM xOP = LM7OP


6. Find x and y if
N1 0Q N yQ N2Q
LM 2 1OP LMx OP = LM7OP
N1 0Q N yQ N2Q
LM 2 x + y OP = LM7OP 2x + y = 7
N1a xf + 0 a yfQ N2Q
.
x + 0 = 2

x = 2 x = 2.
2x + y = 7

a f
2 2 + y = 7
112 Basic Mathematics

y = 7+4
y = 11.
So, x = 2 and y = 11

LM 1 2OP LMx OP = LM2OP


7. Find x and y given that
N3 7Q N yQ N1Q
LM 1 2OP LMx OP B = LM2OP

Given
N3 7Q N yQ N1Q
LM x + 2 yOP = LM2OP
N3x + 7yQ N1Q
x + 2y = 2 ...(1)

3x + 7 y = 1 ...(2)
(1) 3 + (2)
3x + 6 y = 6
3x + 7 y = 1
13 y = 7

7
y=
13

7
Substituting y= in (1)
13

F 7I =2
x+2
H 13K
14
x=2
13
26 14
x=
13

12
x= .
13

LM7 0OP and A B = LM3 0OP .


8. Find A and B if A + B =
N2 5 Q N8 3Q
Matrices & Determinants 113

LM7 0OP
A+ B=
N 2 5Q
A B= M
L3 0OP
N8 3Q
A+ B+ A B= M
L7 0OP + LM3 0OP
Adding
N2 5Q N8 3Q
L10 0OP
2A = M
N10 8Q
1 L10 0 O L5 0 O
2 N10 8 PQ MN5 4 PQ
A= M =

A+ B =M
L7 0OP
Given
N 2 5Q
LM5 0OP + B = LM7 0OP
N 5 4 Q N 2 5Q
B=M
L7 0OP LM5 0OP

N2 5 Q N5 4 Q
B= M
L 2 0OP
N3 1Q
Verif
erif ica
ification:
ication:

LM5 0OP + LM 2 0OP = LM7 0OP


A+ B=
N5 4Q N3 1Q N2 5Q
9. If A =
LM4 0OP
N1 2 Q
, then prove that A2 2A 8I = 0 where 0 is the null matrix.

A=
LM4 0 OP
Given:
N1 2 Q
L4 0OP LM4 0OP B

A = A A = M
N 1 2Q N 1 2Q
2

L 4 4 + 0 1 4 0 + 0 a 2fOP
=M
N1a4f + a2fa1f 1a0f + a2fa 2fQ
114 Basic Mathematics

=
LM16 + 0 0 + 0OP = LM16 0OP
N 4 2 0 + 4Q N 2 4Q
2A = 2 M
L4 0OP = LM 8 0OP
N 1 2 Q N 2 4 Q
8I = 8 M
L 1 0OP = LM 8 0OP
N0 1Q N0 8Q
Consider LHS,

A 2 2 A 8I =
LM16 OP LM 0OP LM8 0OP
0

8
N2 Q N 4Q N0 8Q
4 2

LM16 8 8 0 0 0 O L 0 0O
4 a 4f 8PQ MN 0 0PQ
= = = R.H.S.
N 220
Hence proved.

10. If A =
LM1 2OP
,B =
1 LM 0 OP
and C =
2 LM 1OP
N3 1 Q 2 N 1 Q 1 N 1 Q
,

then prove that A (B + C) = AB + AC.

B+C =
LM 1 0OP + LM 2 1OP = LM3 1OP
Consider
N2 1Q N1 1Q N1 2Q
L1 2OP LM3 1OP B = LM3 + 2 OP

1+ 4
A B+C = M
N3 1Q N1 2Q N 9 1 32 Q
L5 5OP
A a B + Cf = M
N8 1Q ...(1)

Now

L1 OP LM 1 0OP B = LM1 + 4 OP LM OP


0+2
AB = M
2 5 2
=
N3 1 Q N2 1Q N3 2 0 1Q N1 1 Q
L1 OP LM 2 OP B = LM2 2 OP LM OP

1+ 2
AC = M
2 1 0 3
=
N3 1 Q N 1 1 Q N6 +1 3 1 Q N
7 2 Q
L5
AB + AC = M
2 O L0 3O L5 5O
1PQ MN 7 2PQ MN8 1PQ
+ =
N1 ...(2)

From (1) and (2) A (B + C) = AB + AC


Matrices & Determinants 115

L1 OP 1 LM OP
11. If A = M
3 2
N1 Q MM PP
, B = 6 and C = 2 1 , then prove that A (BC) = (AB) C
0 2
2 NQ
LM1OP LM1 2 1 1OP

Consider 2 1 B = M6 2 6 1P
MM2PP
BC = 6
NQ NM2 2 2 1PQ
LM 2 1OP
BC = 12 6
MM 4 2PP
N Q
L1 3 2O M
L 2 1OP L1 2 + 3 12 + 2 4

OP
A a BCf = M
N1 0 2PQ MMN124 26PQP B = MN1 2 + 0 12 + 2 4
11+ 3 6 + 2 2
11+ 0 6 + 2 2 Q
L2 + 36 + 8 1 + 18 + 4OP = LM46 23OP
A a BC f = M
N 2 + 0 + 8 1 + 0 + 4 Q N10 5 Q ...(1)

Now

L1 OP LM16OP B = LM1 1 + 3 6 + 2 2 OP = LM1 + 18 + 4OP = LM1 + 18 + 4OP = LM23OP



AB = M
3 2
N1 0 2 Q MMN2PPQ N1 1 + 0 6 + 2 2Q N 1 + 0 + 4 Q N 1 + 0 + 4 Q N 5 Q

a ABf C = LMN235OPQ B = LMN235 22 OP LM OP




23 1 46 23
=
Q N Q
2 1 ...(2)
5 1 10 5

From (1) and (2) A (BC) = (AB) C.


Hence proved.

12. If A =
LM3 4OP and B = LM 1 1 OP a f
, then prove that A + B = A + B
N0 5 Q N2 1 Q
Consider

A+ B=
LM3 4 OP LM
+
1 1
=
OP LM
4 3 OP
N0 5 Q N
2 1 Q N
2 6 Q
a A + Bf = LMN43 2O
6 PQ
...(1)
116 Basic Mathematics

A =
LM 3 0 OP
and B =
1LM OP
2
N4 5 Q 1 N 1Q
A + B = M
L3 0O L 1 OP LM OP
5PQ MN 1
2 4 2
+ =
N4 1 3 Q N 6 Q ...(2)

From (1) and (2)

a A + Bf = A + B
Hence proved.

13. If A =
LM 1 2OP , then find X such that AX = I where I is identity matrix of order 2 2.
N 1 7 Q
A=M
L 1 2OP, To find X such that AX = I
Given
N1 7Q
L bO
LM3 A is of order 2 2 OP
X=M
N c d PQ MM IXisisofoforder P
a
2 2, So
N 2 2. PQ
Let
order

Consider
AX = I

LM 1 2OP LMa OP B = LM a + 2c OP LM OP

b b + 2d 1 0
=
N 1 7Q N c d Q N a + 7c b + 7d 0 Q N 1 Q
a + 2c = 1 ; b + 2d = 0
a + 7c = 0 ; b + 7d = 1
9c = 1 9d = 1
c = 1 9. d = 1 9.

Substituting c = 1/9 Substituting d = 1/9

F 1I = 1 F 1I = 0
a+2
H 9K b+2
H 9K
2 2
a =1 b=
9 9
7
a= .
9
Matrices & Determinants 117

L OP
LM OP MM PP LMN OP
7 2
a b 2
Hence X = = 9 9 =1 7
c N d Q M
1 1 9 1 1 Q
9N 9 Q
14. If A =
LM3 4OP , B = LM1 1 OP
N0 5 Q N2 4 Q
, then find X such that A + 2X = B.

Solution. Since A is of order 2 2. B is of order 2 2. X is of order 2 2.


Now
A + 2X = B
2X = B A
1
X= B A
2

X=
1 1 LM OP LM
1

3 4 OP
2 2 N 4Q N0 5 Q
X=
LM
1 2 OP
5
2 2N Q
1

LM1 5 2O
1 2 PQ
X=
N1 .

6.6 DETERMINANTS:
Every square matrix is associated with a unique real number called its determinant value.

Determinant of a Square Matrix of Order 2:

If A =
LM a
1 a2OP
Na3 a4 Q
, then determinant A is denoted by |A| or det A and its value is a1a 4 a2 a3 .

Example: If A =
LM1 2OP , then A = 1 7 2 3 = 7 6 = 1.
N3 7Q
Determinant of a Square Matrix of Order 3:

a1LM a2 a3 OP
A = a4
MM a5
PP
a6 , then
N Q
If
a7 a8 a9
118 Basic Mathematics

a5 a6 a4 a6 a4 a6
A = a1 a2 + a3
a8 a9 a7 a9 a7 a9

b g b
A = a1 a5 a9 a6 a8 a2 a4 a9 a7 a6 + a3 a4 a9 a6 a7g b g
For example,

1 LM 2 3OP
If A= 2
3 MN 1
1 2 PQ
1 , then

1 1 2 1 2 1
A =1 2 +3
1 2 3 2 3 1

a f a f a f
= 1 2 +1 2 4 + 3 + 3 2 3

1 a3f 2 a7f + 3 a 1f

3 14 3 = 14.
Singular and non-singular matr
matr ices: If A = 0 for a matrix A then A is said to singular matrix. If
trices:
A 0 then A is called non-singular matrix.

WORKED EXAMPLES:

1. If A =
LM 1 2OP
N4 1 Q
, Then find A .

Solution: Given A =
LM 1 OP
2
N4 1Q
a f af
A = 1 1 2 4
1 8 = 9.

2. If A =
LM1 2OP and B = LM 2 1OP , then find AB .
N0 1 Q N1 1Q
L 1 2OP LM 2 OP B

AB = M
1
Solution:
N0 1Q N1 1Q
=M
L1a2f + 2 a1f 1a1f + 2 a1fO
P
N0 a2f + 1a1f 0 a1f + 1 a1fQ
Matrices & Determinants 119

AB =
LM2 2 1 + 2OP = LM 0 3OP
N 0 1 0 + 1Q N1 1Q
AB = 0 a 3f = 3.

OR

1 2 2 1
AB = A B =
0 1 1 1

af a f a f
= 1 1 2 0 2 1

a1fa2 + 1f = 1a3f = 3.
LM1 2 1O
1 P is singular
3. Find x if the matrix 0
MM1 2x P
6 PQ
N
LM1 2 1OP
Given 0
MM1 2x 61PP is singular.
N Q
1 2 1
0 x 1 = 0.
1 2 6

x 1 0 1 0 x
1 2 +1 =0
2 6 1 6 1 2

a f a f a f
1 6x 2 2 0 1 + 1 0 x = 0

6 x 2 2 a 1f + 1 a x f = 0

6 x 2/ + 2/ x = 0

5x = 0 x=0 .

1 2 2
4. If 1 3 x = 2, then find x.
0 6 2
120 Basic Mathematics

1 2 2
Given 1 3 x =2
0 6 2

3 x 1 x 1 3
1 2 +2 =2
6 2 0 2 0 6

a f a
1 6 6x 2 2 0 + 2 6 = 2 f af
af
6 6 x 2 2 + 12 = 2
6 6 x 4 + 12 = 2
6 x + 14 = 2

6 x = 2 14 x=2 .

x 2 1
5. Solve for x : 2 5 x = 0.
1 2 x

a f a f a f
x 5x 2 x 2 2 x + x 1 4 + 5 = 0

x a3 x f 2 a 3 x f a 9 f = 0

3x 2 6 x 9 = 0

3x 2 9 x + 3x 9 = 0
a f a f
3x x 3 + 3 x 3 = 0

a3x + 3fa x 3f = 0
< +3 x
9 x
3x + 3 = 0 or x 3= 0 27 x 2
6 x
3 x = 3
x = 1 or x = +3.
Hence x = 1 or x = 3.

6.7 PROPERTIES OF DETERMINANTS:


1. Prove that the value of a determinant remains same when its rows and columns are interchanged
i.e., prove that A = A .

LMa 1 a2 a3 OP
Proof: Let
Proof:
MMc
A= b 1 b2 b3
PP
N 1 c2 c3 Q
Matrices & Determinants 121

LMa1 b1 c1 OP
Then MM
A = a2 b2 c2
PP
N
a3 b3 c3 Q
To prove: A = A .

a1 a2 a3
Consider A = b1 b2 b3
c1 c2 c3

b2 b3 b b3 b b2
= a1 a2 1 + a3 1
c2 c3 c1 c3 c1 c2

b g b g b
a1 b2 c3 b3c2 a2 b1c3 b3 c1 + a3 b1c2 b2 c1 g
a1b2 c3 a1b3c2 a2 b1c3 + a2 b3c1 + a3 b1c2 a3 b2 c1 ...(1)
Now

a1 b1 c1
A = a2 b2 b g b g b
c2 = a1 b2 c3 b3c2 b1 a2 c3 a3 c2 + c1 a2 b3 a3 b2 g
a3 b3 c3

a1 b1 c1
A = a2 b2 c2 = a1b2 c3 a1b3c2 a2 b1c3 + a3 b1c2 + a2 b3 c1 a3 b2 c1 ...(2)
a3 b3 c3

From (1) and (2) |A| = |A|. Hence proved.


2. Prove that a determinant changes its sign when 2 of its rows are interchanged.

LMa 1 a2 a3 OP
Proof: Let
Proof:
MMc
A= b 1 b2 b3
PP
N 1 c2 c3 Q
LM
b1 b2 b3 OP
B = a1
MM a2 a3
PP is the matrix obtained by interchanging 1st and 2nd rows.
c1 N c2 c3 Q
To prove: A = B .
Now
122 Basic Mathematics

a1 a2 a3
A = b1 b2 b3
c1 c2 c3

b g b
= a1 b2 c3 b3 c2 a2 b1c3 b3c1 + a3 b1c2 b2 c1g b g
= a1b2 c3 a1b3c2 a2 b1c3 + a2 b3c1 + a3 b1c2 a3 b2 c1 ...(1)

b1 b2 b3
B = a1 a2 b g b g b
a3 = b1 a2 c3 a3 c2 b2 a1c3 a3 c1 + b3 a1c2 a2 c1 g
c1 c2 c3

= a2 b1c3 a3 b1c2 a1b2 c3 + a3 b2 c1 + a1b3c2 a2 b3c1

= a2 b1c3 + a3 b1c2 + a1b2 c3 a3 b2 c1 a1b3 c2 + a2 b3 c1 ...(2)


From (1) and (2)
B =A

A =B .
Hence proved.
3. Prove that the value of a determinant is zero if any two of its rows are identical.

a1 a2 a3
Proof: Let A = a1
Proof: a2 a3 be the determinant whose 2 rows, (1st and 2nd) are identical.
b1 b2 b3

To prove A = 0

a1 a2 a3
Now A = a1 a2 b g b g
a3 = a1 a2 b3 a3 b2 a2 a1b3 a3b1 + a3 a1b2 a2 b1 b g
b1 b2 b3

= a1a2 b3 a1a3 b2 a1a2 b3 + a2 a3 b1 + a1a3 b2 a2 a3 b1


= 0.
Hence proved.
4. If every element of any row of a determinant is multiplied by a non zero constant k then prove that
the whole determinant is multiplied by k.

LMa 1 a2 a3OP a1 LM a2 a3 OP
Proof: Let
Proof:
MMc
A= b 1 b2
PP
b3 and B = b1
MM b2 b3
PP
N 1 c2 c3 Q kc1 N kc2 kc3 Q
Matrices & Determinants 123

which is obtained by multiplying 3rd row of A by k.


To prove B = kA.

a1 a2 a3
Now A = b1 b2 b g b
b3 = a1 b2 c3 b3c2 a2 b1c3 b3c1 + a3 b1c2 b2 c1 g b g ...(1)
c1 c2 c3

a1 a2 a3
B = b1 b2 b g b
b3 = a1 kb2 c3 kb3c2 a2 kb1c3 kb3 c1 + a3 kb1c2 kb2 c1 g b g
kc1 kc2 kc3

b g b
= k a1 b2 c3 b3 c2 a2 b1c3 b3 c1 + a3 b1c2 b2 c1 g b g
B = k . A from 1 af
Hence proved.
5. If each element of any row of a determinant is the sum of two terms, then prove that the determi-
nants can be expressed as the sum of two determinants i.e.,

a1 + x a2 + y a3 + y a1 a2 a3 x y z
Prove that b1 b2 b3 = b1 b2 b3 + b1 b2 b3
c1 c2 c3 c1 c2 c3 c1 c2 c3

a1 + x a2 + y a3 + z
b1 b2 b3
Proof: L.H.S.:
Proof:
c1 c2 c3

b
= a1 + x g bc
2

2
b3
c3
b b
a2 + y 1
c1
g b3
c3
b b
+ a3 + z 1g
c1
b2
c2

= a1
b2 b3
+x
b2 b3
a2
RS
b1 b3
+y
b1 b3 UV
+ a3
b1 b2
+z
b1 b2
c2 c3 c2 c3 c1T c3 c1 c3 W c1 c2 c1 c2

Rearranging,
b2 b3 b1 b3 b1 b2 b2 c3 b1 b3 b1 b2
= a1 a2 + a3 +x y +z
c2 c3 c1 c3 c1 c2 c2 c3 c1 c3 c1 c2

a1 a2 a3 x y z
= b1 b2 b3 + b1 b2 b3 = RHS.
c1 c2 c3 c1 c2 c3

Hence proved.
124 Basic Mathematics

6. Prove that the value of a determinant is not altered if to the elements of any row the same
multiples of the corresponding elements of any other row are added.

a1 a2 a3 a1 + kb1 a2 + kb2 a3 + kb3


i.e., prove that b1 b2 b3 = b1 b2 b3 bhere R1 = R1 + kR2 g
c1 c2 c3 c1 c2 c3

a1 + kb1 a2 + kb2 a3 + kb3


Proof: R.H.S.: b1 b2 b3
Proof:
c1 c2 c3

a1 a2 a3 kb1 kb2 kb3


= b1 b2 b3 + b1 b2 a
b3 using property 5 . f
c1 c2 c3 c1 c2 c3

a1 a2 a3 b1 b2 b3
= b1 b2 b3 + k b1 b2 a
b3 using property 4 . f
c1 c2 c3 c1 c2 c3

a1 a2 a3
= b1 b2 a fa
b3 + k 0 3 2 rows are identical . f
c1 c2 c3

a1 a2 a3
= b1 b2 b3 = L.H.S.
c1 c2 c3

Hence proved.
7. Prove that if all the elements in a row of a determinant is zero then the determinant value is zero.

a1 a2 a3
Proof: Let A = b1
Proof: b2 b3 whose 3rd row is zero row.
0 0 0

b2 b3 b b3 b b2
A = a1 a2 1 + a3 1
0 0 0 0 0 0

af af af
= a1 0 a2 0 + a3 0 = 0. Hence proved.
Matrices & Determinants 125

8. In a determinant if all the elements on one side of the principal diagonal are zeros, then prove that
the value of the determinant is equal to the product of the elements in the principal diagonal.

a1 a2 a3
Proof: Let
Proof: A = 0 b1 b2 be the determinant in which all the elements to the left of prin-
0 0 c1
cipal diagonal are zeros.
To prove: A = a1b1c1

a1 a2 a3
b1 b2 0 b2 0 b1
Consider 0 b1 b2 = a1 a2 + a3
0 c1 0 c1 0 0
0 0 c1

b g
= a1 b1c1 0 a2 0 + a3 0 af af
= a1b1c1 . Hence proved.
Note: The above properties which are proved for the rows also holds good for columns because
|A| = |A| from property 1. So
The determinant changes its sign when 2 of its columns are interchanged i.e.,

a1 a2 a3 a2 a1 a3
b1 b2 b3 = b2 b1 b3
c1 c2 c3 c2 c1 c3

a1 a1 a3
The value of the determinant is zero when 2 of its columns are identical i.e., b1 b1 b3 = 0 .
c1 c1 c3

If every element of any column of a determinant is multiplied by constant k. Then the whole
ka1 a2 a3 a1 a2 a3
determinant is multiplied by k i.e., kb1 b2 b3 = k b1 b2 b3
kc1 c2 c3 c1 c2 c3

If each element of any column of a determinant is sum of two terms then the determinant can be
expressed as the sum of 2 determinants.

a1 + x a2 a3 a1 a2 a3 x a2 a3
b1 + y b2 b3 = b1 b2 b3 + y b2 b3
i.e.,
c1 + z c2 c3 c1 c2 c3 z c2 c3

The value of a determinant is not altered if to the elements of any column the same multiples of
the corresponding element of any other column are added.
126 Basic Mathematics

a1 a2 a3 a1 a2 + ka3 a3
b1 b2 b3 = b1 b2 + kb3 b3
i.e.,
c1 c2 c3 c1 c2 + kc3 c3 c2 = c2 + kc3

If all the elements in a column of a determinant are zero then the determinant is zero.

a1 0 a2
b1 0 b2 = 0
i.e.,
c1 0 c2

WORKED EXAMPLES:

3860 3861
1. Find the value of: 3862 3863

3860 3861
R = R2 R1 Using property 6 with k = 1
3862 3863 2

3860 3861 3860 3861


= =2
2 2 1 1

= 2 3860 3861 = 2 1 = 2 a f
81 82 83
2. Evaluate: 84 85 86
87 88 89

Solution:

81 82 83
R2 = R2 R1
84 85 86
R3 = R3 R2
87 88 89

81 82 83
= 3 3 a
3 = 0 3 2 rows are identical f
3 3 3

xy yz zx
3. Without expanding prove that y z zx xy =0
zx xy yz
Matrices & Determinants 127

xy yz zx
LHS: yz zx x y R1 = R1 + R2 + R3
zx xy y z

0 0 0
yz zx xy=0 3 1st row is zero.
zx xy yz

99 101 104
4. Evaluate: 100 102 105
101 103 106

R2 = R2 R1

R3 = R3 R1

99 101 104 99 101 104


= 1 1 1 =2 1 1 1 = 0 Since 2 rows are equal.
2 2 2 1 1 1

5. Solve for x:

x+2 3 4
2 x +3 4 =0
2 3 x+4

Taking C1 = C1 + C2 + C3

x+9 3 4
x+9 x+3 4 =0
x+9 3 x+4
Taking x + 9 common

1 3 4
a f
x+9 1 x +3 4 =0
1 3 x+4

1 3 4
x+9=0
or 1 x +3 4 =0
x = 9
1 3 x+4

R2 = R2 R1
128 Basic Mathematics

and R3 = R3 R1

1 3 4
0 x 0 =0
0 0 x

1 x x = 0

x2 = 0
x=0
Hence
x = 9 or x = 0.

1+ x y z
6. Prove that x 1+ y z =1+ x + y + z
x y 1+ z

1+ x y z
LHS: x 1+ y z C1 = C1 + C2 + C3
x y 1+ z

1+ x + y + z y z
= 1+ x + y + z 1+ y z
1+ x + y + z y 1+ z

1 y z
a1+ x + y + z 1 f 1+ y z
R2 = R2 R1
R3 = R3 R1
1 y 1+ z

1 y z
a1+ x + y + z 0 f 1 0
0 0 1

a1 + x + y + zfa1fa1fa1f = 1 + x + y + z = R.H.S.
1 a a2
7. Prove that 1 b a fa fa f
b2 = a b b c c a
1 c c2
Matrices & Determinants 129

1 a a2
R2 = R2 R3
1 b b2
LHS: R3 = R3 R1
1 c c2

1 a a2 1 a a2
=0 bc b c = 0
2 2
bc a fa f
bc b+c
0 ca c2 a2 0 ca a fa f
ca c+a

Taking (b c) and (c a) common from R2 and R3

1 a a2
a fa f
bc ca 0 1 b+c R3 = R3 R2
0 1 c+a

1 a a2
a fa f
bc ca 0 1 b+c
0 0 ab

Taking (a b) common from R3

1 a a2
a fa fa f
ab bc ca 0 1 b+c
0 0 1

aa bfab cfac af a1f a1f a1f


= aa b fa b cfac a f = RHS. Hence proved.

x p q
8. Prove that p x a fa
q = x p x q x + p+q fa f
p q x

x p q
LHS: p x q C1 = C1 + C2 + C3
p q x

x+ p+q p q 1 p q
= x+ p+q x a
q = x+ p+q 1 f x q
R2 = R2 R1
R3 = R3 R2
x+ p+q q x 1 q x
130 Basic Mathematics

1 p q
a
= x+ p+q 0 f xp 0
0 qx xq

1 p q
a
= x + p+q 0 f x p 0
0 a
xq f a xq f
Taking (x p) common from R2 and (x q) from R3

1 p q
a fa
x p xq x+ p+q 0 fa f 1 0 R3 = R3 + R2
0 1 1

1 p q
a
= x p xq x+ p+q 0fa fa f 1 0
0 0 1

a fa fa f
= x p x q x + p + q 1 11

= a x pfa x qfa x + p + qf = RHS.

1 1 1
9. Prove that x y a
z = xy yz zx x+y+z fa fa fa f
x3 y3 z3

1 1 1
C1 = C1 C2
x y z
LHS: C2 = C2 C3
x3 y3 z3

0 0 1
= xy yz z
x 3 y3 y3 z3 z3

0 0 1
= xy yz z
a fd
x y x 2 + xy + y 2 i a fd
x y y 2 + yz + z 2 i z3

0 0 1
a
= xy yz fa f 1 1 z
x + xy + y 2
2
y + yz + z 2
2
z3
Matrices & Determinants 131

Expanding

a x yfay zf 0 0 + 1d y + yz + z i d x + xy + y i
2 2 2 2

a x yfa y zf y + yz + z x xy y
2 2 2 2

a x yfa y zf az xfa z + xf + y a z xf
a x yfa y zf a z x fa x + y + zf
= a x yfa y zfa z x fa x + y + z f .
Hence proved.

a + b + 2c a b
10. Prove that c b + c + 2a b =2 a+b+c a f 3

c a c + a + 2b

a + b + 2c a b
LHS: c b + c + 2a b C1 = C1 + C2 + C3
c a c + a + 2b

2a + 2b + 2c a b
= 2a + 2b + 2c b + c + 2a b
2a + 2b + 2c a c + a + 2b

1 a b
a
=2 a+b+c 1 f b + c + 2a b
R2 = R2 R1
R3 = R3 R1
1 a c + a + 2b

1 a b
a
=2 a+b+c 0 f a+b+c 0
0 0 a+b+c

a fa fa
=2 a+b+c 1 a+b+c a+b+c fa f
= 2 (a + b + c)3. Hence proved.

a2 ab ac
11. Prove that ab b2 bc = 4a 2 b 2 c 2
ac bc c 2

a 2 ab ac
LHS:
ab b2 bc
ac bc c2
132 Basic Mathematics

Taking a, b, c common from C1, C2 and C3 respectively we get

a a a
abc b b b
c c c

Taking a, b, c common from R1, R2 and R3 respectively

1 1 1
aabcfaabcf 1 1
R = R2 + R1
1 2
R = R3 + R1
1 1 1 3

1 1 1
a b c 0
2 2 2
0 2
0 2 0

Expanding

a f af af
a 2 b 2 c 2 1 0 4 1 0 + 1 0

a2b 2c2 a4f = 4a b c = RHS.


2 2 2

1+ a 1 1
F
1 1 1 I
12. Prove that 1
1
1+ b
1
1 = abc 1 + + +
1+ c
H
a b c K
1+ a 1 1
LHS: 1 1+ b 1
1 1 1+ c

Taking a, b, c common from R1, R2 and R3 respectively we get

1+ a 1 1
a a a
1 1+ b 1
abc
b b b
1 1 1+ c
c c c

1 1 1
1+
a a a
1 1 1
abc 1+
b b b
1 1 1
1+
c c c
Matrices & Determinants 133

1 1 1 1 1 1 1 1 1
1+ + + 1+ + + 1+ + +
a b c a b c a b c
1 1 1
abc 1+ R1 = R1 + R2 + R3
b b b
1 1 1
1+
c c c

1 1 1

aabcf FH1 + 1a + b1 + 1c IK 1
b
1+
1
b
1
b
C2 = C2 C1
C3 = C3 C1
1 1 1
1+
c c c

1 0 0

aabcf FH1 + a1 + b1 + 1c IK 1
b
1 0
1
0 1
c

aabcf FH1 + a1 + b1 + 1c IK 1 1 1
= a abcf F 1 + + + I Hence proved.
1 1 1
H a b cK
a b c
13. Prove that a
2
b2 a fa fa fa
c 2 = ab + bc + ca b c c a a b . f
bc ca ab

a b c
C1 = C1 C2
a2 b2 c2
LHS: C2 = C2 C3
bc ca ab

ab bc c
a2 b2 b2 c2 c2
bc ca ca ab ab

ab bc c
a ab a+b fa f a fa f bc b+c c2
a
c a b f a f a b c ab
Taking (a b) common from C1, (b c) common from C2
134 Basic Mathematics

1 1 c
a fa f
= ab bc a+b b+c c 2 C1 = C1 C2
c a ab

0 1 c
a fa f
= ab bc ac b+c c2
ac a ab

Taking (c a) common from C1

0 1 c
a fa fa f
= a b b c c a 1 b+c c 2 R2 = R2 R3
1 a ab

0 1 c
a fa fa f
= ab bc ca 0 a+b+c c 2 ab
1 a ab

Expanding

a fa fa f d i a
= a b b c c a 0 1 0 + c 2 ab + c 0 + a + b + c f
= aa bfa b cfac af c/ + ab + ac + bc + c/
2 2

aa bfab cfac afaab + bc + caf. Hence proved.


1 x x2
14. Prove that x
2
1 d 2
x = 1 x3 . i
x x2 1

1 x x2
x2 1 x C1 = C1 + C2 + C3
LHS:
x x2 1

1 + x + x2 x x2
= 1 + x + x2 1 x
1 + x + x2 x2 1

1 x x2
d
= 1+ x + x 2
i 1 1 x
1 x2 1
Matrices & Determinants 135

1 x x2
d
= 1 + x + x2 0 i 1 x x x2 R2 = R2 R1
R3 = R3 R1
0 x2 x 1 x2

1 x x2
d
= 1 + x + x2 i 0 1 x a f x 1 x
0 a f d i
x 1 x 1 x2

Taking (1 x) common from R2, (1 x) from R3

a fa f
3 1 x 2 = 12 x 2 = 1 x 1 + x

1 x x2
d
= 1 + x + x2 ia f a f
1 x 1 x 0 1 x
0 x 1+ x

expanding, we get

d i a f d1 + x + x i
= 1 + x + x2 1 x 2 2

= d1 + x + x i a1 x f d1 + x + x i
2 2 2

d1 + x + x i a1 xf2 2 2
a fd i
3 1 x 1 + x + x 2 = 13 x 3 = 1 x 3

= d1 x i = RHS.
3 2

Hence proved.

x x2 1 + x3
15. If x, y, z are all different and y y2 1 + y 3 = 0, then prove that 1 +xyz = 0.
z z2 1 + z3

x x2 1 + x3
Given y y2 1 + y 3 = 0.
z z2 1 + z3

x x2 1 x x2 x3
y y2 1+ y y2 y3 = 0 (using property of determinants)
z z2 1 z z2 z3
136 Basic Mathematics

x x2 1 1 x x2
y y2 1 + xyz 1 y y2 = 0
z z2 1 1 z z2

x x2 1 x
1 + xyz y
x2 1
1=0
LMBy interchanging R and R OP
1 2

y
z
y2
z2 1 z
y2
z2 1 N Then R and R Q2 3

x x2 1
a1 + xyz y f y2 1=0
z z2 1

x x2 1
1 + xyz = 0. 3 x, y and z are all different, y y2 10.
z z2 1

Hence proved.

6.8 MINOR, CO-FACTOR, ADJOINT AND INVERSE OF A


SQUARE MATRIX:
Let [aij] be a square matrix of order n n. Then minor of an element aij is the determinant obtained by
deleting the row and the column containing it (i.e. ith row & jth column). If minors are multiplied by
(1)i + j i.e., with proper signs + or we get co-factors. Adjoint of a matrix is the transpose of co-factor
matrix.
Illustration:

1. If A =
LM1 2 OP
N3 4 Q
, then

minor of 1 = 4
minor of 2 = 3
minor of 3 = 2
minor of 4 = 1

LM 4 3OP + LM OP
Co-factor matrix =
N2 1 Q
Multiplying by
N + Q
Adjoint of A =
LM 4 2 OP
N 3 1 Q
Taking transpose of co-factor matrix i.e., interchanging rows and

columns.
Matrices & Determinants 137

LM
1 2 3OP
MM
2. If A = 2 3
PP
1 , then
3N 1 2 Q
3 1
LM
1 2 3OP
minor of 1 = = 6 1 = 5. 2
MM 3 1
PP
1 2
3 N 1 2 Q
L1 3O
= 4 3 = 1. M2 1P
2
2 1
minor of 2 =
MM3 3
2PQ
P
3 2
N 1

2 3
LM1 2 3 OP
minor of 3 = = 2 9 = 7. 2
MM3 3 1
PP
3 1
N 1 2 Q
2 3
1 LM 2 3 OP
Similarly minor of 2 :
1 2
= 4 3 = 1. 2
3
MN 3
1
1
2
PQ
1 3
minor of 3 : = 2 9 = 7.
3 2

1 2
minor of 1 = = 1 6 = 5.
3 1

2 3
1 LM 2 3 OP
minor of 3 = = 2 9 = 7. 2
MM 3 1
PP
3 1
3 N 1 2 Q
1 3
minor of 1 = = 1 6 = 5.
2 1

1 2
minor of 2 = = 3 4 = 1.
2 3

LM 5 1 7 OP
Hence matrix of minors =
MM71 7 5
PP
N 5 1 Q
138 Basic Mathematics

LM+ + OP
Multiplying with
MM+ +
PP
the corresponding elements
N + Q
5 LM 1 7 OP
We get co-factor matrix = 1
7 MN 7
+5
+5
1 PQ
Taking transpose, we get Adjoint matrix

LM 5 1 7 OP
MM7
Adjoint of A = 1 7 5
PP
N 5 1 Q
INVERSE OF A SQUARE MATRIX:
If A is a non-singular square matrix of order n n then there exists a square matrix B of order n n such
that AB = BA = I where I is the identity matrix of order n n. Here B is called inverse of A. It is denoted
by A1.
A A 1 = A 1 A = I
Inverse of A can be found by using the formula
adjoint of A
A 1 = .
A

adj A
i.e., A 1 = .
A
Illustration:

1. To find inverse of a matrix of order 2:

LM 1 1OP
N2 2 Q
A=M
L 1 1OP
Let
N2 2Q
A = 1 a2 f a 1fa 2f
= 2 + 2 = 4 0.
A is non-singular. Hence A1 exists.
adj A
A 1 = formula
A
Matrices & Determinants 139

LM 1 1 OP
Now minor of 1 = 2
N2 2 Q
minor of 1 = 2
minor of 2 = 1
minor of 2 = 1
LM2 2 OP + LM OP
Co-factor matrix =
N1 1 Q
. By multiplying minors with
N + Q
Taking transpose we get
LM 2 1OP
Adjoint of A =
N2 1Q
LM 1O M 2
L 1 1 OP
1PQ M 1
MN 2 14 PPQ
1 2
A 1 =
N = 4 .
4 2

Note: We can write adjoint of a square matrix of order 2 2 directly by interchanging the principal
diagonal elements and changing the signs of secondary diagonal elements.

Examples:

(i) If A =
LM 1 7OP , then
N2 3Q
L 3 7OP interchange 1 and 3, change the signs of 2 and 7.
Adj A = M
N2 1Q
L 1 7OP, then adj A = LM 6 7OP
(ii) If A = M
N2 6 Q N2 1Q
L 1 5OP, then adj A = LM7 5OP
(iii) If A = M
N 2 7Q N2 1Q
(iv) If A = M
L 0 5OP , then adj A = LM2 5OP
N6 2Q N 6 0Q
(v) If A = M
L 1 6OP , then adj A = LM7 6OP
N 8 7Q N 8 1Q
2. To find inverse of a square matrix of order 3:

LM1 3 2 OP
MM2
A= 1 2 2
PP
N Q
Let
0 0
140 Basic Mathematics

1 3 2
2 2 1 2 1 2
A=1 2 2 =1 3 +2
0 0 2 0 2 0
2 0 0

af a f a f
=1 0 3 0 4 +2 04

= 1 a0 f 3 a0 4 f + 2 a 0 4 f

0 + 12 8 = 4 0
Hence A1 exists.
adj A
A 1 = .
A

LM1 3 2 OP
MM2
A= 1 2 2
PP
N Q
Now
0 0

2 2
minor of 1 = =0
0 0

1 2
minor of 3 = 2 = 0 4 = 4
0

1 2
minor of 2 = 2 = 0 4 = 4
0

3 2
Similarly minor of 1 = 0 =0
0

1 2
minor of 2 = = 0 4 = 4
2 0

1 3
minor of 2 = 2 = 0 6 = 6
0

3 2
Similarly minor of 2 = =64=2
2 2

1 2
minor of 0 = =22=0
1 2

1 3
minor of 0 = = 2 3 = 1
1 2
Matrices & Determinants 141

LM0 4 4 OP + LM + OP
Matrix of minors = 0
MM2 4
PP
6 Multiplying with
MM +
PP
N 0 1 Q + N + Q
We get

LM0 4 4 OP
Co-factors matrix = 0
MM2 4 6
PP
N 0 1 Q
Taking transpose we get

LM 0 0 2 OP
MM4
Adj A = 4 4 0
PP
N 6 1 Q
Adj A 1
LM 0 0 2OP
A 1 = =
MM44 4 0
PP
A 4
N 6 1 Q
LM 0 0 12 OP
MM1 PP
1
A = 1 1 0
N 32 1 4 Q
Note: We can write the adjoint of a square matrix directly by writing the minors multiplied by proper
signs (+ or ) column wise.

LM 1 3 2 OP
i.e. If A = M 1 PP
MN 2
2 2
0 0 Q
LM 2 2

3 2 3 2 OP
MM 0 0 0 0 2 2 PP
Adj A = M PP
1 2 1 2 1 2

MM
Then
2 0 2 0 1 2

MN
1 2

1 3 1 3 PP
2 0 2 0 1 2 Q
LM 0 0 2 OP
MM4
Adj A = 4 4 0
PP
N 6 1 Q
142 Basic Mathematics

WORKED EXAMPLES:

1. Find A1 if A =
LM 1 7 OP
N2 6 Q
Solution: A 1 =
adj A
A
a
formula f

L 6 OP F By interchanging principal diagonalI


adj A = M
7 GG elements and changing signs of JJ
Now
N 2 1Q H secondary diagonal elements. K
1 7
A= = 6 14 = 20
2 6

A 1 =
LM 7OP
1 6

N
20 2 1Q

LM 6 7O L3
P M
7O
= M 20 20 P
A 1 1P M1 1 P.
20 = 10
MN 202 P M
20 Q N10
P
20 Q

2. Verify A. adj A = |A|.I for the matrix A =


LM1 2OP
N3 1Q
A=M
L1 2OP
Given:
N3 1Q
adj A = M
L1 2OP
Hence
N3 1Q
L1 2OP = LM1 2OP B

A adj A = M
Consider
N3 1Q N3 1Q
L 1a1f + 2 a3f 1a2f + 2 a1fOP
=M
N3a1f + a1fa3f 3a2f + a1fa1fQ
=M
L1 6 2 + 2OP = LM7 0OP
N3 + 3 6 1Q N 0 7Q
Matrices & Determinants 143

LM 1 0OP = 7I
A adj A = 7
MN0 1PQ
= 1a1f 3 a2 f = 1 6 = 7.
1 2
But A=
3 1
A.adjA = |A|.I.

LM 1 2 3OP
3. Find A1
MM 1
if A = 0 1 1
PP
N 1 2 Q
adj A
Solution: A 1 =
A

1 2 3
A= 0 1 1
Now
1 1 2

= 1 (2 1) 2 (0 1) + 3 (0 + 1)
= 3 + 2 + 3 = 2 0.
Hence A1 exists.

LM 1 2 OP
3
Now MM 1
A= 0 1 1
PP
N 1 2 Q
LM 1 1 2 3 2 3 OP
MM 01 21 11 23 11 1
PP
MM 1 2 1 2 0 PP
3
Adj A =
1
MM 01 11 11 21 01 2 PP
N 1 Q
LM2 1 a4 3f 2 + 3OP
= a1f
MM 0 + 1 a1223f 11PP
N Q
LM3 1 5OP
MM 1 1 1PP
Adj A = 1 1 1
N Q
144 Basic Mathematics

1 adj A 1 LM3 1 5 OP
A =
A
=
2 MN 11 1
1
1
1 PQ
4. Verify AA 1 = A 1 A = I if A =
LM 1 2OP .
N1 3Q
A=
LM 1 2OP
Solution: Given
N1 3Q
= 3 a2 f = 5.
1 2
A=
1 3

A 1 =
adj A 1 3
=
LM 2 OP

A 5 1 N 1 Q
= LM
1 2O 1 L3 2 O
= LM OP

Now AA 1
1 3P M 1 1 P
N Q 5 N Q 5 N3 + 3 B 1 3+2 2 + 2
2+3 Q
1 L 5 0 O L 1 0O
AA = M
5PQ MN 0 1PQ
1
= =I
5 N0

LM OP LM OP B


1 1 3 2 1 2
Similarly, A A=
5 1 N
1 1 QN 3 Q
LM a f 6 6OP
1 3 2
=
N a f 2 + 3Q
5 1 + 1

1 L 5 0 O L 1 0O
= M
5PQ MN0 1PQ
= =I
5 N0
Hence AA1 = A1A = I

6.9 CHARACTERISTIC EQUATION OF A SQUARE MATRIX:


If A is any square matrix of order n n and I is identity matrix of the same order then |A I| = 0 where
is a constant is called characteristic equation of a square matrix A. The roots of the equation |A I|
= 0 (i.e., value of which satisfies this equation) are called characteristic roots or eigen values.

Examples:

1. If A =
LM1 4OP
N2 3 Q
then
Matrices & Determinants 145

A I =
LM1 4 OP LM 0OP

1
N2 3 Q N 1Q
0

LM1 4OP LM 0 O L1
PQ = MN 2 3 OPQ
4
=
N2 3 Q N 0
1 4
A I =
2 3

Characteristic equation A I = 0

1 4
=0
2 3

a1 f a3 f 8 = 0
3 3 + 2 8 = 0

2 4 5 = 0. This is the characteristic equation.

a f a f
5 +1 5 = 0

a 5fa + 1f = 0
= 5 or = 1.
Hence the characteristic roots (or eigen values) are 5 and 1.

LM 4 0 1 OP
MM2
A = 2 1 0
PP
N Q
2. If
0 1

4 0 1
Then A I = 0 2 1 0 = 0.
2 0 1

a4 f 1 0 0
1
0 +1
2
2
1
0
=0

a4 fa1 fa1 f + 0 + 2 a1 f = 0
a1 f a4 fa1 f + 2 = 0
1 = 0 or a4 fa1 f + 2 = 0
146 Basic Mathematics

= 1 or 4 4 + 2 + 2 = 0

2 5 + 6 = 0

2 3 2 + 6 = 0

a f a f
3 2 3 = 0

a 3fa 2f = 0
= 1 or = 3 or = 2.
Characteristic roots or eigen values are 1, 2 and 3.

6.10 CAYLEY HAMILTON THEOREM:

Statement: Every square matrix A satisfies its characteristic equation, A I = 0.

Illustration: If A =
LM1 2 OP
N2 3 Q
Then characteristic equation is A I = 0.

1 2
i.e., =0
2 3

a1 fa3 f 4 = 0
3 3 + 2 4 = 0

2 4 1 = 0.
This is characteristic equation.

By Cayley Hamilton theorem A satisfies this equation. So A 4 A I = 0


2
...(i)
where I is identity matrix of same order as that of A.
Now to verify Cayley Hamilton theorem,
We have to verify A2 4A I = 0.

OP LM1 2OP B
L1

=M
2
Q N2 3 Q
N2
2
Now A
3

A =M
L1 + 4 2 + 6OP = LM5 8 OP
N2 + 6 4 + 9Q N8 13Q
2

L1 2OP = LM4 8 OP
4A = 4 M
N2 3Q N8 12Q
Matrices & Determinants 147

LM1 0OP
I=
N0 1 Q
4A I = M
L5 8 OP LM4 8 OP LM1 0OP
A2
N8 13Q N8 12Q N0 1Q
=M
L5 4 1 8 8 0 OP = LM0 0OP .
N8 8 0 13 12 1Q N0 0Q
Hence verified.
Note: From (1) A2 4 A I = 0.
Operating by A1
A 1 A 2 4 A 1 A A 1 I = 0.

A 4 I A 1 = 0
A 1 = A 4 I .

A 1 =
LM1 2 OP LM 0OP
4
1
N2 3 Q N 1Q
0

=M
L1 2O L4 0 O
3 PQ MN 0 4 PQ
A 1
N2
=M
L3 2O
1PQ
A 1
N2
Hence we can find inverse of a square matrix by using Cayley Hamilton theorem.

WORKED EXAMPLES:

LM1 4OP .
1. Find the eigen values of the matrix
N3 2 Q
Solution: Characteristic equation is
A I = 0

1 4
=0
3 2

a1 fa2 f 4 a3f = 0
2 2 + 2 12 = 0
148 Basic Mathematics

2 3 10 = 0

2 5 + 2 10 = 0

a f a
5 +2 5 = 0 f
= 5 or = 2.
Hence the eigen values are = 5 and = 2.

LM1 1 OP
2. Find the characteristic roots of the matrix
N3 1
.
Q
Solution: Characteristic equation is A I = 0.

1 1
= 0.
3 1

a1 fa1 f 3 = 0
a1 fa1 + f 3 = 0

d1 i 3 = 0
2 2

1 + 2 3 = 0

2 4 = 0 2 = 4 = 2.
Hence characteristic roots are +2 and 2.

LM 1 1OP
3. Verify Cayley Hamilton theorem for the matrix
N2 6 Q
.

Solution: Characteristic equation is A I = 0

1 1
= 0.
2 6

a1 fa6 f 2 a1f = 0
6 6 + 2 + 2 = 0

2 7 + 8 = 0 .
This is characteristic equation.
By Cayley Hamilton theorem every square matrix obeys its characteristic equation.
Hence it is required to verify A2 7A + 8I = 0
Matrices & Determinants 149

L1 OP LM 1 1OP B

1
= A A = M
N2 Q N2 6 Q
2
Now A
6

LM 1 2 1 6 OP = LM1 7OP
N2 + 12 2 + 36Q N14 34Q
7 A = 7 LM
1O L 7 7O
N2 6QP = NM14 42QP
1

8I = 8 M
L 1 0OP = LM8 0OP
N0 1Q N0 8Q
Now LHS: A2 7 A + 8I

=
LM1 7

OP LM 7OP + LM8
7 0 OP
N14 34 Q N 42Q N0
14 8 Q
LM 1 7 + 8 7 a7f + 0O L0 OP
34 42 + 8 PQ MN0
0
= = = R.H.S.
N14 14 + 0 0 Q
Hence verified.

LM 1 1 OP
0
4.
MM 1
Verify Cayley Hamilton theorem for the matrix A = 2 1 0
PP
N 2 1 Q
Solution: Characteristic equation is A I = 0

1 1 0
2 1 0 =0
1 2 1

a1 f 1 2 0
1
1
2
1
a f 0
1
+0=0

a1 f a1 fa1 f 0 + 1c2 a1 f 0h + 0 = 0
a1 f 1 2 a1 + f = 0
2 2

a1 f d1 i 2 2 = 0
2

d1 + i 2 2 = 0
2 3
150 Basic Mathematics

3 + 2 3 = 0.
This is characteristic equation.
To verify Cayley Hamilton theorem, we have to verify A 3 + A 2 A 3 I = 0

LM 1 OP LM 1 OP

1 0 1 0
= A A = 2
MM 1 PP MM21 PP B
2
Now A 1 0 1 0
N 2 1 QN 2 1 Q
LM 1 2 + 0 1 1 + 0 0+0+0 OP
MM 1 + 4 1
= 2+2+0 2 + 1 + 0 0+0+0
PP
N 1 + 2 2 0 + 0 +1 Q
LM1 2 0 OP
MM 4
= 4 1
PP
2
A 0
N 1 1 Q
LM1 OP LM 1 OP

2 0 1 0
A =A
MM 4
A= 4 1
PP MM21 PP B
3 2
0 1 0
N 1 1 QN 2 1 Q
LM1 4 + 0 1 2 + 0 0+0+0 OP
MM 4 2 + 1
= 42+0 4 1 + 0 0+0+0
PP
N 4 1 + 2 0 + 0 1 Q
LM5 1 0 OP
MM 3
= 2 5
PP
3
A 0
N 3 1 Q
LM5 1 0 OP 1 LM 0 0 OP
3 LM 0 0 OP
Now A=
MM 23 5
PP
0 and 3I = 3 0
MM 1 0 = 0
PP MM 3 0
PP
N 3 1 Q 0 N 0 1 0 Q N 0 3 Q
Consider LHS,
A3 + A2 A 3I

LM5 1 0 OP1 LM 2 OP
0 3 LM 1 0 3 OP LM 0 0 OP
MM 3
= 2 5
PP
0 + 4
MM 1
PP
0 2
MM 1 0 0
PP MM 3 0
PP
N 3 1 Q4 N 1 1 Q 1 N 2 1 0 Q N 0 3 Q
Matrices & Determinants 151

LM5 11 3 1 2 +1 0 0+000 OP
MM
= 2 + 4 2 0 5 11 3 0+000
PP
N
3 + 4 1 + 0 3 1 2 + 0 1+1+ 1 3 Q
0 LM 0 0 OP
= 0
MM 0 0
PP
0 N 0 0 Q
Hence Cayley Hamilton theorem is verified.

5. By using Cayley Hamilton theorem find A1 if A =


LM 1 7OP
N 6 5Q
Characteristic equation = A I = 0

1 7
=0
6 5

a1 fa5 f a6fa7f = 0
5 5 + 2 + 42 = 0

2 6 + 47 = 0.
By Cayley Hamilton theorem, A2 6A + 47I = 0
Operating by A1,
A 1 A 2 6 A 1 A + 47 A 1 I = 0

A 6 I + 47 A 1 = 0
47 A 1 = 6 I A
1
A 1 = 6I A
47

6I = 6
LM1 0OP = LM6 0OP
Now
N0 1 Q N0 6 Q
L6 0OP LM 1 7OP = LM5
6I A = M
7 OP
N0 6 Q N 6 5Q N6 1 Q
1 L 5 7O
47 MN6 1PQ
1
A = .
152 Basic Mathematics

LM3 2 1OP
6. Find A1
MM 1
if A = 4 1
PP
0 by using Cayley Hamilton theorem.
N 3 1 Q
Solution: Characteristic equation: A I = 0

3 2 1
4 1 0 =0
1 3 1

a3 f 13 1 0 2 41 1 0 + 1 41 13 = 0
a3 f a1 fa1 f 0 2 4 a1 f 0 + 1 12 a1 f = 0
a3 f d1 i 8 a1 f + 12 + 1 + = 0
2

3 + 32 + 3 8 + 8 + 13 + = 0

3 + 32 + 10 + 2 = 0
According to Cayley Hamilton theorem,
A3 + 3 A 2 + 10 A + 2 I = 0
Operating by A1
A3 A 1 + 3 A2 A 1 + 10 AA1 + 2 A 1 I = 0
A 2 + 3 A + 10 I + 2 A 1 = 0
1 2
A 1 = A 3 A 10 I .
2
Now,

LM 3 OP LM 3 OP

2 1 2 1
A = A A = 4
MM 1 1
PP MM41 1
PP B
2
0 0
N 3 1 QN 3 1 Q
LM 9 + 8 + 1 62+3 3+ 0 +1 OP
MM 3 + 12 + 1
= 12 4 + 0 8 +1+ 0 4+0+0
PP
N 23+ 3 1+ 0 +1 Q
LM18 7 4 OP
A2 =
MM168 9 4
PP
N 2 2 Q
Matrices & Determinants 153

LM 3 2 1 OP LM
9 6 3OP
3A = 3 4
MM 1 1
PP MM
0 = 12 3 0
PP
N 3 1 3Q N 9 3 Q
LM 1 0 0 O L10 0O
1 0P = M 0 0P
0
10 I = 10 0
MM0 P M
1PQ MN 0
10
0 10 PQ
P
N 0

LM18 7 4O L 9
P4P MM12 3
3O L10 OP
0P M 0
6 0 0

MM16 2
3 A 10 I = 8 9
P M PP
2
A 10 0
N 2 PQ MN 3 9 3PQ MN 0 0 10 Q
1 LM 1 1 OP
= 4
MM 2 4
PP
13 N 7 11 Q
1 2 1
1 LM 1 1 OP
MM PP
1
A = A 3 A 10 I = 4 2 4
2 2
13 N 7 11 Q
6.11 SOLUTION OF LINEAR SYSTEM OF EQUATIONS:
Consider 3 equations in 3 variables:
a1x + b1y + c1z = d1
a2x + b2y + c2z = d2
a3x + b3y + c3z = d3.
The solution is the value of x, y and z which simultaneously satisfy the above equations. The solution
can be obtained by various methods. Cramers rule and matrix method are 2 such methods of solving
the system of equations.

1. Cramers rule:

a1 b1 c1
= a2 b2 c2
Let
a3 b3 c3

Multiplying both sides by x

a1 b1 c1
x = x a2 b2 c2
a3 b3 c3
154 Basic Mathematics

a1 x b1 c1
x = a2 x b2 c2 C1 = C1 + C2 y + C3 z
a3 x b3 c3

a1 x + b1 y + c1 z b1 c1
x = a2 x + b2 y + c2 z b2 c2
a3 x + b3 y + c3 z b3 c3

d1 b1 c1
x = d2 b2 c2 = x say a f
d3 b3 c3

x = x

x
x= .

a1 b1 c1
= a2 b2 c2
Similarly,
a3 b3 c3

Multiplying by y

a1 b1 y c1
y = a2 b2 y c2 C2 = C2 + xC1 + zC3
a3 b3 y c3

a1 b1 y + a1 x + c1 z c1
y = a2 b2 y + a2 x + c2 z c2
a3 b3 y + a3 x + c3 z c3

a1 d1 c1
y = a2 d2 c2 = y say a f
a3 d3 c3

y = y

y
y=

Matrices & Determinants 155

z
Similarly z= .

Note: If there are 2 equations in 2 variables,
Say a1x + b1y = d1
a2 x + b2 y = d2
a1 b1
Then assume =
a2 b2

d1 b1
x =
d2 b2 which is obtained by replacing the 1st column by d1 and d2

a1 d1
y = nd
a2 d2 which is obtained by replacing 2 column by d1 and d2.

By Cramers rule,

x y
x= and y = .

2. Matrix method:
To solve
a1x + b1y + c1z = d1
a2x + b2y + c2z = d2
a3x + b3y + c3z = d3

LM a 1 b1 c1 OP x LM OP
d1 LM OP
Assume
MMa
A= a b2
PP MM PP
c2 , X = y and D = d2
MM PP
NQ
2

N 3 b3 c3 Q z d3 N Q
Then matrix equation is AX = D
Operating by A1 [3 A is non-singular, A1 exists]

A 1 AX = A 1 D
IX = A 1 D
X = A 1 D

LM xOP
MM yzPP = A
1
D.
NQ
i.e.,
156 Basic Mathematics

Here A1 can be calculated either by using Cayley Hamilton theorem or by using the formula
adj A
A 1 = .
A

WORKED EXAMPLES:

I. Solve the following by Cramers rule:


1. 3x + 4y = 11
2 x + 3y = 8

Let =
3
2
4
3
af af
= 3 3 2 4 = 9 8 = 1.

x =
11
8
4
3
af af
= 11 3 4 8 = 33 32 = 1

y =
3
2
11
8
af af
= 3 8 11 2 = 24 22 = 2

x y
x= and y = .

1 2
x= and y =
1 1
x = 1 and y = 2.
2. 2x + 4y = 7
x 7y = 6.

Let =
2
1
4
7
a f af
= 2 7 4 1 = 14 4 = 18.

x =
7
6
4
7
a f af
= 7 7 4 6 = 49 24 = 73.

y =
2
1
7
6
a f af
= 2 6 7 1 = 12 7 = 5.

x y
x= and y =

73 5
x= and y =
18 18
73 5
x= and y = .
18 18
Matrices & Determinants 157

3. 2x 4y + 3z = 3
3x + 3y + 2z = 15
5x 2y + 2z = 7

2 4 3
Let = 3 3 2
5 2 2

=2
3
2
2
2
a f
4
3
5
2
2
+3
3
5
3
2

a f a f a f
2 6 + 4 + 4 6 10 + 3 6 15

2 a10f + 4 a4 f + 3 a21f
20 16 63
= 20 79 = 59.

3 4 3
x = 15 3 2
7 2 2

=3
3
2
2
2
a f
4
15
7
2
2
+3
15
7
3
2
= 3 (6 + 4) + 4 (30 14) + 3 (30 21)
3 (10) + 4 (16) + 3 (51)
30 + 64 153
x = 94 153 = 59.

2 3 3
y = 3 15 2
5 7 2

15 2 3 2 3 15
=2 3 +3
7 2 5 2 5 7
= 2 (30 14) 3 (6 10) + 3 (21 75)
= 2 (16) 3(4) + 3(54)
= 32 + 12 162
y = 44 162 = 118.
158 Basic Mathematics

2 4 3
z = 3 3 15
5 2 7

=2
3
2
15
7
a f
4
3
5
15
7
+3
3
5
3
2
= 2 (21 + 30) + 4 (21 75) + 3 (6 15)
= 2 (51) + 4 (54) + 3 (21)
102 216 63
z = 102 279 = 177.
x 59
x= = =1
59

y 118
y= = =2
59

z 177
z= = =3
59
x = 1, y = 2 and z = 3.
4. x y 2z = 3
2x + y + z = 5
4x y 2z = 11

1 1 2
Let = 2 1 1
4 1 2

=1
1
1
1
2
1
2
4
a f 1
2
a f
+ 2
2
4
1
1
= 1 (2 (1)) + 1(4 4) 2 (2 4)
= 1 (2 + 1) + 1(8) 2 (6)
= 1 8 + 12 = +3.

3 1 2
x = 5 1 1
11 1 2

=3
1
1
1
2
a f
1
5
11
1
2
+ 2
5
11
a f 1
1
Matrices & Determinants 159

b a fg a f a f
3 2 1 + 1 10 11 2 5 11

3 a2 + 1f + 1 a21f 2 a16 f

3 a1f + 1a 21f 2 a16f

3 21 + 32
24 + 32 = 8.

1 3 2
y = 2 5 1
4 11 2

=1
5
11 2
1
3
4 2
2
a
1
f 2
+ 2
4 11
5

= 1a10 11f 3 a4 4 f 2 a22 20 f

= 21 3 a 8f 2 a2 f
= 21 + 24 4
= 25 + 24 = 1.

1 1 3
z = 2 1 5 =1
1
1
5
11
a f
1
2
4
5
11
+3
2
4
1
1
4 1 11

b a fg a f a
1 11 1 5 + 1 22 20 + 3 2 4 f
11 + 5 + 1 a2 f + 3 a6 f

16 + 2 18 = 0.

x y
x= ,y= and z = z

8 1 0
x= ,y= and z =
3 3 3
8 1
x= , y= and z = 0.
3 3

II. Using matrix method solve the following system of equations:


1. 2x 3y = 4
3 x + 2y = 5
160 Basic Mathematics

A=
LM2 3 OP
,X=
x
and D =
4LM OP LM OP
Let
N3 2 Q y 5 NQ NQ
Matrix equation AX = D
X = A1D.
adj A
Now A 1 =
A

Adj A =
LM 2 3OP and A = 2 3
a f
= 4 9 = 4 + 9 = 13
N3 2Q 3 2

adj A 1 L 2 3O
A 1 =
A
= M
13 N 3 2 PQ
LM OP LM4OP B

1 2
1 3
X= A D=
Now
13 3 N Q N5Q 2

1 L 8 + 15O
13 MN12 + 10PQ
X=

1 L 23O
X= M P
13 N 2 Q

LM 23 OP
LM OP = M 13 P
x
N yQ M 2 P
N 13 Q
23 2
x= ; y= .
13 13
2. 2x 3y = 4
4x 5y = 10.

A=
LM2 3 OP
,X=
x
and D =
LM OP
4 LM OP
Let
N4 5 Q y 10NQ N Q
Matrix equation: AX = D
X = A1D.
adj A
Now A 1 =
A
Matrices & Determinants 161

Adj A =
LM5 3OP and A = 2 a5f a3f 4 = 10 + 12 = 2.
N4 2Q
adj A 1 L 5 3O
= M
2 N 4 2 PQ
1
A =
A

LM OP LM 4 OP B

1 5 3
X = A 1 D =
2 4 N Q N10Q2

1 L 20 + 30 O
= M
2 N 16 + 20 PQ

1 L10 O L 5O
X= M P=M P
2 N 4 Q N2 Q

LM x OP = LM5OP
N y Q N2 Q
x = 5 and y = 2.
3. x + y = 1
y+z=7
z+x=2
Given equations: x + y + 0z = 1
0x + y + z = 7
x + 0y + z = 2

LM1 1 0OP x LM OP
1 LM OP
MM1
A= 0 1
PP MM PP
1 , X = y and D = 7 .
MM PP
N Q NQ NQ
Let
0 1 z 2

Matrix equation: AX = D
X = A1D
adj A
Now A 1 =
A

1LM 1 0 OP
A= 0
MM 1 1
PP
N Q
Consider
1 0 1

af a f
A = 1 1 1 1 = 1 + 1 = 2.
162 Basic Mathematics

LM 1 1

1 0 1 0 OP
MM 00 1
1
0
1
1
0
1
1
1
0
PP
Adj A =
MM 1 1 1 1

0 1 PP
MM 01 1

1 1 1 1 PP
N 0 1 0 0 1 Q
LM 1 1 1OP L 1 1 1O
adj A = a1f
MM 1 a11f 11PP = MMM11 11 11PPP
N Q N Q
adj A 1 M
L 1 1 1OP
2 M
MN1 1 1PPQ
1
A = = 1 1 1
A

L 1 1 1OP LM1OP 1 LM 1 7 + 2 OP
1M
2M
MN1 1 1PQ MN2PQ MN1 + 7 + 2PPQ
P M P 2M
1
X = A D= 1 1 1 7 = 1+ 7 2

L4O L2O
1M P M P
2M P M P
X= 6 = 3
MN 8PQ MN 4PQ
LM x OP LM2OP
MM yz PP = MM 43PP x = 2, y = 3 and z = 4.
NQ N Q
x = 2, y = 3 and z = 4.
4. x + y + z = 3
x + 2y + 3z = 4
x + 4y + 9z = 6.

LM
1 1 1 OP x LM OP 3 LM OP
A= 1
MM 2
PP MM PP
3 , X = y and D = 4
MM PP
N Q NQ NQ
Let
1 4 9 z 6

Matrix equation: AX = D
X = A1D.
adj A
Now A 1 =
A
Matrices & Determinants 163

LM1 1 1OP
MM1 4 9PP
A= 1 2 3
N Q
Consider

A = 1 a18 12f 1 a9 3f + 1 a 4 2 f
A = 6 6 + 2 = 2.

LM+ 2 3 1 1 + 1 1 OP
MM 41 93 41 91 21 31 PP
adj A =
MM 1 9 + 1 9 1 3 PP
MM + 11 24 11 41 + 11 21 PP
N Q
LM 18 12 a9 4f a3 2f OP
adj A = a9 3f
MM 4 2 a9411f a2311fPP
N Q
LM 6 5 1 OP
adj A = 6
MM 2 83 +21PP
N Q
adj A 1 M
L 6 5 1OP
2 M
MN 2 3 +1PPQ
1
A = = 6 8 2
A

L6 OP LM3OP

5
1M
1

2M PP MM46PP B
1
Now X=A D = 6 8 2
MN 2 3 +1 QN Q
1
LM
18 20 + 6
1
4 OP LM OP
MM
X = 18 + 32 12 = 2
PP MM PP
2
N
6 12 + 6
2
0 Q NQ
LM x OP LM2OP
MM z PP MM0PP
X= y = 1 .
NQ NQ
x = 2; y = 1; z = 0.
164 Basic Mathematics

6.12 APPLICATION OF MATRICES IN BUSINESS PROBLEMS:


1. A company sold 22 scooters, 15 bikes and 10 mopeds in January and 20 scooters, 20 bikes and
12 mopeds respectively in June. Represent the data in matrix form.
Scooters Bikes Mopeds
January 22 15 10
June 20 20 12
22LM 15 10 OP
Corresponding matrix = 20
N 20 12 Q
2. Suppose the matrices A and B represent the number of items of different kinds produced by 2
manufacturing units in one day.

LM OP
2 1LM OP
MM PP MM PP
A = 5 and B = 3 Compute 2A + 5B what does 2A + 5B represent?
7NQ 6 NQ
Solution: 2A + 5B

LM OP LM OP
2 1
MM PP MM PP
=2 5 +5 3
7 NQ NQ
6

LM 4OP LM 5OP LM 4 + 5OP


MM14PP MM30PP MM14 + 30PP
= 10 + 15 = 10 + 15
N Q N Q N Q
L 9O
= M 25P .
MM44PP
N Q
2A + 5B represents the number of items produced by one unit in two days and another in 5 days
together.
3. A man buys 3 kgs of dal, 2 kgs of rice and 5 kgs of oil. If the cost of each kg is Rs. 35, Rs. 20
and Rs. 75 respectively. Then find the total cost by matrix method.
Solution: Matrix representing the commodities
= [3 2 5]

35 LM OP
Matrix representing the costs = 20
MM PP
75 N Q
Matrices & Determinants 165

LM35OP
5 20 B

Total cost = 3 2
MM75PP
N Q
= 3 35 + 2 20 + 5 75

= 105 + 40 + 375 = 520


Total cost = Rs. 520.
4. A company sold 30 metal chairs, 40 wooden chairs and 25 plastic chairs in February and 60, 50
and 75 respectively in March. The selling price of a metal chair is Rs. 150, that of wooden chair
is Rs. 500 and plastic chair is Rs. 300. Find the total revenue in February and March using matrix
method.
Solution:
Metal chair Wooden Chair Plastic chair
February: 30 40 25
March: 60 50 75
30 LM 40 OP
25
Corresponding matrix = 60
N 50 75Q
LM OP
150

MM PP
Matrix representing the price = 500
300N Q
L30 40 25OP LM150 OP

Total revenue = M
N60 50 75Q MMN500 300PQ
PB

=M
L30 150 + 40 500 + 25 300OP
N60 150 + 50 500 + 75 300Q
=M
L 4500 + 20000 + 7500 OP
N9000 + 25000 + 22500Q
=M
L32,000OP
N56,500Q
Total revenue in February = Rs. 32,000/-
and Total revenue in March = Rs. 56,500/-
Total revenue = 32,000 + 56,500
= Rs. 88,500/-
166 Basic Mathematics

5. Matrix A and matrix B give the daily sales and selling price of soft drinks for a shopkeeper.

Pepsi Coke Thumbsup

Mon LM6 3 1 OP Pepsi 7 LM OP


A=
Tue
MM21 3 0
PP B = Coke 6
MM PP
Wed
Thu MN2
1
1
7
5 PQ Thumbsup 6 NQ
Find the total revenue from four days.
Solution: Revenue matrix = AB.

LM6 OP L7O

3 1

=M
2
MM 1
3 0
PP MM6PP B
PQ MN6PQ
1 7
N2 1 5

LM 6 7 + 3 6 + 1 6 OP LM42 + 18 + 6OP LM66OP Mon


=M P =M P =M P
27+36+06 14 + 18 + 0 32 Tue
MM 1 7 + 1 6 + 7 6 PP MM 7 + 6 + 42 PP MM55PP Wed
N 2 7 + 1 6 + 5 6 Q N14 + 6 + 30Q N50Q Thu
Total revenue for 4 days = 66 + 32 + 55 + 50
= Rs. 203.
5. In a certain town there are 4 colleges and 12 schools. Each school has 8 peons, 5 clerks and 2
cashiers. Each college has 10 peons, 7 clerks and 3 cashiers. In addition, each college has 1
section officer and one librarian. The monthly salary of each of them is as follows: Peon: Rs.
2000; Clerk Rs. 3000, Cashier Rs. 5000, Section Officer Rs. 6000 and Librarian Rs. 4500. Using
matrix notation find (1) Total number of posts of each kind in schools and colleges taken together
(2) Monthly salary bill of all the schools and colleges taken together.
Solution: Let A = [5 12] represents the number of colleges and schools in that order.
Peon Clerk Cashier S. off. Librarian

B=
College 10 LM 7 3 1 1 OP
N Q
Let
School 8 5 2 0 0

Peon LM2000OP
Clerk
C = Cashier
MM3000 P
5000 P
S. off. MM6000PP
Librarian N4500Q
Matrices & Determinants 167

Number of employees = AB


LM10 7 3 1 1 OP B
i.e., 5 12
N8 5 2 0 0 Q
= 5 10 + 12 8 5 7 + 12 5 5 3 + 12 2 5 1 + 12 0 51+1 0

= 50 + 96 35 + 60 15 + 24 5+0 5+0

= 146 95 39 5 5
1st element i.e. 146 represent number of peons, 2nd element, 95 represent number of clerks and so
on.
6. Total monthly salary bill of each school and college is given by matrix BC.

LM2000OP
LM10 1O M P
3000
P0Q MM5000PP B
7 3 1
N8 5 2 0
MM6000PP
N4500Q
LM10 2000 + 7 3000 + 3 5000 + 1 6000 + 1 4500OP
N 8 2000 + 5 3000 + 2 5000 + 0 + 0 Q
=M
L20,000 + 21,000 + 15,000 + 6,000 + 4,500OP
N 16,000 + 15,000 + 10,000 + 0 + 0 Q
=M
L66,500OP
N41,000 Q
Total monthly salary bill of all colleges and schools taken together

a f
= A BC = 5
LM66,500OP
12
N41,000Q
= 5 66,500 + 12 41,000

= 332500 + 492000
= [8,24,500]
Total salary = Rs. 8,24,500.

Alieter:
Total monthly salary of all colleges and schools
168 Basic Mathematics

= Total number of employees salary

LM2000OP
= 146

M 3000
5 M5000 P B
P
95 39 5
MM6000PP
MN4500PQ
= 146 2000 + 95 3000 + 39 5000 + 5 6000 + 5 4500

= 292000 + 285000 + 195000 + 30000 + 22500

= 8,24,500
Hence total salary = Rs. 8,24,500.
6. A salesman has the following record of sales during 3 months for 3 items A, B, C which have
different rates of commission.

Month Sales of units Total Commission

A B C

Jan 100 100 200 900


Feb 300 200 100 1000
Mar 100 200 300 1400

Find out the rates of commissions on items A, B and C.


Solution: Let x, y and z denote the rates of commission in Rupees per unit for A, B and C items
respectively. Then the data given can be expressed as a system of linear equation.
100 x + 100 y + 200 z = 900
300 x + 200 y + 100 z = 1000

100 x + 200 y + 300z = 1400


or
x + y + 2z = 9

3x + 2 y + z = 10
x + 2 y + 3z = 14
Solving these equations by matrix method:

LM1 1 2OP x LM OP 9 LM OP
Let MM1
A= 3 2
PP MM PP
1 , X = y and D = 10
MM PP
N 2 3 Q z NQ 14 N Q
Matrices & Determinants 169

AX = D
X = A 1 D
adj A
Now A 1 =
A

1 1 2
A=3 2 a f a
1 = 1 2 3 2 1 3 3 1 + 2 6 2 f a f
1 2 3

a f af
= 6 2 91 + 2 4
4 8 + 8 = 4 0. A1 exist.

LM 2 1

1 2 1 2 OP
MM 23 3 2 3 2 1
PP
MM 1 PP
1 1 2 1 2
Now adj A =
3 1 3 3 1
MM 31 2

1 1 1 1 PP
N 2 1 2 3 2 Q
AX = D
X = A1D

LM 4 1 3 OP
MM 4
adj A = 8 1 15
PP
N Q
Now
1 1

adj A 1
4 LM 1 OP
3
MM PP
1
A = = 8 1 +5
A 4
4 N 1 1 Q
1
LM
4 OP LM 9 OP
1 3
X = A 1 D =
MM
8
PP MM1014PP
1 +5
4
4N 1QN Q 1

L 4 9 + 1 10 3 14 O
X = M8 9 + 1 10 + 5 14P
4M
MN 4 9 1 10 1 14 PPQ
1

1M
L 36 + 10 42 OP 1 LM 4 OP
4M
MN 36 10 14 PPQ 4 MMN12PPQ
X= 72 + 10 + 70 = 8
170 Basic Mathematics

LM1OP
X= 2
MM3PP
NQ
LM xOP LM1OP
MM yzPP = MM23PP
NQ NQ
x = 1, y = 2 and z = 3.
i.e., Rate of commission per unit for the 3 items is Re. 1, Rs. 2 and Rs. 3 respectively.
7. The prices of 3 commodities X, Y and Z are x, y and z respectively. A sells 1 unit of X, 1 unit of
Y and 1 unit of Z. B sells 3 units of X, 1 unit of Y and purchases 1 unit of z. C sells 1 unit of X,
3 units of Y and purchases 1 unit of Z. In the process A, B and C earns Rs. 9000, Rs. 1000 and Rs.
5000 respectively. Using matrices find the prices per unit of the commodities. (Note that selling
the units is positive earning and buying the units is negative).
Solution. The above data can be written in the form of simultaneous equations:
A: x + y + z = 9000
B: 3x + y z = 1000
C: x + 3y z = 5000.
Solving these equations, by Cramers rule:

1 1 1
Let = 3 1 a f a f a f
1 = 1 1 + 3 1 3 + 1 + 1 9 1
1 3 1

a f af
2 1 2 + 1 8

2 + 2 + 8 = 12.

9000 1 1
x = 1000 1 1
5000 3 1

a f a f a
9000 1 + 3 1 1000 + 5000 + 1 3000 5000 f
9000 a2f 1 a 4000 f + 1 a 2000 f

18000 4000 2000


= 12000

1 9000 1
y = 3 1000 1
1 5000 1
Matrices & Determinants 171

a f a f a
= 1 1000 + 5000 9000 3 + 1 + 1 15000 1000 f
4000 9000 a 2 f + 1 a14000 f
4000 + 18000 + 14000
= 36000.

1 1 9000
z = 3 1 1000
1 3 5000

a f a
= 1 5000 3000 1 15000 1000 + 9000 9 1 f a f
2000 14000 + 72000
z = 60000.

x 12000
Hence x= = = 1000
12

y 36000
y= = = 3000
12

z 60000
z= = = 5000
12
Hence price per unit of x = Rs. 1000, that for y = Rs. 3000 and for z = Rs. 5000.
8. A company is considering which of the 3 methods of production it should use in producing 3
products X, Y and Z. The amount of each product and produced by each method is as shown
below.

Method Product X Product Y Product Z

I 4 8 2
II 5 7 2
III 3 6 5

Further information relating to profit per unit is as follows:

Product Profit/Unit

X 10
Y 5
Z 7

Using matrix multiplication find which method maximises the total profit:
172 Basic Mathematics

LM4 8 2OP 10 LM OP
Let
MM3
A= 5 7
PP
2 B= 5
MM PP
N 6 5 Q 7 N Q
Total profit = AB

LM4 8 2OP LM10OP


MM3 6 5PP MM 57 PP
= 5 7 2
N QN Q
LM4 10 + 8 5 + 2 7OP
MM3 10 + 6 5 + 5 7PP
= 5 10 + 7 5 + 2 7
N Q
L40 + 40 + 14O L94O
A = M 50 + 35 + 14 P = M99P
MM30 + 30 + 35PP MM95PP
N Q N Q
Total profit in method I = 94.
That in method II = 99 and in method III = 95.
Method II maximises the profit.

REMEMBER:
Matrix is an arrangement of numbers in horizontal rows and vertical columns.
Two matrices of the same order are said to be equal if and only if the corresponding elements are
equal.
Two matrices can be added subtracted if they have same order. It is obtained by adding/subtract-
ing the corresponding elements.
Multiplication of a matrix by a scalar is obtained by multiplying each and every element by a
scalar.
It A is of order m n and B is of order n p then AB is of order m p, i.e., matrix multiplication
is possible only when number of column in 1st matrix is equal to number of rows in the 2nd
matrix.
Transpose of a matrix is obtained by interchanging rows and columns.
A unique value associated with every square matrix is called its determinant value.
If det A = 0, i.e., |A| = 0 for a matrix A, then A is called singular matrix. Otherwise it is called non-
singular matrix.
The value of the determinant is unaltered if its rows and columns are interchanged.
If 2 rows or columns are interchanged the value of the determinant changes its sign.
If in a determinant 2 rows or columns are identical then the value of the determinant is zero.
Matrices & Determinants 173

If the elements of any row (or column) is multiplied by k, the value of the determinant is multi-
plied by k.
If to the elements of any row (or column) of a determinant the same multiples of the correspond-
ing elements of other rows (or columns) of the determinant are added the value of the determinant
remains the same.
The determinant obtained by deleting the row and column containing the element is called minor
of that element. If the minors are multiplied with (1)i + j [where i = row number and j = column
number of the element] we get co-factors. The transpose of the co-factor matrix is called adjoint
of the matrix.
For a non-singular matrix A.
adj A
A 1 = .
A
If A is a square matrix and I is the identity matrix of the same order.
The characteristic equation: |A I| = 0. The values of obtained is called eigen values or
characteristic roots.
Every square matrix satisfies its characteristic equation, |A I| = 0. This is Cayley Hamilton
theorem.
The solution of system of equations
a1 x + b1 y + c1 z = d1
a2 x + b2 y + c2 z = d 2

a3 x + b3 y + c3 z = d3
By Cramers rule:

a1 b1 c1 d1 b1 c1
Let
= a2 b2 c2 , x = d 2 b2 c2
a3 b3 c3 d3 b3 c3

a1 d1 c1 a1 b1 d1
y = a2 d2 c2 , z = a2 b2 d2
a3 d3 c3 a3 b3 d3

x y
Then x= , y= and z = z .

By matrix method:

LM a1 b1 c1 OP x LM OP d1 LM OP
Let MMa
A= a b2
PP MM PP
c2 , X = y and D = d2
MM PP
NQ
2

N 3 b3 c3 Q z d3 N Q
174 Basic Mathematics

Then matrix equation


AX = D
X = A1D.
adj A
and A 1 = or A1 can be found by using Cayley Hamilton theorem.
A

EXERCISE

1. If A =
LM 1 3 OP
and B =
6 LM 5OP
, find A + B .
N2 1 Q 1 N 0 Q
2. If A = M
L1 6 0OP
and B =
0LM 1 7OP
N1 7 1 Q 6 N 5 2 Q
,

find A + 3B.

3. If A =
LM 1 OP
1 LM 1OP , find matrix X such that A + X = B.
and B =
0
N2 Q
1 N 1Q
1

4. Find x and y if M
L4 7OP + LM y + 2 1OP = LM 1 8OP .
N x 7 Q N 5 0 Q N6 7Q
L 1 2OP and B = LM 1 3OP, verify that (A + B) = A + B.
5. If A = M
N7 6 Q N 7 8Q
L x 4OP + L2 x 5O = L3 9O
6. Solve for x: M
N 1 7Q MN 8 3PQ MN9 10PQ
2

7. If A is of order 4 5 and B is of order 5 3, does AB and BA exists? If so what are their order?

8. If A =
LM1 7OP
and B =
1 LM 0OP, then find AB.
N5 2 Q 6 N 5Q
9. If A = M
L1 2O
N2 4 PQ
, find AA and AA. Is AA = AA?

L1 0 0 O L x O L1 O
10. If M 0 1 0 P M y P = M2 P , find x, y and z.
MM0 0 PM P M P
1 PQ MN z PQ MN 3PQ
N
11. If A = M
L1 0O
1PQ
, then prove that AA = AA = I.
N0
Matrices & Determinants 175

12. If A =
LM0 OPi
Ni Q0
, then find A2.

L 1 3OP LM 2OP = LM x OP , then find x and y.


13. If M
N7 1Q N3Q N y Q
LM 1 2 3OP LM 1 7 2OP
MM 1 2 7PP MM 5 20 61PP , then verify A (B + C) = AB + AC.
14. If A = 1 5 3 , B = 1
N Q N Q

15. If A = M
L1 2 3OP , B = LM13OP and C = 2 1 , then verify A (BC) = (AB) C.
N1 5 2Q MMN2PPQ
L 1 2 3OP
16. If A = M0
MM 1 11 21PP , then find A . 3

N Q
17. If A = M
L 1 2OP , then prove that A 4A + 17I = 0. Where I is identity matrix of order 2 2.
N7 3Q
2

LM 1 2 0OP
18. If A = 2
MM 1 31 66PP , then prove that A A + 13A 9I = 0.
3 2

N Q
19. If A = M
L1 2OP , B = LM2 1OP , then prove that (AB) = BA.
N3 1Q N3 2Q
LM 1 2 2OP
20. Prove that A = 2
MM2 21 21PP satisfies A 4A 5I = 0.
2

N Q
21. Evaluate:

1 2 1 3 1 7
(a) 7 5 (b) 6 5 (c) 2 6

5 3 7 5
(d) 6 7 (e) 5 2
176 Basic Mathematics

22. Evaluate:

1 2 4 1 2 7 1 1 2
(a) 2 3 5 (b) 3 5 4 (c) 2 1 1
3 7 8 2 0 8 4 1 2

23. Find x if the matrix A is singular.

LM x 5 OP LM 2 3 4 OP
(a) A =
N20 Q (b) MM 5
A = 4 x 8
PP
N Q
x
6 7

24. Solve for x:

x 2 1
5 x 3 x
(a) 2 =3 (b) 5 =8 (c) 2 5 x =0
7 x
1 2 x

25. Using the properties of determinants find


3485 3486 6001 6002
(a) 3487 3488 (b) 6004 6005

241 242 243 2100 2101 2102


(c) 244 245 246 (d) 2110 2111 2112
247 248 249 2120 2121 2122

ab bc ca
26. Prove that b c ca a b = 0.
ca ab ba

1 a a3
27. Prove that 1 b a fa fa fa
b3 = a b b c c a a + b + c f
1 c c3

1 a a2
28. Prove that 1 b a fa fa f
b2 = a b b c c a
1 c c2

a2 ab ac
29. Prove that ab b2 bc = 4a 2 b 2 c 2
ac bc c 2
Matrices & Determinants 177

x p q
30. Prove that p x a
q = x p xq x+ p+qfa fa f
p q x

1+ a b c
31. Prove that a 1+ b c = 1+ a + b + c
a b 1+ c

32. Prove that x = 9 is a root of equation

x +1 3 5
2 x+3 5 =0
2 3 x+4

a + b + 2c a b
33. Prove that c b + c + 2a b =2 a+b+c a f 3

c a c + a + 2b

34. Find the adjoint of:

LM1 2OP LM 1 2 OP LM1 7 OP LM6 5 OP LM5 OP


3
(a)
N3 4 Q (b )
N 7 6 Q (c )
N5 6 Q (d )
N0 4 Q ( e)
N 2 1Q
35. Find the inverse of:

LM4 3 OP LM6 5OP L5


(c) M 6
OP
1
(a)
N1 1 Q N Q
0 7 (b)
N 2Q
L1
36. If A = M
3O L1 2OP , verify (AB) = B A .
7PQ
and B = M 1 1 1
N4 N 2 7Q
LM3 2 5OP
MM1 3 5PP
37. Find the adjoint of A = 4 5 6 and hence find A . 1

N Q
L 0 1 1O
38. Find the inverse of M 4 2 0 P .
MM 3 1 4PP
N Q
LM1 4OP
39. Find the characteristic polynomial and characteristic roots of the matrix
N2 3 Q
.
178 Basic Mathematics

LM1 7OP .
40. Verify Cayley Hamilton theorem for the matrix
N5 2 Q
LM 1 2 0OP
41. Verify Cayley Hamilton theorem for the matrix 1 3
MM 1 6 25PP .
N Q
L1 0 OP
42. Find A using Cayley Hamilton theorem for the matrix M2
2

MM3 21 PP
1
0 .
N 1 Q
LM 1 2 3OP
43. Find A using Cayley Hamilton theorem if A = 0
3
MM 1 11 21PP .
N Q
44. Solve the following system of equations (a) By Cramers rule (b) By matrix method:
(i) 7 x y = 16 (ii) 3x + 4 y = 10
2 x + 3y = 2 4 x 5y = 3

(iii) 2 x 3y = 4 (iv) x + y = 7
3x + 2 y = 5 2x + y = 8

(v) 3 x + 4 y = 7 (vi) x + y + 2 z = 9
7x y = 6 3x + 2 y + z = 10
x + 2 y + 3z = 14

(vii) x 4 y 3z = 9 (viii) x y 2 z = 3
5 x + y = 19 2x + y + z = 5
2 x 5z = 3 4 x y 2 z = 11

(ix) x + y + 2 z = 9 (x) x + y = 1
3x + 2 y + z = 10 y+z=7
x + 2 y + 3z = 14 z+x=2
45. A man buys 8 dozens of mangoes; 10 dozens of apples and 4 dozen of bananas. Mangoes cost
Rs. 18 per dozen, apple Rs. 9 per dozen and bananas Rs. 6 per dozen. Represent the quantities
bought by a row matrix and prices by column matrix and hence find the total cost.
46. A company is considering which of the 3 methods of production it should use in producing 3
goods X, Y and Z. The amount of each good produced by each method is shown in the matrix.
Matrices & Determinants 179

X Y Z
LM
I 4 8 2 OP
II 5
MM 7 1
PP
III 5N 3 9 Q
The vector [10 4 6] represents the profit per unit for the goods X, Y and Z in that order. Find
which method maximises profit.
47. Matrix A and B give the daily sales and sale price of chocolates for a shopkeeper.

Bar one Dairy milk Five star

Mon LM6 3 1 OP Bar one 5LM OP


A=
Tue
MM21 3 0
PP B = Dairy milk 6
MM PP
Wed
Thu MN1
2
4
4
3 PQ Five star 7 NQ
Find the total revenue for four days.
48. A salesman has the following record of sales during 3 months for 3 items, A, B and C which have
different rates of commission.

Months Sales of units Total commission


in Rs.
A B C

January 90 100 20 800


February 130 50 40 900
March 60 100 30 850

Find out the rates of commission on A, B and C.


49. The prices of the 3 commodities X, Y and Z are x, y and z per unit respectively. A purchases 4 units
of z and 3 units of x and 5 units of y. B purchases 3 units of y and sells 2 units of x and 1 unit
of z. C purchases 1 unit of x and sell 4 units of y and 6 units of z. In the process A, B and C earn
Rs. 6000, 5000 and 13,000 respectively. Using matrices, find the prices per unit of the 3 com-
modities. (Note that selling the unit is positive earnings and buying the unit is negative earning).
50. Suppose the matrices X and Y represent the number of items of different kinds produced by 2
manufacturing units in one day.

4 LM OP 3 LM OP
X = 5
MM PP
and Y= 4 .
MM PP
6 NQ 5 NQ
Compute 2X + 3Y, what does 2X + 3Y represent?
180 Basic Mathematics

ANSWERS

LM7 OP
4 LM 1 9 21 OP LM1 2OP
1.
N7 1Q 2.
N19 22 5 Q 3.
N 1 0 Q
4. x = 1, y = 5 6. x = 1 or x = 3
7. AB exists and is of order 4 3. BA does not exist.

LM1 0OP LM 5 10 OP
8.
N1 0Q 9. AA =
N10 20
= AA
Q 10. x = 1, y = 2 and z = 3.

LM1 0 OP LM 11 22 19 OP
12.
N0 1 Q 13. x = 7, y = 11 16.
MM49 5 9
PP
N 17 16 Q
21. ( a ) 9 ( b ) 23 (c ) 8 ( d ) 53 (e) 39. 22. ( a ) 7 ( b ) 94 (c) 3

23. ( a) x = 10, (b) x = 6 24. ( a) 19 (b) 4 (c ) 3 or 1


25. ( a ) 2 ( b ) 3 (c ) 0 ( d ) 0

LM 4 2OP (b) LM6 2OP (c) LM6 7OP (d ) LM4 5OP (e) LM1 3 OP
34. ( a )
N3 1Q N7 1Q N 5 1Q N0 6Q N 2 5 Q
1 L 1 3O 1 L 7 5O 1 L 2 1O
35. (a) M P
7 N 1 4 Q
( b) M
42 N 0 P
6Q
( c) M
4 N 6 5PQ

1 M
L 7 5 13OP 1 M
L8 5 2OP
28 M P 38. 11 M 4
MN 7 3 1PPQ
14 10 2 3 1
MN 7 7 7PQ
37.

39. 2 4 5 = 0, roots : 5 and 1.

1
1 LM 4 OP
2 LM 11 22 19 OP
42. 3 2 MM 5 4
PP MM49 5 9
PP
N Q N Q
43.
1 2 1 +17 16

23 2
44. (i ) x = 2, y = 2 (ii ) x = 2, y = 1 (iii ) x = ,y=
13 13
(iv ) x = 1, y = 6 (v ) x = 1, y = 1 ( vi ) x = 1, y = 2, z = 3

( vii) x = 4, y = 1, z = 1 ( viii ) x = 8 3, y = 1 3 , z = 0
Matrices & Determinants 181

(ix ) x = 1, y = 2, z = 3 ( x ) x = 2, y = 3, z = 4.

LM18OP
45. 8 10 4
MM 69 PP = 258 .
N Q
Cost = Rs. 258.
46. Method III. Total profit 116.
47. Rs. 178.
48. Rs. 2, 4 and 11 respectively.
49. Price per unit of X = Rs. 3000
Y = Rs. 1000 and Z = Rs. 2000.

LM17 OP
50.
MM2227PP It represent the number of items produced by one unit in 2 days and another in 3 days
N Q
together.
182 Basic Mathematics

7
Ratio and Proportions,
Variations
7.1 INTRODUCTION:
Two quantities of the same kind can be compared either by subtraction method or by division method.
In subtraction method we find how much more (or less) is one quantity than the other, and in division
method, we find how many times (or what fractional part) is one quantity of the other. The quotient here
is nothing but the ratio of the two quantities. For example, if I have Rs. 100 and you have Rs. 600 then
we can compare the money by subtraction method and say You have Rs. 500 more than what I have.
Or we can compare by division method and say you have 6 times the money what I have.
If the ratio of 2 mutual quantities are equal then they are said to be proportional.

7.2 RATIO:
A ratio is a relation or comparison between two quantities of the same kind. The comparison is made
by considering what multiple, part or parts the first quantity is of the second.
N
The ratio of 2 quantities x and y is denoted by x : y or . The first term x is called antecedent and
O
the second term y is called consequent.
Note:
1. A ratio is a pure number. Hence it has no units.
2. When the terms of the ratio are multiplied or divided by the same quantity the ratio is not
altered.
For instance 2 : 3 = 4 : 6 = 40 : 60 = 80 : 120...
3. If a : b and c : d are two ratios, then the ratio ac : bd is called their compound ratio.
Example : The compound ratio of 5 : 2 and 3 : 7 is 5 3 : 2 7 i.e., 15 : 14.
4. If a : b is the given ratio then the ratio
Ratio and Proportions, Variations 183

(i) a2 : b2 is called its duplicate ratio.

(ii) = : > is called its subduplicate ratio.


(iii) a3 : b3 is called its triplicate ratio.
3
(iv) = : 3 > is called its sub-triplicate ratio.

WORKED EXAMPLES:
1. Express the following ratios in their simplest form
(a) 16 : 26 (b) 16 : 64 (c) 6 : 90 (d) 90 : 10
Solution:
16 8
(a) 16 : 26 = = = 8 : 13
26 13
16 1
(b) 16 : 64 = = = 1: 4
64 4
6 2 1
(c) 6 : 90 = = = = 1 : 15
90 30 15
90 9
(d) 90 : 10 = = = 9 :1
10 1
2. Compare the following ratios :
(a) 2 : 5 and 3 : 7 (b) 12 : 13 and 13 : 2
(c) 5 : 11 and 11 : 6 (d) 8 : 3 and 4 : 7
Solution: (a) To compare 2 : 5 and 3 : 7
2
2:5 = and
5
3
3: 7 =
7
To compare 2 fractions first we make denominators equal by multiplying with suitable numbers.
2 7 14
=
5 7 35
3 5 15
=
7 5 35
14 15
Clearly is less than
35 35
i.e. 14 : 35 < 15 : 35
i.e., 2:7<3:7
184 Basic Mathematics

(b) 12 : 13 and 13 : 2
12 2 24
12 :13 = =
13 2 26
13 13 169
13: 2 = =
2 13 26
24 169
Clearly <
26 26
12 : 13 < 13 : 2
(c) 5 : 11 and 11 : 6
5 6 30
5:11 = =
11 6 66
11 11 121
11: 6 = =
6 11 66
30 121
Clearly < ie, 30 : 66 < 121 : 66
66 66
5 : 11 < 11 : 6
(d) 8 : 3 and 4 : 7
8 7 56
8:3 = =
3 7 21
4 3 12
4:7 = =
7 3 21
56 12
Clearly >
21 21
8 : 3 > 4 : 7.
3. Find the ratio between
(a) 1 hr 10 min and 140 min.
(b) 3 kg 30 gm and 1 kg 260 gms.
(c) 6 Rs. 50 ps. and 8 Rs. 75 ps.
Solution: (a) 1 hr 10 min = 60 + 10 min. = 70 min.
70
Ratio between 70 min. and 140 min. =
140
= 70 : 140 = 1 : 2
(b) 3 kg 30 gm and 1 kg 260 gms.
3 kg 30 gm = 3000 + 30 gm
= 3030 gm
Ratio and Proportions, Variations 185

1 kg 260 gms = 1000 + 260 = 1260 gm


Ratio between 3030 and 1260
3030 101
= =
1260 42
= 101 : 42.
(c) 6 Rs. 50 ps. and 8 Rs. 75 ps.
6 Rs. 50 ps = 650 ps.
8 Rs. 75 ps = 875 ps.
Ratio between 650ps and 875 ps
650 26
= = = 26 : 35.
875 35
4. Write the duplicate and subduplicate of the ratio 1 : 9
Solution: Duplicate of a : b is a2 : b2
Duplicate of 1 : 9 is 12 : 92 = 1 : 81
Subduplicate of a : b = a: b

Subduplicate of 1 : 9 = 1 : 9 = 1: 3.
5. Write the triplicate and subtriplicate of the ratio 1 : 8.
Solution: Triplicate of a : b is a3 : b3
Triplicate of 1 : 8 is 13 : 83 = 1 : 512.
3
Subtriplicate of a : b is a: 3 b
3
Subtriplicate of 1 : 8 is 1: 3 8
= 1 : 2.
6. Rajus monthly salary is Rs. 3000 and Ramas annual income is Rs. 60,000. What is the ratio of
their incomes?
Solution: Rajus monthly salary = Rs. 3000
Rajus annual income = 3,000 12
= 36,000
Given Ramas annual income = Rs. 60,000
Ratio of their income
= 36,000 : 60,000
= 6 : 10
=3:5

OR

Given Rajus monthly salary = Rs. 3000


186 Basic Mathematics

Ramas annual income = Rs. 60,000


Ramas monthly income = 60,000/12 = Rs. 5,000
Ratio of their income
= 3000 : 5000
= 3 : 5.
7. A number is divided into 3 parts in the ratio 2 : 3 : 4. If the 3rd part is 32. Find the other 2 parts.
Solution: Let 2x, 3x and 4x be the parts of the number.
Given 3rd part = 32
4x = 32
x=8
1st part = 2x = 2 (8) = 16
2nd part = 3x = 3 (8) = 24.
8. A bag contains rupee, 50 paise and 25 paise coins in the ratio 5 : 6 : 8. If the total amount is
Rs. 840, find the number of coins of each type.
Solution: Ratio of 1 Re., 50 ps., and 25 ps. coins = 5 : 6 : 8
5 6 8
Ratio of values = : :
1 2 4
Ratio of values = 5 : 3 : 2
Now Divide Rs. 840 in the ratio 5 : 3 : 2
Sum of the terms of ratio = 5 + 3 + 2 = 10
5
1st part = Rs. 840 = Rs. 420.
10
3
2nd part = Rs. 840 = Rs. 252.
10
2
3rd part = Rs. 840 = Rs. 168.
10
Number of 1 Re. coins = 420
Number of 50 ps. coins = 252 2 = 504
Number of 25 ps. coins = 168 4 = 672.
9. In a mixture of 35 litres, the ratio of milk and water is 4 : 1. If 7 litres of water is added to the
mixture, then find the ratio of milk and water in the new mixture.
Solution: Given milk : water = 4 : 1
Sum of terms = 4 + 1 = 5
4
Milk in 35 litres mixture = 35
5
= 28 litres.
Ratio and Proportions, Variations 187

Water in mixture = 35 28 = 7 litres.


Now if 7 litres of water is added.
Water in mixture = 7 + 7 = 14.
Milk in the mixture = 28.
Milk : Water = 28 : 14 = 2 : 1.
10. A mixture contains nuts and screws in the ratio 4 : 3. If 7 screws are added to the mixture, the ratio
becomes 3 : 4. Find the number of nuts in the mixture.
Solution: Let the number of nuts and screws be 4x and 3x.
Given : If 7 screws are added the ratio becomes 3 : 4.
i.e., 4x : 3x + 7 = 3 : 4
4x 3
i.e., =
3x + 7 4
Cross multiplying
16x = 3 (3x + 7)
16x = 9x + 21
16x 9x = 21 7x = 21 x = 3.
Number of nuts = 4x = 4 (3) = 12.
11. 5 years ago, Aruns fathers age was 5 times his sons age. After 2 years he will be 3 times Aruns
age. Find the ratio of their present ages.
Solution: Let Aruns age 5 years ago be x yrs.
Then his fathers age = 5x years.
After 2 years,
Aruns age = x + 5 + 2 = x + 7
His fathers age = 5x + 5 + 2 = 5x + 7
Given : 5x + 7 = 3 (x + 7)
5x + 7 = 3x + 21
5x 3x = 21 7
2x = 14 x = 7.
Fathers age now = 5x + 5 = 5 (7) + 5 = 40 yrs.
Aruns age now = x + 5 = 7 + 5 = 12 yrs.
Ratio of their ages = 40 : 12 = 10 : 3.
12. If 4 kgs of tea is worth 3 kgs of sugar, 5 kgs of sugar is worth 16 kgs of flour and 7 kgs of flour
is worth 2 kgs of coffee. How many kgs of tea is worth 24 kgs of coffee.
Solution: Given
Tea : Sugar = 4 : 3
Sugar : Flour = 5 : 16
188 Basic Mathematics

Flour : Coffee = 7 : 2
Let Tea : Coffee = x : 24
Tea Tea Sugar Flour
Now =
Coffee Sugar Flour Coffee

x 4 5 7
=
24 3 16 2
24 4 5 7
x= = 35
3 16 2
x = 35
Tea : Coffee = 35 : 24.
35 kgs of tea is worth 24 kgs of coffee.
13. A, B and C starts a business with a capital of Rs. 10,500 of this Rs. 4400 is contributed by A. Rs.
3700 is contributed by B and the rest by C. After 5 months C withdraws Rs. 800 capital, while
A and B each adds Rs. 400. At the end of the year, profit of the original capital is shared in the
ratio of capitals. Find to the nearest rupee the amount to be received by each
Solution:
Total capital : Rs. 10,500
As contribution : Rs. 4400
Bs contribution : Rs. 3700
Cs contribution : Rs. 10,500 (4400 + 3700)
= Rs. 2400.
As share in capital
= Rs. 4400 used for 12 months and Rs. 400 used for (12 5) months.
i.e., 4400 12 + 400 7 = Rs. 55,600.
Similarly Bs share in capital
Rs. 3700 used for 12 months and Rs. 400 used for (12 5) months
Rs. 3700 12 + 400 7 = Rs. 47,200.
Cs share in capital :
Rs. 2400 used for 5 months and (Rs. 2400 Rs. 800) used for (12 5) months
i.e., 2400 5 + 1600 7 = Rs. 23,200
Now Ratio of capital of A, B and C
= 55600 : 47200 : 23200
= 139 : 118 : 58
Total : 139 + 118 + 58 = 315
Now profit = 13% of capital
Ratio and Proportions, Variations 189

55
= 10,500 = 1443.75
4 100
139
As share in profit = 1443.75 = Rs. 637
315
118
Bs share in profit = 1443.75 = Rs. 541
315
58
Cs share in profit = 1443.75 = Rs. 266.
315
14. Three utensils contains equal mixture of milk and water in the ratio 6 : 1, 5 : 2, and 3 : 1
respectively. If all the solutions are mixed together, find the ratio of milk and water in the final
mixture.
Solution: In 1st utensil milk : water = 6 : 1
6
Quantity of milk in 1st utensil =
7
1
and Quantity of water in 1st utensil =
7
5 2
Similarly 2nd utensil contains milk and water.
7 7
3
Similarly quantity of milk in 3rd utensil =
4
1
and water = .
4
If all solutions are mixed,
6 5 3
quantity of milk = + +
7 7 4
24 + 20 + 21 65
= =
28 28
1 2 1
quantity of water = + +
7 7 4
4 + 8 + 7 19
= =
28 28
65 19
Milk : Water = :
28 28
i.e., Milk : Water = 65 : 19.
190 Basic Mathematics

7.3 PROPORTION:
If 2 ratios are equal then the 4 quantities comprising them form a proportion i.e. if the ratio a : b is equal
to c : d, then 4 quantities a, b, c, d are in proportion.
Example: 1, 2, 4, 8 are in proportion since 1 : 2 = 4 : 8.
Example
Note : 1. a : b = c : d is also denoted by a : b : : c : d. Extremes

2. In a proportion a : b = c : d, the first and


the last terms i.e., a and d are called ex- a : b = c : d
tremes and the second and 3rd terms i.e. b
and c are called means.
means
3. In every proportion, the product of the
means is equal to the product of the ex- Fig. 7.1
tremes. i.e., ad = bc
Conversely, if 4 quantities a, b, c, d are such that ad = bc, then they are said to be in proportion.
a : b = c : d ad = bc is called rule of 3. This rule is used to solve a proportion when one of the
terms is unknown.
For example: 4 : 5 = x : 15
5 x = 4 15
4 15
x= = 12.
5
1. Inver
Inv tendo: If a : b = c : d, then prove that b : a = d : c
ertendo:
oof: Given a : b = c : d
Proof:
Pr
bc = ad

b d
=
a c
b: a = d :c
Hence proved.
2. Alternendo: If a : b = c : d, then prove that a : c = b : d
Alternendo:
oof: Given a : b = c : d
Proof:
Pr
bc = ad

b a
=
d c
a:c = b: d
Hence proved.
3. Componendo: If a : b = c : d, then prove that a + b : b = c + d : d
oof: Given a : b = c : d
Proof:
Pr
bc = ad
Ratio and Proportions, Variations 191

Adding bd to both sides


bc + bd = ad + bd
a
b c+d =d a+bf a f
c+d a+b
=
d b
a+b c+d
i.e, =
b d
a + b:b = c + d :d
Hence proved.
4. Dividendo: If a : b = c : d, then prove that a b : b = c d : d.
Dividendo:
Pr oof: Given a : b = c : d
Proof:
ad = bc
Subtracting bd on both sides.
ad bd = bc bd
a f a
d ab =b cd f
ab cd
=
b d
a b:b = c d : d
Hence proved.
5. Componendo and Di videndo: If a : b = c : d, then prove that a + b : a b = c + d : c d
Dividendo:
oof: Given a : b = c : d
Proof:
Pr
a+b:b=c+d:d (Componendo)
a+b c+d
= ...(1)
b d
Also a b:b = c d :d (Dividendo)

ab cd
i.e., = ...(2)
b d
Dividing (1) by (2) we get

a+b c+d
b = d
ab cd
b d
a+b c+d
=
ab cd
192 Basic Mathematics

a + b:a b = c + d :c d
Hence proved.

CONTINUED PROPORTION:
a b c
Quantities a, b, c, d, ... are said to be in continued proportion if = = ... .
b c d
i.e., a : b = b : c = c : d = ...
Three quantities a, b, c are in continued proportion if a : b = b : c. Here b is called mean propor-
tional and c is called 3rd proportional.
For instance 7, 14, 28, 56, ... are in continued proportion since 7 : 14 = 14 : 28 = 28 : 56 ...

7.4 DIRECT PROPORTION OR DIRECT VARIATION:


Quantities are said to be in direct proportion when an increase (or decrease) in one kind is accompanied
by an increase (or decrease) in the other.
Example: Number of chocolates 1 2 3 4 ...
Cost of chocolates 3 6 9 12 ...
Here we notice as the number of chocolates increases cost of chocolates also increases. So number
of chocolates is directly proportional to cost of chocolates.

INVERSE PROPORTION OR INVERSE VARIATION:


Quantities are said to be in inverse proportion when an increase (or decrease) in one kind is accompa-
nied by decrease (or increase) in the other.
Example: Number of workers 1 2 3 4...
Number of days 12 6 4 3...
Here we notice as the number of workers increase, number of days required to finish the work
decreases and vice versa. So it is inverse proportion.
Note: If a : b = c : d represent a direct proportion, then a : b = d : c or (b : a = c : d) represent an
inverse proportion.

WORKED EXAMPLES:
1. Find the missing term in the proportion:
(a) x : 4 = 27 : 12
(b) 10 : 50 = ? : 250.
Solution: (a) Given: x : 4 = 27 : 12
12 x = 27 4
Ratio and Proportions, Variations 193

27 4
x=
12
x = 9.

(b) Given: 10 : 50 = x : 250


50 x = 10 250
10 250
x=
50
x = 50.

2. Find the 3rd proportional to (a) 5 : 10 (b) 3 : 27


(a) Let the 3rd proportional be x.
Then 5 : 10 = 10 : x

5 x = 10 10
10 10
x= = 20.
5
(b) Let the 3rd proportional be y.
Then 3 : 27 = 27 : y
3 y = 27 27

27 27
y= = 243.
3
3. Find the fourth proportional to
(a) 1 : 2 : 3
(b) 9 : 8 : 18
Solution: (a) Let the 4th proportional be x.
Then 1: 2 = 3: x

x = 2 3 = 6.
(b) Let the 4th proportional be y.
Then 9 : 8 = 18 : y
9 y = 8 18

8 18
y= = 16.
8
4. Find the mean proportional to (a) 2 : 8 (b) 5 : 45.
194 Basic Mathematics

Solution: (a) Let the mean proportional be x.

Then 2: x = x :8

x2 = 8 2

x 2 = 16
x=4.
(b) Let the mean proportional be y.
Then 5 : y = y : 45

y 2 = 5 45

y 2 = 225

y = 15.
5. If 15x = 12y. Then what is x : y?
Given: 15x = 12y
x 12
=
y 15

x 4
=
y 5

x : y = 4 : 5.
6. If 4x 7y : 3x + y = 2 : 3. Then find x : y.

Given: 4 x 7y : 3x + y = 2 : 3

a f a
3 4 x 7 y = 2 3x + y f
12 x 21y = 6 x + 2 y

12 x 6 x = 2 y + 21y
6 x = 23 y

x 23
=
y 6

x : y = 23 : 6.
7. If x : y = 7 : 5. Then find 4x 2y : x + 3y
Given : x : y = 7 : 5
Ratio and Proportions, Variations 195

x 7
=
y 5
Consider 4x 2y : x + 3y

LM FG x IJ 2OP
N H yK Q
y 4
4x 2y
=
x + 3y
=
Lx O
y M + 3P
Ny Q
F 7I 2
=
4
H 5K
7
+3
5

28 28 10
2
= 5 = 5
7 7 + 15
+3
5 5

18 9
= = .
22 11
4 x 2b : a + 3b = 9 : 11.
8. What number must be subtracted from each of 9, 11, 15 and 19, so that the difference will be
proportional.
Solution: Let the number subtracted be x,
So that 9 x, 11 x, 15 x and 19 x will be in proportion.

9 x : 11 x = 15 x : 19 x

a9 x f a19 x f = a11 xf a15 xf


171 19 x 9 x + x 2 = 165 15 x 11x + x 2

171 28 x + x 2 165 + 26 x x 2 = 0
6 2x = 0
2 x = 6 x = 3.
9. What must be added to each of 9, 42, 3 and 18 so that the sums will be in proportion.
Solution: Let the number added be x.
9 + x, 42 + x, 3 + x and 18 + x are in proportion.
196 Basic Mathematics

9 + x : 42 + x = 3 + x :18 + x

a9 + x f a18 + x f = a42 + x fa3 + x f


162 + 18 x + 9 x + x 2 = 126 + 3 x + 42 x + x 2

162 + 27 x + x 2 126 45x x 2 = 0


36 18 x = 0
x = 2.
10. If a : b = 2 : 1, b : c = 3 : 2, then find a : b : c.

Given: a:b = 2 : 1
b:c = 3 : 2
Multiplying 1st ratio by 3 and 2nd ratio by 1 to make the value of b same.
a:b= 6 :3

b:c= 3:2

a : b : c = 6 : 3 : 2.
11. If x : y = 1 : 2, y : z = 3 : 4 and z : w = 5 : 1, then find x : y : z : w.
x: y = 1:2
y : z = 3: 4
To make the value of y same, multiplying 1st ratio by 3 and 2nd ratio by 2.
x:y = 3 :6
y:z = 6:8

x : y: z=3:6:8
Given: z:w= 5 : 1.

To make the value of z same, multiplying first ratio by 5 and 2nd ratio by 8.
x : y : z = 15 : 30 : 40
z:w= 40 : 8

x : y : z : w = 15 : 30 : 40 : 8.
12. Divide Rs. 2360 among A, B, C so that A : B = 3 : 4, B : C = 5 : 6.
Given: A: B = 3 :4
B:C = 5:6

A : B : C = 15 : 20 : 24.
Ratio and Proportions, Variations 197

Sum = 15 + 20 + 24 =59.
Out of 59. As share = 15
2360 15
Out of 2360 As share =
59
= Rs. 600.
Out of 59, Bs share = 20
20
Out of 2360 Bs share = 2360
59
= Rs. 800.
Similarly
24
Cs share = 2360 = Rs. 960.
59

Verification:
As share + Bs share + Cs share
= Rs. 600 + Rs. 800 + Rs. 960 = Rs. 2360 = Total.
13. Divide 166 into 3 parts such that 4 times the first part, 5 times the 2nd part and 7 times the 3rd
part are equal.
Solution: Let 1st, 2nd and 3rd part be a, b and c.
Given: 4a = 5b = 7c = x (say)
Then 4a = x, 5b = x
x x x
a= , b = and c = .
4 5 7
x x x
a:b:c= : : .
4 5 7
i.e., a : b : c = 35 : 28 : 20.
Sum = 35 + 28 + 20 = 83.
35
1st part = 166 = 70
83
28
2nd part = 166 = 56
83
20
3rd part = 166 = 40.
83
Verif
erif ica
ification: 70 + 56 + 40 = 166.
ication:
14. If the cost of 10 metres of cloth is Rs. 225, find the cost of 22 metres of cloth.
198 Basic Mathematics

Solution:

Length mts.a f a f
Cost Rs.
10 225
22 x
As the length of the cloth increases. Cost also increases. Length and Cost are directly propor-
tional. To denote direct proportion, we use 2 arrows, with same direction.

10 : 22 = 225 : x

10 x = 22 225
22 225
x=
10
x = 495.
22 mts of cloth costs Rs. 495.
15. If 60 men can complete a job in 12 days, how many days will 36 men take to complete the same
job?

Solution: Men days


60 12
36 x
As the number of men increases, the days required to complete the job decreases Men and days
are inversely proportional. To denote this we use 2 arrows with opposite direction.

60 : 36 = x : 12

36 x = 60 12
60 12
x= = 20.
36
36 men can complete a job in 20 days.
16. If 10 men can earn Rs. 105 in 7 days, in how many days will 15 men earn Rs. 225?

Solution: Men Money days


10 105 7

15 225 x

Here number of days is unknown.


Leaving money, or keeping money constant, let us first consider men and days. As the number of
men increases, days required to complete the job decreases. Men and days are inversely proportional.
Now, leaving men, let us consider money and days. As the days increase, money earned also in-
creases. Money and days are directly proportional.
Ratio and Proportions, Variations 199

Corresponding compound proportion is


10 : 15
=x:7
225 : 105

15 105 x = 7 10 225
7 10 225
x=
15 105
x = 10
15 men earn Rs. 225 in 10 days.
17. 5 carpenters can earn Rs. 3600 in 6 days, working 9 hrs. a day. How much will 8 carpenters earn
in 12 days working 6 hrs. a day.
Solution:

Carpenter days hrs. Money


5 6 9 3600

8 12 6 x

Here money is unknown.


Taking carpenter and money alone, as the carpenters increase, the money earned by them also
increases. So it is direct proportion. Taking days and money, as the day increases money also increases.
So it is direct proportion. Now taking hours and money, as the number of hours increases, money also
increases. It is direct proportion.
So corresponding compound proportion is
5:8
6 : 12 = 3600 : x
9:6
5 6 9 x = 8 12 6 3600
8 12 6 3600
x=
569
x = 7680.
8 carpenters earn Rs. 7680 in 12 days working 6 hours a day.
18. A contractor undertook to make 15 kms of roadways in 40 weeks. In 10 weeks 3 kms were
completed by 180 men working 8 hours a day. Then the men agreed to work 1 hour a day overtime
and some boys were engaged to assist them. The work was finished in the stipulated time (that is
40 weeks). How many boys were employed if the work of 3 boys is equal to that of 2 men.
Solution:
Men weeks hrs. kms.
180 10 8 3
x a40 10f = 30 a8 + 1f = 9 a15 3f = 12
200 Basic Mathematics

men and Kms are directly proportional.


men and hrs. are inversely proportional. men and week are inversely proportional.
Required compound proportion is
180 : x = 30 : 10
9: 8
3 : 12

x 30 9 3 = 180 10 8 12
180 10 8 12 640
x= = .
30 9 2 3
Already 180 men are there,
640 100
180 =
3 3
Given 3 boys 2 men.
3
1 man boys
2
100 100 3
men boys
3 3 2
= 50 boys.
50 boys were engaged to assist them.
19. If 15 men build a wall 40 ft long, 2 and 1/2 ft. thick and 21 ft. height in 18 days working 10 and
1/2 hrs. each day. In how many days working 15 hrs. a day will 45 men build a wall 200 ft. long,
5 ft thick and 20 ft. height.
Solution: Men Length Breadth Height hours days
(thickness)
15 40 1 21 1 18
2 10
2 2
45 200 5 20 15 x

As the men increase, days decrease, so inverse proportion.


As length increase, days also increase, as breadth increase, days also increase.
As height increase, days also increase.
So, It is direct proportion.
As the number of hrs. increase, days required to construct decrease. So it is inverse proportion.
Corresponding Compound proportion is:
Ratio and Proportions, Variations 201

15 : 45 U|
200 : 40 |
1|
5 : 2 |V = x : 18
2
20 : 21 |
|
10 : 15 |
1 |
2 W
1 1
x 45 40 2 21 15 = 18 15 200 5 20 10 .
2 2

1
18 15 200 5 20 10
x= 2
1
45 40 2 21 15
2
x = 40
40 days are required.
20. If 12 pumps working 6 hours a day can draw 2000 gallons of water in 20 days, find in how many
days will 20 pumps working 9 hours a day draw 3000 gallons of water?

Solution: Pumps Hours Quantity Days


12 6 2000 20
20 9 3000 x
As the number of pumps increase, the days required decreases. So it is inverse proportion. As the
number of hours increases the days required to draw water decreases. So it is inverse proportion. As the
number of gallons increase, number of days required also increases, so it is direct proportion.
Corresponding compound proportion is

12 : 20 U|
6: 9 V| = x : 20
3000 : 2000 W
x 20 9 2000 = 20 12 6 3000
20 12 6 3000
x=
20 9 2000
x = 12
12 days are required.
202 Basic Mathematics

7.5 PROBLEMS ON TIME AND WORK:


1
Note: 1. If A can do a piece of work in n days. Then work done by A in 1 day = .
n
1
2. If Bs 1 days work = . Then B can finish the work in x days.
x
3. If A is twice as good a workman as B,
then
Ratio of work done by A and B = 2 : 1
Ratio of time taken by A and B = 1 : 2
4. If A can do a piece of work in x days, and B can do it in y days, then A and B working together
xy
will do the same work in x + y days.

5. If A and B together can do a piece of work in z days and A alone can do it in x days then B
zx
alone can do it in .
xz

SOLVED EXAMPLES:
1. Ram can reap a field in 6 days which Raju alone can reap in 8 days. In how many days both
together can reap this field?
1
Solution: Rams 1 days work =
6
1
Rajus 1 days work =
8
1 1
Ram and Rajus 1 days work = +
6 8
4+3 7
= = .
24 24
24 3
Both together can reap the field in = 3 days.
7 7

OR

We know, if A can do a piece of work in x days and B can do it in y days then A and B together
xy
can do it in days.
x+y
Ratio and Proportions, Variations 203

68
Ram and Raju together can reap the field in days.
6+8
48 24 3
= = = 3 days.
14 7 7
2. X and Y together can dig a trench in 10 days which X alone can dig in 30 days. In how many days
Y alone can dig it?
1
Solution: X and Ys 1 days work =
10
1
Xs 1 days work =
30
1 1
Ys 1 days work =
10 30
3 1 2
=
30 30
30
Y alone can dig a trench in = 15 days.
2

OR

We know, if A and B together can do a piece of work in z days and A alone can do it in x days then
zx
B alone can do it in days.
xz
30 10
Y alone can dig a trench in
30 10
300
= = 15 days.
20
3. A and B can do a piece of work in 12 days; B and C in 15 days; C and A in 20 days. In how many
days will they finish it together and separately?
1
(A + B)s 1 days work =
12
1
(B + C)s 1 days work =
15
1
(C + A)s 1 days work =
20
204 Basic Mathematics

Adding
1 1 1
2 [A + B + C]s 1 days work = + +
12 15 20
5+4+3
2 (A + B + C)s 1 days work =
60
1
2 (A + B + C)s 1 days work =
5
1
(A + B + C)s 1 days work =
10
So A, B and C together finish the work in 10 days.
Now, As 1 days work = (A + B + C)s 1 days work (B + C)s 1 days work.
1 1 1
= =
10 15 30
A alone can finish the work in 30 days.
Similarly,
1 1 1
Bs 1 days work = =
10 20 20
B alone can finish the work in 20 days.
Similarly,
1 1 1
Cs one days work = =
10 12 60
C alone can finish the work in 60 days.

OR

A, B and C can do together in


2xyz
days
xy + yz + zx

=
a
2 12 15 20 f
12 15 + 15 20 + 20 12

a
2 3600 f2 3600 a f
=
180 + 300 + 240
=
720 a f
= 10 days.

A, B and C can together finish the work in 10 days.


4. A can do a piece of work in 25 days which B alone can finish in 20 days. Both work for 5 days
and then A leaves off. How many days will B take to finish the remaining work.
Ratio and Proportions, Variations 205

1
Solution: As work in 1 day =
25
1
Bs work in 1 day =
20

1 1
(A + B)s 1 days work = +
25 20

LM 1 + 1 OP
(A + B)s 5 day' s work = 5
N 25 20 Q
9
=
20

Remaining work = 1
9
20
a
Note this step f
11
=
20
1
Now work is done by B in 1 day.
20

11
11 1
work will be done by B in 20 = 11 days.
20 1
20
B takes 11 days to finish the remaining work.
5. X is thrice as good a work man as Y and is therefore able to finish the piece of work in 60 days
less than Y. Find the time in which they can do it, working together.
Solution: Ratio of work done by x and y in same time = 3 : 1
Ratio of time taken = 1 : 3
If Y takes y days to finish a work.
Then X takes y 60 days to finish.
Now
y 60 : y = 1 : 3

a f
3 y 60 = y

3 y y = 180

2 y = 180 y = 90
206 Basic Mathematics

Time taken by Y to finish the work = 90 days and time taken by X to finish the work = 90 60 =
30 days.
1
Xs 1 days work =
30
1
Ys 1 days work =
90
1 1 2
(X + Y)s 1 days work = + =
90 30 45
45 1
Both X and Y can finish the work in = 22 days.
2 2
6.
A can build a wall in 30 days which B alone can build in 40 days. If they build it together and get
a payment of Rs. 1400 what is As share and Bs share?
1
Solution: As 1 days work =
30
1
Bs 1 days work =
40
1 1
Ratio of their work = :
30 40
=4:3
Total = 4 + 3 =7
4
As share = 1400 = Rs. 800
7
3
Bs share = 1400 = Rs. 600
7
7. A can do a piece of work in 10 days, while B alone can do it in 15 days. They work together for
5 days and the rest of the work is done by C in 2 days. If they get Rs. 1200 for the whole work
how should they divide the money?
Solution:
1
As 1 days work =
10
1
Bs 1 days work =
15
1 1 1
(A + B)s 1 days work = + =
10 15 6
Ratio and Proportions, Variations 207

F 1I = 5
(A + B)s 5 days work = 5 H 6K 6
5 1
Remaining work = 1 = (Note this step)
6 6
Given: C completes rest of the work in 2 days.
1
Cs 2 days work =
6
Now Consider
As 5 days work : Bs 5 days work : Cs 2 days work.

F 1 I : 5F 1 I : 1
5
H 10 K H 15K 6
1 1 1
: :
2 3 6
=3:2:1
Total = 3 + 2 + 1 = 6.
3
As share = 1200 = Rs. 600
6
2
Bs share = 1200 = Rs. 400
6
1
Cs share = 1200 = Rs. 200.
6
8. A certain number of men complete a piece of work in 60 days. If there were 8 men more the work
could be finished in 10 days less. How many men were originally there?
Solution: Let number of men = x
8 men more means x + 8

Given: Men days


x 60
x +8 60 10 = 50
As the number of men increases, the days required to complete the job decreases.
It is inverse proportion.
x : x + 8 = 50 : 60

a f a
x 60 = x + 8 50 f
60 x 50 x = 400
208 Basic Mathematics

10 x = 400 x = 40.
40 men were originally there.
9. 16 men or 28 boys can fence a farm in 40 days. In how many days will 24 men and 14 boys
complete the same work?
Solution: Given,
16 mens work 28 boys work.
i.e., 16 : 28
i.e., 8 : 14
8 men 14 boys.
Now 24 men and 14 boys 24 men + 8 men
32 men.
Given:

Men days
16 40
32 x
As the number of men increases, the days required to fence a farm decreases. It is inverse
proportion.
16 : 32 = x : 40
32 x = 16 40
16 40
x= = 20.
32
20 days are required to complete the work.
10. 2 men and 4 boys can do a work in 33 days. 3 men and 5 boys can do the same work in 24 days.
How long shall 5 men and 2 boys take to finish it?
Given: 2 men and 4 boys can do the work in 33 days.
2 33 men and 4 33 boys can do it in 1 day.
66 men and 132 boys can do it in 1 day ...(1)
Similarly
3 men and 5 boys can do the work in 24 days.
3 24 men and 5 24 boys can do it in 1 day
i.e., 72 men and 120 boys can do it in 1 day ...(2)
From (1) and (2)
66M + 132B 72M + 120B
132B 120B 72M 66M
Ratio and Proportions, Variations 209

12B 6M
M 2B.
One mans work is equivalent to 2 boys work.
Now given: 2 men and 4 boys can finish the work in 33 days.
i.e., 2M + 2M can finish the work in 33 days.
4 men can finish the work in 33 days.
Now,
5 men and 2 boys 5M + 1M = 6 Men.

Men days
4 33
6 x
As men increases, days required to finish the work decreases so it is inverse proportion.

4 : 6 = x : 33

6 x = 33 4

33 4
x= == 22.
6
5 men and 2 boys can finish the work in 22 days.

7.6 PROBLEM ON TIME AND DISTANCE:


Note :
Distance
(1) Speed =
Time

F 5I
H
(2) x km hr = x
18Kmts sec

F 5 I
(3) x mts sec = x H 18 K
km hr.

WORKED EXAMPLES:
1. The distance between 2 stations A and B is 450 kms. A train starts at 4 p.m. from A and moves
towards B at an average speed of 60 km/hr. Another train starts from B at 3 : 20 p.m. and moves
towards A at an average speed of 80 km/hr. How far from A will the two trains meet and at what
time?
210 Basic Mathematics

Solution: Let the trains meet at a distance x kms. from A.


Let trains from A to B and B to A be X and Y respectively.
Given: Speed of X = 60 km/hr
Speed of Y = 80 km/hr.
x
Time required to cover x kms. by X =
60
450 x
Time required to cover (450 x) kms. by Y =
80
Difference between time
450 x x
= = 3:20 p.m. to 4 p.m.
80 60
450 x x
= 40 min.
80 60
450 x x 40
= hrs.
80 60 60
450 x x 40
=
80 60 60

a f
3 450 x 4 x 2
=
240 3
cross multiplying,
a f
9 450 x 12 x = 480
4050 9 x 12 x 480 = 0
3570 21x = 0
3570
x= = 170.
21
The trains meet at a distance of 170 kms from A.
170
Time taken by X to cover 170 kms. = hrs.
60
= 2hrs. 50 min.
So the trains meet at 4 pm + 2 hrs. 50 min.
= 6 : 50 pm.
3
2. Cycling of his usual speed, a student is 10 min. late to his class. Find his usual time to cover
4
the distance.
Ratio and Proportions, Variations 211

Let the usual time taken be x min.

3 4 F I LM3 Distance = Speed, Time = Distance OP


Time taken at
4
of the usual speed =
3 H K
x min.
N Time Speed Q

4
Given: x x = 10
3
4 x 3 x = 30
x = 30.
Usual time taken = 30 min.

F 3I th
3. A bullock cart has to cover a distance of 80 km in 10 hrs. If it covers half of the journey in H 5K
time. What should be its speed to cover the remaining distance in the time left?
Solution: Total distance = 80 kms.
Total time = 10 hrs.
80
Distance left = = 40 kms.
2
1 3
Given: Time taken to cover distance = 10 hrs. = 6 hrs.
2 5
Remaining time = 10 6 = 4 hrs.
Distance 40
Speed = = km hr
Time 4
Speed = 10 km/hr.
4. A man travels 360 km in 4 hrs. partly by air and partly by train. If he had travelled all the way by
4
air, he would have saved of the time he was in train and would have arrived at his destination
5
2 hrs. early. Find the distance he travelled by air and train.
Solution: Total time = 4 hrs.
4
Given: of total time in train = 2 hrs.
5
25 5
Total time in train = = hrs.
4 2
Given: If 360 km is covered by air then time taken is 4 2 = 2 hrs.
3
When is spent in air,
2
212 Basic Mathematics

360 3
Distance covered = = 270 kms.
2 2
Distance covered in train = 360 270 = 90 km.
5. An aeroplane started 30 minutes later than the scheduled time from a place 1500 km away from
its destination. To reach the destination at the scheduled time, the pilot had to increase the speed
by 250 km/hr. What was the speed of the aeroplane per hour during the journey?
Solution: Let the time taken by aeroplane in later case = x hrs.
Distance
We know Speed =
Time
Distance = 1500 kms.
1500 1500
Given: = + 250
x 1
x+
2

1500 2 1500
=
a f
+ 250
x 2x + 1

1500 3000 + 250 2 x + 1


=
a f
x 2x + 1
Cross multiplying
a f a
1500 2 x + 1 = x 3000 + 500 x + 250 f
3000 x + 1500 3000 x 500 x 2 250 x = 0

500 x 2 + 250 x 1500 = 0


by 250

2x2 + x 6 = 0 +4 x
12 x 2
2 x 2 + 4 x 3x 6 = 0 3x

a f a f
2x x + 2 3 x + 2 = 0 +x

2 x a x + 2 f 3 a x + 2f = 0

a2 x 3fa x + 2f = 0
2x = 3
3
x= or x = 2
2
3
3 x cannot be negative, x =
2
Ratio and Proportions, Variations 213

3 1
The plane takes hrs. = 1 hrs. in later case
2 2
1 1
So in Normal case it takes 1 + = 2 hrs.
2 2
1500
Normal speed = = 750 km hr.
2

7.7 PROBLEMS ON MIXTURE:


Note: 1. The word Alligation literally means linking. The rule takes its name from the lines or links
used in working out questions on mixture.
2. Alligation method is applied for percentage value, ratio, rate, prices, speed etc. and not for
absolute values.
3. Alligation is the rule that enables us to find the proportion in which two or more ingredients at
the given price must be mixed to produce a mixture at a given price.
Cost price of unit quantity of the mixture is called the mean price.
4. Rule of alligation: If 2 quantities are mixed in a ratio, then
Quantity of cheaper a f a
C. P. of dearer Mean Price f
Quantity of dearer
=
a f a
Mean Price C.P. of Cheaper f
i.e., cheaper quantity: dearer quantity = d m: m c.
We represent it as
Cost price of C.P. of dearer
cheaper (c) (d)

Mean Price (m)

(d m) mc

WORKED EXAMPLES:
1. The price of first quality of rice is Rs. 16 per kg and that of second quantity rice is Rs. 10. In what
ratio these two should be mixed so that the mixture can be sold for Rs. 12 per kg.
Solution:
Cost price of 1 kg C.P. of 1 kg
cheaper Rice dearer rice
10 (c) 16 (d)
Mean Price
12 (m)
(d m) = 16 12 mc
=4 = 12 10
=2
214 Basic Mathematics

Quantity of cheaper rice : Quantity of dearer rice.


=4:2
=2:1

OR

Let us mix these 2 types of rice in the ratio x : y.


The price of x kgs. cheaper rice = 10 x
Price of y kgs. dearer rice = 16 y
The price of (x + y) kgs. of mixture = 10x + 16y
10 x + 16 y
Price of 1 kg of mixture = x+y
Given: Price of 1 kg of mixture = 12
10 x + 16 y
= 12
x+y
10 x + 16 y = 12 x + 12 y

16 y 12 y = 12 x 10 x
4y = 2 x
4 x
=
2 y
x: y=4:2
x : y = 2 :1.
2. Arjun travelled a distance of 80 km in 7 hrs. partly in bullock cart at the rate of 8 km/hr and partly
in tonga at 16 km/hr. Find the distance travelled in bullock cart.
Solution: Average distance travelled in
80 km 80
1 hr = = km hr.
7 hrs. 7
Distance travelled Distance travelled
in 1 hr in bullock cart in 1 hr in tonga
8 km (c) 16 km (d)
Average in 1 hr
80
7
ma f
(d m) (m c)
80 32 80 24
16 = 8=
7 7 7 7
Ratio and Proportions, Variations 215

Time taken by bullock cart d m


=
Time taken by tonga mc

32
= 7
24
7

32
= 4 : 3.
24
Out of 7 hrs., he took 4 hrs. to travel in bullock cart and 3 hrs. in tonga.
Distance covered by bullock cart = 4 8 km = 32 km.
3. In what ratio must a person mix three kinds of rice costing Rs. 12.00, Rs. 14.00 and Rs. 17.40 per
kg. So that mixture may be worth Rs. 14.10 per kg.
Solution: Step 1: Mix 1st and 3rd kind of rice to get a mixture worth Rs. 14.10.
C.P. of 1 kg rice C.P. of 1 kg of rice
of 1st kind of 3rd kind
Rs. 12.00 (c) Rs. 17.40 (d)

Mean Price
Rs. 14.10

(dm) = Rs. 3.30 (mc) Rs. 2.10


By alligation rule
Quantity of 1st kind of rice d m
=
Quantity of 3rd kind of rice m c

3.30 11
= = .
2.10 7
They must be mixed in the ratio 11 : 7.
Step II: Mix rice of 1st and 2nd kind to obtain a mixture worth Rs. 14.10 per kg.
C.P. of 1 kg rice C.P. of 1 kg of rice
of 1st kind of 2nd kind
Rs. 12.00 (c) Rs. 14.40 (d)

Mean Price (m)


Rs. 14.10

(dm) = Rs. 0.30 (mc) = Rs. 2.10


216 Basic Mathematics

By alligation rule,
Quantity of 1st kind of rice d m 0.30
= =
Quantity of 2nd kind of rice m c 2.10
1
= .
7
They must be mixed in the ratio 1 : 7.
Now
1st kind : 2nd kind = 1 : 7
1st kind : 3rd kind = 11 : 7
To make the value of 1st kind equal,
Multiply 1st ratio by 11 and 2nd ratio by 1.
1st kind : 2nd kind = 11 : 77
1st kind : 3rd kind = 11 : 7
1st kind : 2nd kind : 3rd kind = 11 : 77 : 7.
4. Gauri possessing Rs. 84,000 lent a part of it at 8% simple interest and the remaining at 6 and
2/3% simple interest. Her total income after 1 and 1/2 years was Rs. 8820. Find the sum lent at
different rates.
Solution: Given:
P = 84000
I = 8820
1 3
T= 1 yrs.= yrs.
2 2
R=?
We know,
PRT 100 I
I= R=
100 PT

100 8820
R= = 7%
3
84000
2
Rate of Interest Rate of Interest
cheaper (c) dearer (d)
6 and 2/3% 8%

(m) Average rate


7%
2 1
(d m) = 8 7 = 1 m c 76 =
3 3
Ratio and Proportions, Variations 217

By alligation rule,

2
Money given at 6 % SI
3 1 3
= = .
Money given at 8% SI 1 1
3
=3:1
Sum = 3 + 1 = 4.
3
Money lent at 6 and 2/3% SI = 84000
4
= Rs. 63000.
1
Money lent at 8% SI = 84000
4
= Rs. 21000.
5. Adarsh buys 2 horses for Rs. 1350 and sells one at 6% loss and other at 7.5% gain and on the
whole, he neither gains, no loses. What does each horse cost?
Solution:
Cheaper horse Dearer horse
6% (c) 7.5% (d)

Mean
O
(d m) (m c)
7.5 6
Cost of 1st horse 7.5 75 5
= = =
Cost of 2nd horse 6 60 4
1st horse : 2nd horse = 5 : 4
Sum = 5 + 4 = 9
5
Cost of 1st horse = 1350 = Rs. 750
9
4
Cost of 2nd horse = 1350 = Rs. 600.
9

REMEMBER:

For the given ratio a : b, duplicate ratio is a2 : b2, subduplicate ratio is a : b , Triplicate ratio
3
is a3 : b3 and subtriplicate ratio is a :3 b .
218 Basic Mathematics

a : b : : c : d iff ad = bc.
If a : b = b : c. Then b is called mean proportional and c is called 3rd proportional.
If a : b = c : d. Then d is called fourth proportional.
If a : b = c : d. Then
(i) b : a = d : c (Invertendo)
(ii) a : c = b : d (Alternendo)
(iii) a + b : b = c + d : d (Componendo)
(iv) a b : b = c d : d (Dividendo)
(v) a + b : a b = c + d : c d
(Componendo and dividendo)
If a : b = c : d represent a direct proportion then a : b = d : c or b : a = c : d represent an inverse
proportion and vice-versa.
Quantity of cheaper C.P. of Dearer Mean
Rule of alligation: Quantity of dearer = Mean C. P. of cheaper

acf adf
amf i.e., cheaper : dearer = d m : m c.

dm mc

EXERCISE

1. Express the following ratios in their simplest form :


(a) 8 : 24 (b) 24 : 88 (c) 9 : 900
2. Compare the following ratios :
(a) 6 : 5 and 3 : 2 (b) 1 : 5 and 6 : 7 (c) 3 : 2 and 2 : 7
3. Find the ratio between :
(a) 1 hr 15 min and 105 min
(b) 3 kg 50 gm and 4 kg 500 gm
4. A number is divided into 3 parts in the ratio 2 : 3 : 4. If the 3rd part is 32. Find the other parts.
5. Find the third proportional to 2 : 6.
6. Find the mean proportional between 49 and 64.
7. Find the fourth proportional to 4, 5 and 12.
8. What number must be subtracted from each of 9, 11, 15 and 19. So that the difference will be
proportional.
9. (a) If x : y = 2 : 3, y : z = 4 : 5. Then find z : x.
(b) If x : y = 2 : 3, y : z = 4 : 5 and z : w = 6 : 7. Then find x : w.
(c) If a : b = 2 : 3, b : c = 4 : 5; c : d = 6 : 7. Then find a : b : c : d.
Ratio and Proportions, Variations 219

10. Two numbers are in the ratio 5 : 8. If 9 is added to each then they are in the ratio 8 : 11. Find the
numbers.
11. The ratio between the ages of Khan and Ranjith is 6 : 5 and the sum of their ages is 44 years. Find
the ratio of their ages after 8 years.
12. One year ago the ratio of between Sarala and Saraswathis salary was 3 : 4. The ratio of their
individual salaries between last years and this years salaries are 4 : 5 and 2 : 3 respectively. At
present the total of their salary is Rs. 4160. Find the salary of Sarala now.
13. The ratio between Sumits and Prakashs age at present is 2 : 3. Sumit is 6 years younger than
Prakash. Find the ratio of Sumits age to Prakashs age after 6 years.
14. 8 labourers can build a wall in 6 days. In how many days, 12 labourers can do the same work?
15. 6 carpenters working 7 hrs. a day can complete 24 tables in 20 days. How many days will 12
carpenters working 6 hrs. a day take to complete 36 tables.
16. A, B, C start a business with investments of Rs. 25,000, Rs. 16,000 and Rs. 12,000 respectively.
If the profit for the year amounts to Rs. 6850. Find the share of each partner.
17. Ram can reap a field in 9 days which Deepak alone can reap in 12 days. In how many days both
together can reap this field.

18. A can do F I
1
rd
F 2I
of the work in 5 days and B can do H K
th

H 3K 5
of the work in 10 days. In how many

days both A and B together can do the work?


19. Sunil completes a work in 4 days, whereas Dinesh completes the work in 6 days. Ramesh works
1 and 1/2 times as fast as Sunil. How many days it will take for the 3 together to complete the
work.
20. A can complete a job in 9 days, B in 10 days and C in 15 days. B and C start the work and are
forced to leave after 2 days. Find the time taken to complete the remaining work.
21. If 3 men or 4 women can construct a wall in 43 days. Then find the number of days that 7 men
and 5 women take to construct it.
22. 8 men can dig a pit in 20 days. If a man works half as much again as a boy then in how many days
4 men and 9 boys can dig a similar pit.
23. A train leaves Meerut at 6 a.m. and reaches Delhi at 10 a.m. Another train leaves Delhi at 8 am.
and reaches Meerut at 11 : 30 a.m. At what time do the 2 trains cross each another.
24. A boy goes to school with a speed of 3 km/hr and returns to the village with a speed of 2 km/hr.
If he takes 5 hrs. in all, find the distance between village and the school.
25. In what proportion must Ragi at Rs. 3.10 per kg be mixed with Ragi at Rs. 3.60 per kg so that the
mixture be worth Rs. 3.25 a kg.
26. A man possessing Rs. 1000 lent a part of it at 6% S.I. and the other at 8% SI the yearly income
is Rs. 75. Find the sum lent at 8% SI.
27. Kantilal mixes 80 kgs of sugar worth Rs. 6.75 per kg with 120 kg worth Rs. 8 per kg. At what rate
he sell the mixture to gain 20%.
28. A jar contains a mixture of 2 liquids A and B in the ratio 7 : 5. When 9 litres of mixture is drawn
off and the jar is filled with B, the ratio of A and B becomes 7 : 9. How many litres of liquid A
was contained by the jar initially?
220 Basic Mathematics

29. A sum of Rs. 41 was divided among 50 boys and girls each boy gets 90 paise and a girl 65 paise.
How many boys are there?

ANSWERS

1. (a) 1 : 3 (b) 3 : 11 (c) 1 : 100


2. (a) 3 : 2 is greater than 6 : 5 (b) 6 : 7 is greater than 1 : 5
(c) 3 : 2 is greater than 2 : 7.
3. (a) 5 : 7 (b) 61 : 90
4. 16 and 24 5. 18 6. 56 7. 15 8. 3
9. (a) 15 : 8 (b) 16 : 35 (c) 16 : 24 : 30 : 35
10. 15 and 24 11. 8 : 11 12. Rs. 1600 13. 3 : 4
14. 4 15. 105 16. Rs. 2650, Rs. 2200, Rs. 2000.
1 3 5
17. 5 days 18. 9 days 19. 1 days 20. 6 days 21. 12 days
7 8 19
22. 16 days 23. 8.56 a.m. 24. 6 km. 25. 7 : 3 26. Rs. 750
27. Rs. 9 per kg 28. 21 litres 29. 34.
8
Averages
8.1 INTRODUCTION:
Condensation of data is necessary in statistical analysis because a large number of big figures are not
only confusing to mind but also difficult to analyse. In order to reduce the complexity of data and to
make them comparable it is essential that the various phenomena which are being compared are re-
duced to one figure each. It is obvious that a figure which is used to represent a whole series should
neither have the lowest value in the series nor the highest value but a value somewhere between these
two limits, possibly in the centre where most of the items of the series cluster. Such figures are called
measures of central tendencies or averages.
Averages are usually of the following types:
(a) Mathematical average
(i) Arithmetic average or mean
(ii) Geometric mean and
(iii) Harmonic mean.
(b) Average of position:
(iv) Median and
(v) Mode.
Of the above mentioned five important averages, we are going to discuss in this book, about arith-
metic average or Mean in detail.

8.2 ARITHMETIC AVERAGE OR MEAN:


Mean of the certain number of quantities which are all of equal weightage or importance is the figure
obtained by dividing the total values of various items by their number.
If the marks obtained by a student in five subjects are 56, 67, 82, 43 and 52, then to find the mean
of these marks we shall add these marks and divide the total so obtained by the number of items which
is 5.
56 + 67 + 82 + 43 + 52
Mean =
5
222 Basic Mathematics

300
Mean = = 60.
5
In general, if x1, x2, x3, ... xn are the values in a data containing n items then their mean, denoted by
x1 + x 2 + x 3 + ... + x n
X is X =
n
n

xi
i.e., X= i =1
n
If all the quantities are not of equal importance i.e. equal weight, we compute the weighted average
as follows:
If w1, w2, w3, ... wn are the weights associated to the values x1, x2 ... xn respectively the weighted
average Xw is given by

x1w1 + x 2 w2 + ... + x n wn
Xw =
w1 + w2 + ... + wn

x w i i
i =1
Xw = n

w i
i =1

For example if Rama scores 35 in 2 subjects, 42 in 4 subjects and 72 in remaining 4 subjects. Then
average marks scored by Rama
35 2 + 42 4 + 72 4
=
2+4+4
70 + 168 + 288
=
10
526
= = 52.6.
10
Note: To compute arithmetic mean in a continuous series, the midpoints of the various class inter-
vals are written down to replace the class intervals. Once it is done, there is no difference
between a continuous series and discrete series.

8.3 COMBINED AVERAGE:


It is nothing but average of averages. Combined average is computed when the data set consists of
different groups and average for each group is known.
Averages 223

If X1 , X2 ... Xn , are the average of groups 1, 2, ..., n and N1, N2, ... Nn are number of quantities in

groups 1, 2, 3, ... n. Then combined average X1, 2, 3 ... n is given by

X1 N1 + X2 N2 + Xn N n
X1, 2, 3 ... n =
N1 + N 2 + ... + N n
For example if the average marks of group of 5 students is 56 and average marks of another group
of 6 students is 72 then combined average marks of 11 students.
56 5 + 72 6
=
5+6
280 + 432 712
= = 64.72.
11 11

WORKED EXAMPLES:
1. Find the arithmetic mean of 56, 87, 36, 72 and 44:

Solution: Arithmetic mean X = x1 + x 2 + x3 + x 4 + x 5


5
56 + 87 + 36 + 72 + 44
=
5
295
= = 59.
5
2. A market survey on demands of chocolates at a local shop provided the following distribution of
daily demand:
Daily demand Frequency
(Number of chocolates) (Number of days)
20 5
30 27
40 62
50 4
60 2

Total: 100
Find the average demand of chocolates in numbers/day.
x1w1 + x 2 w2 + x 3 w3 + ... + x n wn
Solution: Mean =
w1 + w2 + w3 + ... + wn
224 Basic Mathematics

20 5 + 30 27 + 40 62 + 50 4 + 60 2
Mean =
5 + 27 + 62 + 4 + 2
100 + 810 + 2480 + 200 + 120
Mean =
100

X = 37.1.
3. Find the arithmetic mean of the following data:
Class Interval: 0-20 20-50 50-90 90-140 140-200
Frequency: 10 20 40 15 15
C.I. Mid value Frequency X.W
(x) (w)
0-20 10 10 100
20-50 35 20 700
50-90 70 40 2800
90-140 115 15 1725
140-200 170 15 2500
W = 100 XW = 7825
x1w1 + x 2 w2 + ... + x n wn
Xw =
w1 + w2 + ... + wn

Xw 7825
Xw = = = 78.25.
w 100
4. If 10 books are purchased at the rate of Rs. 7 each, 15 books are purchased at the rate of Rs. 9.50
and 20 books are purchased at the rate of Rs. 13.50, find the average price of a book.
Solution: Given: N1 = 10, N2 = 15 and N3 = 20
X1 = 7, X2 = 9.50 and X3 = 13.50.
X1 N1 + X2 N 2 + X3 N3
Combined average =
N1 + N 2 + N3

7 10 + 9.50 15 + 13.50 20
=
10 + 15 + 20
70 + 142.50 + 270
=
45
= 10.7222.
Average price of a book = Rs. 10.72.
5. The average marks of 15 students of class is 45. A student who has secured 17 marks leaves the
class. Find the average marks of the remaining 14 students.
Averages 225

Solution: Average marks of 15 students = 45


Total marks = 45 15 = 675
If the one student with 17 marks leaves the class then total marks of remaining 14
Students = 675 17 = 658
658
Average marks of 14 students = = 47.
14
6. The average age of 10 students is 6 years. The sum of the ages of 9 of them is 52 years. Find the
age of the 10th student.
Solution: Given: Average age of 10 students = 6 years
Total age of 10 students = 10 6 = 60 years
Given, the sum of ages of 9 of them = 52 years.
Age of the 10th student = 60 52 = 8 years.
7. The average age of 10 students is 14 years. Among them, the average age of 5 students is 12 years.
Find the average age of remaining students:
Solution: Given: Combined Average X = 14.
N1 + N2 = 10
N1 = 5, N2 = 5, X1 = 12, X2 = ?
We have

X1 N1 + X2 N 2
X=
N1 + N 2

12 5 + X2 5
14 =
5+5

60 + 5 X2
14 =
10

140 = 60 + 5 X2

5 X2 = 140 60 = 80

80
X2 = = 16.
5
Average age of remaining 5 students = 16 years.
8. A shopkeeper purchased a certain number of dress materials at an average price of Rs. 190 each.
The average price of 10 dress materials was Rs. 175 and that of remaining dress materials was
Rs. 200. Find the total number of dress materials purchased.
Solution: Combined mean X = 190 (Given)
226 Basic Mathematics

N1 = 10, X1 = 175

X2 = 200
Let total number of dress materials purchased = x
N1 + N 2 = x

10 + N 2 = x
N 2 = x 10
We have

N1 X1 + N2 X2
X=
N1 + N 2

190 =
a f a f
10 175 + x 10 200
x
190 x = 1750 + 200 x 2000
190 x = 250 + 200 x
250 = 200 x 190 x
250 = 10 x
250
x= = 25
10
Total number of dress materials purchased = 25.
9. The average weight of a group containing 26 persons is 70 kg. 6 persons with average weight 67
kg leave the group and 5 persons with weights 68, 72, 82, 56 and 54 kgs. joins the group. Find
the average weight of the group now.
Solution: Average weight of 26 persons = 70 kgs. (Given)
Total weight of 26 persons = 26 70 = 1820
6 persons with average weight 67 kgs leave the group. (Given)
i.e., Total weight of 6 persons = 67 6
= 402
Total weight of the remaining 20 persons
= 1820 402 = 1418
5 persons with weight 68, 72, 82, 56 and 54 kgs. join the group.
Total weight of (20 + 5) persons
= 1418 + 332 = 1750
1750
Average weight of the group now = = 70 kgs.
25
Averages 227

10. A batsman realises that by scoring a century in the 11th innings of his test matches he has bettered
his average of the previous 10 innings by 5 runs. What is his average after the 11th inning:
Solution. Let the average runs in 10 innings be x
Then total runs in 10 innings = 10x
Average runs after 11th innings = x + 5 (Given)
Total runs in 11th innings = (x + 5) 11
Also given a batsman scores a century in the 11th innings.
Runs in 11 innings Runs in 10 innings.
= Runs in 11th innings = 100.
a x + 5f11 10 x = 100
11x + 55 10 x = 100
x = 100 55
x = 45.
Average runs after 11th innings = x + 5 = 45 + 5 = 50.
11. Ms. Vani bought 17 books in a discount sale. The average price of books being Rs. 53. The
average price of the eleven Kannada books is Rs. 71. If the prices of the remaining 6 English
books form an increasing arithmetic progression with last term Rs. 25. Find the price of cheapest
English book.
Solution.
Given: X = 53

X1 = 71, N1 = 11

X2 = ?, N2 = 6
We have

N1 X1 + N 2 X2
X=
N1 + N2

53 =
a f d i
11 71 + 6 X2
11 + 6

781 + 6 X2
53 =
17
781 + 6 X2 = 53 17

6 X2 = 901 781

6 X2 = 120

X2 = 20.
228 Basic Mathematics

Total cost of 6 English books = 6 20


= Rs. 120.
Given the cost of 6 English books form an increasing A.P.
Costs are a, a + d, a + 2d, a + 3d, a + 4d and a + 5d.
Given last term = 25
a + 5d = 25
5d = 25 a
Also sum of first n terms in an A.P. is

Sn =
n
2
a f
2a + n 1 d

S6 =
6
2
a f
2a + 6 1 d

S6 = 3 2 a + 5d
Substituting we get
120 = 3 2 a + 25 a

120
= a + 25
3
40 = a + 25
40 25 = a
a = 15.
Cost of the cheapest English book = Rs. 15.
12. At a place, the average temperatures from Monday to Thursday was 35C and from Tuesday to
Friday was 38C. Find the day temperatures on Monday and Friday if the ratio of temperatures
on Monday and Friday is 5:7.
Solution: Average temperatures from Monday to Thursday = 35 (Given)
Temp. on Mon + Tue + Wed + Thu
i.e., = 35
4
Temperature on Mon + Tue + Wed + Thu = 35 4 = 140 ...(1)
Average temperature from tuesday to friday = 38 (Given)

Temp. on Tue + Wed + Thu + Fri


= 38
4
Temperature on Tue + Wed + Thu + Fri = 38 4 = 152 ...(2)
Equation (2) Equation (1)
= Temp on Fri Temp. on Monday = 152 140
Averages 229

Temp. on Fri Temp. on Monday = 12


Temp. on Fri = 12 + Temp. on Monday ...(3)
Also given ratio of temperature on Monday by Friday = 5:7
Temp. on Mon. 5
= .
Temp. of Fri. 7
7 (Temperature on Monday) = 5 (Temperature on Friday)
7 (Temperature on Monday) = 5 (12 + Temperature on Monday)
7 x = 60 + 5 x
7 x 5 x = 60 2 x = 60 x = 30
Temperature on Monday = 30
From (3) Temperature on Friday = 12 + Temperature on Monday
= 12 + 30 = 42.
13. In a hostel of 80 students, the total monthly expenses were increased by Rs. 8000 when 20 more
join the hostel. If the average monthly expenses thereby reduced by Rs. 40 per head, Find the
average monthly expenses per student.
Solution. Let the average monthly expenses per student be x.
Total monthly expenses for 80 students = 80 x = 80x
When 20 more join the hostel, monthly expenses increases by Rs. 8000.
Total monthly expenses of 100 students = 80x + 8000
Monthly average is reduced by 40 x 40.
Monthly expenses of 100 students = 100 (x 40)
a f
100 x 40 = 80 x + 8000

100 x 4000 = 80 x + 8000


100 x 80 x = 8000 + 4000
20 x = 12000
12000
x= = 600.
20
Monthly expenses per student = Rs. 600.
14. Out of 3 numbers, the first is twice the second and is half of the third. If the average of the three
numbers is 56, Find the 3 numbers.
Solution. Let the 3 number be x, y and z.
z x = 2y
Given: x = 2y = i.e.,
2 z = 4y
Also given average = 56.
x+ y+ z
= 56
3
230 Basic Mathematics

x + y + z = 56 3 = 168
2 y + y + 4 y = 168

7 y = 168

168
y= = 24
7
The 3 numbers are x = 2y = 2 (24) = 48
y = 24
and z = 4y = 4 (24) = 96.
15. The average expenditure of a man for the first five months is Rs. 1200 and for the next seven
months it is Rs. 1300. If he saves Rs. 2900 in that year, then find his average monthly income.
Solution. Given average expenditure for 5 months = Rs. 1200
Total expenditure for 5 months = 5 1200
= Rs. 6000.
Also given average expenditure for 7 months = Rs. 1300
Total expenditure for 7 months = 7 1300
= Rs. 9100
Total expenditure for 12 months = Rs. 6000 + Rs. 9100
= Rs. 15,100
Savings = Rs. 2900 (Given)
We know Saving = Income Expenditure
Income = Expenditure + Saving
Total income = Rs. 15100 + 2900
= Rs. 18,000
18000
Average monthly income = = Rs. 1500.
12
16. Ten years ago, the average age of a family of 4 members was 24 years. Two children having been
born, the average age of the family is same today. What is the present age of the youngest child
if they differ in age by 2 years.
Solution: Let the age of youngest child be x yrs.
Age of next child = x + 2 years.
10 years ago
Average age of 4 members = 24
Total age of 4 members = 24 4 = 96.
After 10 years,
Total age of 4 members = 96 + 4 10 [Each members age increases by 10 years]
= 96 + 40 = 136.
Averages 231

Now average of 6 members

=
136 + x + x + 2
6
a
= 24 given f
136 + 2 x + 2 = 24 6
138 + 2 x = 144
2 x = 144 138
2x = 6
x=3
Present age of the youngest child = 3 yrs.
17. The average ages of A and B is 42 yrs., that of B and C is 28 yrs. and that of C and A is 40 yrs.
Find the ages of A, B and C.
Solution: Let the ages of A, B and C be a, b and c. Given average of A and B = 42
a+b
= 42
2
a + b = 84 ...(1)
Given: Average age of B and C = 28 years.
b+c
= 28
2
b + c = 56 ...(2)
Also given average age of C and A is
c+a
i.e., = 20
2
c + a = 40 ...(3)
Solving (1) and (2) we get

a + b = 84
b + c = 56
a fa fa f
...(4)
a c = 28

Solving (3) and (4) we get

c + a = 40
a c = 28
2a = 68
a = 34
a + c = 40 34 + c = 40 c = 6
232 Basic Mathematics

b + c = 46 b + 6 = 46 b = 40
Age of A = 34 yrs. B = 40 yrs. and C = 6 yrs.

REMEMBER:
x1 + x 2 + ... + x n
Mean X =
n

w1 x1 + w2 x 2 + ... + wn x n
Xw =
w1 + w2 + ... + wn

X1 N1 + X2 N2 + ... + XnN n
X123 =
N1 + N 2 + ... + N n

EXERCISES

1. A cricketer makes 72, 59, 101, 18 and 10 runs respectively in 5 matches played by him. Find his
average score.
2. The average weight of a class of 24 students is 35 kgs. If the weight of the teacher is included,
the average rises by 400 gms. Find the weight of the teacher.
3. The average marks of 15 boys of a class is 65 and 11 girls of the same class is 78. Find the average
marks of the students.
4. 3 tests in English, 2 in Hindi, 4 in Kannada and 5 in Sociology are conducted. The average marks
scored by Raju in English is 60, that in Hindi is 56 and in Kannada is 45. If the average marks
of all the subjects taken together is 48. Then find the average marks scored by him in Sociology.
5. 10 shirts and 5 pants were bought for Rs. 6000. If the average price of a shirt is Rs. 450, then find
the average price of a pant.
6. The average of 25 results is 18, that of first 12 is 14 and of the last 12 is 17. Then find the 13th
result.
7. The average age of A, B, C, D five years ago was 45 years. By including X, the present age of
all the five is 49 years. Find the present age of X.
8. A batsman makes a score of 87 runs in the 17th innings and thus increased his average by 3. Find
his average after 17th inning.
9. Miss Radha bought 51 dress materials in a discount sale. The average price of a dress material
being Rs. 318. The average price of 33 polyster dress materials is Rs. 426. If the prices of the
remaining cotton dress materials form an increasing arithmetic progression with last term 150.
Find the price of the cheapest cotton dress material.
10. The average temperature for Monday, Tuesday and Wednesday was 40C. The average tempera-
ture for Tuesday, Wednesday and Thursday was 41C. If the temperature on thursday was 42C.
Then find the temperature on Monday.
Averages 233

11. The average age of a husband and a wife was 23 years when they were married 5 years ago. The
average age of the husband, the wife and a child who was born during the interval is 20 years now.
How old is the child now?
12. The average age of 5 members of a committee is the same as it as 3 years ago, because an old
member has been replaced by a new member. Find the difference between the ages of old and new
member.
13. There were 35 students in a hostel. If the number of students is increased by 7, the expenses of
the mess were increased by Rs. 42 per day while the average expenditure per head diminished by
Re 1. Find the original expenditure of the mess.
14. The average weight of A, B and C is 45 kg. If the average weight of A and B is 40 kgs and that
of B and C is 43 kg. Then find the weight of B.
15. The average salary of 20 workers in an office is Rs. 1900 per month. If the managers salary is
added, the average salary becomes Rs. 2000 per month. What is the managers annual salary?

ANSWERS

1. 52 runs 2. 45 kgs 3. 70.5 4. 40 5. Rs. 300 6. 78


7. 45 years 8. 39 9. Rs. 39 10. 39C 11. 4 years 12. 15 years
13. Rs. 420 14. 31 kg 15. Rs. 48,000.
234 Basic Mathematics

9
Bill Discounting
9.1 INTRODUCTION:
Suppose a merchant A purchases goods worth say Rs. 50,000 from another merchant B at a credit for
certain period say 6 months. Then B draws up a draft i.e. prepares a special bill called Hundi or bill
of exchange. On the receipt of the goods, A gives an agreement dually signed on the bill stating that he
has accepted the bill and the money can be withdrawn from his bank account after 6 months of the date
of bill. On this bill, there is an order from A to his bank asking to pay Rs. 50,000 to B after 6 months.
If B needs the money of this bill earlier than 6 months, then B can sell the bill to a banker or a broker
who pays him the money against the bill but somewhat less than the face value.
Bill discounting is essentially lending by the banker against the bills and the bankers charges a
certain interest for doing this service.

9.2 TERMINOLOGY:
Bill: The document which required an individual to pay a fixed amount after a fixed later date is called
a bill.
Discount: When a bill is cashed in advance of its date of maturity an amount is deducted by the money
lender or bank from the amount of the bill due. This amount deducted is called Discount.
True discount: The present value of a sum of money is that principal which, if placed on stipulated rate
for a specified period will amount to that sum of money at the end of the specified period. The interest
on the present value of the bill is called true discount.
Bank er
erss discount: The interest on the face value or amount of the bill is called Bankers
Banker
discount.
Bank er
erss ggain:
Banker ain: The difference between bankers discount and true discount is called Bankers
gain.
Bill Discounting 235

Illustration: Bill of Exchange:

Stamp No. 1843, K.R. Road,


Bangalore
To, 10.2.2005
Anil,
BTM Layout, Bangalore
Six months later pay me or my order the sum of Rs. 50,000 (Rupees fifty thousand) for value
received.
Anil Akram

In the above bill of exchange.


Drawer is Mr Akram so payee is Akram.
Drawee or Acceptor is Mr. Anil.
Drawing date: 10.2.2005
Bill amount or Face value of Bill = Rs. 50,000 = Nominal value of the bill.
Legally due date: drawing date + Bill period + grace period of 3 days
(It is customary to give 3 days grace period).
So In the above bill,
Legally due date is 13.8.2005

Date of drawing : 10 - 2 - 2005


Bill period : 0 - 6 - 0
Q Grace period : 3 - 0 - 0
Legally due date : 13 - 8 - 2005

9.3 FORMULAE:
If F is the face value of the bill, BD is the bankers discount, t is the time in years, R = Rate of interest
R
r= ; TD = True discount; BG = Bankers gain, P = Present value of the bill, then
100
1. BD = Ftr
2. TD = Ptr
3. BG = BD TD
4. Bankers present worth or the discounted value of bill = F BD
F Ftr
a
F 1 tr f
F
5. True present worth Pw =
1 + tr
236 Basic Mathematics

BD TD
6. F =
BG
7. BG = TD t r

aTDf where P
2

8. BG =
bP g
w
w = True present worth.

9. TD = a P. w.f a B. G.f

WORKED EXAMPLES:
1. Find the present value, true discount, Bankers discount and Bankers gain on a bill of Rs. 10,450
due in 9 months at 6% per annum.
Solution:
Given: F = Rs. 10,450
9 3
t = 9 months = yrs. = yrs.
12 4
6
R = 6% r = = 0.06
100
Bankers discount = BD = Ftr
3
BD = 10,450 0.06
4
BD = 470.25
Bankers discount = Rs. 470.25.
F
True present worth, P= ,
1 + tr
10,450
P=
3
1 + 0.06
4
10,450 4
P=
4 + 0.18
P = 10,000
Present worth = Rs. 10,000.
True discount = Ptr
3
= 10,000 0.06
4
= 450.
Bill Discounting 237

TD = Rs. 450.
Bankers gain = BD TD
= 470.25 450
= 20.25.
Bankers gain = Rs. 20.25.
2. A bill for Rs. 13,000 was drawn on 3rd Feb 2005 at 6 months date and discounted on 13th Mar
2005 at the rate of 8% p.a. For what sum was the bill discounted and how much did the banker
gain on this?

Solution: Date of Bill : 3 - 2 - 2005


Bill period : 0 - 6 - 0
Grace period : 3 - 0 - 0
Legally due date : 6 - 8 - 2005

But bill was discounted on 13th Mar 2005.

Period = 18 days + 30 days + 31 + 30 + 31 +


B B B B B B
6

aMarchf aAprilf aMayf aJunef aJulyf aAugf


146
t = 146 days = yrs.
365
8
R = 8% r = = 0.08
100
F = Rs. 13,000
146
BD = Ftr = 13000 0.08
365
BD = 416.
Bankers discount = Rs. 416.
F
Present worth, P=
1 + tr

13000
P=
146
1+ 0.08
365

13000
P= = 12596.899
1.032
P 12596.90
146
True discount = Ptr = 12596.90 0.08
365
238 Basic Mathematics

TD = 403.10
True discount = Rs. 403.10
Bankers gain = BD TD
= 416 403.10
= 12.90
Bankers gain = Rs. 12.90.
3. A banker pays Rs. 2440 on a bill of Rs. 2500, 73 days before the legally due date. Find the rate
of discount charged by the banker?
Solution: Given: F = Rs. 2500
Bankers discount = Rs. 2500 2440
BD = Rs. 60
73
t= yrs.
365
R=?
We have BD = Ftr
73
60 = 2500 r
365
60 365
r=
2500 73
r = 0.12
R = 100r = 0.12 100
R = 12%.
Rate of discount charged by banker = 12%.
1
4. The bankers gain on a bill is of the bankers discount and the rate of interest is 20% p.a. Find
5
the unexpired period of the bill.
1
Solution: Given: B. G. = BD
5
20
R = 20% r = = 0.2
100
BG = BD TD
1
BD = BD TD
5
1
TD = BD BD
5
Bill Discounting 239

4 F
TD = BD Q P =
5 1 + tr

F F I tr = 4 Ftr
H 1 + tr K 5
1 4
=
1 + tr 5
1 4

a f =
1 + 0.2 t 5
Cross multiplying 5 = 4 + 0.8t
5 4 = 0.8t
1
t= = 1.25 yrs.
0.8
t = 12 + 3 = 15 months.
5. The difference between the bankers discount and true discount on a certain sum of money due
in 4 months is Rs. 10. Find the amount of the bill if the rate of interest is 3% p.a.
Solution: Given BD TD = Rs. 10
4 1
t = 4 months = yrs. = yrs.
12 3
3
R = 3% r = = 0.03
100
Now BD TD = 10

F F I tr = 10
Ftr
H 1 + tr K
Ftr LM1
1 O
N 1 + tr PQ = 10
1M
L OP
F 0.03 M1 P = 10
1
3 M 1 + 0.03 1 P
N 3Q

a f LMN 1.101OPQ = 10
F 0.01 1

F LM
0.0001 O
N 1.01 PQ = 10
240 Basic Mathematics

10 1.01
F=
0.0001
F = 1,01,000
Amount of the bill = Rs. 1,01,000.
6. A bill for Rs. 2920 drawn at 6 months was discounted on 10.4.2000 for Rs. 2916. If the discount
rate is 5% p.a. On what date was the bill drawn.
Solution: Given: F = Rs. 2920.
Discounted value Rs. 2916.
5
Rate = 5% r = = 0.05
100
Discounted values = F (1 tr)
b a fg
2916 = 2920 1 t 0.05

= 1 t a0.05f
2916
2920

a f
t 0.05 = 1
2916
2920
t = 0.027398 yrs.

t = 0.027398 365 days


t = 10 days
Number of days from legally due date to discounted date = 10 days.
Now
Bill was discounted on 10.4.2000
Legally due date = 10.4.2000 + 10 days
= 20.4.2000
Bill period = 6 months
Grace period = 3 days.
Bill drawing date = 20.4.2000
af
3.6.0
Bill drawing date = 17.10.1999
7. A bill was drawn on March 8th at 7 months period and was discounted on May 18th at 5%. If the
bankers gain is Rs. 3 find the true discount, the bankers discount, and the sum of the bill.
Solution: Date on which the bill was drawn = March 8th
i.e., 83
Period (7 months): 7
Grace period: 30
Bill Discounting 241

Legally due date: 11 10 i.e., Oct. 11th.


Date on which the bill was discounted: May 18th
Time for which the bill has yet to run:
May + June + July + Aug + Sep + Oct
13 30 31 31 30 11 = 146 days.
146 2
t= = yrs.
365 5
2
Now Bankers gain = Simple interest on true discount i.e. Rs. 3 is the S.I. on T.D. for at 5%
5
2
P5
3= 5
Then
100
100 3
P= = Rs. 150
2
Bankers discount = T.D. + S.I. on TD
2
= Rs. 150 + SI on 150 for yrs. at 5%
5
2
150 5
150 + 5
100

= 150 + 3 = Rs. 153.

OR

B. D.= TD + B. G.
BD = 150 + 3 = Rs. 153.
Now
BD TD 153 150
Sum = = = Rs. 7650.
BD TD 153 150
8. A bill was drawn on the 10th July 1960 at 3 months after signed and was accepted on presentation
on 1st Aug. 1960. It was discounted on 23rd Aug 69 at 5% p.a. simple interest to realise Rs. 2475.
Find the face values of thee bill and bankers discount.

Drawing date : 10 - 7 -1960

Bill period : 0 - 3 - 0
Solution: Grace period : 3 - 0 - 0

Legally due date : 13 -10 -1960


242 Basic Mathematics

Given: Bill was present on 1st Aug. 1969.


Number of days for which bankers discount is paid =
30 days + 30 + 13 = 73 days.
a f a fa f
Aug Sept Oct

73
t= = 0.2 yrs.
365
R = 5% r = 0.05
Let the face value of bill be Rs. 100.
Then BD = Ftr
BD = 100 0.2 0.05
BD = 1.
For Rs. 100 face value bill, BD = Re 1
i.e., Rs. 99 is realised.
i.e., Rs 99 is realised when face value is Rs. 100
Rs. 2475 is realised when face value is x.
2475 100
x=
99
x = 2500.
Face value of the bill = Rs. 2500.
Now, BD = Ftr
= 2500 0.2 0.05
BD = 25.

OR

For 100, BD = 1
2500 10
For 2500, BD = = Rs. 25.
100
9. The present worth of a bill due sometime hence is Rs. 1100 and the true discount on the bill is
Rs. 110. Find the bankers discount and the extra gain the banker would make in the transaction.
Solution: Here time and rate of interest are not given.
Given: PW = Rs. 1100
TD = Rs. 110

We have TD = a PW f a BGf
Squaring aTDf 2
= PW BG
Bill Discounting 243

BG =
aTDf = a110f
2 2
= Rs. 11.
Pw 1100

Now BG = BD TD
BD = BG + TD
BD = Rs. 11 + 110
BD = Rs. 121.
10. The true discount on a bill of Rs. 1860 due after 8 months is Rs. 60. Find the rate, the bankers
discount and the bankers gain.
Solution: Given: F = Rs. 1860
TD = Rs. 60
8 2
t = 8 months = = yrs.
12 3
r=?
BD = ? and BG = ?
Present worth, P = 1860 60 = Rs. 1800.
TD = Ptr
2
60 = 1800 r
3
60 3
=r
1800 2
r = 0.05
Hence R = 5%.
BD = Ftr
2
BD = 1860 0.05
3
BD = Rs. 62.
BG = BD TD
= 62 60 = Rs. 2.

OR

BG =
aTDf = a60f
2 2
= Rs. 2.
Pw 1800
BD = TD + BG = 60 + 2 = Rs. 62.
244 Basic Mathematics

REMEMBER:
Legally due date = Bill Drawing date + Bil period + 3 days (Grace period).
BD = Ftr [Simple interest on face value of the bill]
TD = Ptr [Simple interest on present worth of the bill]
BG = BD TD
F
Present worth =
1 + tr
BG = TD tr

BG =
aTDf 2
If r and t are not given
Pw

TD = B G Pw

EXERCISE

1. A bill for Rs. 3500 due for 3 months was drawn on 27th March 2000 and was discounted at the
rate of 7% on 18th April 2000. Find the bankers discount and discounted value of the bill.
2. The bankers discount and true discount on a sum of money due four months are respectively
Rs. 510 and Rs. 500. Find the seem and the rate of interest.
3. The difference between BD and TD on a bill due after 6 months at 4% interest per annum is
Rs. 20. Find the true discount bill discount and face value of the bill.
4. The bankers gain on a certain bill due 6 months hence is Rs. 10, the rate of interest being 10%
p.a. Find the face value of the bill.
5. A banker pays Rs. 2340 on a bill of Rs. 2500, 146 days before the legally due date. What is the
rate of discount charged by banker?
6. A bill for Rs. 1460 drawn at 3 months was discounted at 4% p.a. on 9th November for 1454.40.
On what date the bill was drawn?
7. A bill was drawn on April 14th at 8 months after date and was discounted on July 24th at 5% p.a.
If the bankers gain is Rs. 2, what is the face value of the bill.
8. Find the bankers discount and cash value of a bill for Rs. 3400/- drawn on April 25th 1996 at 7
months and discounted on September 16th, 1996 at 5%.
9. The bankers gain of a certain sum due 2 years hence at 5% per annum is Rs. 8. Find the present
worth.
10. The present worth of a sum due sometimes hence is Rs. 576 and bankers gain is Re. 1. Find the
true discount.
11. The bankers gain on a sum due 3 years hence at 5% is Rs. 90. Find the bankers discount.
12. The bankers discount on a bill due 1 year 8 months hence is Rs. 50 and true discount on the same
sum at the same rate percent is Rs. 45. Find the rate of interest.
Bill Discounting 245

13. A bill for Rs. 3500 due for 3 months was drawn on 27th March 2000 and was discounted on 18th
April 2000 at 7% rate of interest. Find the bankers discount and discounted value of the bill.
14. A bill for Rs. 2920 drawn at 6 months was discounted on 10.4.97 for Rs. 2916. If the discount rate
is 5% per annum, on what date was the bill drawn?
15. If the difference of simple interest and true discount of a sum due one year 6 months, hence at 8%
p.a. is Rs. 81.45. Find the sum.

ANSWERS

1. Rs. 3451, Rs. 49 2. 6%, Rs. 255


3. T.D. = Rs. 1000, B.D. Rs. 1,020 F = 51,000 4. Rs. 4200
5. 16% 6. September 11th
7. Rs. 5,100/- 8. Rs.34, Rs. 3,366
9. Rs. 800 10. Rs, 24
11. Rs. 690 12. 6 and 2/3%
13. Rs. 49, Rs. 3451 14. 17.10.96
15. Rs. 6335.
246 Basic Mathematics

10
Stocks and Shares
10.1 STOCK:
In order to meet the expenses of a certain plan or a big project. Loan is raised from the public at a
certain fixed rate of interest. Bonds or promissory notes of a fixed value are issued for sale to the public.
If a man purchases a bond of Rs. 1000 at which 5% interest has been fixed. Then the holder of such
bond is said to have a Rs. 1000 stock at 5%. Here Rs. 1000 is called the face value of the stock.
Usually a period is fixed for the repayment of the loan i.e., the stock matures at a fixed date only. Now
if the person holding a stock is in need of the money before the date of maturity of stock, he can sell
the bond to some other person, where by the claim of interest is transferred to that person.

10.2 SHARES:
To start a big concern or a business a large amount of money is needed. This is usually beyond the
capacity of one or two individuals. Therefore a group of individuals get together and form the company.
The company issues a prospectus and invites the public to subscribe. The required capital is divided
into equal small parts called shares, each of a particular fixed value. The person who possesses one or
more share is called a share holder. Sometimes the company asks its share holders to pay some amount
immediately and balance after some period. The total money raised immediately is called the paid up
capital.

10.3 DISTINCTION BETWEEN STOCK AND SHARES:


The main distinction between stocks and shares is
(i) Shares can be issued directly, but stocks cannot be issued directly. Firstly amount is collected on
shares from the public. Stock certificates are issued only after collecting full amount of shares.
Stocks are never issued unless the shares are issued and subscribed in full by the public.
(ii) Shares need not be fully paid, but stocks must be fully paid. For example the value of a share is
Rs. 100. This is known as face value of the share. A share of Rs. 100 each may be called up at
Rs. 50 each by the company and the balance of Rs. 50 may be paid at later stage by the public as
and when demanded by the company. Stock must be fully paid means face value of Rs. 100 on
each share must be paid in full by the public in order to get the stock certificate from the company.
Stocks and Shares 247

10.4 TERMINOLOGY:
De bentur
Debentur es: Debentures are long term loans taken by the company from the public. Every person who
bentures:
lends such an amount is given a certificate of loan called debentures.
Face Value of Shar
Shares:es: It is the price at which shares are first issued by a company. It is the price printed
on the share certificate.
Market Price: It is the price at which the share can be brought or sold on the stock/share market.
Mark
Par Value of Shares: When the shares are issued to the public at the face value, it is called par value
Shares:
of share.
Example: When Rs. 10 share is issued at its face value, it is called par value.
Abo
Abov ve Par: If the market value of the share is more than the face value, it is said to be above par or
at premium.
Example: When Rs. 10 shares are issued at Rs. 12 the shares are said to be issued at Rs. 2 premium.
Belo
Beloww Par: If the market value of the share is less than the face value, it is said to be below par or it
is said to be at discount.
Example: When Rs. 100 shares are issued at Rs. 90. Then the shares are said to be issued at 10%
discount.
Dividend: It is the portion of the profit of the company which is distributed to the share holders. The
Dividend:
dividend is always calculated on the face value of the share. Dividends may be cash dividends or share
dividends. Bonus shares are known as stock dividends.
Ex-Di vidend and Cum-di
Ex-Dividend vidend Prices: Interest on bond is payable on pre-determined dates. If the
Cum-dividend
bond is bought or sold on a date closer to the interest due date, the prices may be quoted Ex-interest or
Cum-interest. If the price is ex-interest the selling price of it is not inclusive of interest. If it is quoted
cum interest, the buyers will receive the interest amount. In the case of shares, share dividend are paid
instead of interest. Cum dividend price quotations are usually higher than the Ex-dividend quotations.
Yield: Actual dividend received by the actual amount invested in a stock or shares called yield.
Dividend
i.e., Yield =
Amount invested
Nominal interest
i.e., =
Amount invested
Br ok
Brok er
erss and Br
oker ok
Brok er
oker age: Buying and selling of stocks or shares is done through the person called
era
brokers at stock exchange. They charge certain amount called brokerage. Note that when stock/share
is purchased, brokerage is added to the cost price and when stock or share is sold brokerage is sub-
tracted from the selling price.
Kinds of Shar hares:
es: A company may issue two kinds of shares. They are
(i) Preference shares (ii) Equity shares
(i) Pr
Prefefer
ef erence
er shares:
ence shar es: A preference share holder enjoys a preferential claim with regard to the
payments of dividend and repayment of capital. The rate of dividend is fixed, but it is paid before
profit is distributed to other members.
248 Basic Mathematics

(ii) Equity shar es: An equity share holder has no special rights. The rate of dividend is not fixed. It
shares:
varies from year to year. An equity share holder is paid dividend only after the claims of prefer-
ence share holders are satisfied.

Quotation:
Consider the statement Government paper mills 11% shares at 110. This is a quotation. This means
a share of the mill having face value Rs. 100 is available for sale at Rs. 110. This share fetches him a
dividend of Rs. 11 every year.

WORKED EXAMPLES:
1. Find the cost of 80 shares at 5% if the market value of the share is 93 and its par-value is Rs. 100.
If a person invests Rs. 37200 in such shares then find his annual income.
Solution. Cost of 1 share = Rs. 93
Cost of 80 such shares = 80 93 = Rs. 7440.
Now, the person has invested Rs. 37,200.
By investing Rs. 93, the person gets 1 share.
By investing Rs. 37200 the person gets
37200 1
= = 400.
93
The person possesses 400 shares.
Face value of 1 share = Rs. 100
Face value of 400 shares = 400 Rs. 100 = Rs. 40000
Annual income = 5% of face value of shares
5
= 40,000
100
= Rs. 2,000.
2. Find the yield by investing Rs. 1140 on 15% stock quoted at Rs. 95.
Nominal interest
Solution. Yield =
Amount invested
For Rs. 100 stock, Rs. 95 is the amount invested and Rs. 15 is the nominal interest.
15
Yield = = 0.1578 0.16.
95

OR

By investing Rs. 95, 1 stock is obtained


1140 1
By investing Rs. 1140, = 12 stocks are obtained.
95
Stocks and Shares 249

1 stock has face value 100/-


12 stock has face value 1200/-
Nominal interest = 15% of Face value of stock
= 15% of 1200
15
= 1200 = 180.
100
Nominal interest 180
Yield = =
Amount invested 1140
= 0.1578
0.16.
3. Which of the following is better investment? 5 % stock at 102 or 4 and 3/4% stock at 106.
1
5
Solution. Yield from 5 and % stock at 102 = 2
102

11 11
= = = 0.05392.
2 102 204

3
4
Yield from 4 and 3/4% stock at 106 = 4
106

19 19
= =
4 106 424
= 0.044811.
So yield from 5 and % stock at 102 is greater. Hence it is a better investment.
4. Find the cash required to purchase Rs. 20,000 stock at 105 (brokerage %). Also find the annual
dividend received if the company declares dividend of 8 and %.
Solution. Cash required to purchase Rs. 100 stock.

F 1 I
211
H
= 105 +
2
=
K2

211
20,000
Cash required to purchase Rs. 20,000 stock = 2
100

20,000 211
= = Rs. 21,100.
2 100
Annual dividend for Rs. 100 stock = 8 and Rs.
250 Basic Mathematics

1
20,000 8
Annual dividend for Rs. 20,000 stock = 2
100

17
= 200 = Rs. 1,700.
2
5. A person has invested a certain sum of money in 13% stock at 96. He sold the investment when
the market value went up to 101.5. He gained Rs. 1470 in this process. If he has paid the
brokerage at 2% for all the transaction, what was the amount of cash investment and what was the
stock value of the investment in the first instance.
Solution. Let the amount invested = Rs. x
In the first instance,
Cost of 1 share = 96 + 2 (Brokerage)
i.e., Cost of 1 share = Rs. 98.
Now
For Rs. 100 share Rs. 98 is the amount received.
98 x
For Rs. x share is the amount received.
100
When the market value went upto Rs. 101.5.
Cost of 1 share = 101.5 2 (Brokerage)
= 99.5.
Amount received from Rs. 100 99.5
99.5 x
Amount received for Rs. x
100
The person gained Rs. 1470 in this process. (Given)
98 x 99.5 x
+ = 1470
100 100
1.5 x
= 1470
100
470 100
x=
1.5
x = 98,000
Amount invested by a person = Rs. 98,000.
In the first instance,
For Rs. 98 he gets Rs. 100 worth stock
Stocks and Shares 251

98000 100
For Rs. 98000 he gets
98
Rs. 1,00,000.
Stock value of the investment in 1st instance = Rs. 1,00,000 and cash investment = Rs. 98,000.
1
6. Vivek has Rs. 16,500 stock in 3%. He sells it out at 101 and invests the proceeds in 4% railway
8
7 1
debentures at 131 . Find the change in his income a brokerage of % being charged on each
8 8
transaction.
Vivek has Rs. 16,500 stock in 3%.
Income from Rs. 100 stock = Rs. 3
16500 3
Income from Rs. 16,500 stock =
100
= Rs. 495.
He sells it out at 101 and 1/8 (brokerage 1/8%)
1 1
S.P. of Rs. 100 stock = 101
8 8
S.P. of Rs. 100 stock = Rs. 101
16500 101
Selling price of Rs. 16500 stock =
100
= 16,665.
7 1
Now he invests this Rs. 16665 in 4% railway debentures at 131 . (Brokerage = % )
8 8
i.e,
F7 1 I
H K
By investing Rs. 131 + , income derived = Rs. 4
8 8
16665 4
By investing Rs. 16665, income derived =
132
= Rs 505.
Change in income = Rs. 505 Rs. 495 = Rs. 10.
Income is increased by Rs. 10.
7. Tulasi has invested Rs. 1,00,000 partly in 12% stock at 120 and partly in 15% stock at 75. If the
total income from both is Rs. 15,000. Find the amount invested in 2 types of stocks.
Solution. Let the amount invested in 12% stock at 120 be x.
Then Amount invested in 15% stock at 75
= 1,00,000 x
252 Basic Mathematics

12 x x
Income from 12% stock at 120 = =
120 10

Income from 15% stock at 75 =


15
75
a1,00,000 x f
1,00,000 x
=
5
Given total income = 15,000
x 1,00,000 x
+ = 15,000
10 5
1x 2,00,000 2 x
= 15,000
10
x + 2,00,000 = 1,50,000
x = 2,00,000 1,50,000
x = 50,000
Amount invested in 12% stock at 120
= Rs. 50,000 and that invested in 15% stock at 75 = Rs. 1,00,000 50,000
= Rs. 50,000.
8. Mr. Gauriprasad sold Rs. 2250 stock at 75 and bought stock at 88.50 with proceeds. How much
stock worth does he buy if the brokerage is 2% for selling and 1.5% for buying.
Solution:
Cost of 1 stock = Rs. 75 2 (Brokerage)
Cost of 1 stock = Rs. 73
Amount received by selling stock worth Rs. 100 is Rs. 73.
Rs. 2250 73
Amount received by selling stock worth Rs. 2250 =
100
= Rs. 1642.50.
Now Brokerage for buying = 1.5%
Cost of 1 stock = 88.50 + 1.50 = 90
By investing Rs. 90, stock worth Rs. 100 can be purchased.
1642.50 100
By investing Rs. 1642.50, Stock worth can be purchased.
90
= Rs. 1825.
Mr. Gauriprasad bought stock worth Rs. 1825.
9. Pusphak buys Rs. 2,000 shares paying 9% dividend. If he wants to have an interest of 12% on his
money, then find the market value of each share.
Stocks and Shares 253

Solution. Dividend on Rs. 2000 share in 9% of 2000.


9
= 2000 = 180.
100
He wants to have 12% on his money.
Rs. 12 is an income on Rs. 100 share.
180 100
Rs. 180 is income on = 1500.
12
Market value of each share must be Rs. 1500 in order to have interest of 12% on his money.
10. A man invests some money partly in 3% stock at 96 and partly in 4% stock at 120. What is the
ratio of money he must invest in order to get equal dividends from both.
96
Solution. For an income of Re 1 in 3% stock at 96, investment = Rs. = Rs.32.
3
120
For an income of Re 1 in 4% stock at 120 investment = Rs.
4
= Rs. 30.
Ratio of investments = 32:30
= 16:15.

REMEMBER:
Nominal interest
Yield =
Amount invested
When stock is purchased, brokerage is added to cost price.
When stock is sold, brokerage is subtracted from selling price.
Interest or dividend is paid on the face value of the stock or share not the market value.
4 and % stock at 96 means a stock whose face value is Rs. 100 is available at Rs. 96. Interest
earned in 4 and .
Shares need not be fully paid but stock must be fully paid.

EXERCISE

1. Find the cost of


(a) Rs. 8750 8 and 3/4% stock at 92.
(b) Rs. 8500, 9 and % stock at 6 premium.
(c) Rs. 7200, 10% stock at 7 discount.
(d) Rs. 6400, 8% stock at par (brokerage 1/8%).
2. Find the cash required to purchase Rs. 1600, 8 and % stock at 105 (brokerage %).
254 Basic Mathematics

3. Find the cash realised by selling Rs. 2400, 5 and % stock at 5 premium, brokerage being 1/4%.
4. Which of the following is better investment? 6% at 94 or 8% at 110.
5. Ramu possesses 150 shares of Rs. 25 each, the dividend declared by the company is 12%. What
is the dividend earned by him. If he sells the shares at Rs. 40 and reinvest the proceeds in 7%
shares of par value Rs. 100 at Rs. 80, Find the change in his dividend income.
6. A man invested Rs. 6750 partly in 6% stock at 140 and partly in 5% stock at 125. Find his
investment in cash if the income derived from both the investments is Rs. 280.
7. Mr. Vivek invested Rs. 2200 partly in 10% stock at 120 and partly in 12% stock at 96. Find his
investment in each if the income derived from both the investments is Rs. 200.
8. A man invests some money partly in 6% stock at 96 and partly in 5% stock at 120. In what ratio,
he must invest the money so as to get equal dividends from both.
9. Rs. 2780 is invested partly in 4% stock at 75 and 5% stock at 80. If the income from both
investments are equal, find the investment in 5% stock.
10. Mr. Harish has invested a certain amount of money in 13% stock at 101. He sold it when market
value went down to 96.5. He lost Rs. 3564 on this process. If he has paid the brokerage at
1 and % for all transactions, what was the amount of cash investment? What was the stock value
of the investment in first instance.

ANSWERS

1. (a) Rs. 8050 (b) Rs. 9010 (c) Rs. 6696 (d) Rs. 6408
2. Rs. 1688
3. Rs. 2514
4. 8% at 110
5. Rs. 75
6. Rs. 3500 and Rs. 3250
7. Rs. 1800 and Rs. 400
8. 2:3
9. Rs. 1280
10. Rs. 73,062 and Rs. 72338.60.
11
Learning Curve
11.1 INTRODUCTION:
When an individual performs the same task repeatedly the second and subsequent time will provide
experience to the individual and there by there will be an increase in the degree of efficiency in
performance. This is due to the learning process.
The learning effect improves productivity of individuals especially that of workers engaged in fac-
tories and industries.

11.2 LEARNING CURVE:


It is the curvilinear relationship between the decrease in average labour hours per unit with increase in
the cumulative output. The assumption of learning curve theory is that every time total output of a
product doubles, the cumulative average time taken to produce one unit decreases by a constant per-
centage. For example: 80% learning effect means that when the cumulative output is doubled, the
cumulative average hours per unit will be 80% of the previous level.

11.3 THE LEARNING CURVE RATIO:


It is the ratio of average labour cost of first 2N units and average labour cost of first N units. It is also
known as experience ratio or improvement ratio or efficiency ratio.
Average labour cost of first 2 N units
Experience ratio =
Average labour cost of N units

11.4 GRAPHICAL REPRESENTATION OF LEARNING CURVE:


We know 80% learning effect means that when the cumulative output is doubled the cumulative aver-
age hours per unit will be 80% of the previous level. Let us consider the effect of 80% learning in the
form of a table taking cumulative time per unit as 100.
256 Basic Mathematics

Units Total Cumulative Total number Average hour per


Produced Output time percent of hours additional unit

1 1 unit 100 100 100


160 100
1 2 units 80% of 100 = 80 80 2 = 160 = 60
1
256 160
2 4 units 80% of 80 = 64 64 4 =256 = 48
2
409.6 256
4 8 units 80% of 64 = 51.2 51.2 8 = 409.6 = 38.4
4
655.36 409.6
8 16 units 80% of 51.2 = 40.96 40.96 16 = 655.36 = 30.72
8
1048.57 655.36
16 32 units 80% of 40.96 = 32.76 32.76 32 = 1048.57 = 24.57
16
32 64 80% of ... ... ...
M M M M M

Taking the total output on x-axis and cumulative average time per unit on y-axis, we get the learning
curve as shown in the figure.

x 1 2 4 8 16 32
y 100 60 48 38.4 30.72 24.57

Scale:
x - axis : 1 Unit = 1 cm
100 y - axis : 20 units = 1 cm

80

60

40

20

x
0
1 2 3 4 5 6 7 8 9 10 11 12 13 14 15 16

Fig. 11.1

The curve clearly indicates that there will be fast learning effect in the initial stages and after
sometime there will be a steady state phase in which there is not any significant learning effect.
Learning Curve 257

11.5 LEARNING CURVE EQUATION:


If b = logarithmic of learning ratio to base 2, y = cumulative average time per unit, x = cumulative total
number of units produced a = time for first unit, then the learning curve equation is given by
y = axb.
By considering log on both sides we get

@ E
log y = log ax b

log y = log a + log x b

log y = log a + b log x.

WORKED EXAMPLES:
1. Find the index of learning for 60% learning effect.
Solution: Index of learning b = log260%
log10 60%
b=
log10 2

log10 0.6 1.7782


b= =
log10 2 0.3010

1 + 0.7782 0.2218
= =
0.3010 0.3010
b = 0.7368.
2. A worker takes 10 hrs. to produce the first unit of product. What is the cumulative average time
per unit taken by him for the production of first two units? (Assuming the learning effect to be
80%). Also find the total time for producing the first two units.
Solution: Given: a = 10 hrs.
x=2
80% learning effect.
We have learning curve equation
y = axb
log 0.8
where b = log 2 80% =
log 2

1.9031 1 + 0.9031
b= =
0.3010 0.3010
0.0969
b= = 0.3219
0.3010
258 Basic Mathematics

Now y = 10 2 0.3219

10
y= 0.3219
2
10
y=
1.249
y = 8.0064 8
Cumulative average time = 8 hrs.
Let x = 20.3219
Consider log on b.s.

log x = log 2 0.3219

log x = 0.3219 log 2


log x = 0.3219 0.3010

log x = 0.09689
x = Antilog 0.09689
= 1.249.
Time taken for producing the first 2 units = 8 2 = 16 hrs.
3. The time required to produce the first unit of a product is 1000 hrs. If the manufacturers experi-
ences 80% learning effect, calculate the average time per unit and the time taken to produce
altogether 8 units. Also find the total labour charges for the production of 8 units at the rate of
Rs. 12.50 per hour.
Solution: Given: a = 1000
x=8
log 0.8
b = log280% = = 0.3219
log 2
Formula: y = axb
= 1000 8 0.3219
1000
=
8 0.3219
1000
y= = 512.
1.953
Average time/unit = 512 hrs.
To find x = 80.3219
log x = log80.3219
Learning Curve 259

0.3219 log 8

0.3219 0.9031
log x = 0.2907

x = antilog 0.2907
= 1.953.
Average time/unit = 512 hrs.
Total time to produce
8 units = 512 8 = 4096 hrs.
Labour charges for 4096 hrs.
= 4096 12.50 [Q Labour charges per hour = Rs. 12.50 (given)]
= Rs. 51,200.
OR

Units Total Cumulative Total number Average hour per


Produced Output time percent of hours additional unit

1 1 1000 1000 1000


1600 1000
1 2 80% of 1000 = 800 800 2 = 1600 = 300
2
2560 1600
2 4 80% of 800 = 640 640 4 = 2560 = 240
4
4096 2560
4 8 80% of 640 = 512 512 8 = 4096 = 192
8

Total time to produce 8 units = 4096 hrs.


Cumulative average time per unit = 512
Total time to produce 8 units = 512 8 = 4096.
Given Labour charges for 1 hr. = Rs. 12.50
Labour charges for 4096 hrs.
= 4096 12.50
= Rs. 51,200.
4. An engineering company has won the contract for supplying aircraft engines of a new type. The
prototype constructed to win the contract took 500 hrs. It is expected that there will be 90%
learning effect. Estimate the labour cost of producing 8 engines of new order, if the labour cost
is Rs. 40 per hour.
260 Basic Mathematics

Solution:

Units Total Cumulative Total hours Average hour per


Output average time additional unit

1 1 500 500 500


900 500
1 2 90% of 500 = 450 450 2 = 900 = 200
2
1620 900
2 4 90% of 450 = 405 405 4 = 1620 = 180
4
4 8 90% of 405 = 364.5 364.5 8 = 2916 2916 1620/8 = 162

Total time to produce 8 units = 2916


Given Labour charges per hour = Rs. 40
Labour charge for 2916 hrs.
= 2916 40 = Rs. 1,16,640.

OR

Given: a = 500
x=8
log 0.9
b = log290% =
log 2

1.9542 1 + 0.9542
b= =
0.3010 0.3010
0.0458
b= = 0.1521.
0.3010
Formula: y = axb
= 500 8 0.1521

500 500
= 0 .1521
=
8 1.372
y = 364.43
To find 80.1521
x = 80.1521
logx = 0.1521 log8
= 0.152 0.9031.
Average time/unit = 364.43.
Total time to produce 8 units.
Learning Curve 261

= 364.43 8 = 2915.4518
2916.
Given labour charges/hr = Rs. 40
Labour charges/2916 hrs.
= 2916 40
= 1,16,640.

REMEMBER:

=Average labour cost of first 2N unitsB


The learning curve ratio =
=Average labour cost of first N unitsB
The learning curve equation is
y = axb
where y = cumulative average time/unit.
a = Time for first unit.
b = log of learning ratio to base 2.
x = Cumulative total number of units produced.
Index of learning, b = log of learning ratio to base 2.

EXERCISE

1. Define the term learning curve ratio.


2. What is learning curve?
3. What do you mean by 80% learning effect.
4. Find the index of learning for 80% learning effect.
5. A worker requires 40 hrs. to produce first unit of a product. How much time is required for him
to produce a total of 4 units.
6. A worker requires 20 hrs. to produce the first unit of a product. If the learning effect is 80%
calculate how much time is required to produce 4 units. Also find the labour charges for the
production of 4 units at the rate of Rs. 20 per hr.
7. A company requires 2134.2 hrs. to produce the first 40 machines. If the learning effect is 80%,
find the number of hrs. required to produce next 120 machines.
8. The first unit of a product took 80 hrs. to manufacture. If the workers show 80% learning effect,
find the total time taken to produce 8 units.

ANSWERS

Average labour cost of first 2N units


1. .
Average labour cost of first N units
262 Basic Mathematics

2. Curvilinear relationship between the decrease in average labour hrs. per unit with increase in
curve ratio.
3. When the cumulative output is doubled, cumulative average labour hrs./unit will be 80% of the
previous level.
4. 0.3219
5. 108.4 hrs.
6. 48.8 hrs.
7. 3329.35 hrs.
8. 32.768 hrs.
12
Linear Programming
12.1 INTRODUCTION:
Linear programming is a powerful technique which can indicate a definite conclusion as to the best
utilisation of available resources under given circumstances. Any industrial process may consists of a
number of activities relating to capital to be employed, products to be made and sold, materials to be
used: machines to be run; inventories to the stored and consumed or a combination of the above. Since
utilisation of one affects the utilisation of another and due to the limitation of the total available
resources, these activities are interdependent or interlocking. In such a situation, a large number of
ways exists in which the available resources can be allocated to the competing demands. Linear pro-
gramming enables us to arrange for that combination of resources which optimise the cost, production,
profit etc.
This technique was evolved by George B. Dantzig as a tool for planning the diversification activities
of U.S. Airforce in 1947.

12.2 LINEAR PROGRAMMING:


Def inition: Linear programming is a mathematical technique for determining the optimal allocation of
Definition:
resources and obtaining a particular objective when there are alternative uses of the resources: money,
manpower, material, machine and other facilities. The objective in resource allocation may be cost
minimisation or profit maximisation. The technique of linear programming is applicable to problems in
which the total effectiveness can be expressed as a linear function of individual allocations and the
limitations on resources give rise to linear equalities or inequalities of the individual allocations.
Objecti
Objectiv ve function: The objective function is a quantified statement in linear programming model of
what the best results or the best advantage which is aimed for as the objective of the resource allocation
decision. Objective function will either be to maximise a value or to minimise a value.
The objective function is expressed as a linear function of decision variables. It is usually stated as
maximise or minimise a1x1 + a2x2 + ... +anxn where x1 x2 ... xn are the decision variables in the problem
and a1, a2, ..., an are constant values for each variable.
Decision vvar
aria
ar iab
iables: The decision variables in any activity is a variable which is competing with other
activities for limited resources. These variables are inter-related linearly in terms of utilisation of
resources.
264 Basic Mathematics

Constr aints: The resources like production capacity, manpower, time, space, technology, etc. are
Constraints:
scarce and there are limitations on what can be achieved. These restrictions are a set of conditions
which an optimal solution must satisfy. They are known as constraints. These are expressed as linear
inequalities or equalities in terms of decision variables.
Non-neg
Non-ne tivity
gati vity conditions: All decision variables must assume non-negative values. If any of the
variable is unrestricted in sign, a trick can be employed which will enforce the non-negativity without
changing the original information of the problem.

12.3 SOLUTION TO LINEAR PROGRAMMING PROBLEM:


A set of values of decision variables x1 x2 ... xn which satisfies the constraints of linear programming
problem is called solution.
le solution: Any solution to a linear programming problem which satisfies the non-negativity
easible
Feasib
restriction of the problem is called a feasible solution to the Linear programming problem.
Optimal solution: Any feasible solution which optimises (maximises or minimises) the objective
function of a linear programming problem is called an optimal solution.

Solution of linear programming by graphical method:


If the objective function is a function of 2 variables only then the LPP can be solved by graphical
method.
One variable is taken along x-axis and another along y-axis. Since negative values are not allowed,
the graph contains only first quadrant. That is, 0 and positive values of x and y are considered.

2 I quadrant

1 2 3

Fig. 12.1

Illustration to find feasible region:


Consider the graph of y = 2.

0 1 2 3

Fig. 12.2
Linear Programming 265

This is a straight line parallel to x-axis. All points below this line are represented by the inequality
y < 2 and all points above this line are represented by y > 2 and the corresponding graphs are

y>2
y=2 2
2

y<2 1
1

x x
0 0 1 2
1 2

Fig. 12.3

Now consider the linear equation 3x + 2y = 6


To plot the graph, put x = 0
=B
3 0 + 2y = 6

2y = 6

y=3

and put =B
y = 0, 3 x + 2 0 = 6
3x = 6
x=2
Hence consider the points (0, 3) and (2, 0) and join them to get the graph of linear equation
3x + 2y = 6.

3 (0, 3)
3x

2
+2
y=

1
6

(2, 0)
0 1 2 3

Fig. 12.4

Any value (x, y) which will fall in the shaded area in figure (a) is represented by the inequality
3x + 2y 6 and any value which will fall in the shaded area in figure (b) is represented by the inequality
3x + 2y 6.
266 Basic Mathematics

(0, 3)
3 (3, 0) 3

2 2
3x + 2y 6
3x + 2y 6
1 1
(2, 0)

0 0 1 2 3
1 2 3
(2, 0)
(a) (b)

Fig. 12.5

Remark:
(1) When there are several inequalities, which are true at the same time, the feasible region of
combinations of values x and y must be a region where all the inequalities are satisfied.
(2) Optimum solution to linear programming problem lies at one of the vertices only. Hence find the
co-ordinates of each vertex and substitute in the objective function. The value for which the
objective function is highest/least is the optimum solution for maximisation/minimisation.

WORKED EXAMPLES:
1. Write the following data in the form of LPP:
Machine Chair Table Time
M1 5 2 50 min
M2 2 7 130 min
The price of each chair is Rs. 120 and each table is Rs. 130.
Solution: Let the number of chairs be x and number of tables be y.
Time availability for M1 50 and
Time availability for M2 130.
5 x + 2 y 50 and

2 x + 7 y 130
Objective function is Maximise Z = 120x + 130y
x 0 and y 0.
Hence LPP is given by
Maximise Z = 120x + 130y. Subject to constraints
5 x + 2 y 50; 2 x + 7 y 130 and x 0, y 0.
2. A company manufacturers 2 types of bulbs A and B by using 2 machines M1 and M2. One bulb
of type A require 2 hrs. at machine M1 and 1 hr. at machine M2 and one bulb of type B requires
Linear Programming 267

one hr. at M1 and 2 hrs. at M2. The profit from each bulb of type A is Rs. 2 and that of type B is
Rs. 3. The number of hrs. available per week on machines M1 and M2 are 20 hrs. and 30 hrs.
respectively. Formulate the above problem as LPP. The aim of the company is to maximise the
profit.
Solution: Let the number of bulbs of type A = x and that of type B = y
At machine M1, one bulb of type A requires 2 hrs. and one bulb of type B requires 1 hr. Also number
of hrs. available per week on machine M1 is 20.
2 x + 1y 20
Similarly at machine M2, one bulb of type A requires 1 hr., B requires 2 hrs. Also number of hours
available per week = 30 hrs.
1x + 2 y 30
The profit from each bulb of type A is Rs. 2 and that from type B = Rs. 3
Profit function = 2x + 3y
Mathematical model of LPP is
Maximise, Objective function, Z = 2 x + 3y
Subject to constraints
2 x + 1y 20

1x + 2 y 30
x 0, y 0.
3. Maximise Z = 20x + 30y subject to x + 2y 20, x + 5y 35 and x 0 and y 0.
Indicate the feasible region on the graph:
Solution: To draw x + 2y = 20
Put x = 0; 2y = 20 y = 10
Put y = 0; x + 2 (0) = 20 x = 20.
(0, 10) and (20, 0) are points on straight line x + 2y = 20.
To draw x + 5y = 35
Put x = 0, 5y = 35
y=7
Put y = 0, x = 35.
(0, 7) and (35, 0) are points on the straight line x + 5y = 35.
The co-ordinates of the vertices are obtained by solving

x + 2 y = 20
x + 5 y = 35
= B= B = B

3y = 15

y=5
268 Basic Mathematics

Substituting y = 5 in x + 2 y = 20
=B
x + 2 5 = 20
x + 10 = 20
x = 10 .
(x, y) =(10, 5)

30
Scale:
x-axis : 1 cm = 5 units
25
y-axis : 1 cm = 5 units
20

15
D
10
B P
5
A

0 5 10 15 20 25 30 35 40
C

Fig. 12.6

The shaded area OCPB in the figure is the feasible region.


The co-ordinates of vertices: O (0, 0), B (0, 7), P (10, 5) and C (20, 0).
Objective function: Z = 20x + 30y
for (0, 0) Z = 0.
for (7, 0) Z = 20 (0) + 30 (7) = 210
for (10, 5) = Z = 20 (10) + 30 (5) = 200 + 150 = 350
for (20, 0) = Z =20 (20) + 30 (0) = 400
Z is maximum when x = 20 and y = 0.
i.e., Optimum solution is Z = 400 when x = 20 and y = 0.
4. Solve the following LPP graphically:
Maximise: Z = 5x + 6y Subject to x + y 300 and x + y 500 x 0 and y 0.
Solution: To plot the graph of x + y = 300
Put x = 0 y = 300
Put y = 0 x = 300
(0, 300) and (300, 0) are points on the straight line x + y = 300
To plot graph of x + y = 500
Put x = 0 y = 500 and
y = 0 x = 500
Linear Programming 269

(0, 500) and (500, 0) are points on the straight line x + y = 500.

600
D
500

400

B300

200

100

0 100 200 300 400 500 600


A C

Fig. 12.7

The shaded region OAB in the figure is the feasible region.


To find the optimum solution:
Z = 5 x + 6y
At (0, 0) Z = 0.
= B =B = B
At 0, 300 , Z = 5 0 + 6 300 = 1800

At =300, 0B, Z = 5 =300B + 6 =0B = 1500


Zmax = 1800 at x = 0 and y = 300.
5. A radio factory produces 2 different types of transistor radios (a) cheaper ordinary model and (b)
an expensive special model. For greater efficiency the assembly and finishing operation and
performed in 2 workshop the ordinary model requires 3 hrs. of workshop A and 4 hrs. of work-
shop B. While special model requires 6 hrs. of workshop A and 4 hrs. of workshop B. Due to
limited resources and skilled labour only 180 hrs. of workshop A and 200 hrs. in workshop B. The
factory makes a profit of Rs. 300 on each ordinary model and Rs. 400 on each special model.
Formulate the above as a LPP and solve by graphical method, assuming the company expects
maximum profit.
Solution: Let the number of ordinary models = x and the number of special model = y.
Ordinary Special Total/hrs.

Workshop A: 3 6 180
Workshop B: 4 4 200

Objective function maximise Z = 300x + 400y


Subject to constraints,
270 Basic Mathematics

3 x + 6 y 180
4 x + 4 y 200, x 0 and y 0
To plot 3x + 6y = 180
Put x = 0, 6 y = 180 y = 30
Put y = 0, 3 x = 180 x = 60
(0, 30) and (60, 0) are points on the straight line 3x + 6y = 180
To plot 4x + 4y = 200
x + y = 50
Put x = 0, y = 50
y = 0, x = 50
(0, 50) and (50, 0) are points on the straight line 4x + 4y = 200
To get point of intersection
Solving 3x + 6y = 180 and 4x + 4y = 200

x + 2 y = 60
x + y = 50
i.e., = B= B = B

y = 10

Substituting y = 10 in x + y = 50 we get x = 40.


Point of intersection = (40, 10).

P
50
Scale : 1 cm = 10 units for both x-axis and y-axis
40
C
30

20
B
10

A D
0 10 20 30 40 50 60 70

Fig. 12.8

The shaded area OABC represent the feasible region.


Objective function: Z = 300x + 400y
At 0 (0, 0), Z = 0
At A (50, 0), Z = 300 (50) + 400 (0) = 15000
Linear Programming 271

At B (40, 10), Z = 300 (40) + 400 (10) = 16000


At C (0, 30), Z = 300 (0) + 400 (30) = 12000
Hence optimum solution, i.e.,
Zmax = 16,000 and at x = 40 and y = 10.
i.e., Max profit is obtained if number of ordinary models = 40 and special models = 10.
6. Minimise Z = 3x + 2y subject to x + y 7, 2 x + y 10 x, y 0 by graphical method.
Solution: To plot x + y = 7
Put x = 0, y = 7
y = 0, x = 7
(0, 7) and (7, 0) are points in x + y = 7
Now to plot 2x + y = 10
Put x = 0, y = 10
Put y = 0, 2x = 10 x = 5
(0, 10) and (5, 0) are points on 2x + y = 10

14
Scale :
1 cm = 2 units
12
C
10

4 B

0 2 4 6 A 8 10 12 14 16

Fig. 12.9

To get point of intersection solving

x+ y = 7
x + y = 10
= B= B = B

x = 3

x=3
Substituting x = 3 in x + y = 7 y = 7 3 = 4
272 Basic Mathematics

(3, 4) is the point of intersection.


The shaded region in the figure is feasible region.
To minimise Z = 3x + 2y
At A (7, 0) : Z = 3 (7) + 2 (0) = 21
At B (3, 4) : Z = 3 (3) + 2 (4) = 9 + 8 = 17
At C (0, 10) : Z = 3 (0) + 2 (10) = 0 + 20 = 20
Minimum value of Z = Zmin = 17
It is attained when x = 3 and y = 4
7. Food X contains 20 units of Vitamin A and 40 units of vitamin B. Food Y contains 30 units of each
vitamin A and vitamin B. The daily minimum human requirement of vitamin A and vitamin B are
900 and 1200 units respectively. How many grams of each type of food should be consumed so
as to minimise the cost if food X costs 60 paise per gram and food Y costs 80 per paise per gram.
Solution:

Food A B Cost
X 20 40 60
Y 30 30 80
Requirement: 900 1200

Objective function: Z = 60x + 80y


To minimise 60x + 80y. Subject to the conditions
20x + 30y 900 and 40x + 30y 1200
i.e., 2x + 3y 90 and 4x + 3y 120
x, y 0
To plot 20x + 30y = 900
i.e., 2x + 3y = 90
Put x = 0, 3 y = 90 y = 30.

Put y = 0, 2 x = 90 x = 45.
(0, 30) and (45, 0) are points on the straight line 2x + 3y = 90.
To plot
40x + 30y = 1200
i.e., 4x + 3y = 120
Put x = 0, 3 y = 120 y = 40.

Put y = 0, 4 x = 120 x = 30.


(0, 40) and (30, 0) are points on the straight line 4x + 3y = 120.
To find the point of intersection solving
Linear Programming 273

2x + 3y = 90 and 4x + 3y = 120

2 x + 3 y = 90
4 x + 3y = 120
=B =B =B

2 x = 30

x = 15
Putting x in 2x + 3y = 90
= B
2 15 + 3 y = 90

30 + 3y = 90
3 y = 60

y = 20
Point of intersection = (x, y) = (15, 20)

y
D Scale :
40 10 units = 1 cm

B 30

20 P

10

x
0 10 20 30 40 A 50 60
C

Fig. 12.10

The shaded region in the figure is the feasible region. To minimise Z = 60x + 80y
= B = B =B
At A 45, 0 : Z = 60 45 + 80 0 = 2700

At P =15, 0B : Z = 60 =15B + 80 =20B = 900 + 1600 = 2500

At D =0, 40B : Z = 60 =0B + 80 =40B = 0 + 3200 = 3200


Z is minimum when x = 15 and y = 20
Zmin = 2500.
8. A certain company wishes to plan its advertising strategy to reach certain minimum percentage
of high and low income groups. Two alternatives are considered namely cinema commercials and
magazines. Magazines advertising has an exposure for the high income group of 2% per page but
only 1% per page exposure for the low income group. Cinema commercials exposes 3% of the
low income group per show and only 1% of high income group per show. Magazines advertising
274 Basic Mathematics

costs Rs. 1000 per page and cinema commercial Rs. 4000 per show. If the firm wants a minimal
exposure of 50% of high income group and 30% of low income group, what strategy should it use
to minimize the advertisement cost? Also find the minimum advertisement cost by graphical
method:
Solution:
High income group Low income group Cost
Magazine 2% 1% 1000/page
Cinema 1% 3% 4000/show
Minimal exposure 50% 30%
It is required to minimise
Z = 1000x + 4000y
Subject to conditions 2x + y 50
1x + 3y 30
x 0 and y 0.
To plot 2x + y = 50
Put x = 0, y = 50
y = 0, 2x = 50 x = 25
(0, 50) and (25, 0) are points on straight line 2x + y = 50.
To plot 1x + 3y = 30
Put x = 0, 3y = 30 y = 10
Put y = 0, x = 30
(0, 10) and (30, 0) are points on the straight line x + 3y = 30.

y
70

60 Scale :
D 10 units = 1 cm
50

40

30 Feasible
region
20

B 10
P
A
x
0 10 20 C 30 40 50 60 70

Fig. 12.11
Linear Programming 275

To find point of intersection solving


2x + y = 50 and x + 3y = 30

2 x + y = 50
2 x + 6 y = 60
=B=B =B

5y = 10

y=2
Put y = 2 in 2x + y = 50
2x + 2 = 50
2x = 48 x = 24
(24, 2) is the point of intersection.
The shaded region in the graph is the feasible region.
To minimise Z = 1000x + 4000y
= B = B =B
At A 30, 0 Z = 1000 30 + 4000 0 = 30000

At P =24, 2B Z = 1000 =24B + 4000 =2B = 32000

At D =0, 50B Z = 1000 =0B + 4000 =50B = 200000


Zmin = 30,000
Minimum advertisement cost = Rs. 30,000.

REMEMBER:
The objective function is a quantified statement in linear programming problem of what the best
result is aimed for Objective function will either to maximise or to minimise a value.
If the objective function is a function of 2 variables only, the linear programming problem can be
solved by graphical method.
Optimum solution to linear programming problem lies at one of the vertices. Hence find the co-
ordinates of each vertex and subject in the objective function. The values for which the objective
function is highest/least is the optimum solution for maximisation/minimisation.

EXERCISE

1. Write the following data in the form of LPP:


Machine Product A Product B Time
M1 2 3 60 min.
M2 4 5 100 min.
The price of product A is Rs. 1000 each and product B is Rs. 1500 each.
2. Old hens can be brought at Rs. 200 each and the young ones at Rs. 500 each. The old hen lays
3 eggs and young one lays 5 eggs per week: If there are only Rs. 8000 available to spend on
276 Basic Mathematics

purchase, the hens should be bought in order to have a maximum profit per week, assuming that
the house cannot accommodate more than 20 hens at a time.
3. Maximise: 30x + 20y subject to
10 x + 6 y 1000, 5 x + 4 y 600

x, y 0
4. Maximise 5x + 8y subject to
2 x + y 100 ; x + 2 y 200

x, y 0.
5. A company produces 2 products x and y each of the product require two operations one on
machine A and the other on machine B. The machine hours required by these two products and
the total hours available are given as follows:

Machine hrs. required Product Total machine hrs. available


X Y

A 2 5 19
B 4 3 17

Each unit of the product x and y makes a profit of Rs. 3 and Rs. 4. Find the optimal solution of
the product to obtain the maximum profit.
6. Minimise Z = 4x + 4 subject to 3 x + 4 y 20, x + 5 y 15, x , y 0. .

7. Minimise Z = 10x + 6y subject to 2 x + y 60, 4 x + y 80, x1 y 0. .


8. Minimise 2 = 3x + 5y subject to 5 x + 2 y 0, x + y 4, x + 3 y 6, x1 y 0. .
9. An animal feed company must produce 200 kgs of mixture consisting of ingredients x1 and x2
daily. x1 costs Rs. 3 per kg and x2 Rs. 8 per kg. Not more than 80 kgs. of x, can be used and atleast
60 kgs. of x2 must be used. Find how much of each ingredients should be used if the company
wants to minimise the cost.
10. A chemist provides his customers at least cost, the minimum daily requirement of 2 vitamins A
and B by using a mixture of 2 products M and N. The amount of each vitamin in one gram of each
product, the cost per gram of each product and minimum daily requirements are given below:

Vitamin A Vitamin B Cost per gram

M 6 2 20 ps.
N 2 2 16 ps.
Minimum requirement 12 8

Formulate the problem of finding the least expensive combination which provide the minimum
requirement of the vitamins.
Linear Programming 277

ANSWERS

1. Maximise Z = 1000 x + 1500 y subject to


2 x + 3y 60
4 x + 5 y 100, x 0, y 0.
2. Maximise Z = 3x + 5y
Subject to condition
3. x = 40, y = 100, Zmax = 3,200
4. x = 0, y = 100, Zmax = 800
5. x = 2 and y = 3 Max Profit = 18 Rs.
6. x = 0, y = 5, Zmin = 5
7. x = 10, y = 40, Zmin = 340
8. x = 3, y = 1, Zmin = 14
9. Minimise 3x + 8y subject to
x + y 200, x1 < 01 y 60
x1 y 0
Optimum solution x = 80, y = 120.
Zmin = 1200 Rs.
10. Minimise Z = 20x + 16y subject to
6 x + 2 y 12
2x + 2y 8

x 0, y 0
278 Basic Mathematics

13
Circles
13.1 DEFINITIONS:
Circle is a locus of point which moves such that its distance from a fixed point is constant. The fixed
point is called centr
centree and the distance is called radius of the circle.
The boundary line of a circle is called cir cumf
circumf er
cumfer ence
erence
ence.
Note:
(1) The distance between 2 points (x1, y1) and (x2, y2) is given by distance formula

bx 2 x1 g + by
2
2 y1 g 2
.

y2 y1
(2) Slope of the line joining (x1, y1) and (x2, y2) is
x 2 x1
(3) 2 lines are perpendicular if and only if the product of their slopes = 1
(4) Angle in a semi circle = 90.

13.2 EQUATION OF A CIRCLE:


y
1. Equa tion of the cir
Equation circcle whose centr
whose centree is aatt the orig
orig in
igin
and radius = r units:
Let O (0, 0) be the centre of the circle and r be the radius P (x, y)
of the circle. Let P (x, y) be any point on the circle. r

OP = Radius = Distance between O (0, 0) and P (x, y). x x


O (0, 0)

r= a x 0f + a y 0f2 2

r = x 2 + y2
y
x 2 + y2 = r 2 .
Fig. 13.1
Circles 279

This is the equation of the circle whose centre is at the origin and radius = r units.
2. Equa tion of the cir
Equation circcle w centree is aatt (h, k) and rradius
hose centr
whose adius = r units:
Let C (h, k) be the centre of the circle. r be the radius of the y
y)
circle. P (x, y) be any point on the circle. (x,
P
CP = Radius = Distance between C (h, k) and P (x, y) r

r= a x hf + a y k f
2 2 C (h, k)

Squaring,

a f + a y kf
r2 = x h 2 2

a x hf + a y k f = r .
0
2 2 2
i.e., Fig. 13.2
This is the equation of the circle whose centre is at the origin and radius = r units.
tion of the cir
Equation
3. Equa circcle w hic
whic h is descr
hich ibed on line joining (x1, y1) and (x2, y2) as the diameter:
described
Let A (x1, y1) and B (x2, y2) be the ends of the diam-
eter of a circle. Let P (x, y) be any point on the circle.
P (x, y)
Join AP and BP.
We know,
90
Angle in a semicircle = 90
APB = 90
A (x 1, y 1) A (x2, y 2)
So AP BP
Slope of AP Slope of BP = 1 ...(1)
y y1
Slope of line joining A (x1, y1) and P (x, y) =
x x1
Fig. 13.3
y y2
Slope of line joining B (x2, y2) and P (x, y) =
x x2
Substituting in (1), we get
y y1 y y2
= 1
x x1 x x 2

b y y gb y y g = 1
b x x gb x x g
1 2

1 2

b y y gb y y g = b x x gb x x g
1 2 1 2

b x x gb x x g + b y y gb y y g = 0.
1 2 1 2

This is the equation of the circle which has (x1, y1) and (x2, y2) as ends of a diameter.
280 Basic Mathematics

General Equation of a Circle:


The equation x2 + y2 + 2gx + 2fy + c = 0 always represent a circle.
Consider
x 2 + y 2 + 2gx + 2 fy + c = 0
Rearranging,
x 2 + 2gx + y 2 + 2 fy + c = 0
Adding and subtracting g2 and f 2

x 2 + 2gx + g 2 g 2 + y 2 + 2 fy + f 2 f 2 + c = 0

a x + gf g + a y + f f
2 2 2
f2 +c=0

x a gf + y a f f
2 2
= g2 + f 2 c

a f
x g
2
+ y f a f =e
2
g2 + f 2 c j
2

Comparing this with the equation of circle

a x hf + a y k f
2 2
= r2
We get
a f a
Centre = h, k = g, f f
Radius = r = g 2 + f 2 c .
Hence

x2 +y2 + 2gx + 2fy + c = 0 represent a circle with centre (g, f) and


radius = g2 + f 2 c .

Note:
1. In the equation x2 + y2 + 2gx + 2fy + c = 0 co-efficient of x2 = co-efficient of y2 = 1 and there is
no xy term.
2. Centre is the mid point of diameter.

Fx +x I
y1 + y2
H 2 K
1 2
3. Co-ordinates of mid point of line joining (x1, y1) and (x2, y2) is , .
2
4. The point of intersection of 2 diameters of a circle = Centre

WORKED EXAMPLES:
1. Find the equation of the circle with centre at the origin and radius = 3 units.
Circles 281

Solution: Equation of the circle with centre (0, 0) and radius = r units = x2 + y2 = r2.
Equation of the circle with centre (0, 0) and radius r units = x2 + y2 = 32
x2 + y2 = 9.
2. Find the equation of circle with centre (3, 4) and radius 5 units.

Solution: Equation of circle with centre (h, k) and radius = r units = x h a f + a y kf


2 2
= r2..

a x 3f + a y 4f
2 2
= 52 .

x 2 + 9 6 x + y 2 + 16 8 y = 25.

x 2 + y 2 6 x 8 y + 25 25 = 0

x 2 + y 2 6 x 8 y = 0.
3. Find the equation of circle whose ends of diameter are (3, 1) and (4, 2).
Solution: Equation of circle whose ends of diameter are (x1, y1) and (x2, y2) =

b x x gb x x g + b y y gb y y g = 0
1 2 1 2

Equation of circle
a f b a fg a fa f
= x 3 x 4 + y 1 y 2 = 0

a x 3fax + 4f + ay 1fa y 2f = 0
x 2 3x + 4 x 12 + y 2 y 2 y + 2 = 0

x 2 + y 2 + x 3y 10 = 0.
4. Find the equation of the circle with centre (4, 3) and which passes through (0, 0).
Solution: Centre = (4, 3) (Given)
Circle passes through (0, 0).
Distance between centre and any point on the circle = Radius.
Distance between (4, 3) and (0, 0) = Radius

a0 4f + a0 3f
2 2
= Radius
(4, 3)

(0, 0)
16 + 9 = Radius
Radius = 5 units. Fig. 13.4

Equation of circle with centre (4, 3) and radius 5 units = x 4 a f + a y 3f


2 2
= 52

x 2 + 16 8 x + y 2 + 9 6 y = 25

x 2 + y 2 8x 6 y = 0
282 Basic Mathematics

Alieter:

a f a
Given: Centre = 4, 3 = g, f f
g = 4 and f = 3.
Let the equation of circle be x2 + y2 + 2gx + 2fy + c = 0
Given it passes through (0, 0)
0 + 0 + 2g (0) + 2f (0) + c = 0
c=0
Equation of circle

a f a f
= x 2 + y 2 + 2 4 x + 2 3 y + 0 = 0

x 2 + y 2 8 x 6 y = 0.
5. Find the equation of the circle whose two diameters are x + 2y = 3 and x y = 6 and radius 6 units.
Solution: The point of intersection of 2 diameters = centre.

x + 2y = 3
xy=6
Solving ( ) ( + ) ( )
3 y = 3

y = 1.

Substituting
Fig. 13.5
y = 1 in x + 2 y = 3

a f
x + 2 1 = 3

x2=3
x =3+2

x = 5.

Centre = (x, y) = (5, 1) = (h, k)


Radius = 6 units.

Equation of circle = x h a f + a y kf
2 2
= r2.

a x 5f + b y a1fg = 6 .
2 2 2

x 2 + 52 10 x + y 2 + 1 + 2 y = 36.

x 2 + y 2 10 x + 2 y + 26 36 = 0
Circles 283

x 2 + y 2 10 x + 2 y 10 = 0.

6. If one end of the diameter of the circle x2 + y2 2x 4y + 4 = 0 is (1, 1), find the other end.
Solution: Centre is the mid point of the diameter.
Given: Equation of circle:

x 2 + y2 2x 4y + 4 = 0
Comparing with general equation

x 2 + y 2 + 2gx + 2 fy + c = 0

2 g = 2, 2 f = 4, c = 4
g = 1, f = 2.

a
Centre = g, f = 1, 2 . f a f
Fx +x y1 + y2 I
H 2 K
1 2
Mid point formula: ,
2

a1, 2f = FH 1 +2x 2
,
1 + y2
2
I
K a f
Q 1, 1 is one end of the diameter.

1 + x2 1 + y2
1= and 2 =
2 2
2 = 1 + x2 2 2 = 1 + y2
x2 = 1 4 1 = y2

y2 = 3.
Other end of the diameter = (1, 3).
7. Find the centre and radius of the following circles:
(a) x 2 + y 2 + 2 x 4 y 7 = 0

(b) 2 x 2 + 2 y 2 3 x + 6 y 8 = 0

(c) 3 x 2 + 3 y 2 2 x 33 = 0

Solution: (a) x 2 + y 2 + 2 x 4 y 7 = 0.
Comparing with general equation,

x 2 + y 2 + 2gx + 2 fy + c = 0

We get 2 g = 2, 2 f = 4, c = 7

g = 1, f = 2, c = 7
284 Basic Mathematics

a f a
Centre = g, f = 1, 2 f
g 2 + f 2 c = 12 + a 2f a 7f
2
Radius =

= 1 + 4 + 7 = 12 = 2 3 units.

(b) 2 x + 2 y 3 x + 6 y 8 = 0
2 2

by 2

3x
x 2 + y2 + 3y 4 = 0
2
Comparing with general equation,
3
2 g = , 2 f = 3 and c = 4
2
3 3
g = , f = and c = 4
4 2

a
Centre = g, f = f FH 34 , 23 IK
F 3 I + F 3 I a4f
2 2
Radius = g 2 + f 2 c =
H 4K H 2K
9 9 9 + 36 + 64
= + +4 =
16 4 16

109 109
Radius = = units.
16 4

(c) 3 x 2 + 3 y 2 2 x 33 = 0
by 3.

2
x2 + y2 x 11 = 0
3
Comparing with general equation we get
2
2 g = , 2 f = 0, c = 11.
3
1
g = , c = 11.
3

a
Centre = g, f = f FH 13 , 0IK
Circles 285

F 1I 2
a f
Radius = g 2 + f 2 c =
H 3K + 0 11

1 1 + 99 100 10
= + 11 = = = units.
9 9 9 3
8. Find the equation of the circle whose centre is same as the centre of the circle x2 + y2 2x +
4y 7 = 0 and which has the radius 3 units.
Solution: Centre of the circle,
x 2 + y 2 2 x + 4 y 7 = 0 is

a g, f f = a+1, 2f FGHQ 2gg == 12 2ff ==24IJK


Given: Centre = (1, 2) and radius = 3 units.
Equation of the circle = (x h)2 + (y k)2 = r2
a x 1f + b y a2fg
2 2
= 32

x 2 + 12 2 x + y 2 + 4 + 4 y = 9
x 2 + y2 2 x + 4y 4 = 0 .
9. Find the equation of the circle which is concentric with the circle 2x2 + 2y2 + 3x 4y + 8 = 0 and
having unit radius.
Solution: Circles having same centre are called concentric circles.
Now centre of the circle: 2 x 2 + 2 y 2 + 3x 4 y + 8 = 0
by 2.
3 8
x 2 + y2 + x 2y + = 0
2 2
3
2g = 2 f = 2
2
3
g= f = 1
4

a 3
Centre = g, f = , + 1
4
f FH I
K
Radius = 1 unit.

Equation of circle = x h a f + a y kf
2 2
= r2

LM x F 3 I OP + a y 1f
2

N H 4K Q
2
= 12
286 Basic Mathematics

F x + 3 I + a y 1f
2

H 4K =1
2

9 3
x2 + + 2x + y2 + 1 2 y = 1
16 4
9 3x
x2 + + + y2 + 1 2y 1 = 0
16 2
3x 9
x 2 + y2 + 2y + =0
2 16

or 16 x 2 + 16 y 2 + 24 x 32 y + 9 = 0.

10. Find the equation of the circle whose centre is same as the centre of the circle x 2 + y 2 4 x +
8y 7 = 0 and radius same as that of the circle x 2 + y 2 + 8y 9 = 0 .

Solution: Centre of the circle x 2 + y 2 4 x + 8 y 7 = 0 is (g, f)

a f LMQ 2g = 4 2f =8 OP
= 2, 4
N g = 2 f =4 Q
Radius of the circle: x2 + y2 + 8x 9 = 0
Q 2g = 8
is g2 + f 2 c g=4

2f =0

a f
4 2 9
f =0
c = 9

= 16 + 9 = 25 = 5 units.
Equation of the required circle

a f + a y kf = r
= xh 2 2 2

a x 2f + b y a4fg = 5
2 2 2

x 2 + 2 2 4 x + y 2 + 16 + 8 y = 25

x 2 + y 2 4 x + 8 y 5 = 0.
11. A circle touches x-axis at (1, 0) and passes through (5, 2) find its equation.
Solution: Let the equation of circle be x2 + y2 + 2gx + 2fy + c = 0 ...(1)
Since the circle touches x-axis, y-co-ordinate of the centre = Radius.
Circles 287

f = g2 + f 2 c
Squaring,

f 2 = g2 + f 2 c

g 2 = c. ...(2)
Given (1) passes through (1, 0) y

af af
12 + 0 2 + 2 g 1 + 2 f 0 + c = 0

2g + c = 0 1
2 g + c = 1 ...(3)
Given (1) passes through (5, 2)
c (g, f)

af af
(5, 2)
5 + 2 + 2g 5 + 2 f 2 + c = 0
2 2
f

25 + 4 + 10g + 4 f + c = 0 (1, 0)
x

10 g + 4 f + c = 29 ...(4) Fig. 13.6


From (2) and (3)

g 2 = c and 2 g + c = 1

2 g + g 2 = 1

g2 + 2g + 1 = 0

ag + 1f 2
=0

g = 1.

c = g2

a f
c = 1 2 c = 1.

Substituting g = 1, c = 1 in (4)
10 g + 4 f + c = 29

10 + 4 f + 1 = 29
4 f = 29 + 9

4 f = 20

f = 5.
288 Basic Mathematics

Equation of required circle

x 2 + y 2 + 2gx + 2 fy + c = 0

i.e. a f a f
x 2 + y 2 + 2 1 x + 2 5 y + 1 = 0

x 2 + y 2 2 x 10 y + 1 = 0.
12. Find the equation of circle with centre (4, 6) and which y
touches the y-axis.
Since the circle touches y-axis, radius = x-co-ordinates of
the centre.
4
Radius = 4 units. Q centre = (4, 6).
(4, 6)
Equation of circle with centre (h, k) and radius r units

a f + a y kf = r .
= xh 2 2 2

a x 4 f + a y 6f = 4 .
2 2 2
x
0
x + 16 8 x + y + 36 12 y = 16
2 2
Fig. 13.7
x 2 + y 2 8 x 12 y + 36 = 0.

Alieter:
Let the equation of circle be x2 + y2 + 2gx + 2fy + c = 0 ...(1)
Given: a f a
Centre = 4, 6 = g, f f
g = 4 and f = 6.

Given: Circle touches y-axis.


g = g 2 + f 2 c
Squarring,
g2 = g2 + f 2 c
f 2 = c.
c= f2 = 6 af 2
= 36.

c = 36.

Equation of circle,

a f a f
= x 2 + y 2 + 2 4 x + 2 6 y + 36 = 0

x 2 + y 2 8 x 12 y + 36 = 0.
Circles 289

13. A circle has its centre on the y-axis and passes through (1, 3) and (2, 5). Find its equation.
Solution: Let the equation of the circle be
x2 + y2 + 2gx + 2fy + c = 0 ...(1)
Given: (1) has centre on Y-axis.
On y-axis, x-co-ordinate = 0.
X-co-ordinates of the centre (g, f) = 0

i.e., g = 0 g = 0.

Given: (1) passes through (1, 3)

a1f 2
a f af
+ 32 + 2 g 1 + 2 f 3 + c = 0

1 + 9 + 2 a0f + 6 f + c = 0 aQ g = 0f
6 f + c = 10 ...(2)
Given (1) passes through (2, 5)

af af
2 2 + 52 + 2 g 2 + 2 f 5 + c = 0

4 + 25 + 0 + 10 f + c = 0

10 f + c = 29 ...(3)
Solving (2) and (3)

6 f + c = 10
10 f + c = 29
( ) ( ) ( + )
4f = 19

19
f = .
4

19
Substituting f = in (2)
4
F 19 I + c = 10
6
H 4K
57
+ c = 10
2
57
c = 10 +
2
20 + 57 37
c= =
2 2
290 Basic Mathematics

37
c= .
2
Equation of the circle

af F 19 I y + 37 = 0
= x 2 + y2 + 2 0 x + 2
H 4K 2
Multiplying by 2 we get

2 x 2 + 2 y 2 19 y + 37 = 0.
14. Find the equation of circle which touch both the co-ordinate axis and passes through the point
(2, 1).
Solution: Let the equation of circle be
x2 + y2 + 2gx + 2fy + c = 0 ...(1)
Given (1) touches both co-ordinate axes.
[X-co-ordinate of centre = Radius = Y-co-ordinate of centre]

g = g 2 + f 2 c and f = g 2 + f 2 c r

g = f
r
g = f.

Also g2 = g2 + f 2 c
Fig. 13.8
f 2 = c. Similarly g 2 = c.

g 2 = f 2 = c.
Given (1) passes through (2, 1)
af af
2 2 + 12 + 2 g 2 + 2 f 1 + c = 0

4 + 1 + 4g + 2 f + c = 0
5 + 4g + 2 f + c = 0

5+4f +2f +c = 0 Q g= f
5+6f +c = 0

5+6f + f2 = 0

f2 +6f +5= 0 Q c= f2

f 2 + 5 f +1f + 5 = 0

a f a
f f + 5 +1 f + 5 = 0 f
Circles 291

f = 5 or f = 1
Since g = f,

g = 5 or g = 1

Q c = g2

a f
c = 5 2
a f
or c = 1 2

c = 25 or c = 1.
Equation of required circle

a f a f
= x 2 + y 2 + 2 5 x + 2 5 y + 25 = 0

x 2 + y 2 10 x 10 y + 25 = 0
or

a f a f
x 2 + y 2 + 2 1 x + 2 1 y + 1 = 0

x 2 + y 2 2 x 2 y + 1 = 0.
15. A circle cuts off positive intercepts 5 and 6 on x and y-axes respectively, and passes through the
origin. Find its equation.
Solution: Let the equation of circle be
x2 + y2 + 2gx + 2fy + c = 0 ...(1)
Given (1) makes an intercept 5 on x-axis.
(1) passes through (5, 0)
af af
52 + 0 + 2 g 5 + 2 f 0 + c = 0

25 + 10g + c = 0 ...(2)
Given (1) makes an intercept 6 on y-axis.
(1) passes through (0, 6).
af af
0 + 62 + 2g 0 + 2 f 6 + c = 0

36 + 12 f + c = 0 ...(3)
Given (1) passes through (0, 0)
af af
0 + 0 + 2g 0 + 2 f 0 + c = 0

c = 0.

Substituting c = 0 in (2) we get


25 + 10 g = 0
292 Basic Mathematics

25 5
g= =
10 2
Substituting c = 0 in (3)
36 + 12 f = 0

f = 3.

Equation of required circle

F 5 I x + 2 a3f y + 0 = 0
x 2 + y2 + 2
H 2K
x 2 + y2 5x 6 y = 0 .

13.3 POINT OF INTERSECTION OF A LINE AND A CIRCLE:


chord
Chor
Chord:d: A line which divides the circle into 2 parts is called chord. So chord is a line
joining any 2 distinct points on the circle.
angent:
Tang ent: A line which touches the circle at only one point and which is perpendicular
to the radius drawn from the point of contact is called tangent.
Note:
If ax + by + c = 0 is the equation of the chord AB of the circle x2 + y2 + 2gx + 2fy + c = 0 whose centre
is C.
Then from figure,
CB2 = CD2 + BD2

Now CB = Radius of the circle = g2 + f 2 c C

CD = Length of the perpendicular from the centre


(g, f) to the line ax + by + c = 0. A B
D

=
a f a f
a g + b f + c
a2 + b2 Fig. 13.9

1
BD = AB.
2
Length of the chord AB can be found if we know the equation of the circle and equation of the
chord.

WORKED EXAMPLES:
1. Find the length of the chord 4x 3y = 5 of the circle x2 + y2 + 3x y 10 = 0.
a
Centre: g, f f
Circles 293

F 3 1 I
H
= ,+
2 2 K
Radius of the circle = g2 + f 2 c

F 3 I + F 1 I a 10f
2 2
=
H 2K H 2K
9 1
= + + 10
4 4

50 1 1
= = 50 = 25 2
4 2 2

5 2 5
= = units.
2 2

F 3 1 I
H K
Perpendicular distance from C , , to the line 4x 3y 5 = 0 is
2 2

F 3I 3F 1 I 5
CD =
H 2K H 2K
4

4 + a 3f
2 2 C

A B
12 3 D
5
CD = 2 2
5 Fig. 13.10

25 1 5
= = .
2 5 2
5
CD = units.
2
From figure,
CB 2 = CD 2 + DB 2
DB 2 = CB 2 CD 2

FG 5 IJ F 5 I
2 2
DB =
H 2 K H 2K
294 Basic Mathematics

25 25 50 25
DB = =
2 4 4

25 5
DB = = .
4 2
Length of the chord AB = 2 DB
5
=2 = 5 units.
2
2. If x + 2y 4 = 0 is the equation of the chord to the circle x2 + y2 = 4, then find its length.
Solution: Centre of the circle = (0, 0)
Radius = 4 = 2 units.
Perpendicular distance from centre (0, 0) to the line x + 2y 4 = 0 is

a0 f + 2 a 0 f 4 = 4 =
4
units.
1 +2
2 2 5 5

From figure,
CB2 = CD2 + BD 2
BD 2 = CB 2 CD 2 C

FG 4 IJ 2
BD 2 = 2 2
H 5K A
D
B

16
BD2 = 4
5 Fig. 13.11
20 16
BD 2 =
5
4
BD2 =
5

4 2
BD = = units.
5 5
Length of the chord AB = 2 BD
2 4
=2 = units.
5 5
3. If 3x + y 5 = 0 is a chord to the circle x2 + y2 22x 4y + 25 = 0, then find its length.
Circles 295

Solution:
Centre = (g, f)
a f
= 11, 2

Radius = g 2 + f 2 c = 112 + 2 2 25

= 121 + 4 25 = 100 = 10 units.


Perpendicular distance from the centre (11, 2) to the line 3x + y 5 = 0 is

a f
3 11 + 2 5
=
33 + 2 5
3 +1
2 2 10

30
CD = units.
10
From figure,
CB 2 = CD 2 + BD 2

BD 2 = CB2 CD2

F 30 IJ
G
2 C
BD = 10
H 10 K
2

A B
D
900
= 100
10
Fig. 13.12
BD = 10 units.
AB = Length of the chord = 2 BD

= 2 10

= 2 10 units.
4. Find the point of intersection of the line 4x 3y = 10 and the circle x2 + y2 2x + 4y 20 = 0
and find the length of chord intercepted by the line.

Equation of circle: x 2 + y 2 2 x + 4 y 20 = 0

Equation of line: 4 x 3y = 10
4 x = 10 + 3 y

10 + 3y
x= .
4
296 Basic Mathematics

10 + 3y
Substituting x = in the equation of circle we get the point of intersection.
4

F 10 + 3y I 2
F 10 + 3y I + 4 y 20 = 0
H 4 K + y2 2
H 4 K
a10 + 3yf 2
+ y2 2
a10 + 3yf + 4 y 20 = 0
16 4
Multiplying by 16

a10 + 3yf 2
a f
+ 16 y 2 8 10 + 3y + 64 y 320 = 0

y 2 + 4 y 12 = 0

y 2 + 6 y 2 y 12 = 0

a f a f
y y+6 2 y+6 =0

a y 2fay + 6f = 0
y = 2 or y = 6

10 + 3y
Q x=
4

x=
10 + 3 2 af
or x =
10 + 3 6 a f
4 2
x = 4 or x = 4
The point of intersection are (4, 2) and (4, 6).
Length of the chord = Distance between (4, 2) and (4, 6).

= bx x g + by y g
2 1
2
2 1
2

= a4 4f + a6 2f
2 2
A
(4, 6)
B (4, 2)

= 8 2 + 8 2 = 64 + 64 Fig. 13.13

= 128 = 8 2 units.
5. If x + y = 1 is the chord to the circle x2 + y2 2x 4y 29 = 0, then find its length. Also find the
co-ordinates of middle point of chord intercepted.
Solution:
Equation of the line = x + y = 1
y=1x
Circles 297

Substituting y = 1 x in the equation of the circle we get

a f
x2 + 1 x 2
a f
2 x 4 1 x 29 = 0

x 2 + 1 + x 2 2 x 2 x 4 + 4 x 29 = 0

2 x 2 32 = 0

2 x 2 = 32
32
x2 =
2

x 2 = 16
x = 4.
y =1 x

y = 1 4 y = 5 or y = 3.
Point of intersection of the circle with the line = (4, 3) and (4, 5)

Length of the chord = bx x g + by y g


2 1
2
2 1
2

= a4 4f + b5 a3fg
2 2
A B (4, 3)
(4, 5)
= 64 + 64
Fig. 13.14
= 8 2 units.
Co-ordinates of the middle point of the chord whose end points are (4, 3) and (4, 5)

Fx +x , y +y I
=
H 2 2 K
1 2 1 2

F 4 + a4f , 3 + 5IJ
=G
H 2 2 K
= (0, 1).
x y
6. If the straight line + = 1 cuts the circle x 2 + y 2 = r 2 in 2 points, then prove that the length
a b
of the chord

=2
d
r 2 a 2 + b 2 a 2b2 i
a + b2
2

Proof: Equation of line


Proof:
x y
= + =1
a b
298 Basic Mathematics

bx + ay
=1
ab
bx + ay ab = 0
The equation x2 + y2 = r2 represent the circle whose centre = (0, 0) and radius = r units.
Length of the perpendicular from (0, 0) to the line bx + ay ab = 0.

OD =
af af
b 0 + a 0 ab
=
ab
.
b +a
2 2
a + b2 2

From figure,
OB2 = OD 2 + BD2
bx + ay ab = 0
BD2 = OB 2 OD 2

F
G
I 2

BD = r
2 2

H a +b
2
ab
2 JK Fig. 13.15

a2 b2
BD2 = r 2
a2 + b2

BD = r 2
a 2b2
=
d
r 2 a2 + b2 a2 b2 i
a2 + b 2 a + b2
2

Length of the chord AB = 2 BD

AB = 2
d
r 2 a2 + b 2 a2b 2 i .
a + b2
2

Hence proved.

13.4 EQUATION OF TANGENT TO THE CIRCLE


x2 + y2 + 2gx + 2fy + c = 0 AT THE POINT (x1, y1) ON IT:
Let P (x1, y1) be any point on the circle x2 + y2 + 2gx + 2fy + c = 0. Let C (g, f) be the centre. PT
be the tangent. Joint CP.
P (x 1, y 1)
We know,
T
y2 y1
Slope of the line joining 2 points (x1, y1) and (x2, y2) = x x
2 1

y1 f a f C (g, f)
Slope of line joining C (g, f) and P (x1, y1) = x g
1 a f
y1 + f
= Fig. 13.16
x1 + g
Circles 299

Since PT perpendicular CP
Slope of PT Slope of CP = 1
1
Slope of PT =
Slope of CP

1
=
y1 + f
x1 + g

FG x + g IJ
Slope of PT =
Hy + fK
1

Equation of a straight line passing through (x1 y1) and having slope m is

b
y y1 = m x x1 g
Equation of tangent PT

FG x + g IJ b x x g
= y y1 =
Hy + fK
1
1
1

b y y gb y + f g = b x + ggb x x g
1 1 1 1

yy1 y12 + yf y1 f = xx1 + gx x12 gx1

yy1 y12 + yf y1 f + xx1 + gx x12 gx1 = 0

xx1 + yy1 + gx + fy = x12 + y12 + gx1 + fy1

adding gx1 + fy1 + c to both sides

xx1 + yy1 + gx + fy + gx1 + fy1 + c = x12 + y12 + 2 g x1 + 2 fy1 + c


Since (x1, y1) lie on the circle,

= x12 + y12 + 2gx1 + 2 fy1 + c = 0

b g b
xx1 + yy1 + g x + x1 + f y + y1 + c = 0. g
This is the equation of the tangent to the circle.

13.5 LENGTH OF THE TANGENT FROM THE POINT (x1, y1) TO THE
CIRCLE x2 + y2 + 2gx + 2fy + c = 0.
Let C (g, f) be the centre of the circle x2 + y2 + 2gx + 2fy + c = 0. Let P (x1, y1) be any point outside
the circle. Let PT be the tangent drawn to the circle from P. Join CT and CP.
300 Basic Mathematics

Since PT is a tangent, T P (x1, y 1)

CT PT.
CTP is a right angled triangle.
By Pythogoras theorem,
CP2 = CT2 + PT2 C ( g, f)

PT2 = CP2 CT2.


Now CP = Distance between C (g, f) and P (x1, y1)

CP = x1 g a f 2
+ y1 f a f 2 Fig. 13.17

CP = b x + gg + b y + f g
1
2
1
2

CT = Radius = g 2 + f 2 c

LM b x + gg + b y + f g OP 2 2

N Q
2 2
PT 2 = 1 1 g2 + f 2 c

PT = b x + g g + b y + f g d g i
2 2
2
1 1
2
+ f2 c

PT 2 = x12 + g 2 + 2 gx1 + y12 + f 2 + 2 fy1 g 2 f 2 + c

PT 2 = x12 + y12 + 2 gx1 + 2 fy1 + c.

PT = x12 + y12 + 2 gx1 + 2 fy1 + c .

This is the length of the tangent from P (x1, y1) to the circle.

13.6 CONDITION FOR THE LINE y = mx + cTO BE A TANGENT TO


THE CIRCLE x2 + y2 = a2 AND POINT OF CONTACT
The line y = mx + c is a tangent to the circle x2 + y2 = a2 if and only if length of the perpendicular from
the centre to the line is equal to radius.
Now centre of the circle = (0, 0)
Radius = a
Length of perpendicular from (0, 0) to the line y = mx + c = Radius
(i.e. mx y + c = 0)

af af
m 0 0 +c
=a
m2 + 1

c
=a
m2 + 1
Circles 301

c
= a
m2 + 1 y = mx + c

c = a m2 + 1 (0, 0)

Squaring,

d
c2 = a 2 m2 + 1 . i
Fig. 13.18
This is the condition for the line y = mx + c to be a tangent to the
circle x2 + y2 = a2.

Equation of tangent = y = mx a m 2 + 1 ...(1)


Also we know that equation of tangent at (x1, y1) on x2 + y2 = a2 is

xx1 + yy1 = a 2

yy1 = xx1 + a 2 ...(2)


Comparing (1) and (2) we get
y1 x1 a2
= =
1 m a m2 + 1
Taking 1st and 3rd ratio,
a
y1 =
m2 + 1
Taking 2nd and 3rd ratio.
x1 a
=
m m2 + 1
am
x1 = m
m2 + 1
The points of contact are
F a I or F am I
GH am
m2 + 1
,
m2
J GH
+1K m2 + 1
,
m2
a
J
+1K
out of the 2 points of contact, one will satisfy the equation of tangent.

13.7 CONDITION FOR THE LINE lx + my + n = 0 TO BE A TANGENT


TO THE CIRCLE x2 + y2 + 2gx + 2fy + c = 00.
lx + my + n = 0 may be a tangent to the circle iff length of the perpendicular from the centre = Radius.

Centre = (g, f) and radius = g2 + f 2 c


302 Basic Mathematics

a f a f
l g + m f + n
= g2 + f 2 c
lx + my + n = 0
i.e.,
l +m
2 2

lg mf + n (g, f)
= g2 + f 2 c
l +m
2 2

Squaring

an lg mf f 2
= g2 + f 2 c Fig. 13.19
l 2 + m2
an lg mf f = dl
2 2
id
+ m2 g 2 + f 2 c i
This is the required condition.
Note:
The length of the tangent from the point P (x1, y1) to A P (x1, y 1)

the circle x2 + y2 + 2gx + 2fy + c = 0 is given by

x12 + y12 + 2 gx1 + 2 fy1 + c .


If P lie out the circle, then AP is positive or

x12 + y12 + 2 gx1 + 2 fy1 + c > 0

or x12 + y12 + 2gx1 + 2 fy1 + c > 0. Fig. 13.20


If P coincide with A, then AP = 0 and the point P lie on the circle.
x12 + y12 + 2gx1 + 2 fy1 + c = 0.

If P lie inside the circle then

x12 + y12 + 2gx1 + 2 fy1 + c < 0.

WORKED EXAMPLES:

1. Find the equation of tangent to the circle 2 x 2 + 2 y 2 + x 3y 12 = 0 at (1, 3).


Solution: Equation of circle:

2 x 2 + 2 y 2 + x 3y 12 = 0
by 2.
x 3
x 2 + y2 + y6 = 0
2 2
1 3
2g = 2f = c = 6
2 2
Circles 303

1 3
g= f = .
4 4

b x , y g = a1, 3f.
1 1

Equation of tangent

b g b g
xx1 + yy1 + g x + x1 + f y + y1 + c = 0

x a1f + y a3f + a x + 1f + F I a y + 3f 6 = 0
1 3
4 H 4K

1 1 3y 9
x + 3y + x+ 6=0
4 4 4 4
4 x + 12 y + x + 1 3 y 9 24
=0
4
Crossmultiplying
5 x + 9 y 32 = 0.

2. Find the equation of the tangent to the circle x 2 + y 2 + 8 x 6 y 11 = 0 parallel to 4x 3y + 1 = 0.


Solution: Any line parallel to 4x 3y + 1 will have the equation 4x 3y + k = 0.
Now 4x 3y + k = 0 will be a tangent to the circle if the length of the perpendicular from the centre
to the line = Radius.
Now Equation of circle = x 2 + y 2 + 8 x 6 y 11 = 0
Centre = (g, f) = (4, 3)

Radius of x + y + 8 x 6 y 11 = 0 is
2 2
4 2 + 3 2 + 11

= 25 + 11 = 6 units. 4x 3y + 1
Length of perpendicular from (4, 3) to 4x 3y + k = 0 = Radius

a f af
4 4 3 3 + k
=6
4 2 + 32

16 9 + k
=6
5
Fig. 13.21

25 + k = 30 [Note: We get 2 tangents parallel to given line]

25 + k = 30
304 Basic Mathematics

k = 30 + 25
k = 55 or k = 5.
Equations of tangents
= 4 x 3 y + 55 = 0 and 4 x 3 y 5 = 0

3. Find the equation of tangent to the circle x 2 + y 2 2 x 4 y 4 = 0 which are perpendicular to


the line 4x + 3y 7 = 0.
Any line perpendicular to 4x + 3y 7 will have the equation 3x 4y + k = 0.
3x 4y + k = 0 will be a tangent to the circle if the length of the perpendicular from the centre =
Radius.
Now equation of circle = x 2 + y 2 2 x 4 y 4 = 0
Centre = (g, f) = (+1, 2)

a f
4x + 3y 7 = 0
Radius = g 2 + f 2 c = 12 + 2 2 4

= 1 + 4 + 4 = 3 units.
Length of the perpendicular from (1, 2) to 3x 4y + k = 0 (Radius)

i.e.,
af a f
3 1 4 2 +k
=3
32 + 4 2

38+ k Fig. 13.22


= =3
5

= 5 + k = 15 [Note: We get 2 tangents perpendicular


to given line]
5 + k = 15
k = 15 + 5
k = 20 or k = 10.
Equation of tangents are
3 x 4 y + 20 = 0 and 3 x 4 y 10 = 0.

4. Find the value of k if x + y + k = 0 touches the circle x 2 + y 2 7 x 5 y + 18 = 0.


Solution: x + y + k touches the circle if the length of the perpendicular from the centre to the line =
Radius.

Now Equation of circle = x 2 + y 2 7 x 5 y + 18 = 0

a
Centre = g, f = f FH 72 , 25 IK
Circles 305

F 7I + F 5I
2 2
Radius = g 2 + f 2 c =
H 2K H 2K 18

49 25
= + 18
4 4

74 72 2 1
Radius = = = units.
4 4 2

F 7 , 5I
Length of the perpendicular from H 2 2K to x + y + k = 0 = Radius

7 5
+ +k
2 2 1
=
12 + 12 2

12
+k
2 1
=
1 +1
2 2 2

12
+k
2 1
=
2 2

6+ k =1

6 + k = 1
k = 1 6
k = 7 or k = 5
Hence k = 7 or 5.
5. Find the length of the tangent to the circle 3x 2 + 3y 2 7 x 6 y 12 = 0 from (6, 7)
Solution: Equation of the circle
3x 2 + 3y 2 7 x 6 y 12 = 0
by 3.
7
x 2 + y2 x 2 y 4 = 0.
3
306 Basic Mathematics

Length of the tangent from (x1, y1)

= x12 + y12 + 2gx1 + 2 fy1 + c


Length of tangent from (6, 7)

= 6 2 + 7 a f 2

7
3
af a f
6 2 7 4

= 36 + 49 14 + 14 4

= 81 = 9 units.
6. Find whether the origin is inside or on or outside the circle
(i) x 2 + y 2 + 4 x 7 y + 8 = 0

(ii) x 2 + y 2 = 7

(iii) x 2 + y 2 + 2 y + 6 x = 0
Solution: (x1, y1) lie inside the circle

x 2 + y 2 + 2gx + 2 fy + c = 0 if

x12 + y12 + 2 gx1 + 2 fy1 + c < 0

Outside if x12 + y12 + 2 gx1 + 2 fy1 + c > 0 and on the circle if

x12 + y12 + 2 gx1 + 2 fy1 + c = 0

(i) Clearly (0, 0) lie outside the circle x 2 + y 2 + 4 x 7 y + 8 = 0

af af
Q 0+0+4 0 7 0 +8=8> 0

(ii) Clearly (0, 0) lie inside the circle x 2 + y 2 7 = 0 . Since 0 + 0 7 = 7 < 0

(iii) (0, 0) lie on the circle x 2 + y 2 + 2 y + 6 x = 0

af af
Q 0+0+2 0 +6 0 = 0
7. Find the equation of the circle so that the lengths of the tangents from (1, 0), (0, 2) and (2, 1)
are respectively 3, 10 and 3 3.
Solution: Let the equation of circle be

x 2 + y 2 + 2gx + 2 fy + c = 0 ...(1)
Given: length of tangent from (1, 0) to circle is 3 units.

a1f 2
a f af
+ 0 2 + 2 g 1 + 2 f 0 + c = 3
Circles 307

1 2g + c = 3
Squaring
1 2g + c = 9

2g + c = 8 ...(2)
Given: Length of tangent from (0, 2) to the circle = 10 units.

af af
0 + 2 2 + 2 g 0 + 2 f 2 + c = 10
Squaring
4 + 4 f + c = 10

4f +c = 6 ...(3)

Given: Length of the tangent from (2, 1) is 3 3

a2f 2
a f af
+ 12 + 2g 2 + 2 f 1 + c = 3 3
Squaring,

4 + 1 4g + 2 f + c = 3 3 d i 2

5 4 g + 2 f + c = 27
4 g + 2 f + c = 22 ...(4)
Solving (3) and (4)

4f +c = 6
4 g + 2 f + c = 22
a + f a f a f a f
4g + 2 f = 16
by 2

2 g + f = 8 ...(5)
Solving (2) and (3)

2g + c = 8
4f +c= 6
( ) ( ) ( ) ...(6)
2g 4 f = 2
308 Basic Mathematics

Solving (5) and (6)


(5): 2g + f = 8
( 6): 2g 4 f = 2
3 f = 6

f = 2.

Substituting f = 2 in (5)
2 g + 2 = 8
2 g = 8 2

2 g = 10

g = 5

Substituting g = 5, f = 2 in (2)
2g + c = 8

a f
2 5 + c = 8

c = 8 10

c = 2

Substituting g = 5, f = 2 and c = 2 in (1).


Equation of circle

a f af a f
= x 2 + y 2 + 2 5 x + 2 2 y + 2 = 0

x 2 + y 2 10 x + 4 y 2 = 0.
8. Find the equation of the circle passing through the points (1, 2) and (3, 4) and touching the line
3x + y 3 = 0.
Solution: Let the equation of the circle be x2 + y2 + 2gx + 2fy + c = 0 ...(1)
Given: (1) passes through (1, 2)
af af
12 + 2 2 + 2 g 1 + 2 f 2 + c = 0
5 + 2g + 4 f + c = 0
2 g + 4 f + c = 5 ...(2)
Given (1) passes through (3, 4)

af af
32 + 4 2 + 2g 3 + 2 f 4 + c = 0
25 + 6 g + 8 f + c = 0
6 g + 8 f + c = 25 ...(3)
Circles 309

Solving (2) and (3) we get

2g + 4 f + c = 5
6g + 8 f + c = 25
a f a f a f a + f
4g 4 f = 20
by 4

g + f = 5

f = 5 g ...(4)
Given: 3x + y 3 = 0 touches the circle.
Length of the perpendicular from centre (g, f) to 3x + y 3 = 0 (Radius)

a f a f
3 g + f 3
= Radius

3 2 + 12

3g f 3
= a1 + gf + a2 + f f
2 2 LMQ Radius = Distance between centreOP
10 N and any point on circumference Q
Substituting f = 5 g we get
(1, 2)
a
3g 5 g 3 f = a1 + gf + a2 5 gf
2 2
10

a
3g 5 g 3 f = 1 + g2 + 2g + 9 + 6g + g2 C (g, f)
10
Squaring,

FG 2g + 2 IJ 2 Fig. 13.23

H 10 K = 10 + 2 g 2 + 8g

4
10
ag 1f 2
= 2 g 2 + 8g + 10

ag 1f 2
= g2 + 4g + 5
5

g 2 2 g + 1 = 5g 2 + 20 g + 25

4 g 2 + 22 g + 24 = 0
by 2
310 Basic Mathematics

2g 2 + 11g + 12 = 0

2 g 2 + 8g + 3g + 12 = 0 +8g
24g 2
a f a
2g g + 4 + 3 g + 4 = 0 f +3g
+11g
2 g = 3
3 or g = 4
g=
2
when g = 4,
Substituting g = 4 in (4)
f = 5 g

f = 5 4 a f
f = 1.

Substituting g = 4, f = 1 in (2)
2 g + 4 f + c = 5

a f a f
2 4 + 4 1 + c = 5

c = 5 + 12

c=7

Similarly we can get f = 7 2 and c = 12 when g = 3/2.


Equation of required circle

a f a f
x 2 + y 2 + 2 4 x + 2 1 y + 7 = 0

x 2 + y 2 8 x 2 y + 7 = 0 or

LM 3 7 OP
N
x 2 + y 2 3x 7 y + 12 = 0. By putting g = , f = and c = 12
2 2 Q
9. Find the equation of the circle which is concentric with the circle x 2 + y 2 8 x + 12 y 20 = 0
and which touches the line 4x 3y 14 = 0.
Solution: Let the equation of the circle be x2 + y2 + 2gx + 2fy + c = 0 ...(1)
Given (1) is concentric with

x 2 + y 2 8 x + 12 y 20 = 0

Centre of (1) = Centre of x 2 + y 2 8 x + 12 y 20 = 0


Circles 311

ag, f f = a4, 6f
g = 4 and f = 6.
Also given (1) touches 4x 3y 14 = 0
Length of the perpendicular from centre = Radius

af a f
4 4 3 6 14
= g2 + f 2 c
4 +3
2 2

16 + 18 14
= = 42 + 62 c
5
20
= 16 + 36 c
5
Squaring,
4 2 = 52 c
16 = 52 c
c = 52 16 = 36
Equation of the required circle
a f af
x 2 + y 2 + 2 4 x + 2 6 y + 36 = 0
x 2 + y 2 8 x + 12 y + 36 = 0.
10. Find the equation of the circle passing the points (4, 1) and (6, 5) and having its centre on the line
4x + y = 16.
Solution: Let the equation of the circle be
x2 + y2 + 2gx + 2fy + c = 0 ...(1)
Given: (1) passes through (4, 1)

af af
4 2 + 12 + 2 g 4 + 2 f 1 + c = 0

16 + 1 + 8g + 2 f + c = 0
8g + 2 f + c = 17 ...(2)
Given: (1) passes through (6, 5)

af af
6 2 + 52 + 2g 6 + 2 f 5 + c = 0

12g + 10 f + c = 61 ...(3)
Also given (1) has centre on 4x + y = 16 i.e., (g, f) lie on 4x + y = 16.
a f a f
4 g + f = 16
4 g f = 16 ...(4)
312 Basic Mathematics

Solving (2) and (3)

8g + 2 f + c = 17
12 g + 10 f + c = 61
( ) ( ) ( ) (+ ) ...(5)
4 g 8 f = 44

Solving (4) and (5)

4 g f = 16
4g 8 f = 44
( +) ( +) ( )
7 f = 28

f =4

Substituting f = 4 in (4)
a f
4 g 4 = 16

4 g + 4 = 16

4 g = 16 4
4 g = 12

g = 12 4 g = 3

Substituting g = 3 and f = 4 in (2)


a f a f
8 3 + 2 4 + c = 17.

24 8 + c = 17
c = 17 + 32

c = 15

Equation of the required circle

a f a f
= x 2 + y 2 + 2 3 x + 2 4 y + 15 = 0

x 2 + y 2 6 x 8 y + 15 = 0.

REMEMBER:
Distance between 2 points (x1, y1) and (x2, y2)

bx 2 x1 g + by
2
2 y1 . g 2
Circles 313

Slope of the line joining (x1, y1) and (x2, y2)


y2 y1
= .
x 2 x1
2 lines are perpendicular iff product of their slopes = 1.
Equation of circle with centre (0, 0) and radius r units: x2 + y2 = r2.

Equation of circle with centre (h, k) and radius r units: x h a f + a y kf


2 2
= r2.
Equation of circle which is described on line joining (x1, y1) and (x2, y2) as diameter =
b x x gb x x g + b y y gb y y g = 0.
1 2 1 2

General equation of the circle x2 + y2 + 2gx + 2fy + c = 0

Centre = (g, f), Radius = g2 + f 2 c .


Centre is the mid point of diameter.
Fx +x y1 + y2 I
H 2 K
1 2
Co-ordinates of mid point of line joining (x1, y1) and (x2, y2) = ,
2
The point of intersection of 2 diameters of a circle = centre.
If a circle x2 + y2 + 2gx + 2fy + c = 0 touches x-axis, then, radius = f and g2 = c.

If a circle x2 + y2 + 2gx + 2fy + c = 0 touches y-axis, then Radius = g and f 2 = c.


Length of the perpendicular from (x1, y1) to the line ax + by + c = 0 is

ax1 + by1 + c
a 2 + b2
Equation of tangent at (x1, y1) on the circle x2 + y2 + 2gx + 2fy + c = 0 is

b g b
xx1 + yy1 + g x + x1 + f y + y1 + c = 0. g
Length of the tangent from (x1, y1) to the circle x2 + y2 + 2gx + 2fy + c = 0 is

x12 + y12 + 2 gx1 + 2 fy1 + c.


Any line is a tangent to the given circle if the length of the perpendicular from centre to the line
= Radius of the circle.
Condition for the line y = mx + c to be a tangent to the circle x2 + y2 = a2 is c = a m + 1 and
2 2 2
d i
F a I or F am I..
point of contact is GH am
m2 + 1
,
m2
J GH
+ 1K m2 + 1
,
a
m2
J
+1K
Condition for the line lx + my + n = 0 to be a tangent to the circle x2 + y2 + 2gx + 2fy + c = 0 is
an lg mf f = dl
2 2
id
+ m2 g 2 + f 2 c . i
314 Basic Mathematics

The point (x1, y1) lie inside the circle x2 + y2 + 2gx + 2fy + c = 0 if x12 + y12 + 2gx1 + 2 fy1 + c < 0.

Outside the circle if x12 + y12 + 2 gx1 + 2 fy1 + c > 0.

On the circle if x12 + y12 + 2 gx1 + 2 fy1 + c = 0.


Any line parallel to ax + by + c = 0 will have the equation ax + by + k = 0.
[Take x and y co-efficients same].
Any line perpendicular to ax + by + c = 0 will have the equation bx ay + k = 0 [interchange x
and y co-efficients and change the sign of anyone].

EXERCISE

I. Find the equa tion of cir


equation circcle in each of the ffollo
each ollowing:
ollowing:
1. Centre: Origin, Radius: 3 Units.
2. Centre: (0, 2), Radius: 2 Units.
3. Centre: (3, 4), Radius: 5 Units.

F 4 , 7 I , Radius:
4. Centre:
H 3 3K 15 units.

5. Two diameters are x + y = 8 and 2x y = 4 and radius 4 units.


6. Diameter is the line joining (5, 0) and (0, 5).
7. Centre: (3, 4), touching the x-axis.
8. Centre: (5, 1), touching the y-axis.
9. Centre: (4, 4), touching the axes.
10. Centre: (3, 2), passes through the origin.
II. centree and rradius
Find the centr adius of the ffollowing cir
ollowing
ollo circcles:
1. x 2 + y 2 8 x + 4 y + 2 = 0

2. 2 x 2 + 2 y 2 6 x 10 y 15 = 0

3. 3x 2 + 3y 2 6 x 12 y 2 = 0.
III.

1. If one end of a diameter of a circle x 2 + y 2 + 4 x 6 y 14 = 0 is (5, 1), find the other end.
2. Find the equation of the tangent to the circle 2 x 2 + 2 y 2 + x 3y 12 = 0 at (1, 3).

3. Find the equation of tangent at the point (3, 5) on the circle x 2 + y 2 + 4 x + 6 y + 8 = 0.


4. Find the value of k if 3x + 4y = k touches the circle x2 + y2 = 16. Also find the point of contact.
5. Find the equation of tangent to the circle x 2 + y 2 2 x + 6 y 15 = 0 which are parallel to the line
3x + 4y 8 = 0.
Circles 315

6. Find the equation of tangent to the circle x 2 + y 2 2 x 4 y + 1 = 0 which are perpendicular to


the line 3x 4y + 8 = 0.

7. Find the length of the chord 4x 3y = 5 of the circle x 2 + y 2 + 3x y 10 = 0.

8. Find the length of the tangent to the circle 3x 2 + 3y 2 7 x 6 y 12 = 0 from (6, 7).

9. Find whether (1, 2) lie inside or on or outside the circle x 2 + y 2 4 x + 8 y 8 = 0 .


10. Find the equation of the circle so that the lengths of the tangents from (1, 2), (2, 3) and (2, 4)
are respectively 2 3, 30 and 17 units.
11. Find the equation of the circle which passes through (1, 1), (2, 2) and whose radius is unity.
12. Find the equation of circle which has centre on x-axis, passes through (1, 3) and whose radius is
18 units.
13. Find the equation of circle which makes an intercepts 8 and 6 on X and Y-axes and which passes
through the origin.
14. Find the equation of circle passing through the point (1, 2), (4, 3) and whose centre lies on
3x + 4y 7 = 0.

15. Find the equation of circle concentric with x + y 6 x + 8 y + 16 = 0 and touching the line
2 2

5 x 12 y + 15 = 0.
16. Find the equation of circle which touches the y-axis at (0, 9) and cuts x-axis at (3, 0) and
(9, 0).

ANSWERS

I. 1. x 2 + y 2 = 9 2. x 2 + y 2 4 y = 0

3. x 2 + y 2 6 x + 8 y = 0 4. 9 x 2 + 9 y 2 24 x 42 y 70 = 0

5. x 2 + y 2 8 x 8 y + 16 = 0 6. x 2 + y 2 5 x 5 y = 0.

7. x 2 + y 2 6 x 8 y + 9 = 0 8. x 2 + y 2 10 x + 2 y + 1 = 0

9. x 2 + y 2 8 x 8 y + 16 = 0 10. x 2 + y 2 + 6 x 4 y = 0

a f F 3 , 5 I , r = 4 units.
II. 1. C 4, 2 , r = 3 2 units. 2. C
H 2 2K
3. C a1, 2f, r = 17 3 units.

III. 1. a1, 7f 2. 5 x + 9 y 32 = 0
316 Basic Mathematics

LM 12 16 OP
3. x + 2 y + 13 = 0 4. k = 20,
N 5
,
5 Q
5. 3x + 4 y + 34 = 0 and 2 x + 4 y 16 = 0. 6. 4 x + 3y 20 = 0; 4 x + 3y = 0

7. 5 units. 8. 9 units.

9. Inside 10. x 2 + y 2 6 x + 4 y 7 = 0

11. x 2 + y 2 4 x 2 y + 4 = 0 12. x 2 + y 2 + 4 x 14 = 0

13. x 2 + y 2 + 8 x 6 y = 0 14. 15x 2 + 15 y 2 94 x + 18 y + 55 = 0

15. x 2 + y 2 6 x + 8 y 11 = 0 16. x 2 + y 2 + 12 x 18 y + 27 = 0.
14
Parabola
14.1 INTRODUCTION:
When a solid cone is cut by a plane, the curves which lies on the surface of the cone and the plane,
are the curves circle, parabola, ellipse and hyperbola. These curves are called as conic sections.

Definitions:
Let l be a fixed line and S be fixed point. A point P moves
in a plane containing S and l such that its distance from S M P
bears a constant ratio to its distance from the line l, i.e.,
SP
= a constant.
PM S

The locus of the point P is called a conic.


A conic is the locus of the point which moves such that l

the ratio of its distance from a fixed point in the plane to its
distance from a fixed line in a plane is constant. Fig. 14.1

SP
The fixed point is called focus and the fixed line is called directrix and the constant ratio is
PM
called eccentricity of the conic.
If the eccentricity of the conic is less than 1 then the conic is called ellipse.
If the eccentricity is greater than 1 then it is called hyperbola.
If the eccentricity is equal to one then the conic is called parabola.
Circle is regarded as the conic of eccentricity zero. It is a particular case of an ellipse.

14.2 PARABOLA:
Parabola is the locus of point which moves in a plane such that its distance from a fixed point is equal
to its distance from a fixed line. The fixed point is called focus. The fixed line is called directrix.
318 Basic Mathematics

14.3 EQUATION OF THE PARABOLA IN THE STANDARD FORM:


Let S be the focus, l be the directrix.
Draw SZ l.
Let O be the midpoint of SZ. Let SZ = 2a. So that OS = OZ = a.
Choose O as origin the 2 mutually perpendicular lines OX and OY as co-ordinate axes.
Let P (x, y) be any point on the parabola.
Draw PM r to l and PN r X-axis. Join PS.

l y

P
M

x z O N S x

Fig. 14.1

SP
From definition =1
PM
SP = PM ...(1)
Since S is on X-axis and OS = a,
Co-ordinates of S = (a, 0)
SP = Distance between S (a, 0) and P (x, y).

F Distance formula: b x x g + b y y g I
H K
2 2
2 1 2 1

SP = a x af + a y 0f = a x af + y
2 2 2 2
...(2)

Now
PM = NZ = ON + OZ
PM = x + a ...(3)
Substituting (2) and (3) in (1) we get

a x af 2
+ y2 = x + a
Parabola 319

Squaring,

x 2 + a 2 2ax + y 2 = x 2 + a 2 + 2 ax

y 2 = 2ax + 2ax

y 2 = 4 ax.

This is the standard equation of parabola.

THE PARABOLA y2 = 4ax.


For the parabola y2 = 4ax we have the following:
1. Shape:
Since (0, 0) satisfy the equation y2 = 4ax, the curve passes through the origin.
The equation y2 = 4ax remains unchanged if we replace y by y. So the curve is symmetric
about x-axis.
For negative values of x, we get imaginary values for y. So the curve entirely lies on the right
side of y-axis if a is +ve or a > 0. (and if a is ve, the curve entirely lies on the left side of
y-axis).
Hence the shape of the parabola is

x
O S

Fig. 14.2

2. The point O(0, 0) is called vertex of the parabola.


3. The line OX, the +ve x-axis is called the axis of the parabola.
4. S is the focus and its co-ordinate = (a, 0).
5. The line l is the directrix and its equation is x = a.
6. The line LSL which is perpendicular to axis and passing through the focus is called Latus rectum.
L and L are ends of latus rectum. L and L has x-co-ordinate a. To get y-co-ordinate, since L and
L lie on the parabola y2 = 4ax it satisfies the equation.
y 2 = 4 ax
320 Basic Mathematics

af
y 2 = 4a a 3x = a

y 2 = 4 a 2 y = 4a 2

y = 2 a.
Ends of latus rectum = (a, 2a).
Equation of latus rectum: x = a
Length of latus rectum = 2a + 2a = 4a.

14.4 DIFFERENT FORMS OF PARABOLA WITH VERTEX (0, 0):


1. The parabola y2 = 4ax.
Shape:

x S x

l
y

Fig. 14.3

Vertex: (0, 0)
Axis: Negative x-axis.
y
Co-ordinates of focus = (a, 0)
Equation of directrix = x = +a
Ends of latus rectum = (a, 2a)
L
Equation of latus rectum: x = a L
S
Length of latus rectum: 4a.
2. The parabola x2 = 4ay x x
Shape:
Vertex: (0, 0) l
Axis: Positive y-axis.
Co-ordinates of focus = (0, a) y
Equation of directrix = y = a
Fig. 14.4
Parabola 321

Ends of latus rectum = (2a, a) y

Equation of latus rectum = y = a


l
Length of latus rectum = 4a.
3. The parabola x2 = 4ay
S
Shape: x x

Vertex: (0, 0)
Axis: Negative y-axis L L
Co-ordinate of focus = (0, a)
Equation of directrix = y = a
Ends of latus rectum = (2a, a)
Equation of latus rectum = y =a y

Length of latus rectum = 4a Fig. 14.5

14.5 DIFFERENT FORMS OF PARABOLA WITH VERTEX (h, k):

1. a y kf 2
a
= 4a x h f
y

(h, k) s

O x

Fig. 14.6
Let OX and OY be x and y-axis respectively change the origin to the point (h, k) without changing
the direction. Then the new co-ordinates (X, Y) are given by
x = X + h and y = Y + k
X = x h and Y = y k

a y kf 2
a
= 4a x h f
= 4aX Y2
which is the equation of parabola, which opens to the right.

For the parabola y k a f 2


a f
= 4 a x h we have the following
Vertex = (h, k)
Axis: y = k
322 Basic Mathematics

Co-ordinates of focus = (a + h, k)
Equation of directrix = x = a + h.
Ends of Latus rectum = (a + h, 2a + k)
Equation of Latus rectum = x = a + h
Length of Latus rectum = 4a.

2. a y kf 2
a
= 4 a x h f
Shape:

h, k

Fig. 14.7

Parabola which opens to the left in any quadrant depending on value of h and k.
Vertex: (h, k)
Axis: y = k
Co-ordinates of focus = (a + h, k)
Equation of directrix: x = a + h
Ends of latus rectum = ( a + h, 2a +k)
Equation of Latus rectum: x = a + h
Length of Latus rectum = 4a

3. a x hf 2
a
= 4a y k f
Shape:

(h, k)

Fig. 14.8

Parabola which opens upward in any quadrant depending on value of h and k.


Parabola 323

Vertex : (h, k)
Axis: x = h
Focus: (h, a + k)
Ends of latus rectum = (2a + h, a + k)
Equation of latus rectum = y = a + k
Length of latus rectum = 4a

4. a x hf 2
a
= 4 a y k f
Shape:

(h, k)

Fig. 14.8

Parabola which open downward is any quadrant depending on value of h and k.


Vertex : (h, k)
Axis : x = h
Focus = (h, a + k)
Equation of directrix: y = a + k
Ends of Latus rectum: (2a + h, a + k)
Equation of latus rectum: y = a + k
Length of latus rectum: 4a.
Note: In any parabola, the focus is inside the curve (i.e. on the axis of the parabola) and directrix is
away from the curve (i.e. directrix never meet the parabola).
2. The distance between the vertex and the focus is equal to the distance between the vertex and
the directrix is equal to a.
3. The distance of a point on the parabola from the focus is called the focal distance of the point.
4. A chord drawn through the focus is called focal chord and the focal chord perpendicular to the
axis is Latus rectum.

WORKED EXAMPLES:
I. Find the vertex, axis, focus, equation of directrix, Ends of latus rectum, Length of latus
rectum, equation of latus rectum of the following parabolas.
1. x2 = 8y
324 Basic Mathematics

Comparing with x2 = 4ay we get


4a = 8 a = 2.
Vertex = (0, 0)
Axis: y-axis or x = 0.
Focus: (0, a) = (0, 2)
Equation of directrix: y = a
y = 2
Ends of latus rectum = (2a, a)
= (2(2), 2)
= (4, 2).
Length of latus rectum = 4a
= 4 (2) = 8.
Equation of latus rectum = y = a
y = 2.
2. y = 6x.
2

Comparing with y2 = 4ax we get


4a = 6
6 3
a= = .
4 2
Vertex: (0, 0)
Axis: x-axis or y = 0

F
3 I
H
Focus = (a, 0) = , 0 .
2 K
Equation of directrix = x = a
3
x= .
2
Ends of Latus rectum: (a, 2a)

FG
3 3 F I IJ
H
= , 2
2 2 H KK
= F , 3I
3
H 2 K
Equation of latus rectum x = a
3
x= .
2
Parabola 325

Length of latus rectum = 4a

F 3 I = 6.
=4
H 2K
3. x2 = y
Comparing with x2 = 4ay we get
1
4a = 1 a =
4
Vertex = (0, 0)
Axis: x-axis or x = 0.

F 1 I
H
Focus = (0, a) = 0,
4 K
Equation of directrix : y = a
1
y= .
4

a
Ends of latus rectum = 2a, a f
F F 1 I 1 I F 1 1 I
= GH 2 H K , JK = H , K
4 4 2 4
Equation of latus rectum: y = a
1
y=
4
Length of latus rectum = 4a

F 1 I = 1.
=4
H 4K
4. y2 = 4 (x + 1)

a
y0 f 2
b a fg
= 4 x 1
Comparing with (y k)2 = 4a (x h)
We get (h, k) = (1, 0)
4a = 4 a = 1
Vertex = (h, k) = (1, 0)
Axis: y = k
y = 0.
Focus: (a + h, k) = (1 + (1), 0)
= (0, 0).
326 Basic Mathematics

Equation of directrix x = a + h
x = 1 1
x = 2
Ends of latus rectum = (a + h, 2a + k)
= (1 + (1), 2 (1) + 0)
= (0, 2).
Equation of latus rectum = x = a + h
x = 1 + (1)
x = 0.
Length of latus rectum = 4a
= 4 (1) = 4.

5. a y + 2f 2
= 3x + 1

a y + 2f F 1 I
H K
2
=3 x+
3

a f LM F 1I OP
N H 3K Q
2
y 2 =3 x

Comparing with (y k)2 = 4a (x h)


We get

ah, kf = FH 13 , 2IK
3
4a = 3 a =
4

a f FH
1
Vertex = h, k = , 2
3
I
K
Axis: y = k, y = 2
Focus: (a + h, k)

F 3 1 , 2I
=
H4 3 K
=F
94 I F5 I
H 12 , 2K = H 12 , 2K
Equation of directrix: x = a + h
3 1
x= .
4 3
Parabola 327

13
x= .
12
Ends of latus rectum = (a + h, 2a + k)

FG 3 1 , 2 F 3 I 2IJ
=
H 4 3 H 4K K
F 5 , 3 2I = F 5 , 7 I and F 5 , 1 I .
H 12 2 K H 12 2 K H 12 2 K
Equation of Latus rectum x = a + h
3 1 5
x= = .
4 3 12

F 3 I = 3.
Length of latus rectum = 4a = 4
H 4K
6. a x + 3f = 24 a y 1f
2

b x a 3fg = 24 a y 1f
2

Comparing with (x h)2 = 4a (y k)


We get (h, k) = (3, 1)
4a = 24 a = 6.
Vertex = (h, k) = (3, 1)
Axis: x = h, x = 3.
Focus = (h, a + k) = (3, 6 + 1)
= (3, 5)
Equation of directrix: y = a + k
y=6+1=7
Ends of latus rectum = (2a + h, a + k)

b af a f
= 2 6 + 3 , 6 + 1 g
= a 12 3, 5f

= a 15, 5f and a9, 5f.


Equation of latus rectum: y = a + k
y = 6 + 1 = 5
Length of latus rectum = 4a
= 4 (6) = 24.
328 Basic Mathematics

7. x 2 + 8 x + 12 y + 4 = 0
Consider

x 2 + 8 x + 12 y + 4 = 0 LMCo - efficient of x 8 OP
+ 12 y + 4 = 0 M = 4 Square it, Add & Subtract P
8
x 2 + 8x + 4 2 4 2
N2 Q
a x + 4f 2
16 + 12 y + 4 = 0
Comparing with (x h)2 = 4a (y k)
We get
ah, kf = a4, 1f
4 a = 12 a = 3.
a f a f
Vertex: h, k = 4, 1

Axis: x = h, x = 4.

a f a
Focus: h, a + k = 4, 3 + 1 = 4, 2 f a f
Equation of directrix: y = a + k
y=3+1=4
Ends of latus rectum: (2a + h, a + k)

b af a f
= 2 3 + 4 , 3 + 1 g
= a 6 4, 2f

= a 10, 2f & a2, 2f.


Equation of latus rectum, y = a + k
y = 3 + 1
y = 2
Length of latus rectum = 4a
= 4 (3) = 12.
8. y 2 4 y 6 x + 13 = 0
Consider

y 2 4 y 6 x + 13 = 0

y 2 4 y + 2 2 2 2 6 x + 13 = 0

a y 2f 2
4 6 x + 13 = 0
Parabola 329

a y 2f 6 x + 9 = 0
2

a y 2f = 6 x 9.
2

a y 2f = 6 FH x 96 IK
2

a y 2f = 6 FH x 23 IK
2

Comparing this with (y k)2 = 4a (x h)

We get ah, kf = FH 32 , 2IK


6 3
4a = 6 a = =
4 2

a f FH 32 , 2IK
Vertex: h, k =

Axis: y = k , y = 2.

a
Focus: a + h, k =f FH 23 + 32 , 2IK
= F , 2I = a3, 2f.
6
H2 K
Equation of directrix: x = a + h
3 3
x= +
2 2
x = 0.
Ends of Latus rectum = (a + h, 2a + k)

F 3 + 3 , 2 3 + 2I
=
H2 2 2 K
a f a f a f
= 3, 3 + 2 = 3, 5 & 3, 1 .

Equation of latus rectum: x = a + h


3 3 6
x= + = = 3.
2 2 2
330 Basic Mathematics

Length of latus rectum: 4a

F 3 I = 6.
=4
H 2K
9. 3 y 2 + 6 y + 8x 5 = 0.
Consider
3 y 2 + 6 y + 8 x 5 = 0.

d i
3 y2 + 2 y + 8x 5 = 0

3 y 2 + 2 y + 12 12 + 8x 5 = 0

a f 2
3 y + 1 1 + 8x 5 = 0

3 a y + 1f 3 + 8 x 5 = 0
2

3 a y + 1f + 8 x 8 = 0
2

3 a y + 1f = 8 x + 8
2

3 a y + 1f = 8 a x 1f
2

a y + 1f = 83 a x 1f
2

b y a1fg 2
=
8
3
x 1

8 2
Comparing with (y k)2 = 4a (x h) we get (h, k) = (1, 1), 4a = a=
3 3
Vertex = (h, k) = (1, 1)
Axis: y = k, y = 1

F 2 + 1, 1I
Focus: (a + h, k) =
H 3 K
= F , 1I
1
H3 K
Equation of directrix : x = a + h
2
x= +1
3
5
x= .
3
Parabola 331

Ends of latus rectum:


aa + h, 2a + kf
F 2 I
= G + 1, 2 F I 1J
2
H 3 H 3K K
= F , 1I = F , I and F , I .
1 4 1 7 1 1
H 3 3 K H 3 3 K H 3 3K
Equation of latus rectum
x = a + h ,
2 1
x= +1= .
3 3

F 2I = 8 .
Length of latus rectum: 4
H 3K 3
10. 2 x 2 5 x + 3y + 4 = 0

Consider 2 x 2 5 x + 3y + 4 = 0

F 5 I
H
2 x2
2 K
x + 3y + 4 = 0

LM 5 5 F I F 5 I OP + 3y + 4 = 0
2 2
2 x2
MN 2
x+
4 H K H 4 K PQ
L 5 25 O
2 MF x I P + 3y + 4 = 0
2

MNH 4 K 16 PQ
2F x I
2
5 25
H 4 K 8 + 3y + 4 = 0
2 F x I + 3y + = 0
2
5 7
H 4K 8

2 LM x OP = 3y
2
5 7
N Q 4 8

2 LM x OP = 3 LM y + OP
2
5 7
N 4 Q N 24 Q
332 Basic Mathematics

F x 5I 2
3FG F 7 I IJ
H 4K =
2 H H
y
24 KK
Comparing with (x h)2 = 4a (y k) we get

ah, kf = FH 45 , 247 IK and 4a = 23 , a = 83 .


F5, 7 I
Vertex = (h, k) = H 4 24 K
Axis: x = h

5
x=
4
Focus: (h, a + k)

F 5, 3 7 I
=
H 4 8 24 K
=F ,
5 9 7 I
H 4 24 K
= F , I = F , I.
5 16 5 2
H 4 24 K H 4 3 K
Equation of directrix:
y=a+k
3 7
y=
8 24
97 2 1
y= = = .
24 24 12
Ends of latus rectum

a2a + h, a + k f = FGH 2 FH 83IK + 45 , 83 247 IJK

F 3 5 16 I
H
= + ,
4 4 24
.
K
F 2, 2 I and F 1 , 2 I .
H 3K H 2 3K
Parabola 333

Equation of latus rectum,


3 7
y = a + h =
8 24
2
y= .
3
Length of latus rectum = 4a

F 3I = 3 .
=4
H 8K 2
II. Find the Equation of Parabola given that:
1. Vertex: (0, 0), Focus: (5, 0)
Solution: Since the focus lies inside the parabola
and vertex is given as origin. The parabola opens to
the right. (5, 0)

Equation of the parabola with vertex (0, 0) and


which opens to right = y2 = 4ax
Co-ordinates of focus = (a, 0) = (5, 0) [given]
Fig. 14.9
a = 5.
Equation of the parabola = y2 = 4 (5) x
(0, 4)
y2 = 20x
2. Vertex (0, 0), Focus (0, 4)
Solution: Since focus = (0, 4) and vertex: (0, 0) and
focus lies inside the parabola, the parabola opens
upward.
Equation of parabola: x2 = 4ay
Given focus = (0, 4) = (0, a) Fig. 14.10
a=4
Equation of parabola: x2 = 4 (4) y
x2 = 16y
3. Vertex: (0, 0), Axis = Negative Y-axis and
length of latus rectum = 5.
Since axis = Negative Y-axis, and vertex = (0, 0)
parabola opens downward as shown in fig.
Equation of parabola: x2 = 4ay
Given length of latus rectum = 5
4a = 5
Fig. 14.11
334 Basic Mathematics

Equation of required parabola. Y l


x2 = 5y.
4. Vertex (0, 0), Directrix x = 4.
Since directrix x = 4, and directrix is away from the pa- x
rabola and vertex = (0, 0), Parabola opens to the left.
Equation of parabola: y2 = 4ax. x=4

For y2 = 4ax parabola, Equation of directrix is


x = a. Fig. 14.12
a = 4 (3 x = 4 is given)

af
Equation of the parabola: y 2 = 4 4 x y 2 = 16 x
5. Vertex (1, 2), Focus (1, 1) Y
Since vertex = (1, 2) and focus = (1, 1) and focus lie
inside the parabola, the parabola open upwards. Equation of
a
parabola = x h f 2
a
= 4a y k f
Given vertex (1, 2) x
0
x

(h, k) = (1, 2)
S
h = 1, k = 2.
V
Focus = (h, a + k) = (1, 1) Y
a + k = 1
a + (2) = 1 Fig. 14.13

a = 1 + 2
a=1

a = +1.

Equation of parabola

a x 1f = 4 a1f b y a2fg
2

a x 1f = 4 a y + 2f
2

x2 + 1 2x = 4y + 8

x 2 2 x 4 y 7 = 0.

Alieter:
Given: Vertex = (1, 2) = (h, k)
h = 1, k = 2
Parabola 335

We know,
Distance between vertex and focus = a
i.e. distance between (1, 2) and (1, 1) = a

a1 1f + a1 + 2f
2 2
=a

12 = a a = 1.

Equation of parabola

a x hf = 4a a y k f
2

a x 1f = 4 a1fa y + 2f
2

x2 2x 4y 7 = 0 .
6. Focus (1, 1), directrix x + 8 = 0.
Let S (1, 1) be the focus and l: x + 8 = 0 be the directrix.
M P
Let P (x, y) be any point on the parabola. Join SP, draw
PM l. We know from definition,
l
SP
=1
PM S

SP = PM
SP = Distance between S (1, 1) and P (x, y)

= a x 1f + a y 1f
2 2
Fig. 14.14
[Using distance formula]
PM = perpendicular distance of P (x, y) from x + 8 = 0

x+8
=
12 + 0

a x 1f + a y 1f
2 2
=
x+8
1
Squaring,

a x 1f + a y 1f = a x +18f
2
2 2

x 2 + 1 2 x + y 2 + 1 2 y = x 2 + 64 + 16 x

y 2 2 y 18x 62 = 0.
336 Basic Mathematics

7. Focus (4, 0) and directrix x = 6.


Let P (x, y) be any point on the parabola whose focus is S (4, 0) and directrix is x 6 = 0. Join, SP and
draw PM l.
From definition
SP
=1
PM
P (x, y)
SP = PM M

a f a f
SP = Distance between S 4, 0 and P x, y

= a x 4f + a y 0f = a x 4f + y
2 2 2 2
x6=0
S (4, 0)

PM = perpendicular distance of P (x, y) from x 6 = 0.


l
x6 x6
= = = x 6.
12 1
Fig. 14.15

a x 4f 2
+ y2 = x 6
Squaring,
a x 4f 2
a
+ y2 = x 6 f 2

x 2 + 16 8 x + y 2 = x 2 + 36 12 x

y 2 + 4 x 20 = 0.
8. Ends of latus rectum (5, 2) and (3, 2)

(5, 2) (3, 2)
S
L L

x
x

Fig. 14.16

Since the ends of latus rectum are (5, 2) and (3, 2). There are 2 parabolas, open upwards and open
downwards.
So equations are

a x hf 2
a
= 4 a y k . f
Parabola 337

To find h, k and a:
Length of latus rectum = 4a i.e., Distance between (3, 2) and (5, 2) = 4a.

b5 a3fg + a2 2f = 4a
2 2

a 5 + 3f + 0 = 4a
2

a2f = 4a 2

4 = 4a

1
a= .
2
Focus = Mid point of latus rectum
Focus = mid point of (3, 2) and (5, 2)

FG 3 + a5f , 2 + 2 IJ
Focus =
H 2 2 K

= F , I = a 4, 2f.
8 4
H 2 2K
But Focus = (h, a + k)
(h, a + h) = (4, 2)
h = 4, a + k = 2
1
+k=2
2
1
k =2 .
2
5 3
k= or k =
2 2
(for ve a) (for +ve a)
Equations of parabolas:

a f F 1 I F y 3I
H 2K H 2K
2
(i) x 4 =4

a x + 4f = 2 FH y 23 IK
2

x 2 + 16 + 8 x = 2 y 3

x 2 + 8 x 2 y + 19 = 0 .
338 Basic Mathematics

a f F 1 I F y 5I
H 2 K H 2K
2
(ii) x 4 = 4

a x + 4f = 2 FH y 25 IK
2

a x + 4f = 2 y + 5
2

x 2 + 8 x + 16 + 2 y 5 = 0

x 2 + 8 x + 2 y + 11 = 0
9. Vertex: (3, 4) Directrix y = 1
Given: Vertex = (3, 4)
(h, k) = (3, 4) h = 3 and k = 4.
(3, 4)
Since directrix is away from the parabola and per- y=1

pendicular to axis, the parabola opens upwards.


Equation of parabola is (x h)2 = 4a (y k)
Equation of directrix: y = a + k
3 y = 1 is directrix.
1 = a + k
1 = a + 4 Fig. 14.17
a = 4 1

a = 3.

a
Equation of parabola: x 3 f 2
a
= 4.3 y 4 f
x 2 + 9 6 x = 12 y 48
x 2 6 x 12 y + 57 = 0.
10. Directrix: x + 3 = 0, axis y = 2 and length of latus rectum: 6.
Given: Axis y = 2, and directrix x + 3 = 0, x = 3.
x=3
From figure,
l
Since directrix is away from parabola, the parabola axis
opens to the right. y=2
Equation of parabola.

a y kf 2
a
= 4a x h f
Axis: y = k , 3 y = 2.

k = 2. Fig. 14.18
Parabola 339

Length of latus rectum = 4a


4a = 6 (Given)
6
a=
4

3
a= .
2
Directrix: x = a + h
3
x= +h
2
3
3 = +h
2
3
3 + =h
2
6 + 3
=h
2

3
h= .
2
Equation of parabola:

a y 2f F 3 I FG x F 3 I IJ
H 2K H H 2KK
=4
2

a y 2f = 6 FH x + 23 IK
2

y2 + 4 4y = 6x + 9

y2 4y 6x 5 = 0
REMEMBER:

340
Basic Mathematics
Equation Figure Vertex Focus Equation Axis Ends Equation Length
of of of of
directrix Latus rectum Latus rectum Latus rectum

y2 = 4ax (0, 0) (a, 0) x = a x-axis (a, 2a) x=a 4a


y=0

y2 = 4ax (0, 0) (a, 0) x=a x-axis (a, 2a) x = a 4a


y=0

x2 = 4ay (0, 0) (0, a) y = a y-axis ( 2a, a) y=a 4a


x=0

x2 = 4ay (0, 0) (0, a) y=a y-axis ( 2a, a) y = a 4a


x=0
(h, k)

(y k)2 = 4a (x h) (h, k) (a + h, k) x = a + h y=k (a + h, 2a + k) x=a+h 4a

(h, k)

(y k)2 = 4a (x h) (h, k) (a + h, k) x=a+h y=k (a + h, 2a + k) x = a + h 4a

(x h)2 = 4a (y k) (h, k) (h, k) (h, a + k) y = a + k x=h ( 2a + h, a + k) y=a+k 4a

(x h)2 = 4a (y k) (h, k) (h, k) (h, a + k) y=a+k x=h ( 2a + h, a + k) y = a + k 4a


Parabola 341

In any parabola, focus is inside the curve and directrix is away from the parabola.
Distance between vertex and focus = a.
For the given ends of latus rectum, there are 2 possible parabolas.
Focus is the mid point of latus rectum.
Axis is perpendicular to the directrix. Distance between directrix and vertex = a.

EXERCISE

I. Find the vver


erte
ertex, F
tex, ocus, equa
Focus, tion of dir
equation ectr
directrix, axis, length of la
ectrix, tus rrectum,
latus ectum, ends of la tus
latus
rectum and equa tion of la
equation tus rrectum
latus ectum of the ffollo
ollowing par
ollowing abolas:
para
1. y 2 = 4 x 2. y 2 = 8 x

3. x 2 = 3 y 4. x 2 = 7 y

5. a y 1f 2
= 4x 6. a x + 1f
2
a f
= 6 y +1

7. x 2 = 12 y 7 a f 8. a y + 3f 2
= 8 a x 7f

9. y 2 20 x 10 y + 15 = 0 10. y 2 4 y + 6 x 8 = 0

11. 2 y 2 5 y + 3x + 4 = 0 12. x 2 + 4 x + 10 y + 8 = 0

13. x 2 + 4 y 2 x + 3 = 0 14. x 2 8 x + 16 y = 0

15. x 2 4 x 5y 1 = 0
II. Find the equa tion of par
equation paraabola ggiiven that:
that:
1. Vertex: (0, 0), Focus (3, 0)
2. Vertex: (0, 0), Directrix : y = 8
3. Vertex : (0, 0), Length of Latus rectum = 3, axis positive x-axis.
4. Vertex: (0, 0), Equation of latus rectum x = 4.
5. Vertex : (0, 0), Axis x-axis and passing through (3, 1)
6. Focus (0, 5) and directrix y = 4
7. Focus (4, 0), Directrix x = 6
8. Vertex: (1, 1), Focus (4, 1)
9. Ends of latus rectum (3, 1) and (1, 1)
10. Directrix x + 2 = 0, axis y = 3 and length of latus rectum = 6.
11. Vertex: (3, 4), Directrix y = 1
12. Vertex: (3, 3), Directrix y = 7 and axis parallel to y-axis.
13. Directrix 4x 1 = 0, Axis y = 2 and length of latus rectum 8.
14. Ends of latus rectum (8, 4) and (8, 2)
15. Directrix: 4x + 1 = 0, axis 2y + 1 = 0 and length of latus rectum 24.
342 Basic Mathematics

ANSWERS
I.

Question Vertex Focus Directrix Axis Length of Ends of Equation of


No. Latus rectum Latus rectum Latus rectum

1 (0, 0) (1, 0) x = 1 x-axis y = 0 4 (1, 2) x=1


2 (0, 0) (2, 0) x=2 x-axis y = 0 8 (2, 4) x = 2
3 (0, 0) (0, 3/4) y = 3/4 y-axis x = 0 3 (3/2, 3/4) y = 3/4
4 (0, 0) (0, 7/4) y = 7/4 y-axis x = 0 7 (7/2, 7/4) y = 7/4
5 (0, 1) (1, 1) x = 1 y=1 4 (1, 2 + 1) x=1
6 (1, 1) (1, 1/2) y = 5/2 x = 1 6 (13, 1/2) y = 1/2
7 (0, 7) (0, 10) y=4 x=0 12 (8, 10)
8 (7, 3) (5, 3) x = 5 y = 3 8 (5, 4 3) x=5
9 (1/2, 5) (9/2, 5) x = 11/2 y=5 20 (9/2, 25) x = 9/2
(9/2, 15)
10 (2, 2) (1/2, 2) x = 7/2 y =2 6 (1/2, 5) x = 1/2
(1/2, 1)
11 (7/24, 5/4) (2/3, 5/4) x = 1/12 y = 5/4 3/2 (2/3, 2) x = 2/3
(2/3, 1/2)
12 (2, 2/5) (2, 29/10) y = 21/10 x=2 10 (3, 29/10) y = 29/10
(7, 29/10)
13 (1, 1/2) (1, 3/2) y = 1/2 x=1 4 (3, 3/2) y = 3/2
(1, 3/2)
14 (4, 1) (4, 3) y=5 x=4 16 (12, 1) y=1
(4, 1)
15 (2, 1) (2, 1/4) y = 9/4 x=2 5 (9/2, 1/4) y = 1/4
(1/2, 1/4)

II.

1. y 2 = 12 x 2. x 2 = 24 y 3. y 2 = 3x

F 1 I
H K
1
5. y = 6. x = 18 y
2 2
4. y 2 = 16 x x
3 2

7. y 2 = 4 x 5 a f 8. a y 1f 2
a f
= 12 x 1 9. a x + 1f 2
a
= 4 y 2 f
10. a y 3f = 6 FH x + 12 IK
2
11. a x 3f 2
af
= 12 y 4 12. a x 3f 2
a f
= 16 y + 3

13. a y 2f = 8 FH x + 74 IK
2
14. a y 1f = 6 FH x 132IK
2
15. a y 2f = 8 FH x + 74 IK
2
15
Limits and Continuity
15.1 INTRODUCTION:
The discovery of calculus was done independently and almost during the same time by Sir Isaac
Newton and Gottfried Wilhelm Leibnitz. Calculus is developed on the basis of a more fundamental
concept called the Limit.

15.2 CONSTANTS AND VARIABLES:


A quantity which remains the same throughout any mathematical discussion is called a constant.
A quantity which takes different values in any mathematical discussion is called a variable. For
instance, if we increase the production by using more raw materials, the cost of the machine doesnt
change, but the costs of raw materials, Labour sales change. So cost of the machine which remains the
same value throughout is constant and cost of raw materials and labour which assumes any numerical
value out of given set of values are variables.
Constants are represented by a, b, c, d, e and variables are represented by u, v, w, x, y, z.

15.3 FUNCTION:
Let A and B be 2 non empty sets. A rule f which associates each element x of A to an unique element
y of B is called a function. It is denoted by f : A B. If x A is related to y B, then we write y = f (x).

Algebraic function:
A function y = f(x) is said to be an algebraic function if f(x) is a polynomial function (eg: x2 + 3x 1)
F 3x + 8 I
or rational function G
2

H 4 x 1 JK
3 or irrational function (e.g.: (1 x)2/3).

Note: The functions other than algebraic functions are called transcendental functions. They include,
trigonometric functions, logarithmic function, inverse trigonometric functions, exponential func-
tions, hyperbolic functions etc.
344 Basic Mathematics

Exponential function:
The function which associates every real number x to the real number ex is called exponential function.
Where e is the sum of the infinite series
1 1 1 1
1+ + + + + ...
1! 2 ! 3! 4 !
Its value is 2.7182818284.... e is an irrational number.

Logarithmic function:
The function which associates every positive real number x to the real number loge x is called the
logarithmic function.

15.4 LIMITS:
Consider the function

x2 1
y=
x 1

12 1 0
when x = 1, y = = which is not defined.
11 0
Instead of giving x = 1, Let us give, x a value which is slightly less than 1 or slightly greater than 1.
Then
when x = 0.9, y = 1.9
x = 0.99, y = 1.99
x = 0.999, y = 1.999

when x = 1.0001, y = 2.0001


x = 1.001, y = 2.001
x = 1.01, y = 2.01
x = 1.1, y = 2.1
From the above set of values, we can observe that the value of y is slightly less than 2 or slightly
more than 2 when the value of x is slightly less than 1 or slightly more than 1. But when x = 1, y has
no value. In otherwords, when x is very nearly equal to 1, y is very nearly equal to 2. In symbols this
x2 1
is expressed as lim =2
x 1 x 1

i.e. lim y = 2
x 1

Read it as the limit of y as x tends to 1 is 2.


The statement simply imply when x 1 y 2 even though it doesnt exist when x = 1.
Limits and Continuity 345

Definition of Limit:
A function f(x) is said to tend to a limit l as x tends to a if the numerical difference between f(x) and
l can be made as small as we please by taking the numerical difference between x and a as very small.
In other words f(x) is very nearly equal to l when x is very nearly equal to a.
af
It is denoted by lim f x = l or
xa

af
lt f x = l.
x a

Left Hand and Right Hand Limits:


When x a, through the values which are smaller than a then we write x a 0 this
af a f
lim f x or lim f a h is called Left Hand Limit [LHL] of a function f (x).
x a 0 h 0

For example: if x 3 by taking the values 2.9, 2.99, 2.999, 2.9999... then x 3 0 or Simply
x 3
When x a, through the values which are greater than a then we write x a + 0 this lim f x
x a +0
af
a f
or lim f a + h is called Right Hand Limit [RHL] of a function f(x).
h0

For example: if x 3 by taking the values 3.1, 3.01, 3.001... then x 3 + 0 or Simply x 3+

15.5 STANDARD LIMITS:

x n an
1. Prove that lim = na n 1 , where n is any rational number.
x a xa
Proof. Case (i): Let n be a positive integer. We know

a fd
x n a n = x a x n 1 + x n 2 a + x n 3 a 2 + ... + a n 1 i
lim
x n an
= lim
a fd
x a x n 1 + x n 2 a + x n 3 a 2 + ... + a n 1 i
xa x a x a xa

= lim x n 1 + x n 2 a + x n 3 a 2 + ... + a n 1 (n terms)


x a

= a n 1 + a n 2 a + a n 3 a 2 + ... + a n 1

= a n 1 + a n 1 + a n 1 + ... (n terms)

x n an
lim = na n 1
x a x a

Case (ii): Let n be a negative integer, say n = m, where m is a +ve integer.


346 Basic Mathematics

Consider

x n an x m a m
lim = lim
x a x a x a xa

1 1
m m 1
= lim x a As a m =
x a xa am

am xm
m m
= lim x a
x a xa

am xm
= lim
xa a
x m am x a f
= lim
LM x a OP
m m
1 LMmUsing case (i) as O
xa
N xa Q x m
am N is + ve integerQP
x n an 1
lim = ma m 1 m m
x a x a a a

ma m 1
=
a2m

= ma m 1 2 m

= ma m 1
n = m

x n an
lim = na n 1
x a x a

Case (iii) Let be a fraction, positive or negative.


p
Take n = where p is an integer and q 0.
q
Now

x n an xp q ap q
lim = lim
x a x a xa xa
Taking x = yq and a = bq
x1/q = y and a1/q = b
Limits and Continuity 347

So that

x a yq bq

y b.

xp q ap q yp bp
lim = lim q
x a xa yb y b q

Dividing both numerator and denominator by y b

yp bp
lim q
yb
=
pb p 1 LM lim
x n an
= na n 1
OP
x a y b q qb q 1
yb
N x =a xa Q
p P1 q +1
b
q

p p q
b
q
1
But q
b=a

x p q ap q p 1 q
lim
x a xa
= a
q
d i p q

p
x p q a p q p q 1
lim = a
x a xa q

x n an
lim = na n 1 for all rational values of n.
x a x a

Some Standard Limits:


Here we state some standard limits (without proof) and use them while solving problems.

F 1 I n
a f 1
1. (a) lim 1 +
n H n K =e (b) lim 1 + n
n 0
n =e

F k I n
= ek
2.
n H
lim 1 +
n K
ex 1
3. lim =1
x0 x
348 Basic Mathematics

ax 1
4. lim = log e a where a > 0.
x 0 x

Algebra of Limits:
If f(x) and g(x) are 2 functions of x and k is any scalar
Then

af af
1. lim f x g x = lim f x lim g x
xa x a
af x a
af
2. lim k f a x f = k lim f a x f
xa xa

3. lim f a x f g a x f = lim f a x f lim g a x f


x a xa x a

f a gf lim f a x f
4. lim = x a
provided lim g a x f 0 .
xa g a x f lim g a x f xa
xa

Indeterminate Forms:

0 0
In mathematics the forms like , , 0 , 1 ... are called indeterminate forms.
0

x2 1 0 12 1 0
Ex: 1. If y = then the value of y = when x = 1 y = =
x 1 0 11 0

a f
2. If y = 1 + n
1
n then value of y = 1 + 0 a f 1
0 y = 1 when n = 0.

Evaluation of Limits:

af
To evaluate lim f x , first find f(a).
x a

(i) If f (a) an indeterminate form then f (a) itself is the limit.


(ii) If f (a) = an indeterminate form then suitable method / formula is applied to reduce f (x) so that
it will not take an indeterminate form when x is replaced by a.

WORKED EXAMPLES:

4 x 2 + 3x + 1
1. Evaluate: lim
x0 3x 2 4 x 1
Solution: By Putting x = 0. We get

lim
af af
4 x 2 + 3x + 1 4 0 + 3 0 + 1 1
= = = 1 an intermediate.
x 0 af af
3 x 2 4 x 1 3 0 4 0 1 1
Limits and Continuity 349

4 x 2 + 3x + 1
Hence lim = 1
x 0 3x 2 4 x 1

x 2 5x + 4
2. Evaluate: lim
x 1 x 2 4x + 3

af
12 5 1 + 4 0
= = an indeterminate
Solution: By putting x = 1 we get
af
12 4 1 + 3 0
Hence consider

x 2 5x + 4
lim by factorising both numerator and denominator we get
x 1 x 2 4x + 3

lim
x 2 4 x 1x + 4
= lim
a f a f
x x 4 1 x 4
x 1 2
a f a f
x 3 x 1x + 3 x1 x x 3 1 x 3

= lim
a x 1fa x 4f = lim x 4
x 1 a x 1fa x 3f x3x 1

Now substituting x = 1 or applying lim we get


x1

1 4 3 3
= = .
1 3 2 2

x 2 5x + 4 3
lim = .
x 1 x2 4x + 3 2

3+ x 3 x
3. Evaluate: lim
x 0 x
Solution: By putting x = 0 we get

3+0 30 3 3 0
= = = an indeterminate
x 0 0
Consider
3+ x 3 x
lim By rationalising the numerator,
x 0 x

3+ x 3 x 3+ x + 3 x
lim
x 0 x 3+ x + 3 x

lim
d 3+ x i d
2
3 xi 2

x 0 x d 3+ x + 3 xi
350 Basic Mathematics

3+ x 3+ x
= lim
x 0 x d 3+ x + 3 x i
2x
= lim
x0 x d 3+ x + 3 x i
2
= lim
x 0 3+ x + 3 x
Now applying lim ,
x0

2 2
=
3+ 0 + 3 0 3+ 3

2 1
= =
2 3 3

3+ x 3 x 1
lim = ,
x0 x 3

x 5 243
4. Evaluate: lim
x 3 x 3

x 5 243
= lim
x 5 35 LMFormula: lim x a
n n
= na n 1
OP
Solution: lim
x 3 x 3 x 3 x 3 N xa xa Q
= 5 351 = 5 34 = 5 81 = 405a f
x 5 243
lim = 405
x 3 x 3

3
x 3 5
5. Evaluate: lim
x 5 x 5
Consider

x 1 3 51 3
lim
x 5 x 1 2 51 2

Dividing both numerator and denominator by x 5.

x 1 3 51 3 x 1 3 51 3
lim
lim x5 = x 5 1 x2 51 2 [using algebra of limits]
x 5 x 1 2 51 2 x 5
lim
x5 x5 x5
Limits and Continuity 351

Now applying the formula,

x n an
lim = na n 1 for numerator and denominator separately we get
x a xa

1 2
1 3 1 1 3
5 5
3 = 3
1 1
1 2 1 1 2
5 5
2 2

2 1
2 3+2
= 5
3
4 + 3 1
2 2 2
5 6 = 5 6 = 6
3 3 3 5

3
x 3 5 2
lim = 6 .
x 5 x 5 3 5

x 5 + 32
6. Evaluate: lim
x 2 x8 28

lim
x 5 + 32
= lim
x 5 2 a f 5

x 2 x 8 2 8 x 2 x 8 2 a f 8

Dividing both Numerator and Denominator by a f = 32


2
5

x (2) we get a2f = +2


8 8

a f
x 5 2
5
a f
x 5 2 5

lim
a f
x 2
=
lim
x 2 a f
x 2
x a2f x a2f
x 2 8 8 8 8

x a2f
lim
x 2 x a2f

5 a 2f 5 a 2f
5 1 4
= =
8 a 2f 8 1
8 a 2f 7

= a 2f = a 2f
5 47 5 3
8 8
5 1 5
=
8 8 a f
64
352 Basic Mathematics

n2 + n + 1
7. Evaluate: lim .
n 3n 2 + 2n 1

n2 + n + 1
Solution: lim 2
= = an indeterminate .
n 3n + 2 n 1
Consider

F 1+ 1I
n2 1 +
lim
n2 + n + 1
= lim
H n nK 2

n 3n 2 + 2 n 1 n
n2
F3 + 2 1 I
H n nK 2

1 1
1+ + 2
= lim n n
n 2 1
3+ 2
n n

Now applying the lim we get


n

1+ 0 + 0 1
=
3+ 0+ 0 3

n2 + n + 1 1
lim = .
n 3n 2 + 2n 1 3

n 12 + 2 2 + 32 + ... n 2
8. Evaluate: lim .
n 13 + 2 3 + 33 + ... n 3

n
LM n an + 1fa2n + 1f OP
lim N 6 Q a fa
n n + 1 2n + 1 f
n an + 1f 12 + 2 2 + 32 + ...n 2 =
n 2 2
6
4

lim
a fa
n 2 n + 1 2n + 1
2
4 f 3 3 3 a f
n2 n + 1
2

n 6 n n +1 2 a f and 1 + 2 + ...n =
4

F 1I F 1 I
lim
H
n2 n 1 +
nK H
n 2+
n

K 4
n 6 F 1 I 2

H
n2 n2 1 +
n K
Limits and Continuity 353

F1 + 1 I F 2 + 1 I
lim
H nK H nK 4
n 6 F 1 I 2

H 1+
n K
Applying the lim
n

a1 + 0fa2 + 0f 4
=
2 4 4
=
6 a f1+ 0
2
6 1 3

n 12 + 2 2 + ... n 2 4
lim 3 3 3
= .
n 1 + 2 + ... n 3

n +1
9. Evaluate: nlim

4n 2 3

F 1I
lim
n +1
= lim
H nK
n 1+

4n2 3 n F4 I
n n 3
2
H nK 2

1 F I
= lim
n
n 1+
H K
n 3
n 4 2
n
Applying the limit, we get
1+ 0 1 1
= = .
40 4 2

n +1 1
lim =
n
4n 3 2 2

F n + 1I 4n
10. Evaluate: lim
n H nK
F n + 1 I = lim F1 + 1 I
4n 4n
lim
n H n nK H nK n

LF 1 I OP = e .
= lim M 1 +
n 4
4 F 1 I n

nMNH n K PQ lim 1 +
n H n K =e
354 Basic Mathematics

F nI 6n
11. Evaluate: lim
n H n + 1K
Dividing both numerator and denominator by n

F n I 6n
F I 6n

= lim GG n JJ = lim GG
1 J
n 1J
n
GH n n+ 1 JK n
GH n + n JK
F I 6n

lim GG
1 J 16 n
GH 1 + 1n JJK
=
n F1 + 1 I 6n

H nK
16 n 1
= lim = = = e 6 .
n LMF1 + 1 I OP n 6 e6

NH n K Q
1 1 1 1
12. Evaluate: lim + 2 + 3 + ... + n .
n 6 6 6 6

1
1 1 1 1 1 2 1
Solution: + 2 + 3 + ... + n is a G.P. with a = , r = 6 =
6 6 6 6 6 1 6
6

Sn =
d
a 1 rn i [formula]
1 r

1 1 1 1
lim + 2 + 3 + ... + n .
n 6 6 6 6

LM F I OP 1 L F 1I O
1 F I
n n
1 1
M P
n
1
= lim
6
1
6 N H K Q = lim N
6
1
H 6 K Q = 6KH
n 1 n 5 5
1
6 6
Applying the lim, we get
1 0 1 1
= = .
5 5 5
Limits and Continuity 355

1 1 1
13. Evaluate: lim 1 + + ... n terms.
n 2 4 8
1 1 1
Solution: 1 + ...n terms is a G.P.
2 4 8

1
1
with a = 1, r = 2 =
1 2

Sn =
d
a 1 rn i Formula
1 r
1 1 1
lim 1 + + ... n terms.
n 2 4 8

F F 1I I
1 F I
n n
GH H 2 K JK
1 1
H
1
2K
lim = lim
1 F I
n 1 1
n
1+
H 2K 2

1 0 2
Applying the lim = .
n 3 3
2

ax bx
14. Evaluate: lim
x 0 x

ax bx +1 1
lim Adding and Subtracting 1
x 0 x

= lim
ax 1 1 bx 1 d i
x 0 x

ax 1 bx 1
lim
x 0 x x

ax 1 bx 1
lim lim
x 0 x x0 x
= log e a log e b

F aI
= log e
H bK
356 Basic Mathematics

3x 1
15. Evaluate lim
x 0 x +1 1
Consider

3x 1
Rationalising the denominator,
x +1 1

3x 1 x +1 +1

x +1 1 x +1 +1

=
d3 1id
x
x +1 +1 i
x +11

d3 1id
x
x +1 +1 i
x

lim
3x 1
= lim
d
3x 1 x + 1 + 1 id i
x 0 x + 1 1 x 0 x

Applying the lim


x0

3x 1
log e 3 d 0 +1 +1 i lim
x 0 x
= log e 3

log e 3 2 af
= 2 log e 3

15.6 CONTINUOUS FUNCTIONS:

A function y = f (x) is said to be continuous at x = a if lim f x = f a = lim+ f x


xa
af af x a
af
i.e., Left Hand Limit = f (a) = Right Hand Limit
or a f af a
lim f a h = f a = lim f a + h
h 0 h 0
f
In other words, y = f(x) is continuous at x = a if lim f x = f a
xa
af af
Geometrically y = f (x) is continuous at x = a means that there is no break in the graph of y = f (x)
at x = a.
Note: A function which is not continuous at x = a is said to be discontinuous at x = a.
Limits and Continuity 357

WORKED EXAMPLES:

Rx + 3 when x 2
a f |S 5 when x = 2 is continuous at x = 2.
1. Prove that the function f x =
T|3x 1 when x 2
Proof. For a function y = f (x) to be continuous at x = a we have LHL = f (a) = RHL.
Here a = 2.

x 2
af
LHL = lim f x = lim x + 3 = 2 + 3 = 5
x2

f(a) = f(2) = 5 (Given)


af
RHL = lim+ f x = lim 3 x 1 = 3 2 1 = 5
x 2 x 2
af
af
LHL = f a = RHL
Hence f(x) is continuous at x = 2.

af R|x +2 3 when
when x > 1
2. Verify whether the function f x = S|2 x 1 when xx <= 11 is continuous at x = 1
T
Solution: For a function y = f (x) to be continuous at x = a we have LHL = f (a) = RHL.
Here a = 1
af
LHL = lim f x = lim 2 x 1 = 2 1 1 = 1
x 1 x 1
af
f (a) = f (1) = 2 (Given)

x 1
af
RHL = lim+ f x = lim x + 3 = 1 + 3 = 4
x 1

af
LHL f a RHL
So f (x) is discontinuous at x = 1
3. Prove that the function defined by

R| x 64 when x 64
3

f a x f = S x 16
2
|T26 when x = 4
is not continuous at x = 4.

Proof. For a function f (x) to be continuous at x = a we have LHL = f (a) = RHL.


Here a = 4

x 3 64
LHL = RHL = lim x 4 x < 4 or x > 4
x4 x 2 16
358 Basic Mathematics

x 3 43
3.4 31 3 4 2 3 4
= lim x2 4 2 = = = =6
x4 x 4 2.4 2 1 24 2
x4
Given f(a) = f(4) = 26
LHL = RHL f(a).
The function f(x) is not continuous at x = 4.

R|
f a x f = Sa1 + 2 x f
1
x for x 0 is continuous at x = 0.
4. Prove that
|Te
2
for x = 0
Proof. For a function to be continuous at x = a we have LHL = f(a) = RHL.
Here a = 0

LHL = RHL = lim 1 + 2 x


x 0
a f 1
x

a
lim 1 + 2 x
x 0
f 2
2x

L O
= lim Ma1 + 2 x f P
1 2

x0 N Q 2x

LM lim a1 + nf 1
=e
OP
=e 2
N x 0
n
Q
af af
f a = f 0 = e 2 given a f
LHL = RHL = f 0 af
Hence f (x) is continuous at x = 0.
5. If f (x) = + 3x 1, then prove that f (x) is continuous at x = 1.
x2
Proof. For a function f (x) to be continuous at x = a we have LHL = f (a) = RHL.
Here a = 1.
af
LHL = lim f x = lim f 1 h
x 1 h 0
a f
= lim a1 hf + 3 a1 hf 1 af
2
f x = x 2 + 3x 1
h 0

= lim 1 + h 2 2 h + 3 3h 1
h 0

= lim h 2 5h + 3
h 0
Limits and Continuity 359

Applying the lim we get


h0

LHL = 0 5(0) + 3 = 3.
af
RHL: lim+ f x = lim f 1 + h
x 1 h 0
a f
d i a f
= lim 1 + h 2 + 3 1 + h 1
h 0

= lim 1 + h 2 + 2 h + 3 + 3h 1
h 0

= h 2 + 5h + 3
Applying the lim we get
h0

RHL = 3.

a f af af
f a = f 1 = 13 + 3 1 1 = 1 + 3 1 = 3 .

LHL = f a af = RHL.
Hence the function y = f(x) is continuous at x = 1.

R| x 2
5x + 6
6. Find K if the function f axf = S x2
if x 2 is continuous at x = 2.
|T K if x = 2

Given f (x) is continuous at x = 2.


LHL = f (a) = RHL.

x 2 5x + 6
i.e. lim =K
x 2 x2

lim
ax 3fa x 2f = K
x 2 a x 2f
lim x 3 = K
x 2

23= K
K = 1

af a
7. Define f (0) so that f x = 1 + 3 x f 1
x x 0 is continuous at x = 0.
Given: f (x) is continuous at x = 0.
af af
lim f x = f 0
x 0

lim 1 + 3 x
x 0
a f 1
x af
= f 0
360 Basic Mathematics

a
lim 1 + 3x
x 0
f 3
3x = f 0af
LMa
lim 1 + 3 x f OPQ = f a0f
1 3

x0 N 3x

e3 = f a0 f
n 0
a f
lim 1 + n
1
n =e

R| 1+ x x
if x 0
8. Find K if f a xf = S x is continuous at x = 0.
|T K +3 if x = 0

Given f (x) is continuous at x = 0

lim f x = f 0
x 0
af af
1+ x 1
lim = K+3
x 0 x

1+ x 1 1+ x +1
lim = K +3
x 0 x 1+ x +1

lim
d 1+ x i 1 = K + 3 2 2

x 0 x d 1 + x + 1i

1+ x 1
lim = K +3
x 0 x d 1+ x +1 i
x
lim = K +3
x 0 x d 1 + x + 1i
Applying the lim we get
x0

1
= K+3
1+ 0 +1

1
= K +3
1+1
1
3= K
2
Limits and Continuity 361

1 6
=K
2
5
K=
2

af {
2 x + a if x 2
9. Find a if f a = x 1 if x > 2 is continuous at x = 2.

Given: f(x) is continuous at x = 2.


LHL = RHL = f(a)
af af
lim f x = f 2 = lim+ f x
x 2 x 2
af
lim 2 x + a = lim x 1
x 2 x2

af
2 2 + a = 2 1

4+ a =1

a =1 4

a = 3

REMEMBER:

x n an
lim = na n 1
x a xa

ex 1
lim =1
x0 x

ax 1
lim = log e a
x 0 x

F 1 I n
lim 1 +
n H n K =e

n 0
a f
lim 1 + n
1
n =e

af af
A function y = f (x) is said to be continuous at x = a if lim f x = f a = lim+ f x
x a xa
af
i.e., LHL = f (a) = RHL.
Limit of a function exists at x = a if
362 Basic Mathematics

af
lim f x = lim+ f x
x a xa
af
f(x) is continuous at x = a iff
af af
lim f x = f a
x a

EXERCISE

Evaluate: x 5 + 32
13. lim
1. xlim
2
a fa
2 x + 1 3x + 2 f x 2 x+2

1 1
x 3 + 4 x 2 3x + 3 3
2. lim x 64
14. lim
x 1 7x 2 x 1 x4 x4

3x 2 + 8 x 9 x 16
3. lim 15. lim
x 0 4 x2 9x 3 x 25 x 17

x 3 + 3x 2 + 2 x 1 x 4 81
4. lim 16. lim
x 1 x 3 + 3x 2 + 6 x 3 x 3

x 2 5x + 6 x3 2 1
5. lim 17. lim
x 3 3x 2 + 3x 7 x 1 x 1

3+ x 3 x 3 125
6. lim 18. lim
x 0 x x 5 x2 6x + 5

a x a+x 6 z 2 5z 6
7. lim lim
x 0 x 19. z
3 8 z 2 14 z + 3
2

x 3
8. lim
x 3 x2 4x 20. lim
a n + 2f d n 2
i
+ 4n + 3
2
n 1 n 1
3 5+ x
9. lim 2 y 3 3y 2 3 y + 2
x 4 x4
21. ylim
1 3 y 3 + 2 y 2 11y 10
x 3 27
10. lim 1 a
x 3 3x 4 2 x 1 22. lim 2
x a x a x ax
2
x 4
11. lim 1 1
x 2 3x 2 x + 2 3
lim x 27
23. x 3 1 1
3x 4 4 x 4
12. lim x 81
x 2 x2
Limits and Continuity 363

x 3 27
24. lim
x 3 x x 3 3

25. lim
a x 3f
32
5

x 5 x5

ax bx
26. lim
x 0 x

e x ex
27. lim
x 0 x

2x 1
28. lim
x0 x

2x 1
29. lim
x 0 x +1 1

3n 2 6n + 8
30. lim
n 4n 2 8

31. lim
a1 + 2 + 3 + ...+nf
n 3n 2

12 + 2 2 + 32 + ...+ n 2
32. lim
n 3n 3 8n + 1

2
33. lim n n 4 n
n

1 1 1
34. lim + + ... + n
n 3 32 3

13 + 2 3 + ... + n 3
35. nlim
a
n+4
4
f
3 n +1 + 1
36. lim
n 3n + 2 + 2

af {
3x + 2 if x < 2
af
37. If f x = 7 x 6 if x > 2 , then find lim f x .
x 2

x
38. Prove that lim does not exist
x 0 x
364 Basic Mathematics

39. Evaluate (a) nlim



1+ 2 na f 3n
(b) nlim
0
a1 + 3x f 4x

40. Examine the Continuity of the following functions:


(a) f(x) = x2 + x + 1 at x = 1

af R|4 x1+ 1 ifif xx =< 00 at x = 0


(b) f x = S|2 + 3x if x > 0
T
R| x 9
2

(c) f a x f = S x 4 x + 3 for x 3 at x = 3.
2
|T 3 for x = 3
R| x for x 0
(d) f a xf = S 1 + x 1 at x = 0
|T 3 for x = 0
R| log a1 + bxf when x 0
(e) f a xf = S
|T xb when x = 0 at x = 0
41. Prove that the function f(x) = |x| is continuous at x = 0.

Rx
42. Prove that f x = a f |S x for x 0
is discontinuous at x = 0.
T|0 for x = 0

af R|a f for x 0
1

43. Find K if the function f x =


1 + 4x S| x
is continuous at x = 0.
e2 K T for x = 0

R| a + x a x for x 0
44. Find K if the function f axf = S
| x
is continuous at x = 0.
|| 1
for x = 0
T K

R| 1 1
4

45. Find a if f a xf = S
| x
1

81
1
for x 3
is continuous at x = 3.
|| x3
27
T K for x = 3
Limits and Continuity 365

ANSWERS

1 1
1. 40 2. 1 3. 3 4. 5. 0 6. 2 3
2

a 1
7. 8. 1 9. 10. 54 5 11. 8 12. 2
2 6
3 1 3 75
13. 80 14. 15. 16. 108 17. 18.
256 2 2 4
5 3 1 9
19. 20. 1 21. 22. 23. 24. 6 3
2 2 a 4
a 3
25. 80 26. log 27. 2 28. log 2 29. log 4 30.
b 4
1 1 1 1 4
31. 32. 33. 2 34. 35. 36.
6 9 2 4 11
37. 8 39. (a) e6 (b) e12
40. (a) Continuous (b) discontinuous (c) Continuous (d) discontinuous (e) Continuous.
4
43. e2 44. a 45.
9
366 Basic Mathematics

16
Differential Calculus
16.1 INTRODUCTION:
Differential calculus was discovered by Sir Isaac Newton of England and Wilhelm Leibnitz of Ger-
many. It deals with the study of rate of change of one quantity with another.

16.2 DERIVATIVE OF A FUNCTION:


Let y = f(x).
Let x be an increment given to x.
y be the corresponding change in y.
y + y = f(x + x)
y = f(x + x) y
y = f(x + x) f(x)
Dividing by x and taking lim .
x0

lim
y
= lim
a f af
f x + x f x
x 0 x x 0 x

If lim
a f af
f x + x f x
exists and finite then the function y = f (x) is said to be differentiable at x
x 0 x

and lim
a f af
f x + x f x
is called derivative or differential co-efficient of y with respect to x. It is
x 0 x
dy
denoted by or y or y1 or f(x).
dx

lim
y dy
= = lim
a f af
f x + x f x
x 0 x dx x 0 x
Differential Calculus 367

16.3 DERIVATIVE OF SOME STANDARD FUNCTIONS FROM


FIRST PRINCIPLES:
1. xn.
Let y = xn.
Let x be an increment given to x.
y be the corresponding increment in y
y + y = (x + x)n
y = (x + x)n y
y = (x + x)n xn.
Divide by x and take lim
x0

lim
y
= lim
a
x + x x n f n

x 0 x x 0 x
Add and subtract x in the denominator of RHS.

lim
y
= lim
a
x + x x n f n

x 0 x x 0 x + x x
As lim x 0,
lim x + x x.

x n an
Also lim = na n 1 [formula].
x a xa

lim
y
= lim
a
x + x x n f n
= nx n 1
x 0 x x 0 x + x x
dy
= nx n 1
dx
d n
i.e.,
dx
d i
x = nx n 1

Hence derivative or differential co-efficient of xn is nxn1.


2. ex.
Let y = ex.
Let x be an increment given to x.
y be the corresponding increment in y.

y + y = e x +x

y = e x + x y
368 Basic Mathematics

y = e x + x e x

y = e x e x e x a m+n = a m a n

e
y = e x e x 1 j
Dividing by x and taking lim .
x0

lim
y
= lim
e
e x e x 1 j
x 0 x x 0 x

ex 1
But we have lim = 1 (formula)
x 0 x

lim
y
= lim
e
e x e x 1 j
x 0 x x 0 x

dy
= e x 1
dx
dy
= ex
dx

d x
i.e.,
dx
d i
e = ex.

Hence derivative or differential co-efficient of ex is ex.


3. ax.
Let y = ax.
Let x be an increment given to x. y be the corresponding increment in y.

y + y = a x +x

y = a x +x y

y = a x + x a x

y = a x a x a x a m+n = a m a n

e
y = a x a x 1 j
Dividing by x and taking lim
x0
Differential Calculus 369

lim
y
= lim
e
a x a x 1 j
x 0 x x 0 x

ax 1
lim = log e a (formula)
x 0 x

lim
y
= lim
a x a x 1 e j
x 0 x x 0 x

dy
= a x log e a.
dx

d x

dx
d i
a = a x log e a.

Hence derivative or differential co-efficient of ax is ax.logea.


4. logex.
Let y = logex
Let x be an increment given to x.
y be the corresponding increment in y.
a f
y + y = log e x + x

y = log a x + x f y
e

y = log a x + x f log
F mI
e e x but log m log n = log
H nK
FG x + x IJ
y = log e
H x K
F x x I
y = log G + J
Hx x K
e

F x I
y = log G1 + J
H xK
e

Divide by x and take lim


x0

FG x IJ
lim
y
= lim
H
log e 1 +
x K
x 0 x x 0 x
370 Basic Mathematics

y 1 FG
x IJ
lim
x 0 x
= lim
x 0 x
log e 1 +
xH K
Multiplying and dividing by x in RHS.

y 1 x x FG IJ
lim
x 0 x
= lim log 1 +
x 0 x x x H K
But n log m = log m n

x
y 1 x FG IJ x
lim
x 0 x
= lim log e 1 +
x 0 x x H K
x
F x I
lim G1 + J
x
As x 0H xK =e

x
y 1 xFG IJ x
lim
x 0 x
= lim log e 1 +
x 0 x x H K
dy 1
= log e e But logee = 1
dx x
dy 1
= 1
dx x
dy 1
= .
dx x

i.e.,
d
dx
a f
log x =
1
x
1
Derivative or differential Co-efficient of logx is .
x
5. Constant function:
Let y = c where c is a constant.
Let x be an increment given to x. y be the corresponding increment in y. But Since c is a constant
function any change in x will not cause change in y. In other words y = 0.
y 0
lim = =0
x 0 x x
dy
=0
dx
Differential Calculus 371

Hence
d
dx
a f
constant = 0.

derivative of a constant is zero.


d F I5 d
af d
af
Ex.:
dxH K

2
=0;
dx
5 =0
dx
a = 0 if a is constant.

16.4 RULES OF DIFFERENTIATION:

1. Derivative of Product of Constant and a Function:


Let y = Ku where K is a constant and u is a function of x.
Let x be an increment given to x.
u be the increment in u and
y be the corresponding increment in y.
a
y + y = K u + u f
y + y = Ku + Ku
y = Ku + Ku y

y = Ku + Ku Ku
y = Ku

Divide by x and take lim


x0

y u
lim = lim K
x 0 x x 0 x

dy du
=K
dx dx

i.e.,
d
dx
a f
Ku = K
du
dx

Hence derivative or differential co-efficient of constant multiple of a function is constant into deriva-
tive of the function.

Examples:

(a)
d
dx
a f
4 log x = 4
d
dx
a f
log x

1 4
= 4 = .
x x
372 Basic Mathematics

d
(b)
dx
d i
8e x = 8 d i
d x
dx
e

= 8ex

2. Derivative of Sum of 2 Functions:


Let y = u + v where u and v are functions of x.
Let x be an increment given to x.
u, v be the increments in u and v.
y be the corresponding increment in y.
y + y = u + u + v + v
y = u + u + v + v y

a f
y = u + u + v + u u + v

y = u/ + u + v/ + v u/ v/

y = u + v

Divide by x and take lim


x0

y u + v
lim = lim
x 0 x x 0 x

y u v
lim = lim +
x 0 x x 0 x x
dy du dv
= +
dx dx dx

i.e.,
d
dx
a f
u+v =
du dv
+
dx dx

Derivative or differential co-efficient of sum of 2 functions is derivative of first function plus the
derivative of the second function.

Examples:

dy d 4
(a) If y = x4 + ex, then =
dx dx
d
x + ex i
d 4
=
dx
d i
x +
d x
dx
e d i
Differential Calculus 373

dy
= 4 x 4 1 + e x
dx
dy
= 4x3 + e x
dx
d
(b)
dx
d
log x + x 2 i
=
d
dx
a f
log x +
d 2
dx
x d i
1
= + 2 x 2 1
x
1
= + 2x
x

3. Derivative of Difference of 2 Functions:


Let y = u v where u and v are functions of x. Let x be an increment given to x.
u, v be the increments in u and v.
y be the corresponding increment in y.
a f
y + y = u + u v + v

y = u + u av + v f y

y = u + u av + vf au v f

y = u + u v v u + v

y = u v

Divide by x and take lim


x0

y u v
lim = lim
x 0 x x 0 x

y u v
lim = lim
x 0 x x 0 x x

y u v
lim = lim lim
x 0 x x 0 x x 0 x

dy du dv
=
dx dx dx
374 Basic Mathematics

i.e.,
d
dx
a f
uv =
du dv

dx dx
Derivative of difference of 2 functions is the derivative of first function minus the derivative of the
second function.

Examples:

d 3
(a)
dx
d
x x2 i
d 3 d 2
=
dx
d i
x
dx
x d i
= 3 x 31 2 x 2 1

3x 2 2 x
(b) If y = logx ex, then
dy d
=
dx dx
d
log x e x i
=
d
dx
a f
log x
d x
dx
e d i
1
= ex .
x

4. Derivative of Product of 2 Functions:


Let y = uv where u and v are functions of x. Let x be an increment given to x, u, v be the increments
in u and v. Let y be the corresponding increment in y.
a fa f
y + y = u + u v + v

y = au + ufav + v f y

y = uv + uv + vu + uv uv

y = uv + vu + uv

Divide by x and take lim


x0

y uv + vu + uv
lim = lim
x 0 x x 0 x

v u uv
= lim u +v +
x 0 x x x
Differential Calculus 375

uv
As lim , u and v are small. Hence the product u v is very very small. So the term can
x0 x
be neglected.
y v u
lim = lim u + v
x 0 x x 0 x x
dy dv du
=u +v
dx dx dx

i.e.,
d
dx
a f dv
uv = u + v
dx
du
dx

So Differential Co-efficient or derivative of product of 2 function is first function into derivative of


2nd function plus second function into derivation of the first function.
d
i.e., [I function II function]
dx
d d
= (I function) (II function) + (II function) (I function).
dx dx
This rule is known as product rule.

Examples:

d 3 x d d 3
(a)
dx
d i
x e = x3 ex + ex
dx dx
x d i
= x 3 e x + e x 3 x 31

= x 3e x + e x 3 x 2
(b) If y = x4 log x, then
dy d 4
=
dx dx
d
x log x i
= x4
d
dx
a f a f d i
log x + log x
d 4
dx
x

1
= x4 + log x 4 x 4 1
x

x 3 + log x 4 x 3

x 3 + 4 x 3 log x
376 Basic Mathematics

5. Derivative of Quotient of 2 Functions:

u
Let y = where u and v are functions of x. Let x be an increment given to x.
v
u, v be the increments in u and v.
y be the corresponding increment in y.
u + u
y + y =
v + v

u + u
y = y
v + v

u + u u
y =
v + v v

y =
a f a f
v u + u u v + v
a fa f
v + v v

uv + vu uv uv
y =
v v + va f
vu uv
y =
a
v v + v f
Divide by x and take lim
x0

y vu uv
lim = lim
x 0 x x 0 a
v v + v x f
u v
v u
y
lim = lim x x (note this step)
x 0 x x 0 v v + v a f
As lim , v 0
x0

u v
v u
y x x
lim = lim
x 0 x x 0 v v + v a f
du dv
v u
dy dx dx
=
dx a f
v v+0
Differential Calculus 377

du dv
v u
dy dx dx
=
dx v2

du dv
u
d u FI v
dx dx
i.e.,
dx v
=
HK v2
Hence Derivative or differential co-efficient of quotient of 2 functions is Denominator into differen-
tial co-efficient of Numerator, minus Numerator into differential co-efficient of Denominator, whole
divided by square of the denominator.

F I
d Nr.
a Dr.f aNr.f aNr.f aDr.f
d
dx
d
dx
i.e.,
H K
dx Dr.
=
aDr.f 2

Nr.: Numerator
Dr.: Denominator
This rule is known as quotient rule.

Examples:

F
d log x I ex
d
dx
a f a f d i
log x log x
d x
dx
e
(a)
H
dx e x
=
K de i x 2

1
ex log x e x
= x
de ix 2

x3
(b) If y = , then
log x

dy dFx3 I log x d i
d 3
x x3
d
log x a f
= GH
dx dx log x
JK
= dx
a f
log x
2
dx

1
log x 3 x 2 x 3
= x
alog x f 2

log x 3 x 2 x 2
=
alog x f 2
378 Basic Mathematics

d n
Some particular cases of
dx
d i
x = nx n 1

1. When n = 0
d 0
dx
d i
x = 0 x 0 1 But x0 = 1.

d
dx
af
1 =0

Also we have
d
dx
a f
K u = K
du
dx

d
dx
a f
K 1 = K
d
dx
af
1 = K 0 = 0

d
dx
af
K =0

2. When n = 1,
d 1
dx
d i
x = 1 x 11 = 1x 0 = 1

d
dx
af
x =1

3. When n = 2,
d 2
dx
d i
x = 2 x 2 1 = 2 x

d 2

dx
d i
x = 2x

4. When n = 3,
d 3
dx
d i
x = 3x 31

d 3
dx
d i
x = 3x 2

d 4
Similarly,
dx
d i
x = 4x 3,
d 5
dx
d i
x = 5 x 4 ...
d 100
dx
x d i
= 100 x 99 and so on.

1
5. When n = ,
2
Differential Calculus 379

d F I
1
1 1
1

dx
GH JK
x2 = x2
2

1
d 1
d i
1
x = x 2 x2 = x
dx 2

d
dx
d xi = 2 1x am =
1
am

d 1
x=
dx 2 x
6. When n = 1
d 1
dx
d i
x = 1 x 11

d F 1I 2
dx H x K
= 1 x

d F 1 I 1
dx H x K x
= 2

7. When n = 2
d 2
dx
xd i
= 2 x 2 1 = 2 x 3

d F 1I 2
dx H x K
2
= 3
x
Similarly
F I
d 1 F I
3 d 1 4
H K
dx x 3
= 4,
H K
x dx x 4
= 5
x

F I
d 1 5
H K
dx x 5
= 6 and so on.
x
380 Basic Mathematics

List of Formulae:

dy
y
dx

(1) x n nx n1
(a) x 1
(b) x 2 2x

(c) x 3 3x 2
(d) x 4 4x3
(e) x 5 5x 4
1
(f) x 2 x

1 1
(g)
x x2
1 2
(h)
x2 x3
1 3
(i)
x3 x4
1 4
(j)
x4 x5
(k) K (Constant) 0
x
2. e ex
3. a x a x log e a

1
4. log x
x
du dv
5. u v
dx dx

6. (I function) (II function) I function


d
dx
af
II + II
d
dx
af
I

Nr. Dr.
d
dx
a f a f a f
Nr. Nr.
d
dx
Dr.
7.
Dr. a f
Dr
2
Differential Calculus 381

Note: Sum, Difference and product rule can be extended i.e.,


d
dx
a f
u v w ... =
du dv dw

dx du du
...

d
dx
a
uvw ... = f
du
dx
dv
vw ...+ uw ...+
dx
dw
dx
uv ... + ...

WORKED EXAMPLES:
dy
1. Find if y = x3 3x + 7
dx

Consider y = x 3 3x + 7, differentiate with respect to (w.r.t) x.

dy d 3
= x 3x + 7
dx dx

=
d 3
dx
d i
x
d
dx
3x +
d
dx
7a f af
3x 2 3
d
dx
af
x +0

3x 2 3 a1f + 0

dy
= 3x 2 3.
dx

2. Find y if y = 7e x 4 log e x.
x x
Given: y = 7e 4 log e
diff. w.r.t.x. (differentiate with respect to x)

y =
d
dx
d i
7e x
d
dx
b
4 log e x g
1
y = 7 e x 4
x
4
y = 7e x
x
3. Find f (x) if f(x) = 7x + 8 ex 9
Consider f(x) = 7x + 8 ex 9
diff. w.r.t. x.
382 Basic Mathematics

a f dxd d7 i + 8 dxd e dxd a9f


f x = x x

f a x f = 7 log 7 + 8 e 0
x x

f a x f = 7 log 7 + 8e
x x

1
4. If y = x + , then find y1
x

1
y= x +
x
diff. w.r.t. x.

y1 =
d
dx
d x i + dxd FGH 1x IJK
d 12
= d i d i
dx
x +
d 1 2
dx
x

1
1
F 1 I
1
+ G x
1
1

H 2 JK
= x 2 2
2

1 1 2 1 3 2
y1 = x x
2 2
1 1
y1 =
2 x 2x3 2

Alieter:

1
y= x +
x
x +1
y=
x
diff. w.r.t. x. using quotient rule.

x a f a f d xi
d
dx
x +1 x +1
d
dx
y1 =
d xi 2

x 1 + 0 a x + 1f
1
2 x
x
Differential Calculus 383

x 1
x
2 x 2 x
x

x x 1
=
x 2 x x 2 x x

1 1 1
=
x 2 x 2x x

2 1 1
= 1+1 2
2 x 2x

1 1
y1 = .
2 x 2x3 2

dy
5. Find if y = x38x.
dx

y = x 3 8x .
diff. w.r.t. x using product rule.
dy d x
dx
= x3
dx
d i
8 + 8x
d 3
dx
x d i
dy
= x 3 8 x log 8 + 8 x 3 x 2
dx

x2 + 1
af
6. If f x =
x3 7
, then find f (x)

x2 + 1
af
f x =
x3 7
diff. w.r.t. x using quotient rule.

dx 7i dxd dx + 1i d x + 1i dxd dx 7i
3 2 2 3

f a x f =
d x 7i3 2

d x 7i LMN dxd d x i + dxd a1fOPQ d x + 1i LMN dxd d x i dxd a7fOPQ


3 2 2 3

=
d x 7i3 2
384 Basic Mathematics

d x 7i a2 xf dx + 1id3x i
3 2 2

d x 7i 3 2

2 x 4 14 x 3 x 4 3 x 2
af
f x =
d x 7i
3 2

x 4 3 x 2 14 x
af
f x =
d x 7i
3 2

x 2 dy
7. If y = + 2 x 2 , then find .
2 x dx
x 2
y= + 2x 2
2 x
diff. w.r.t.x.
dy d x F I
d 2 d F I d i
2x2
=
dx dx 2
+
H K
dx x

dx H K
=
1 d
2 dx
a xf + 2 dxd FH 1x IK 2 dxd dx i 2

1 F 1 I 2 a2 x f
= 1 + 2
2 H xK 2

dy 1 2
= 4 x.
dx 2 x 2

xe x dy
8. If y = , then find .
a f
log x + 7 dx

xe x
y= .
log x + 7
diff. w.r.t. x using quotient rule.

dy
alog x + 7f dxd dxe i d xe i dxd alog x + 7f
x x

=
dx alog x + 7f 2

alog x + 7f LMNx dxd e + e dxd a xfOPQ xe LMN dxd alog xf + dxd a7fOPQ
x x x

=
alog x + 7f 2
Differential Calculus 385

alog x + 7f x e + e 1 xe LMN 1x + 0OPQ


x x x

=
alog x + 7f 2

dy alog x + 7f d xe + e i e
x x x
=
dx alog x + 7f 2

xn nx dy
9. If y = x , then find .
e dx

xn nx
y=
ex
diff. w.r.t. x using quotient rule.

d n
dy
ex
dx
dx nx xn nx
d x
dx
i d
e i d i
=
dx de ix 2

ex
LM d dx i d dn iOP d x
n x n
i
nx ex
= N dx dx Q
de i x 2

=
e x nx n 1 n x log n x n n x e x d i
de i x 2

e x nx n 1 n x log n x n + n x
=
de i x 2

dy nx n 1 n x log n x n + n x
= .
dx ex
10. If f(x) = (x2 + 1) (x3 + 7x + 8) ex, then find f (0).

af d id
f x = x 2 + 1 x 3 + 7x + 8 e x i
diff. w.r.t. x using extended product rule.

af d id
f x = x2 + 1 x3 + 7x + 8 i dxd de i + dx + 1i e
x 2 x

d 3
dx
d i d
x + 7x + 8 + x 3 + 7x + 8 e x
d 2
dx
i
x +1 d i
386 Basic Mathematics

a f d id i d i d i d i
f x = x 2 + 1 x 3 + 7 x + 8 e x + x 2 + 1 e x 3x 2 + 7 + x 3 + 7x + 8 e x 2 xa f
f a0f = a0 + 1fa0 + 8f e + a0 + 1f e a0 + 7f + a0 + 0 + 8f e a2 0f
0 0 0

=8+7+0

af
f 0 = 15

16.5 DIFFERENTIATION OF COMPOSITE FUNCTIONS:


Chain rule: If y = g(u) and u = f (x) are 2 differentiable functions, then the composite function
y = g[f (x)] can be differentiated by using chain rule, which can be stated as
dy dy du
=
dx du dx

i.e.
dy
dx
af af
= g u f x

i.e.
d
dx
b a f g = g f a x f f a x f
g f x

Examples:
1. If y = log (x2 4x + 8), then
dy 1 d 2
= 2
dx x 4 x + 8 dx
x 4x + 8 d i
= 2
1
x 4x + 8
a
2x 4 f
2x 4
=
x2 4x + 8
Alieter: Consider y = log (x2 4x + 8)
Let y = logu where u = x2 4x + 8
diff. w.r.t. x.
dy 1 du
= u = x 2 4 x + 8 diff. w.r.t. x
dx u dx
dy 1
= 2x 4
dx u
a f
du
dx
= 2x 4

dy 1
= 2x 4
dx u
a f
Differential Calculus 387

dy
=
1
dx x 2 4 x + 8
2x 4 a f
dy 2x 4
= 2
dx x 4 x + 8

af 2
2. If f x = e x , then

af
f x = ex
2 d 2
dx
x d i
af
f x = e x 2 x.
2

Note: Chain rule can be extended.


i.e., If y = f[g(h(x))], then
dy
dx
= f g h x b a fg g bha xfg haxf
Example. If y = e d i
8
x2 +4
, then

dy
=e
d x 2 + 4i d x 2 + 4
8

d i 8

dx dx

dy
=e
d x 2 + 4i 8 x 2 + 4 7 d x 2 + 4
8

d i d i
dx dx

=e
d x + 4i
2 8

d
8 x 2 + 4 2 x. i 7

WORKED EXAMPLES:
dy
1. Find if y = log (xn ex)
dx

y = log x n e x d i
diff. w.r.t. x.
dy
= n
1
dx x e dx
x

d n
x ex d i
1
= nx n 1 e x
x e
n x

nx n 1 e x
= .
xn ex
388 Basic Mathematics

dy
2. Find if y = x 2 + 4 x 9 .
dx

Consider y = x + 4 x 9
2

diff. w.r.t x.
dy
=
1

d 2
dx 2 x + 4 x 9 dx
2
x + 4x 9 d i
dy 1
= 2x + 4 0
dx 2 x + 4 x 9
2

dy
=
1
dx 2 x + 4 x 9
2
2 x + 2 a f
dy x+2
= .
dx x + 4x 9
2

3. If y = 7
d x + 9 x 6 i , then find
3 dy
.
dx

y=7
d x + 9 x 6i
3

diff. w.r.t. x.

d x 3 + 9 x 6 i log 7 d x 3 + 9 x 6
dy
dx
=7
dx
d i
=7
d x + 9 x 6 i log 7 3x 2 + 9 a1f 0
3

dy
dx
= 7 x +9 x 6 log 7 3 x 2 + 3 .
3
d i
af
4. If f x = x 3 e 5 x
2
+8
, then find f (x)

af
Consider f x = x 3 e 5 x
2
+8

diff. w.r.t. x using product rule.

af
f x = x3
dx
e e
d 5 x 2 +8
+ e5 x +8
2
j
d 3
dx
x d i
= x 3 e5 x
2
+8

d
dx
d i
5x 2 + 8 + e 5 x +8 3x 2
2
d i
af
f x = x 3 e5 x
2
+8
a10 x + 0f + e 5x 2 +8
3x 2
Differential Calculus 389

x +1 dy
5. If y =
dx i 3 , then find .
2
+ 8x 9 dx

Consider
x +1
y=
dx 2
+ 8x 9 i 3

diff. w.r.t. x using quotient rule.

dy dx 2
+ 8x 9 i 3 d
dx
a f a f d
x +1 x +1
d 2
dx
x + 8x 9 i 3

=
dx LMd x + 8x 9i OP 3 2

N Q
2

dx 2
i a f a f LMN d
3
+ 8x 9 1 + 0 x + 1 3 x 2 + 8x 9 i dxd d x + 8x 9iOPQ 2 2

=
d x + 8x 9i 2 6

x + 8 x 9i a x + 1f L3 d x + 8x 9i a2 x + 8fO
dy d MN QP .
2 3 2 2

=
dx d x + 8x 9i 2 6

xe 3 x + 7 dy
6. If y =
a
log 7 x 6
, then find
fdx
.

xe 3 x + 7
y=
log 7 x 6 a f
diff. w.r.t. x using quotient rule.

dy a
log 7 x 6 f d
dx
xe 3 x + 7 xe 3 x + 7
d
dx
log 7 x 6 a f
=
dx log 7 x 6
2
a f
a
log 7 x 6 x f LMN dxd de i + e 3x+7 3x + 7 d

dx
a fOPQ
x xe 3 x + 7
1

d
7 x 6 dx
LM
7x 6
N a fOPQ
=
a f
log 7 x 6
2

a
log 7 x 6 x e 3 x + 7 f LNM d
dx
a f
3x + 7 + e 3 x + 7 1 xe 3 x + 7
1 OP
7x 6Q7 LM
N a fOQP
=
log 7 x 6
2
a f
390 Basic Mathematics

dy
a f
log 7 x 6 xe 3 x + 7 3 + e 3 x + 7 xe 3 x + 7
7
7x 6
=
dx a
log 7 x 6 f 2

16.6 DIFFERENTIATION OF IMPLICIT FUNCTIONS:


Consider the function, y = x2 +ex logx. Here y is expressed as a function of x. i.e., y = f (x). Such
functions are called explicit functions.
Now consider the function xy + log y + ex = 0. Here y is not expressed explicitly as a function of x.
Such functions are called implicit functions. They are of the form f (x, y) = 0.

dy dy
To find , differentiate the given function shift all the terms containing to Left Hand Side and
dx dx
dy dy
the remaining terms to Right Hand Side. Take common and shift the co-efficient of to Right
dx dx
Hand Side.

WORKED EXAMPLES:
dy
1. Find if x3 + y3 = a3.
dx
Consider x3 + y3 = a3
diff. w.r.t. x.

LMQ a is constant d da i = 0OP


3

MM d dx
d a yf P
MM dxausing chain ruledxf PPP
d y i = 3y
dy 3 2
3x 2 + 3y 2 =0
dx
MN PQ
dy
3 x 2 + 3y 2 =0
dx
dy
3y 2 = 3x 2
dx

dy 3x 2
=
dx 3y 2

dy x 2
= 2 .
dx y
Differential Calculus 391

dy
2. If y + x2 + ey = 0, then find .
dx
Solution: Consider y + x2 + ey = 0
diff. w.r.t. x.
dy dy
+ 2x + ey =0
dx dx
dy dy
+ ey = 2 x .
dx dx
dy
1 + e y = 2 x .
dx
dy 2 x
= .
dx 1 + e y
dy
3. If ex + ey = logx, then find when x = 1 and y = 0
dx
Solution:
Consider ex + ey = logx.
diff. w.r.t. x.
dy 1
ex + ey =
dx x
dy 1
ey = ex
dx x

1
ex
dy x
=
dx ey

1 1
dy e
when x = 1 and y = 0 = 1 0
dx e

dy 1 e
= =1 e .
dx a1, 0 f 1

4. If y = x + x + x + x + ...

dy
Then find .
dx
392 Basic Mathematics

Solution: y = x + x + x + x + ...

y= x+y
Squaring,

y2 = x + y
diff. w.r.t. x.
dy dy
2y =1+
dx dx
dy dy
2y =1
dx dx
dy
dx
a f
2y 1 = 1
dy
=
1
dx 2 y 1
dy
5. If y = ax ax ax ... , then find .
dx

y = ax ax ax ...

y = axy
Squaring

y 2 = axy.
diff. w.r.t. x (using product rule in RHS, Chain rule in LHS)
dy dy LM d
a fOPQ
2y
dx
=a x
dx N
+ y
dx
x

dy dy LM OP
2y
dx
=a x
dx
+y
N Q
dy dy
2y = ax + ay
dx dx
dy dy
2y ax = ay
dx dx
dy
dx
a
2 y ax = ay f
dy ay
= .
dx 2 y ax
Differential Calculus 393

16.7 DIFFERENTIATION OF PARAMETRIC FUNCTIONS:


If both x and y are expressed as a function of another variable say t then the function y = f (x) is said
to be in parametric form. The variable t is called a parameter.
dy
To find we use.
dx

dy
dy dt
=
dx dx
dt

WORKED EXAMPLES:
dy
1. Find if x = at2 and y = 2 at.
dx
Consider y = 2 at
diff. w.r.t. t.
dy
dt
= 2a 1 af
dy
= 2a
dt
Now Consider x = at2
diff. w.r.t. t.
dx
dt
= a 2ta f
dx
= 2 at
dt
Now

dy
dy dt 2a
= =
dx dx 2 at
dt

dy 1
= .
dx t
dy
2. If x = 7t + et and y = et 7t, then find when t = 0.
dx
Consider y = et 7t.
394 Basic Mathematics

diff. w.r.t. t.
dy
= e t 7t log 7.
dt
Now consider
x = 7t + e t
diff. w.r.t. t.
dx
= 7t log 7 + e t
dt

dy
dy dt
=
dx dx
dt

e t 7 t log 7
=
7t log 7 + e t

dy e 0 7 0 log 7
= 0
dx when t =0 7 log 7 + e 0

dy 1 log 7
=
dx when t = 0 log 7 + 1
3
3. Differentiate e x with respect to log x.
3
Solution. Let u = e x and v = log x.
du
To find:
dv

du
du dx
=
dv dv
dx
3
Consider u = e
x

diff. w.r.t. x.
du
dx
3
= ex
d 3
dx
x d i
du 3
= e x 3x 2
dx
Differential Calculus 395

v = log x
diff. w.r.t. x
dv 1
=
dx x
3
du e x 3 x 2
=
dv 1
x

du 3
= e x 3x 3 .
dv
2
9
4. Differentiate (x2 + 8x 1)4 with respect to e x
2
9
Solution. Let u = (x2 + 8x 1)4 and v = e x
du
To find:
dv

du
du dx
=
dv dv
dx
Consider u = (x2 + 8x 1)4
diff. w.r.t. x.
du
dx
d 3 d
= 4 x 2 + 8x 1
dx
x 2 + 8x 1i d i
du
dx
d 3
= 4 x 2 + 8x 1 2 x + 8 i a f
2
9
Now, v = ex
diff. w.r.t. x.
dv
dx
= e x 9
2 d 2
dx
x 9 d i
dv
= e x 9 2 x
2

dx

d ia f
du 3
du dx 4 x + 8 x 1 2 x + 8
2

Hence = = .
dv dv e x 9 2 x
2

dx
396 Basic Mathematics

5. Differentiate 4 + log x with respect to e


d
log x 2 + 9 i

Let u = 4 + log x and v = e


d
log x 2 + 9 i

v = x2 + 9 e b a fg = f a x f aformula f
log f h

du
To find:
dv

du
du dx
=
dv dv
dx

Consider u = 4 + log x
diff. w.r.t. x.
du
=
1

d
dx 2 4 + log x dx
4 + log x a f
du 1 1
=
dx 2 4 + log x x

du 1
=
dx 2 x 4 + log x
Now v = x2 + 9.
diff. w.r.t. x.
dv
= 2x + 0
dx
dv
= 2x
dx

du 1
du dx 2 x 4 + log x
= =
dv dv 2x
dx

du 1
= 2 .
dv 4 x 4 + log x
Differential Calculus 397

16.8 LOGARITHMIC DIFFERENTIATION:


If it is required to differentiate (function)function or (constant)function then we consider log on both sides,
apply logmn = nlogm and then differentiate.

WORKED EXAMPLES:
dy
1. Find if y = xx
dx

y = xx
Consider log on both sides

log y = log x x

log y = x log x
diff. w.r.t. x.
1 dy

y dx
= x
d
dx
a f
log x + log x
d
dx
x af
1 dy 1
= x + log x 1
y dx x

1 dy
= 1 + log x
y dx

dy
dx
a
= y 1 + log x f

d x
dx
d i a
x = x x 1 + log x f
dy
2. Find if x y = y x .
dx

xy = yx
Consider log on both sides

log x y = log y x

y log x = x log y
diff. w.r.t. x.

y
d
dx
a f
log x + log x
dy
dx
= x
d
dx
a f
log y + log y
d
dx
af
x
398 Basic Mathematics

1 dy 1 dy
y + log x = x + log y 1
x dx y dx

y dy x dy
+ log x = + log y
x dx y dx

dy x dy y
log x = log y
dx y dx x

dy FG x IJ y
dx H
log x
y K
= log y
x

y
log y
dy x.
=
dx log x x
y

1+ x dy
3. If y = , then find .
1 x dx

1+ x
y=
1 x
Consider log on both sides.

1+ x
log y = log
1 x

log y = logF
1+ xI
1

H1 xK
2

log y = logF
1 1+ xI
2 H1 xK

log y = loga1 + x f loga1 x f


1
2
diff. w.r.t. x.

LM
1 dy 1 1 1 d
a fOPQ
=
N
y dx 2 1 + x 1 x dx
1 x

1 dy 1 L 1
= M
1
y dx 2 N 1 + x 1 x
a1fOPQ
Differential Calculus 399

LM
dy y 1 1 OP
N
= +
dx 2 1 + x 1 x Q
dy y L 1 x + 1 + x O
= M P
dx 2 N a1 + x fa1 x f Q

1+ x
dy 1 x 2 LM OP
N a fa f Q
=
dx 2 1+ x 1 x

dy 1+ x 1 1
dx
=
a fa f
1 x 1+ x 1 x
=
1+ x 1 x a f 32

Alieter:

1+ x
y=
1 x
diff. w.r.t. x.

dy 1 d 1+ x F I
dx
=
2

1 + x dx 1 x H K
1 x

dy 1 a1 xf dxd a1 + xf a1 + xf dxd a1 xf
dx
=
2
1+ x

a1 xf 2

1 x

a fa f a fa f
1 x 1 x 1 1 + x 1
2 1+ x

a f
1 x 2

1 x 1 x +1+ x
2 1+ x

1 x 2a f
1 x 2

2 1+ x 1 x a f 2

dy 1 1
= =
dx
a f
1+ x 1 x 2
1
2 a1 + xf a1 xf 3
2
400 Basic Mathematics

dy
4. If yey = xx, then find .
dx
Consider yey = xx
Consider log on both sides

d i
log ye y = log x x Q log mn = log m + log n

log y + log e y = log x x

log y + y log e = x log x


log y + y = x log x

diff. w.r.t. x. Q log e = 1

1 dy dy
+
y dx dx
= x
d
dx
d
log x + log x x
dx
a f
1 dy dy 1
+ = x + log x 1
y dx dx x

dy 1 LM OP
+ 1 = 1 + log x
dx y N Q
dy 1 + log x
=
dx 1
+1
y

a
dy y 1 + log x
= .
f
dx 1+ y
dy
5. If ey = ax+y, then find .
dx

ey = a x+y
Consider log on both sides

log e y = log a x + y

a f
y log e = x + y log a

y 1 = a x + y f log a
diff. w.r.t.x.
dy dyLM OP
dx
= log a 1 +
dx N Q Q log a is a constant.
Differential Calculus 401

dy dy
= log a + log a
dx dx
dy dy
log a = log a.
dx dx
dy
dx
a f
1 log a = log a.

dy log a
= .
dx 1 log a

16.9 SUCCESSIVE DIFFERENTIATION:


If y = f (x), then
dy
dx
af
= y1 = y = f x is first order derivative of y with respect to x. It is a function of x. The deriva-

tive of this function with respect to x is


d2 y
dx 2
af
= y2 = y = f x and is called second order derivative.

F I
d dy d2y
af
i.e., H K
dx dx
= 2 = f x = y = y2 .
dx

WORKED EXAMPLES:

d2y
1. Find if y = x2 + 3x + 8.
dx 2

Solution: y = x 2 + 3 x + 8
diff. w.r.t. x.
dy
dx
af
= 2x + 3 1 + 0

dy
= 2x + 3
dx
diff. again w.r.t. x.

d2y
dx 2
af
=2 1 +0

d2y
= 2.
dx 2
402 Basic Mathematics

d2y
2. If y = x e , then find
2 x
.
dx 2

y = x 2e x .
diff. w.r.t. x using product rule.
dy
dx
= x2
d x
dx
d i
e + ex
d 2
dx
x d i
dy
= x 2 e x + e x 2x
dx
dy
dx
d
= e x x 2 + 2x i
diff. again w.r.t. x using product rule,

d2 y
dx 2
= ex
d 2
dx
d
x + 2x + x2 + 2 x
d x
dx
ei d i
d
= ex 2x + 2 + x2 + 2x ex i
d2 y
2
= ex 2x + 2 + x2 + 2 x
dx
d2 y
2
= e x x2 + 4x + 2 .
dx

d2y
3. If x = t2 and y = 4t, then find at t = 1.
dx 2
Consider x = t2
diff. w.r.t. t.
dx
= 2t
dt
Consider y = 4t.
diff. w.r.t. t.
dy
dt
= 4 1 af
dy
= 4.
dt
dy
dy dt 4
= =
dx dx 2t
dt
Differential Calculus 403

dy 2
=
dx t
diff. again w.r.t. x.

d2 y F I af
1 d
dx 2 H K
=2 2
t dx
t [Note this step]

d2y 2 1 dx
2
= 2 Q = 2t
dx t 2t dt

d2 y 1 dt 1
2
= 3 = .
dx t dx 2t

d2y 1
= 3.
dx 2 when t =1 1

d2 y
= 1.
dx 2 when t =1
4. If y = e2t and x = log 3t, then

d2y
Find .
dx 2
y = e 2t
diff. w.r.t. t
dy
dt
= e2t
d
dt
a f
2t

dy
= e 2 t 2 = 2e 2t
dt
x = log 3t
diff. w.r.t. t.
dx 1 d
=
dt 3t dt
3t a f
dx 1
= 3
dt 3t
dx 1
=
dt t

dy
dy dt 2e 2 t
= =
dx dx 1
dt t
404 Basic Mathematics

dy
= 2te 2 t
dx
diff. w.r.t. t.

d2 y LM d i
d 2t d
a f OP
dx 2
N
= 2 t
dx
e + e 2t
dx
t
Q
= 2 LMt e a2t f + e OP
2
d y d dt
N dx dx Q
2t 2t
2
dx

= 2 LMte 2 + e OP
2
d y dt dt
N dx dx Q
2t 2t
2
dx

d2 y dt
2
= 2 e 2t 2t + 1
dx dx dx 1
Q =
dt t
d2 y dt
2
= 2e 2 t t 2t + 1 =t
dx dx
5. If y = ae mx + be mx , then prove that y2 m 2 y = 0

y = ae mx + be mx
diff. w.r.t. x.

y1 = ae mx
d
dx
a f
mx + be mx
d
dx
mx a f
af
y1 = ae mx m + be mx m a f
y1 = m ae mx be mx

diff. again w.r.t. x.

LM d
a f
mx be mx
d
a fOPQ
N
y2 = m ae mx
dx dx
mx

af a f
y2 = m ae mx m be mx m

y2 = m ae mx amf + be m mx

y2 = m 2 ae mx + be mx

y2 = m 2 y
Differential Calculus 405

y2 m 2 y = 0
Hence proved.

MISCELLANEOUS PROBLEMS:

I. One Mark Problems:


1. Differentiate xe + ex ee with respect to x.
Solution:
Let y = xe + ex ee
diff. w.r.t. x.
dy
= ex e1 + e x 0
dx
dy
= ex e 1 + e x
dx
2. Differentiate 2x + x2 logx with respect to x.
Let y = 2x + x2 logx
diff. w.r.t. x.
dy 1
= 2 x log 2 + 2 x
dx x

3. Differentiate e d
2 log x + x 2 i w.r.t. x.

Let y = e
d
2 log x + x 2 i
diff. w.r.t. x.
2 d log x + x 2 i d
dy
dx
=e
dx
ed
2 log x + x 2 ij
d
2 log x + x 2 i 2 F 1 + 2 xI
=e
Hx K
d
2 log x + x 2 i F 2 + 4 xI .
=e
Hx K
2
+4 x7
4. If y = 5 x , then find y1
2
+4 x7
y = 5x
diff. w.r.t. x.
406 Basic Mathematics

y1 = 5 x
2
+ 4x 7
log 5
d 2
dx
d
x + 4x 7 i
y1 = 5 x
2
+4 x7
a
log 5 2 x + 4 f
5. Differentiate e
d
log x 2 + 4 x i w.r.t. x.

Let y = e
d
log x 2 + 4 x i

y = x2 + 4x af
Q e log f a x f = f x
diff. w.r.t. x.
dy
= 2 x + 4.
dx

1 dy
6. If y = , then find .
3
x 2 dx

2
1 1
y= = =x 3.

d i
3 1
x2 x2 3

2

y=x 3.

diff. w.r.t. x.
2 5
dy 2 1 2
= x 3 = x 3
dx 3 3
2
= .
3x 5 3
dy
7. If y = 38x, then find
dx

y = 38 x
diff. w.r.t. x.
dy
dx
= 38 x log 3
d
dx
8 x a f
dy
dx
= 38 x log 3 8 a f
Differential Calculus 407

8. If y = x 2 4e x , then find y

y = x 2 4e x
diff. w.r.t. x.

y =
1
2 x 2 4e x

d 2
dx
d
x 4e x i
1
y = 2 x 4e x
2 x 4e
2 x

x 2e x
y =
x 2 4e x

af
9. If f x =
4
x 9
2 , then find f (x)

af d
f x = 4 x2 9 i 1

diff. w.r.t. x.

a f LNM d i OQP dxd d x


f x = 4 1 x 2 9
11 2
9 i
f a x f = 4 L d x 9i O 2 x
2
MN 2
PQ
f a x f =
8 x
d x 9i
2 2

1
10. If y = , then find y1
3
3x
1 1
y= = = 3 x 3
d3 i
x 13 3 x3

y = 3 x 3
diff. w.r.t. x.

F I d x
y1 = 3 x 3 log 3
H K dx

3

log 3 F I
1
y1 = 3 x 3
H 3K
408 Basic Mathematics

log 3 log 3
y1 = x 3
= 1 x 3 .
33 3

II. Two Marks Problems:

dy
1. If x2 + y2 = 10, then find at (1, 1)
dx
Consider x2 + y2 = 10
diff. w.r.t. x.
dy
2x + 2y =0
dx
dy
2y = 2 x.
dx

dy 2 x
=
dx 2y

dy x
=
dx y

dy 1
dx a1, 1f
=
a f
1

dy
= 1.
dx a1, 1f

dy
2. If x = 4t and y = 5t2, then find .
dx

dy
dy dt
Solution: =
dx dx
dt
Now
y = 5t2
diff. w.r.t. t.
dy
dt
= 5 2ta f
dy
= 10t
dt
Differential Calculus 409

Next, x = 4 t
diff. w.r.t. t.
dx
= 4 1 = 4.
dt

dy
dy dt 10t 5t
= = = .
dx dx 4 2
dt

af
3. If f x = log x 2 a, then find f (a)

af
f x = log x 2 a =
log e a
log e x 2

log e a
=
2 log e x

af
f x =
log e a
2

1
log e x
diff. w.r.t. x.

F I d log x
af
f x =
log e a
2
GH b

1
log e x g JK dx a f
2

af
f x =
log e a

1

1
2 b
log e x g 2
x

af
f a =
log e a
4 log e a a

af
f a =
1
4a

1 x dy
4. If y = log , then find
1+ x dx

y = log F
1 xI
1

H1+ xK
2

y = log F
1 1 xI
2 H1+ xK
410 Basic Mathematics

y=
1
2
a f a f
log 1 x log 1 + x

diff. w.r.t. x.
dy 1 1LM d
a f1 d
a fOPQ
=
N
dx 2 1 x dx
1 x
1 + x dx
1+ x

dy 1 L 1
= M a 1f 1O
1 + x PQ
1
dx 2 N 1 x

dy 1 L 1 1 O
dx 2 MN1 x 1 + x PQ
=

1 L 1 x 1 + x O
= M P
2 N a1 x fa1 + x f Q

dy 1 L 2 O
dx 2 MN1 x PQ
= 2

dy 1
= .
dx 1 x 2

e 2
j
5. If y = log x + 1 + x , then Prove that
dy
dx
=
1
1 + x2
.

Pr
Proof: e
oof: Consider y = log x + 1 + x
2
j
diff. w.r.t. x.
dy
=
1

dx x + 1 + x dx
2
d
x + 1+ x2 e j
dy
=
1
1+
1 LM
d
1 + x2 d i OP
dx x + 1 + x 2 2 1+ x MN
2 dx
QP
dy
=
1 LM1 + 1 a2 xfOP
dx x + 1 + x 2 MN 2 1 + x 2
PQ
dy
=
1 LM 1 + x + x OP
2

dx x + 1 + x 2 MN 1 + x PQ
2
Differential Calculus 411

dy 1
= .
dx 1 + x2
Hence proved.
6. If xmyn = am+n, then prove that
dy my
= .
dx nx
Consider xmyn = am+n.
Taking log on both sides log (xmyn) = log am+n

log x m + log y n = log a m + n

a
m log x + n log y = m + n log a.f
diff. w.r.t. x.

m
1
x
1 dy
+ n
y dx
=0 a f
Q m + n log a is constant

m n dy
+ =0
x y dx

n dy m
=
y dx x

dy my
= .
dx nx
Hence proved.
dy y log a
7. If y = ax+y, then prove that dx = 1 y log a

Consider y = ax+y
Consider log on both sides

log y = log a x + y

a f
log y = x + y log a
diff. w.r.t. x.
1 dy LM dy OP
y dx N
= log a 1 +
dx Q
1 dy dy
= log a + log a
y dx dx
412 Basic Mathematics

1 dy dy
log a = log a
y dx dx

LM
dy 1 OP
log a = log a
dx yN Q
dy log a
=
dx 1
log a
y

dy log a
=
dx 1 y log a
y

dy y log a
=
dx 1 y log a
Hence proved.
dy y
8. If yey = x, then prove that =
a f
dx x 1 + y
Consider
yey = x.
taking log on both sides

d i
log y e y = log x

log y + log e y = log x

log y + y log e = log x

log y + y = log x Q log e = 1


diff. w.r.t. x.
1 dy dy 1
+ =
y dx dx x

FG IJ
dy 1 1
dx yH K
+1 =
x

dy F 1 + y I 1
G J=
dx H y K x
Differential Calculus 413

dy y
=
dx x 1 + ya f
Hence proved.

Alieter:
Consider yey = x.
diff. w.r.t. x. using product rule.

y
d y
dx
d i
e + ey
dy
dx
=1

dy dy
y ey + ey =1
dx dx

d
dy y
dx
e y + ey = 1i
dy 1
=
a f
x
dx e y y + 1 But ye y = x , e y =
y
dy 1
=
dx x y + 1
y
a f
dy y
=
dx x y + 1a f
Hence proved.

III. 4 Marks Problems:

1. If x 1 + y + y 1 + x = 0 and x y , prove that

dy 1
=
dx 1 + x a f 2

Consider

x 1+ y + y 1+ x = 0

x 1 + y = y 1 + x
Squaring

dx 1+ y i = d y
2
1+ x i 2
414 Basic Mathematics

a f a f
x 2 1 + y = y2 1 + x

x 2 + x 2 y = y2 + y 2 x

x 2 + x 2 y y2 y 2 x = 0

a
x 2 y 2 + xy x y = 0 f
a x yfa x + yf + xy ax yf = 0
a x yf x + y + xy = 0
x y = 0 or x + y + xy = 0

Q x y, x + y + xy = 0
y + xy = x

a f
y 1 + x = x

x
y=
1+ x
diff. w.r.t. x.

dy
a1 + xf FH dxd a xf a xf dxd a1 + xfIK
dx
=
a1 + xf 2

dy a1 + x fa 1f + x a1f 1 x + x 1
dx
=
a1 + xf 2
=
a1 + xf a1 + xf
2
= 2

Hence proved.
2. If x = at2, y = 2at, then prove that

d2y 1
2
=
dx 2 at 3
Consider x = at2
diff. w.r.t. t.
dx
dt
= a 2t a f ...(1)

y = 2at
diff. w.r.t. t.
dy
dx
= 2a 1 af
Differential Calculus 415

dy
= 2a ...(2)
dt

dy
dy dt 2a 1
= = =
dx dx 2 at t
dt

dy 1
=
dx t
diff. w.r.t. x.

d2 y 1 dt
2
= 2
dx t dx

d2y 1 1F I
dx 2
= 2
t H K
2 at

d2y 1
2
=
dx 2 at 3
Hence proved.
3. Differentiate x from 1st principles:

Let y = x
Let x be an increment given to x.
y be the corresponding increment in y

y + y = x + x

y = x + x y

y = x + x x

Divide by x and take lim


x0

y x + x x
lim = lim
x 0 x x 0 x
Add and subtract x in the Denominator of RHS.

y x + x x
lim = lim
x 0 x x 0 x + x x
416 Basic Mathematics

a x + x f 1 1
y 2 x2
lim = lim
x 0 x x 0 x + x x

x n an
RHS is of the form lim
xa x a

with x + x in place of x
x in place of a.
and 1/2 in place of n.
As x 0
x + x x.
By applying the formula.

x n an
lim = na n 1
x a x a

we get

a f
1 1
y x + x 2 x2
lim = lim
x 0 x x 0 x + x x
1
dy 1 2 1
= x
dx 2
1
dy 1 2 1
= x =
dx 2 2 x

d
dx
d x i = 2 1x .
4. If xmyn = (x + y)m+n, then prove that
dy y
= .
dx x
Proof: xmyn = (x + y)m+n
Proof:
Consider log on both sides

d i a f a f
log x m y n = m + n log x + y

log x + log y = a m + nf log a x + y f


m n

m log x + n log y = a m + nf log a x + yf


diff. w.r.t. x.
Differential Calculus 417

m
1
x
+ n
1 dy
y dx
a
= m+n
1

d
x + y dx
x+yf a f
m n dy m + n dy LM OP
+ =
x y dx x + y
1+
dx N Q
m n dy m + n m + n dy
+ = +
x y dx x + y x + y dx

n dy m + n dy m + n m
=
y dx x + y dx x + y x

LM
dy n m + n OP a f a f
m+n xm x+y

N
dx y x + y
=
Q x x+y a f
dy L n a x + yf a m + nf y O mx + nx mx my
M y a x + yf PQ = x a x + yf
dx N

dy L nx + ny my ny O nx my
dx MN y PQ = x
dy L nx my O nx my
dx MN y PQ
=
x

dy y
=
dx x
Hence proved.

e j d i
m
5. If y = x + x 2 + 1 , then prove that 1 + x 2 y2 + xy1 m 2 y = 0 .

Consider

e j
m
y = x + x2 + 1

diff. w.r.t. x

e j e j
m 1 d
y1 = m x + x 2 + 1 x + x2 + 1
dx

L O
e j MM1 + 2 d iPP
m 1 1 d 2
y1 = m x + x 2 + 1 x +1
N x + 1 dx
2
Q
418 Basic Mathematics

L O
e j MMN1 + 2 x1 + 1 2 x PPQ
m 1
y1 = m x + x 2 + 1
2

F x +1 + xI
e + 1j G
H x + 1 JK
m 1 2
y1 = m x + x 2
2

ex + j
m 1+1
x2 + 1
y1 =m
x2 +1

e j
m
m x + x2 + 1
e j
m
y1 = But x + x 2 + 1 =y
x2 + 1

my
y1 =
x2 + 1
Cross multiplying

y1 e x 2 + 1 = my j
Squarring

d
y12 x 2 + 1 = m 2 y 2 i
Diff. again w.r.t. x.

d i
y12 2 x + x 2 + 1 2 y1 y2 = m 2 2 yy1

d
2 y1 xy1 + x 2 + 1 y2 = 2 y1 ym 2 i d i
d
xy1 + x 2 + 1 y2 = m 2 y i
Rearranging

d x + 1i y
2
2 + xy1 m 2 y = 0

Hence proved.
6. If y = ex log x, then prove that
a
xy2 2 x 1 y1 + x 1 y = 0f a f
Given: y = e log x
x

diff. w.r.t. x.
Differential Calculus 419

1
y1 = e x + log x e x
x

ex
y1 = +y Q y = e x log x
x

e x + xy
y1 =
x
Cross multiplying

xy1 = e x + xy ...(1)
diff. again w.r.t. x.

af
xy2 + y1 1 = e x + x y1 + y 1

xy2 + y1 e x xy1 y = 0
But from (1) xy1 = ex + xy

e x = xy1 xy

b g
xy2 + y1 xy1 xy xy1 y = 0

xy2 + y1 xy1 + xy xy1 y = 0

xy2 + y1 2 xy1 + xy y = 0

a f a f
xy2 y1 1 + 2 x + y x 1 = 0

xy2 y a2 x 1f + y a x 1f = 0
1

Hence proved.
7. Differentiate eax from first principles:
Let y = eax
Let x be an increment give to x.
y be the corresponding increment in y

y + y = e b
a x + x g

y = e ax + ax y

y = e ax + ax e ax

y = e ax e ax e ax

y = e ax e ax 1
420 Basic Mathematics

Divide by x and take lim


x0

lim
ax
y e e 1
=
ax
e j
x 0 x x
Multiply and divide by a in RHS.

ex 1
Q lim =1
x0 x

lim
ax
y ae e 1
=
ax
e j
x 0 x ax

dy
= a e ax 1
dx

dx
d i
d ax
e = ae ax

e j d
8. If y = log x + a + x , then prove that a + x y2 + xy1 = 0.
2 2 2 2
i
e
Consider y = log x + a + x
2 2
j
diff. w.r.t. x.

y1 =
x+ a +x
1
2 2

d
dx
e
x + a2 + x 2 j
LM1 + 1 d a + x OP
MN 2 a + x dx d iP
1
y1 = 2 2

x+ a +x 2 2 2
Q 2

y1 =
1 LM1 + 1 a0 + 2 xfOP
x + a2 + x 2 MN 2 a + x 2
PQ 2

y1 =
1 LM a + x + x OP
2 2

x + a2 + x 2 MN a + x PQ2 2

1
y1 = .
a + x2
2

a 2 + x 2 y1 = 1
Differential Calculus 421

Squaring
da 2
+ x 2 y12 = 1i
diff. again with respect to x.
da 2
i
+ x 2 2 y1 y2 + y12 2 x = 0 a f
d
2 y1 a 2 + x 2 y2 + xy1 = 0 i
da 2
i
+ x 2 y1 + xy1 = 0
Hence proved.
9. If xy = exy, then prove that
dy log x
=
dx 1 + log xa f 2

Proof: x y = e x y
Proof:
Consider log on both sides

log x y = log e x y
y log x = x y log e a f
y log x = x y ...(1) Q log e = 1
diff. w.r.t. x.
1 dy dy
y + log x = 1
x dx dx
y dy dy
+ log x + =1
x dx dx
dy
dx
a
log x + 1 = 1
y
x
f
dy
a
log x + 1 =
xy
f
dx x LM Q x y = y log x OP
dy
=
xy
a f MM P
dx x 1 + log x x = y log x + y P
MM PP
MM x = y a1 + log x fP
dy y log x
=
dx x 1 + log x a f P
But x y = y logx from (1) MM x
= a1 + log x f P
PP
dy
=
log x MM y
y 1
P
dx 1 + log xa f 2
N
=
x 1 + log x Q
Hence proved.
422 Basic Mathematics

x...
x
10. Differentiate x with respect to e8x.
x ...
Let u = x x and v = e 8 x .

du
du dx
To find =
dv dv
dx
x ...
Now u = x x

u = xu
Consider log on both sides

log u = log x u

log u = u log x
diff. w.r.t. x.
1 du 1 du
= u + log x
u dx x dx
1 du du u
log x =
u dx dx x

LM
du 1 u OP
dx uN log x = .
x Q
u
du u2
a f
= x =
dx 1 log x x 1 u log x
u
Now v = e8x
diff. w.r.t. x.
dv
dx
= e8 x
d
dx
8x a f
dv
= e 8 x 8.
dx
Hence

u2
a f
du
du dx x 1 u log x
= =
dv dv 8e 8 x
dx
Differential Calculus 423

du u2 N
=
a f
dv 8 x 1 u log x e 8 x where u = x
xx

REMEMBER:

dy
y
dx

xn nxn1
x 1
x2 2x
x3 3x 2
1
x 2 x
1 1

x x2
1 2
x2 x3
1 3
x3 x4
ex ex
ax x
a loga
1
logx
x
du
Cu C
dx
du dv
uv
dx dx

I II I
d
dx
af
II + II
d
dx
af
I

Nr.* Dr.
d
dx
a f a f a f
Nr Nr
d
dx
Dr.
Dr.** a f
Dr. 2

*: Numerator **: Denominator


424 Basic Mathematics

Chain rule: If y = g(u) & u = f(x)


Then y = g[f(x)] is differentiated by chain rule
dy dy du
=
dx du dx
Implicit dif
diffferentia
erentiation: Given function f(x, y) = 0.
entiation:
dy dy
diff. w.r.t. x., take common among the terms containing and shift the remaining terms to
dx dx
dy
RHS. Then find .
dx
Par ametr
arametr
ametricic dif
diffferentia
erentiation:
entiation:
Given x = f (t) and y = g(t).

dy
dy dy dt
Then to find , use dx = dx
dx
dt
Second or der der
order deriivative:
tiv
dy
If y = f (x), then by differentiating we get or y or y1 or f (x). This is a function of x. By
dx
d2y
differentiating this again with respect to x we get or y or f (x) or y2.
dx 2

EXERCISE

(1) Dif
Diffferentia
erentia te the ffollo
entiate ollowing functions fr
ollowing om ffir
from ir st pr
irst inciples:
principles:
(a) x n (b) e ax

(c) logax (d) x


(e) 7x
(2) Dif
Diffferentia
erentiate the ffollo
entiate ollo espect to x:
wing with rrespect
ollowing
(a) x 4 + 3e x 7 (b) 3x 3 + 8 x + 9

1 1
(c) x+ 4 (d) e x + e e e + e
x 3x

(e) a x + x a e a (f) 7 x + x 7 + e 7 7 e
Differential Calculus 425

dy
(3) Find if
dx

d
(a) y = 3x + 8 5x + 7
2 3
id i (b) y = d x +7 id 3
x 6 i
F 1 I x2 + x + 2
(c) y = x 5 x + H 12 K (d) y =
x2 2x + 3

e x ex 1 + x2
(e) y = (f) y =
e x + e x 1 x2
dy
(4) Find if
dx
1
(a) y = e
5x
+ ex + e x (b) y = e 7 x + 86 x + 7

(c) y = e
6x
d
log x 3 + 8 i e
(d) y = log x + a + x
2 2
j
d i 1 F
xa I
(e) y = log e + e
x x
(f) y =
2a
log
H
x+a K
F e + 1I
GH e 1JK
x
(g) y = e log
x
x (h) y = x x + a + a log x + x + a
2 2 2 2 2
e j
Fx I LM F x 2 I OP
GH x + 1 x2
JK
34
(i) y = log (j) y = log e
MN H x + 2 K PQ
x

1 x2

dy
5. Find if
dx

(a) x 2 + y 2 = 10 (b) y = log x + log + log x + ...



x ...
(c) y = x x

a f
6. If (a) y 2 2 a x = x 3 . Then prove tha
pro thatt
dy
dx
a f
at a, a = 2.

dy 2 x 7 y
(b) If x2 + 3y2 7xy = 5 Then prove that dx = 7 x 6 y

dy 1
(c) If x y, x 1 + y + y 1 + x = 0 . Then prove that dx = 1 + x a f 2
426 Basic Mathematics

(d) If ax2/3 + by2/3 = (a2 b2)2/3

F I
1
dy y
H K
3
Then prove that = .
dx x

dy
7. Find if
dx

1 t2 2t
(a) x = t 3 , y = t 2 + 1 (b) x = , y=
1+ t 2
1 + t2

3at 3at 2
(c) x = , y =
1 + t3 1 + t3
8. Dif
Diffferentia
erentiate
entiate
(a) log e x with respect to ex

(b) log10 x with respect to x2


(c) 10x with respect to 5x
dy y
(d) If xmyn = (x + y)m+n, then prove that = .
dx x
dy
(e) If ex + ey = ex+y, then prove that = e y x
dx
dy log x
=
(f) If xy = exy, then prove that
dx a
1 + log x f 2 .

d2y
9. Find if
dx 2

(a) x 2 + y 2 = a 2 (b) y = x 2 e x

x 2 y2
(c) + =1 (d) x 3 y 3 = a 5
a2 b2

e j d i
m
10. (a) If y = x + 1 + x 2 , then prove that 1 + x y2 + xy1 m y = 0 .
2 2

e j
(b) If y = log x + a + x , prove that a + x y2 + xy1 = 0 .
2 2 2 2
d i
(c) If y = aemx + bemx, then prove that y2 m2y = 0.
f a f a
(d) If y = exlogx, then prove that xy2 2 x 1 y1 + x 1 y = 0.

1) , prove that d x 1i y + 2 x a1 nf y 2ny = 0.


n 2
(e) If y = (x2 2 1
Differential Calculus 427

(f) If y = a + x
2 2
d i , prove that d x
6 2
i
+ a 2 y2 10 xy1 12 y = 0.

n +1
(g) If y = ax +
b
xn
, then prove that x 2 y2 + n n + 1 y = 0 . a f
d2 y dy
(h) If y = (a + bt)ent, then prove that 2
+ 2n + n 2 y = 0.
dt dt

e j e j d i
n n
(i) If y = a x + x 1 + b x x2 1 , then prove that x 1 y2 + xy1 n y = 0.
2 2 2

ANSWERS

1 1
1. (a) nxn1 (b) aeax (c) (d) (e) 7x log 7.
x 2 x

1 5

1 13 4 x4
2. (a) 4x3 + 3ex (b) 9x2 +8 (c) + (d) ex
2 x 2x x 4
(e) ax loga + axa1 (f) 7x log7 + 7x6.

F I
d x + 7i GH 13 x JK + d x 6i FGH 2 1 x IJK
2

d3x + 8i15x + d5x + 7i 6 x



2 2 3 3 3
(3) (a) (b)

x a 5f + F 5 x + I
1 1 d x 2 x + 3ia2 x + 1f d x + x + 2i a2 x 2f
2 2

(c) H 12 K 2 x (d)
d x 2 x + 3i 2 2

4 4x
(e)
de x
e x 2
i (f)
d1 x i 2 2

1
ex (b) 7e 7 x + 6 86 x log 8
4. (a) 5e 5 x
2
+ e x
x

e 6x 3x 2 1
(c) 6 3 (d)
2 6x x +8 x 2 + a2

ex + ex 1
(e) (f)
e x ex x a2
2
428 Basic Mathematics

x LM ex + 1OP 2e 2 x
2x
(g) e log
N Q
e 1 e 1
x (h) 2 x 2 + a 2

2 x2 1
(i)
d2 x 1i
2
1 x 2 (j)
x2 4

x 1 y2
5. (a) y a
(b) x 2 y 1 f (c) 1
a
x y log x f
2 t2 1 x 2t t 4
7. (a) (b) or (c)
3t 2t y 1 2t 3

e x 1 10 x log 10
8. (a) (b) 2 x 2 log 10 (c)
x e 5 x log 5

b 4
9. (a)
a2
y3
(b) e
x
dx 2
+ 4x + 2 i (c) 2 3
a y
( d)
15 y
4x2
.
17
Application of Derivatives
Derivatives have many applications. To mention a few, we use differentiation in finding 1. Equation of
tangent and normal. 2. Length of subtangent and subnormal 3. Angle between the curves 4. Rate
measure i.e., variation with respect to time 5. Maximum and minimum values of the function.
Derivatives are also used in finding marginal cost and marginal revenue.

17.1 DERIVATIVE AS A RATE MEASURE:


Velocity: Rate of change of displacement is called velocity. If v is the velocity, s is the displacement
and s = f (t) Then

Velocity =
ds
dt
af
= f t

Acceler
Acceleraation: The rate of change of velocity is called acceleration. If a is the acceleration v is the
velocity. Then

dv d ds F I
d 2s
a= =
dt dt dt H K
= 2
dt
Note:
1. Rate means differentiation with respect to time, t.
dA
Rate of increase of area = .
dt
dV
Rate of decrease of volume = and so on.
dt
2. Formulae:
4 3
(a) Area of square = (side)2 = S2. (d) Volume of sphere = r .
3
(b) Area of circle = (radius)2 = r2. (e) Volume of a cylinder = r2h.
1 2
(c) Surface area of Sphere = 4r2 (f) Volume of a cone = r h.
3
430 Basic Mathematics

WORKED EXAMPLES:
1. If the displacement of a particle at time t seconds is s = 3t2 7t + 6, then find the velocity and
acceleration at t = 1 second.
Given: s = 3t2 7t + 6
diff. w.r.t. t.
ds
dt
a f af
= 3 2t 7 1

ds
Velocity = = 6t 7
dt
Velocity when t = 1 sec. = 6(1) 7.
= 6 7 = 1 unit/sec.
Now

Velocity = 6t 7
v = 6t 7
diff. w.r.t. t.
dv
dt
af
=6 1 0

dv
=6
dt
acceleration = 6 units/sec2
acceleration when t = 1 is 6 units/sec2.
2. When the brakes are applied to the moving car, the car travels a distance s mts in t seconds given
by s = 6t 3t2, when and where does the car stops?
Solution: Car stops when velocity = 0.
Now s = 6t 3t2.
diff. w.r.t. t.
ds
dt
af a f
= 6 1 3 2t

ds
= 6 6t
dt
ds
Velocity = =0
dt
0 = 6 6t
6 = 6t t = 1 .
Application of Derivatives 431

Car stops when t = 1 sec.


Distance travelled in 1 sec.
s = 6t 3t 2 when t = 1 sec.

af af
s=6 1 3 1 2

s =63
s = 3 mts.
So car stops after travelling a distance of 3 mts.
3. With usual notation, if s = at + b. Where a and b are constants then prove that velocity is constant
and acceleration is zero.
Solution: Given s = at + b
diff. w.r.t. t.
ds
dt
af
= a 1 + 0 3 a and b are constants

ds
= velocity = a = a constant.
dt
diff. w.r.t. t.

d 2s
= 0 3 a is constant.
dt 2
acceleration = 0
4. A square plate expands uniformly, the side is increasing at the rate of 6 mm/sec.
What is the rate of increase in area when side is 13 mm.
Solution: Given
Rate of increase of side = 6 mm/sec.
dS
i.e., = 6 mm sec.
dt
dA
To find: when S = 13 mm.
dt
We have Area of square = side side
A = S2
diff. w.r.t. t.
dA dS
= 2S
dt dt
dA
dt
a fa f
= 2 13 6
432 Basic Mathematics

dA
= 156 mm 2 sec.
dt
Rate of increase of area = 156 mm2/sec.
5. A stone is dropped into a pond, waves in the form of circles are generated and the radius of the
outermost wave increase at the rate of 2 mm/sec. How fast is the area increasing (a) when the
radius is 5 mm (b) after 3 sec.
Solution: If r is the radius at time t;
dr
Then given: = 2 mm sec
dt
dA
To find (a) when r = 5 mm
dt
dA
( b) when t = 3 sec
dt
We have
Area of circle = r2
A = r2
diff. w.r.t. t.
dA
dt
a f
= 2r
dr
dt
...(1)

(a)
dA
dt
b a fg
= 2 5 2

dA
= 20
dt
dA
= 20 mm 2 sec
dt

(b)
dA
dt
a f
= 2r
dr
dt
Now we have radius is increasing at the rate of 2 mm/sec.
After 3 seconds, radius = 3 2 = 6 mm.
dr
(3 = 2 mm sec
dt
For 1 sec radius is 2 mm.
For 3 sec. radius is 3 2 = 6 mm)
From (1)
Application of Derivatives 433

dA
dt
a f
= 2r
dr
dt
dA
dt after 3 seconds
a f
= 2 6 2

dA
= 24
dt after 3 sec
After 3 seconds, Area is increasing at the rate of 36 mm2/sec.
6. Water flows into a cylindrical tank of radius 4 mts at 80,000 cc/hr. How fast water level is raising?
Solution: If r is the radius, h is the height. The volume V is given by
V = r 2 h
dV
Given = 80,000 cc hr.
dt
dh
If h is the height of the water level, represent rate of increase in water level.
dt
r = radius = 4 mts = 400 cm = a constant while filling the water in the cylindrical tank.
Consider
V = r 2 h
diff. w.r.t. t.
dV
dt
d i
= r2
dh
dt

a f
80,000 = 400 2
dh
dt
80,000 dh
=
160000 dt
1 dh
= =
2 dt
1
Water level is rising at the rate of cms/hr.
2
7. Sand is poured at the rate of 30 cc/sec and it forms a conical pile in which the diameter of the
circular base is always equal to one third the height. At what rate height of the pile is increasing
when the height is 30 cm.
Solution:
2 Radius = Diameter
434 Basic Mathematics

1
Given Diameter = h
3
1
2 radius = h
3
1
r= h ...(1) 3h
6
Fig. 17.1
dV
= 30 cc sec
dt
dh
To find when h = 30 cm.
dt
1 2
We have volume of the cone = r h.
3
1
V = r 2 h
3

1 1 F I 2
V=
3
h
6 H K h [from (1)]

1 h 2 h h 3
V = =
3 36 108

h3
V=
108
diff. w.r.t. t.
dv LM dh OP
=
dt 108 N
3h 2
dt Q
L
M 3 a30f OP
dh
108 N dt Q
30 =
2

30 108 dh
= 3 900
dt
30 108 dh
=
3 900 dt
dh 6
=
dt 5
Application of Derivatives 435

6
Height of the pile is increasing at the rate of cm/sec.
5
8. A spherical snow ball is forming so that its volume is increasing at the rate of 8 cm/sec. Find the
rate at which the radius is increasing when the snow ball is 2 cm in diameter. Also find the rate
of increase in surface area.
Solution.
dV
Given: = 8 cm sec.
dt
dr
To find: when diameter = 2 cm.
dt
ds 2
and i.e. radius = 1 cm .
dt 2
We have for a sphere,
4 3
V= r
3
diff. w.r.t. t.
dV 4 dr
= 3/ r 2
dt 3/ dt
dV dr
= 4 r 2
dt dt

af
8 = 4 1 2
dr
dt
dr 8 2
= =
dt 4
dr 2
= cm sec.
dt
Now surface area = 4 r2.
diff. w.r.t. t.
ds
dt
= 4 2r a f
dr
dt
ds
dt
= 4 2 1
2

af
ds
= 16 sq cm sec.
dt
436 Basic Mathematics

9. A ladder 13 ft long rests with its ends on a horizontal floor and against a smooth vertical wall. If
the upper end is coming downwards at the rate of 1 ft/min. Find the rate at which the lower end
moves, when the upper end is 5 ft from the ground.
Solution: Let PQ be the ladder.
At time t, Let OP = y,
OQ = x
From figure OP2 + OQ2 = PQ2
P
y2 + x2 = 132
dy
Given = 1ft min. y
dt
dx
To find when y = 5ft. 0
dt x Q

dx Fig. 17.2
i.e., To find when y = 5 ft. and x = 12 ft.
dt

y 2 + x 2 = 132

5 2 + x 2 = 169

x 2 = 169 25

x 2 = 144

x = 144
x = 12
From fig.,

y 2 + x 2 = 132
diff. w.r.t. t.
dy dx
2y + 2x =0
dt dt
dx dy
2x = 2 y
dt dt
dx 2 y dy
=
dt 2 x dt
dx y dy
=
dt x dt
dx 5
= 1
dt 12
Application of Derivatives 437

dx 5
=
dt 12
5
Lower end moves at the rate of ft/min. Negative sign shows as x increases, y decreases.
12
10. An aeroplane at an altitude of 400 kms flying horizontally at 500 km/hr passes directly over an
observer. Find the rate at which it is approaching the observer when it is 500 kms away from him.
Solution. Let at time t A be the position of the
A x B
aeroplane, B be the point on the path directly above
the observer O. Let AB = x. OA = y.
From fig. AB 2 + OB 2 = OA 2 y 400 kms

x 2 + 400 2 = y 2
diff. w.r.t. t. O
dx dy Fig. 17.3
2x + 0 = 2y ...(1)
dt dt
dx
Given = 500 km hr
dt
[ve Sign is taken since x is decreasing]
dy
To find when y = 500 kms.
dt
From fig.

x 2 + 400 2 = y 2

x 2 + 400 2 = 500 2 a3 y = 500f


x 2 = 500 2 400 2

x 2 = 250000 160000

x 2 = 90000
x = 300
dx
Substituting x = 300, y = 500 and = 500 in (1) we get
dt

a fa f a f
2 300 500 = 2 500
dy
dt
dy
300 =
dt
438 Basic Mathematics

dy
= 300 kms hr.
dt
The aeroplane is approaching at the rate of 300 km/hr.

17.2 MAXIMA AND MINIMA:


Let y = f (x) be a continuous function. As x increases, if y also incr
increases, eases then y = f (x) is said to be an
increases
increasing function.
increasing
Ex.: y = x + 3 is an increasing function



increase

x 0 1 2 3 .......
3
y 3 4 5 6 .......
increase

Fig. 17.4

As x incr eases if y decr


increases eases then the function y = f(x) is said to be a decr
decreases easing function
decreasing function.
Ex.: y = 7 x is a decreasing function.



increase

x 0 1 2 3 .......
3 y 7 6 5 4 .......
decrease

dy
Note that for an increasing function, > 0 i.e. positive.
dx Fig. 17.5
dy
For a decreasing function, < 0 i.e., negative.
dx
dy
y is said to be stationary at a point if it neither increases nor decreases. At such point =0.
dx
If a continuous function increases upto a certain value and then decreases from that value, then that
value is called a maximum value of a function.
Similarly if continuous function decreases to a value and then increases, then that value is called
minimum value of the function.
Note that a continuous function may attain maxima/minima at several points or it may neither have
maxima nor minima.
In the figure, the points P, Q, R, S, are points of maxima. The points A, B, C, are points of minima.
Between any 2 maxima there must be a minima and vice versa.
Application of Derivatives 439

Fig. 17.6

DERIVATIVE TEST FOR MAXIMA AND MINIMA:


Let y = f (x) be the given function. To find the maximum and / or minimum value of the function. We
dy
find and equate it to zero. Let x = a, x = b and x = c be the points.
dx

d2 y d2y
We find , if is greater than zero, then x = a is a point of minima. Minimum value
dx 2 at x = a dx 2 at x = a
of the function is y = f (a).

d2 y d2 y
Next we find if is less than zero, then x = b is a point of maxima. Maximum
dx 2 at x = b dx 2 at x = b
value of the function is y = f (b).

d2y
If is equal to zero, then at x = c, the function neither attains maxima, nor minima. The
dx 2 at x = c
point x = c is called a point of inflexion.

WORKED EXAMPLES:
1. Find the maximum and minimum value of the function 2x3 15x2 + 36x + 10.
Solution: Let y = 2x3 15x2 + 36x + 10.
diff. w.r.t. x.
dy
dx
d i a f af
= 2 3 x 2 15 2 x + 36 1 + 0

dy
= 6 x 2 30 x + 36. ...(1)
dx
440 Basic Mathematics

dy
At extremum, =0.
dx

0 = 6 x 2 30 x + 36 by 6.
x 2 5x + 6 = 0

x 2 3x 2 x + 6 = 0
3 x
a f a f
x x3 2 x3 =0 6x 2
2 x
a x 2fa x 3f = 0 5 x
x = 2 or x = 3 .
Consider equation (1)
dy
= 6 x 2 30 x + 36
dx
diff. w.r.t. x.
d2 y
dx 2
a f
= 6 2 x 30 1 + 0 af
d2 y
= 12 x 30
dx 2
d2 y
dx 2 at x = 2
af
= 12 2 30

= 6 < 0 negative.
x = 2 is a point of maxima.
Maximum value of the function
2 x 3 15 x 2 + 36 x + 10
is af
2 2 3
15 2af 2
af
+ 36 2 + 10
af
2 8 60 + 72 + 10
16 60 + 72 + 10
= 98 60 = 38.
Now
d2 y
= 12 x 30
dx 2
d2 y
dx 2 at x = 3
af
= 12 3 30 = 6 > 0 positive

x =3 is a point of minima.
Application of Derivatives 441

Minimum value of the function


2x 3 15 x 2 + 36 x + 10 is

af af af
2 3 3 15 3 2 + 36 3 + 10

2a27f 15a9f + 108 + 10


54 135 + 118
172 135 = 37.
2. Find two numbers such that their sum is 20 and their product is maximum.
Solution. Let the 2 numbers be x and y.
Given: Their sum = 20.
x + y = 20 y = 20 x
To find: x and y such that their product is maximum.
Their product = xy
P = xy

a
P = x 20 x f
P = 20 x x 2
dP
For P to be extremum, =0
dx

P = 20 x x 2
diff. w.r.t. x.
af
0 = 20 1 2 x
2 x = 20
x = 10.
So the numbers are x = 10 and y = 20 10 = 10. Reqd. Numbers are: 10 and 10.
3. Find two numbers whose sum is 16 and sum of their cubes is minimum.
Solution: Let the 2 numbers be x and y.
Given: Their Sum = 16.
x + y = 16
y = 16 x ...(1)
To find: x and y such that sum of their cubes is minimum.

i.e., x 3 + y 3 Should be minimum.


442 Basic Mathematics

Let P = x 3 + y3

P = x 3 + 16 x a f 3
[from (1)]
For P to be extremum,
dP
=0
dx

Now P = x 3 + 16 x a f 3

diff. w.r.t. x.
dP
dx
a
= 3x 2 + 3 16 x f 2

d
dx
a
16 x f
dP
dx
a f a1f
= 3x 2 + 3 16 x 2

0 = 3 x 3 a16 x f
2 2

0 = 3 x d16 + x 32 x i
2 2 2

0 = x 2 256 x 2 + 32 x
0 = 256 + 32 x
32 x = 256
256 16
x= = = 8.
32 2
So the numbers are x = 8 and y = 16 8 = 8 i.e. 8 and 8.
4. Prove that logx do not have maxima or minima.
oof: Let y = log x.
Proof:
Pr
dy
For maxima or minima, =0
dx
y = log x

dy 1
=
dx x
1
0=
x
1
=0
x
Application of Derivatives 443

x has no finite value.


logx do not have maxima or minima.
5. The product of 2 positive numbers is 36. Find the minimum value of their sum.
Solution: Let the 2 number be x and y.
Given: Their product = 36.
xy = 36

36
y=
x
Their sum = x + y.
36
S=x+
x
dS
For S to be extremum =0
dx
36
S=x+
x
diff. w.r.t. x.

dS 1 F I
dx
= 1 + 36 2
x H K
36
0 = 1
x2
x 2 36
0=
x2
x 2 36 = 0

x 2 36 x = 6

dS 36
=1 2
dx x
diff. w.r.t. x.

Now
d2S
dx 2
d
= 36 2 x 3 i
d 2 S 72
=
dx 2 x 3
d2S 72
2 = 3 > 0 positive.
dx at x = 6 6
444 Basic Mathematics

x = 6 is a point of minima.
36 36
y= = = 6.
x 6
36
Minimum value of their sum = 6 +
6
= 6 + 6 = 12.
6. Prove that a maximum rectangle that can be drawn with a constant perimeter is a square.
oof: Let x be the length and y be the breadth of a rectangle.
Proof:
Pr
Given: Perimeter = constant.
Sum of all sides = constant.
x + y + x + y = 2k say a f D x C

2 x + 2 y = 2k

a f
y y
2 x + y = 2k

x+y=k A x B

y=kx Fig. 17.7


Area of rectangle = Length Breadth
A = xy

A=x kx a f
A = kx x 2
dA
For area to be extremum, =0
dx

A = kx x 2
diff. w.r.t. x.
dA
dx
af
= k 1 2x

0 = k 2x
2x = k
k
x=
2

dA d2A
Consider = k 2 x diff. again w.r.t. x, = 2 < 0 for all
dx dx 2
values of x.
Application of Derivatives 445

A attains maxima for all values of x.


Now,
k 2k k k
y=kx=k = =
2 2 2
k
Hence x=y=
2
k
Length 2x = 2 =k
2
k
Breadth 2y = 2 =k
2
Rectangle has Length = Breadth.
Hence it is a square.
7. Prove that the maximum rectangle that can be inscribed in a circle is a square.

D x C

y y

A x B

Fig. 17.8

Let ABCD be the rectangle with length x and breadth y inscribed in a circle of radius say a.
OB = OD = a
BD = 2 a
From right angled triangle, ABD,

AB 2 + AD2 = BD2

a f
x 2 + y 2 = 2a 2

x 2 + y2 = 4a 2

y 2 = 4a 2 x 2

y = 4a 2 x 2
446 Basic Mathematics

Now, Area of the rectangle = Length Breadth


A= xy

A = x 4a 2 x 2

A = x 4a 2 x 2

dA
For A to be maximum/minimum =0
dx

A = x 4a 2 x 2
diff. w.r.t. x.
dA
dx
= x
d
dx
e j
4 a 2 x 2 + 4a 2 x 2
d
dx
af
x

dA
= x
1

d
d
4a 2 x 2 + 4a 2 x 2 1 i af
dx 2 4a x
2 2 dx

dA
=
x
dx 2 4a x
2 2
a f
2 x + 4 a 2 x 2

2 x 2
0= + 4a2 x 2
2 4a x 2 2

x 2 + 4a 2 x 2
0=
4a 2 x 2

0 = 2 x 2 + 4 a 2
2 x 2 = 4a2

x 2 = 2a 2

x= 2a

d2 A
Also is < 0.
dx 2 at x = 2a

A attains maxima at x = 2 a .

Now y = 4a 2 x 2 = 4a 2 d 2 ai 2

y = 4a 2 2 a 2 = 2 a
Application of Derivatives 447

Rectangle has maximum area when its length = 2 a and Breadth = 2 a , i.e., when its length =
Breadth = 2 a , i.e., when it is a square.
8. What is the largest size rectangle that can be inscribed in a semicircle of radius r unit so that 2
vertices lie on the diameter.
Solution: Let AB be the diameter of a semicircle. S 2x R
Let 2 vertices P and Q of the rectangle PQRS lie on
the diameter.
Let PQ = RS = 2 x
2y 2y
PS = QR = 2 y
O
Join OR. Given: r is the radius of the semicircle.
A P 2x Q B
From fig.
OQ 2 + QR 2 = OR 2 Fig. 17.9

x2 + 2ya f 2
= r2

x 2 + 4y2 = r 2

4y2 = r2 x 2

1 2
y2 = r x2 ...(1)
4
Now,
Area of the rectangle = Length Breadth
A = 2x 2y
A = 4 xy

1 2
A = 4x r x2 from (1)
2

A = 2x r2 x2

dA
Area is extremum when =0
dx

Now A = 2x r2 x2
diff. w.r.t. x.
dA dLM e jOQP + d
a f
dx
= 2x
dxN r2 x2 r2 x2
dx
2x
448 Basic Mathematics

dA
dx
= 2x
1
2 r x
2

d 2
2 dx
d
r x2 + r2 x2 2 i
dA
=
x
a f
2 x + 2 r 2 x 2
dx r x2
2

0=
d
2 x 2 + 2 r 2 x 2 i
r x
2 2

0 = 2 x 2 + 2r 2 2 x 2

4 x 2 = 2r 2

r2
x2 =
2
r
x= .
2

d2A
Also is negative.
dx 2 at x = r
2

Hence A attains maxima when x = 2 r.

r
Area of rectangle is maximum when its length = 2 x = 2 = 2r
2

1 2 FG r IJ 2
Breadth = 2 y = 2
2
r
H 2K
r2 r
Breadth = r 2 = .
2 2

Maximum area = Length Breadth

r
=r 2
2

= r2.
9. A box is constructed from a square metal sheet of side 60 cm by cutting out identical squares from
the four corners and turning up the sides. Find the length of the side of the square to be cut out
so that the box is of maximum volume.
Application of Derivatives 449

Solution: Let the side of the cut out square be x.


Now length of the box = 60 2x x x
Breadth of the box = 60 2x
x x
Height of the box = x.
Volume = Length Breadth height. 60 2x

a fa
V = 60 2 x 60 2 x x f x x
V = a60 2 xf x
2
x x
dV
V attains extrema when =0
dx
60 2x
Consider

a
V = 60 2 x 2 xf Fig. 17.10

diff. w.r.t. x.
dV
dx
a
= 60 2 x f 2

d
dx
af
x + x
d
dx
a
60 2 x f 2

dV
dx
a
= 60 2 x f 2
a
1 + x 2 60 2 x f dxd a60 2 xf
dV
dx
a
= 60 2 x f 2
a
+ 2 x 60 2 x 2 fa f
dV
dx
a
= 60 2 x f 2
240 x + 8 x 2

dV
= 60 2 + 4 x 2 240 x 240 x + 8 x 2
dx
0 = 3600 + 12 x 2 480 x by 12.

300 + x 2 40 x = 0
30 x
x 2 40 x + 300 = 0 300 x 2

x 30 x 10 x + 300 = 0
2 10 x
40 x
a f
x x 30 10 x 30 = 0 a f
a x 30fa x 10f = 0
x = 30 or x = 10
Now
dV
= 3600 + 12 x 2 480 x
dx
450 Basic Mathematics

d 2V
= 24 x 480.
dx 2
d 2V
is negative.
dx 2 at x =10
V attains maxima when x = 10.
Length of the square to be cut = 10 cm.

F
10. Prove that G
1
I
JK
1
are the points of inflection for the curve y = e x .
2

H
,e 2
2

Proof: Consider
Proof:
y = ex
2

diff. w.r.t. x.
dy
dx
= ex
2 d
dx
x2 d i
dy
dx
a f
= e x 2 x
2

diff. again w.r.t. x.

d2 y
dx 2
= e x
2 d
dx
a f a f e j
2 x + 2 x
d x2
dx
e

d2 y
dx 2
2
a f 2
e ja f
= e x 2 2 x e x 2 x

d2 y
= 2e x + 4 x 2 e x
2 2

2
dx
y attains neither maxima nor minima when

d2y
= 0 i.e.
dx 2

0 = 2e x + 4 x 2 e x
2 2

0 = 2 e x 1 + 2 x 2
2

e x = 0 or 1 + 2 x 2 = 0
2

2x2 = 1
x 2 = 0 or 1
x=0 x=
2
Application of Derivatives 451

Now

G
F 1 IJ 2

H 2K
1
1
, y = e x = e
2
when x= =e 2
2

1
1
, y = ex = e 2 .
2
when x=
2

F 1 , e I and F 1 1
I
GH 2 JK GH JK
1
The points of inflection for the curve y = e x are
2
2 ,e 2 .
2

Hence proved.

REMEMBER:
ds
Velocity =
dt

dV d 2 s
Acceleration = =
dt dt 2
Rate means differentiation w.r.t. t.
dA
rate of change of area =
dt
Area of Square = S2
Area of circle = r2
Surface area of sphere = 4r2
4 3
Volume of Sphere = r
3
Volume of a cylinder = r2h
1 2
Volume of a cone = r h
3

dy dy
For an increasing function > 0 and for a decreasing function < 0.
dx dx
dy
To find maximum and/or minimum value of the function y = f (x), find , equate it to zero. Let
dx
d2 y d2y d2y
x = a, x = b, x = c be the points. Find , and .
dx 2 at x = a dx 2 at x = b dx 2 at x = c.
452 Basic Mathematics

d2y
if > 0, then x = a is a point of minima. Minimum value of the function is y = f (a).
dx 2 at x = a

d2 y
if is less than zero, x = b is a point of maxima. Maximum value of the function is
dx 2 at x = b
y = f (b).

d2y
if is equal to zero, then x = c is called point of inflection. At x = c the function neither
dx 2 at x = c
attains maxima nor minima.

EXERCISE

1. The distance s is metres moved by a particle in t seconds is given by s = 45t + 11 t2 t3. Find
the time when the particle comes to rest?
2. The displacement s of a particle at time t seconds is given by 2t3 3t2 36 t + 90. Find the (a)
velocity after 4 seconds (b) displacement and acceleration when the velocity vanishes (c) accel-
eration after 4 seconds.
3. With usual notation if.
If s2 = at2 + 2bt + c, then prove that
(a) the acceleration is inversely proportional to s3.

a v2
(b) the acceleration is where v is the velocity.
s
4. The equation of motion of a particle is given by s = 9t2 t3. Find the displacement when velocity
is zero and velocity when the acceleration is zero.
5. If s = at3 + bt, find a and b given that when t = 3 velocity is zero and acceleration is 14 units.
6. When breaks are applied to the moving car, the car travels a distance S feet in t seconds given by
s = 20t 40 t2. When does the car stop?
7. The side of a square sheet metal is increasing at 3 mm/min. At what rate is the area increasing
when the side is 10 mm long.
8. A circular patch of oil spreads on water, the area is growing at the rate of 2 sq cm/hr. How fast
are the radius and the circumference increasing when the diameter is 24 cm.
9. A drop of ink spreads over a blotting paper so that the circumference of the blot which is circular
increases at the rate of 3 cm/min. Find the rate of increase of the radius and area when its
circumference is 4 cm.
10. A stone is dropped into a pond, waves in the form of circles are generated and the radius of the
outermost ripple increases at the rate of 2 mm/min. How fast is the area increasing when the
radius is 5 mm, after 5 min?
11. A cylindrical tank is 10 mts in diameter, water is flowing in it at the rate of 24 m3/min. at what
rate height of the water is rising?
Application of Derivatives 453

12. Water is flowing into a right circular cylindrical tank of radius 50 cm at the rate 500 cc/min.
Find how fast is the level of water rising?
13. A ladder 20 ft long rests with its ends on a smooth horizontal floor and against a smooth vertical
wall. If the lower end is moved at the rate of 5 ft/min. Find the rate at which the upper end moves
when the lower end is 12 ft. from the wall.
14. The radius of the sphere is decreasing at the rate of 3 cm/sec. Find the rate at which surface area
is decreasing when radius is 12 cm.
15. The height of circular cone is 30 cm and it is constant. The radius of the base is increasing at the
rate of 0.25 cm/sec. Find the rate of increase of volume of the cone when the radius is 10 cm.
16. A man 6 ft tall is moving directly away from a lamp at a height of 10 ft above the floor. If he is
moving at the rate of 6 ft/sec find the rate at which the length of his shadow is increasing?
17. Find the maximum and minimum value of the function 4x3 15x2 + 12 x + 7.
18. Find the maximum and minimum value of the function x3 3x2 9x + 17.
1
19. Prove that the function xex attains maxima at x = 1 and its maximum value is .
e
1
20. Prove that xx is minimum when x = .
e

F 1I x
21. Prove that the maximum value of H xK is e1/e.

22. Prove that x3 3x2 9x + 9 has a point of inflection at x = 1


23. Prove that y = a(x b)2/5 has no point of inflection.
24. Find two number whose sum is 24 and their product is maximum.
25. The sum of two numbers is 20. If the product of square of the first and cube of the 2nd is
maximum. Find the numbers.
26. Prove that the area of the right angled triangle of given hypotenuse is maximum when the triangle
is isoceles.
27. Prove that among all rectangles of a given area the square has minimum perimeter.
28. Prove that among all rectangles of given perimeter, the square is the one with shortest diagonal.
29. What is the largest rectangle that can be inscribed in a semicircle of radius 1 so that two vertices
lie on the diameter.
30. A rectangular field is to be fenced off along the bank of a river, no fencing is required along the
river. If the material for fence cost Rs. 8 per running foot for 2 ends and Rs. 12 per running foot
for the side parallel to the river, find the dimension of the field of largest possible area that can
be enclosed with Rs. 3600 worth of fences.

ANSWERS

1. 9 Seconds
2. (a) 0 (b) 9 cm; 30 cm/sec2 (c) 42 cm/sec2,
454 Basic Mathematics

7
4. 108 units; 27 units/sec. 5. a = , b = 21
9
1
6. sec. 7. 60 mm2/min.
4
1 1 3
8. cm/hr; cm/hr. 9. cm/min; 6 sq cm/min
12 6 2
24
10. 20 mm2/m 40 mm2/min 11. m/min
25
1 15
12. cm/min 13. ft/sec.
5 4
14. 288 sq cm/sec. 15. 50 cm3/sec.
39
16. 9 ft/sec. 17. ;3
4
18. 22; 10 24. 12; 12
1
25. 8 and 12 30. 112 ft and 150 ft.
2
18
Integration
If
d
af af
f x =g x ,
d
af af
f x + c = g x . Then g(x) is called integrand and f(x) or f(x) + c is called

zaf
dx dx

integral or primitive or antiderivative of g(x) with respect to x. It is denoted by g x dx.

z af
g x dx = f x + c af
18.1 STANDARD INTEGRALS:
By using the definition of integration as the reverse operation of differentiation, the following formulae
may be obtained.

1. z x n dx =
x n +1
n +1
+c

Provided n 1

3
LM
d x n +1
+c =
OP a f
n + 1 x n +11
+ 0 = xn
N
dx n + 1 Qn +1

if n = 1,

z x 1 dx = z 1
x
dx = log x + c

d 1 1
Since log x + c = + 0 =
dx x x

2. z e x dx = e x + c

3
d x
dx
d
e + c = ex i
456 Basic Mathematics

3. z a x dx =
ax
log a
+c

3
d ax LM +c =
a x log a OP
+0
N
dx log a log a Q
= ax

4. z k dx = kx + c

3
d
dx
a f af
kx + c = k 1 + 0 = k

18.2 ALGEBRA OF INTEGRALS:


1. If K is constant, f(x) and g(x) are 2 integrable functions, then

(i) z af z af
k f x dx = k f x dx

i.e., Integral of constant multiplied by function is constant multiplied by integral of the function.

(ii) z b a f a fg z a f z a f
f x g x dx = f x dx g x dx

i.e., Integral of sum or difference of 2 or more functions = Integral of the first function integral
of the second function.

WORKED EXAMPLES:

1. Evaluate: zd i
4 x 3 1 dx

Solution: zd i z z
4 x 3 1 dx = 4 x 3 dx 1 dx

z z
= 4 x 3 dx 1 dx

x4
4 x + c.
4
where c is the constant of integration

zd i
4 x 3 1 dx = x 4 x + c.

1
2. Integrate + e x with respect to x.
x
Integration 457

z FH 1
x
I
K
+ e x dx = z z
1
x
dx + e x dx

= log x + e x + c.

3. Evaluate: z FH 1 1
+
4
x x2 x3
dx I
K
Solution:
z FH 1 1 4
2 + 3 dx =
x x x
I
K z z
1
x
dx
1
x2
dx + z 4
x3
dx

x 2 +1 4 x 3+1
log x + +c
2 + 1 3 + 1

x 1 4 x 2
log x + +c
1 2
1 2
log x + + c.
x x2

4. Evaluate: z F x + 1I
H xK
2
dx

z F x + 1I
H xK
2

z FH
dx = x2 +
1
x2
I
1
K
+ 2 x dx
x

z z
= x 2 dx +
1
x2
dx + 2 dxz
x 2 +1 x 2 +1
= + + 2x + c
2 + 1 2 + 1

x 3 x 1
= + + 2x + c
3 1

x3 1
+ 2x + c .
3 x

5. Evaluate: z Fx
GH
2
+ 3x 1
x
I
JK
dx

Solution: z F x + 3x
GH x x
2
1 I
J dx
xK
458 Basic Mathematics

= x z z z 2
1
2 dx + 3x
1
1
2 dx x

1
2 dx

z z z 3
x 2 dx + 3 x 2 dx x
1

1
2 dx

3 1 1
+1 +1 +1
x2 3x 2 x 2
+ +c
3 1 1
+1 +1 +1
2 2 2

5 3 1
x2 3x 2 x2
= + +c
5 3 1
2 2 2

5
3 1
2x 2
+ 2 x 2 2 x 2 + c.
5
6. Integrate (x2 + 1) (2x3 6x) with respect to x.

zd id
x 2 + 1 2 x 3 6 x dx = i zd 2 x 5 6 x 3 + 2 x 3 6 x dx i
= zd 2 x 5 4 x 3 6 x dx i
z z
= 2 x 5 dx 4 x 3 dx 6 x dx z
2x 6 4x 4 6 x2
= +c
6 4 2

x6
= x 4 3x 2 + c.
3

7. Evaluate: zd 2 x e x + 3x 2 dxi
z z z 2 x dx e x dx + 3x 2 dx

2x 3x 3
ex + +c
log 2 3

2x
e x + x 3 + c.
log 2
Integration 459

8. Evaluate: z F x 1I dx
Hx K 2
2

z F x 1I
Hx K2
2
dx = zF x 1 I dx
Hx x K
2 2
2

= zF 1 1 I dx
Hx x K 2
2

= zF 1 + 1 2 1 1 I dx
Hx x x x K
2 4 2

= z z
1
x2
1 1
dx + 4 dx 2 3 dx
x x z
x 2 +1 x 4 +1 x 3+1
+ 2 +c
2 + 1 4 + 1 3 + 1

x 1 x 3 2 x 2
+ +c
1 3 2
1 1 1
3 + 2 + c.
x 3x x

Note: If z af af
f x dx = g x + c , then za f
f ax + b dx = g ax + b a f 1
a
+ c where a and b are constants.

Examples 1. ze x dx = e x + c

z 1
e 3 x + 7 dx = e 3 x + 7 + c
3

2. zx 3 dx =
x 3+1
3+1
+c=
x4
4
+c

za 6 4xf
3
dx =
a6 4 x f
4
4

1
4
+c

=
a6 4 x f 4
+ c.
16

WORKED EXAMPLES:

1. Evaluate: z 1
8x 6
dx.
460 Basic Mathematics

z 1
8x 6
1
a f
dx = log 8 x 6 + c
8

log a8 x 6f
= + c.
8

2. Evaluate: zd i
e 3 x 76 x dx.

z z
= e 3 x dx 76 x dx

e 3x 76 x 1
= +c.
3 log 7 6

3. Evaluate: za f
4 x 6 7 dx

z a 4 x 6f 7
dx =
a 4 x 6f
7 +1
7+1

1
4
+c

=
a4 x 6f 8
+ c.
32

4. Integrate 6 4 x with respect to x.

Solution: z 6 4 x dx = za 6 4x f 1
2 dx.

=
a6 4 x f 1
2
+1

1
+c
1
+1 4
2

=
a6 4 xf 3
2

1
+c
3 4
2

=
a6 4xf 3
2
+c
6
1
5. Integrate 8 x 9 with respect to x.
e

z 1
e 8 x 9 z
dx = e a8 x 9 f dx
Integration 461

z
= e 8 x + 9 dx 3
1
am
= a m

e 8 x + 9
= +c.
8

18.3 SUBSTITUTION METHOD:

To evaluate integrals of the type z af


f x
n
af
f x dx, we put f (x) = t

Diff. w.r.t. x.

a f dxdt
f x =

f a x f dx = dt.
Substituting we get

z af
f x
n
af z
f x dx = t n dt

t n+1
+ c provided n 1
n +1
if n = 1,

z z
t n dt = t 1dt = log t + c

z af n +1
f x af n
af
f x dx =
f x
n +1
+ c if n 1

af
log f x + c if n = 1
Note: To evaluate ef(x)f (x)dx, put f(x) = t and proceed the same way.

WORKED EXAMPLES:

1. Evaluate: z dx 2
i a
4
+ x 1 2 x + 1 dx f
Solution: zd i a
4
x 2 + x 1 2 x + 1 dx f
Put x2 + x + 1 = t
diff. w.r.t. x.
dt
2x + 1 =
dx
462 Basic Mathematics

a2 x + 1f dx = dt
zd z
Substituting we get

x2 + x + 1 i a2 x + 1f dx =
4
t 4 dt

t5
= +c
5

=
dx 2
i
+ x +1
5

+ c.
5

2. Evaluate: z 4x3
x4 9
dx.

Solution: Put x4 9 = t
diff. w.r.t. x.
dt
4x3 =
dx

4 x 3 dx = dt

z 4x3
x 9
4
dx =
dt
t z
= log t + c

d
= log x 4 9 + c. i
3. Evaluate: z x
4x + 7
2
dx.

Solution: Put 4x2 + 7 = t


diff. w.r.t. x.
dt
8x =
dx
8x dx = dt

1
x dx = dt.
8
Substituting,

z x
4x + 7
2
dx = z 1
8t
dt

1
= log t + c
8
Integration 463

1
d i
= log 4 x 2 + 7 + c.
8

4. Evaluate: z 9
x log x
dx

Solution: Put log x = t.


diff. w.r.t. x.
1 dt
=
x dx
dx
= dt.
x
Substituting,

z 9
x log x
dx = 9
tz
dt
= 9 log t + c

a f
= 9 log log x + c.

ex
5. Integrate with respect to x.
6 4e x

Solution: z ex
6 4e x
dx

Put 6 4ex = t
diff. w.r.t. x.
dt
0 4e x =
dx
4e x dx = dt
dt
e x dx =
4
Substituting,

z ex
6 4e x
dx = z dt
4t
1
= log t + c
4
1
d
log 6 4e x + c.
4
i
6. Evaluate: z 3
e x x 2 dx

Solution: Put x3 = t
464 Basic Mathematics

diff. w.r.t. x.
dt
3x 2 =
dx

3 x 2 dx = dt
dt
x 2 dx =
3

z 3

z
e x x 2 dx = e t dt = e t + c

3
= e x + c.

7. Evaluate: z e 4x
2
+8 x 7
a x + 1f dx
Solution: Put 4x2 + 8x 7 = t
diff. w.r.t. x.
dt
8x + 8 =
dx

a8x + 8f dx = dt
8 a x + 1f dx = dt

a x + 1f dx = dt8
Substituting,

z e4x
2
+8x 7
a f
x + 1 dx = e t z dt
8

=
1
8 z 1
e t dt = e t + c
8
1 4 x 2 +8x 7
= e +c.
8

8. Evaluate: z xe 2 x dx
2

Solution: Put 2x2 = t


diff. w.r.t. x.
dt
4x =
dx
4x dx = dt
Integration 465

dt
x dx =
4
Substituting,

z 2
xe 2 x dx = e t z dt 1 t
=
4 4
e dt z
1 t 1 2
= e + c = e 2 x + c.
4 4

9. Evaluate: z x e1 + e x 1
xe + ex
dx

Solution: Put xe + ex = t
diff. w.r.t. x.
dt
ex e 1 + e x =
dx

LM
e x e1 +
ex
=
OP
dt
N e Q
dx

LM
e x e1 +
ex OP
dx = dt
N e Q
dx e 1
+ e x 1 i dx = dte .
Substituting

z x e1 + e x 1
x +e
e x dx =
e
dt
t z af
1
= log t + c
e
1
e
d
log x e + e x + c . i
10. Evaluate: z dx
x x log x

Solution: z dx
x x log x
= za dx
x 1 log x f
Put 1 logx = t
466 Basic Mathematics

diff. w.r.t. x.
1 dt
=
x dx
dx
= dt
x
Substituting

za dx
x 1 log x
dt
f z
= = log t + c
t

a
= log 1 log x + c. f
18.4 INTEGRATION BY PARTIAL FRACTION METHOD:

To evaluate integrals of the type za px + q


fa
ax + b cx + d
or
f za px + q
fa
ax + b 2 cx + d f
, first resolve into partial

fractions. i.e.,
px + q A B
Express
a fa
ax + b cx + d
=
f a +
ax + b cx + d f
px + q A B C
or a fa
ax + b x + d
2 =
f +
ax + b ax + b a f a
2 +
cx + d f
(where A, B and C are constants to be determined) and then integrate.

WORKED EXAMPLES:

1. Evaluate: za 3x 1
fa f
x 3 x +1
dx.

3x 1
a fa f
Consider x 3 x + 1 ,

Resolve into partial fractions


3x 1 A B
a fa f a f
x 3 x +1
= +
x 3 x +1 ...(1)

3x 1 A a x + 1f + B a x 3f
a x 3fa x + 1f a x 3fa x + 1f
=

3 x 1 = A a x + 1f + B a x 3f
Put x+1=0
Integration 467

x = 1
a f af a
3 1 1 = A 0 + B 1 3 f
4 = B a 4 f

B =1

a f a
3x 1 = A x + 1 + B x 3 f
Put x3=0
x=3
af a f af
3 3 1= A 3+1 + B 0

8 = A a4f

A = 2.

Substituting A = 2 and B = 1 in (1) we get


3x 1 2 1
a x 3fa x + 1f = x 3 + x + 1
integrating with respect to x.

za 3x 1
fa f
x 3 x +1
dx = z 2
x3
dx + z 1
x +1
dx.

a f a f
= 2 log x 3 + log x + 1 + c.

= log ea x 3f a x + 1fj + c.
2

2. Evaluate: z 4x + 6
x2 1
dx.

Consider
4x + 6 4x + 6
x 1
2
=
a fa f
x 1 x +1
Resolve into partial fractions.
4x + 6 A B
a x 1fax + 1f = a x 1f + a x + 1f . ...(1)

4 x + 6 = A a x + 1f + B a x 1f
Put x+1=0
x = 1
468 Basic Mathematics

a f af a f
4 1 + 6 = A 0 + B 1 1

4 + 6 = B a 2 f

2 = B a 2 f B = 1
Now
a f a f
4x + 6 = A x + 1 + B x 1
Put x1=0
x =1
af a f af
4 1 + 6 = A 1+1 + B 0

10 = A a2 f

A=5

Substituting A = 5 and B = 1 in (1)


4x + 6 5 1
a fa f
x 1 x +1
= +
x 1 x +1
integrating,

za 4x + 6
fa f
x 1 x +1
dx = z 5
x 1
dx + z 1
x +1
dx

=5 z 1
x 1
dx z 1
x +1
dx

za 4x + 6
fa f
x 1 x +1
a f
dx = 5 log x 1 log x + 1 + c. a f

3. Evaluate: za fa
x2 + 2
fa
x + 1 2x + 3 4x 1
dx .
f
Solution:
Consider

x2 + 1
a fa
x + 1 2x + 3 4 x 1 fa f
Resolve into partial fractions,

x2 + 1 A B C
a fa
x + 1 2x + 3 4x 1fa
= +
f +
x +1 2x + 3 4x 1
...(1)
Integration 469

a fa f a fa f a fa
x2 + 1 = A 2x + 3 4x 1 + B x + 1 4x 1 + C x + 1 2 x + 3 f
Put x + 1 = 1
x = 1

a1f 2
b a f gb a f g a f a f
+ 1 = A 2 1 + 3 4 1 1 + B 0 + C 0

2 = A a1fa 5f

2
A=
5
Put 2x + 3 = 0
2 x = 3
3
x=
2

F 3I a f FH 23 + 1IK FGH 4 FH 23IK 1IJK + C a0f


2

H2K +1 = A 0 + B

+ 1 = BF
9 3 + 2 I F 12 2 I
4 H 2 KH 2 K

= BF I
13 14
4 H 4K
13
B=
14
Put 4x 1 = 0
4x = 1
x = 1/4

F 1I 2
a f a f FH 14 + 1IK FH 2 14 + 3IK
H 4K +1= A 0 + B 0 + C

+1 = CF I F I
1 5 14
16 H 4K H 4 K
= CF I
17 70
16 H 16 K

17
C=
70
470 Basic Mathematics

Substituting the values of A, B and C in (1)

2

x2 + 1 13 14 17 70
a fa
x + 1 2x + 3 4x 1 fa
= 5 +
f
+
x +1 2x + 3 4x 1
integrating w.r.t. x.

za x2 + 1
fa fa
x + 1 2 x + 3 4x 1
dx =
f z 2 5
x +1
dx +z13 14
2x + 3
dx +z17 70
4x 1
dx

=
2
a f
log x + 1 +
a f a f
13 log 2 x + 3 17 70 log 4 x 1
+ +c
5 28 4

4. Evaluate: za 2x 3
x x +3 x 2
dx
fa f
Consider
2x 3
a fa
x x +3 x 2 f
, Resolve into partial fractions.

2x 3 A B C
a fa
x x+3 x2
= + +
x x +3 x 2 f
2x 3 a fa
A x +3 x2 + B x x2 +C x x+3 f a fa f a fa f
a
x x+3 x2
=
fa f x x+3 x2 a fa f
2 x 3 = A a x + 3fa x 2 f + Bx a x 2 f + Cx a x + 3f
Put x + 3 = 0
x = 3
a f a f a fa
2 3 3 = A 0 + B 3 3 2 + C 0 f af
6 3 = B a 3fa 5f

9 = B a15f

9 3
B= B=
15 5
Put x 2 = 0
x=2
af a f a f a fa
2 2 3= A 0 + B 0 +C 2 2+3 f
4 3 = C a2 fa5f
Integration 471

a f
1 = C 10

1
C=
10
Put x = 0,
af a fa
2 0 3= A 0+3 02 + B 0 +C 0 f af af
3 = A a 6f

3 1
A= A=
6 2

Substituting A =
1
2
, B=
3
5
and C =
1
10
in 1 af
2x 3 1 2 3 5 1 10
a fa
x x+3 x2
=
x
+
f
+
x+3 x2
integrating with respect to x,

za z z z
1 3 1
2x 3
fa f
dx = 2 dx 5 dx + 10 dx
x x+3 x2 x x+3 x2

=
1
2
3
5
a f1
log x log x + 3 + log x 2 + c
10
a f
5. Evaluate: za 3x + 4
fa f
x 2 2 x +1

3x + 4
a fa f
Consider x 2 2 x + 1

Resolve into partial fractions:


3x + 4 A B C
a fa f a
x 2 x +1
2 =
x2
+
x2 f a f 2 +
x +1 ...(1)

Multiplying by (x 2)2 (x + 1)

a fa f a f a
3x + 4 = A x 2 x + 1 + B x + 1 + C x 2 f 2

Put x 2 = 0
x=2
af af a f af
3 2 + 4 = A 0 + B 2 +1 + C 0

6 + 4 = B a3f B =
10
3
472 Basic Mathematics

Put x + 1 = 0
x = 1

a f af af a
3 1 + 4 = A 0 + B 0 + C 1 2 f 2

1 = C a 9f C =
1
9
10 1
Put x = 0, B = and C =
3 9

a fa f
4 = A 2 1 +
10
3
af a f
1
1 + 2
9
2

10 4
4 = 2 A + +
3 9
10 4
4 = 2A
3 9
36 30 4
= 2A
9
2 1
+ = 2A A =
9 9

Substituting A =
1
9
, B=
10
3
1
and C = in 1
9
af
1 1
3x + 4 9 10 3
+ 9
a x 2f a f
2
x +1
=
x 2
+
x2 2
ax +1 f
integrating we get,

za 3x + 4
fa f
x 2 2 x +1
dx = z 19
x2
dx + za 10 3
x2 2 f
dx + z 19
x +1
dx

=
1
a f
10 x 2
log x 2 +
a f 2 +1
+
1
a f
log x + 1 + c
9 3 2 + 1 9

=
1
9
a
log x 2 f a
10 1
f
+ log x + 1 + c.
3 x2 9
a f
6. Evaluate: za 4x + 8
fa f
2x 1 2 x + 3
Integration 473

Consider
4x + 8
a fa f
2 x 1 2 x + 3 Resolve into partial fractions

4x + 8 A B C
a fa f
2x 1 x + 3
2 = +
2x 1 2x 1 a
2 +
x +3 f ...(1)

Multiplying by (2x 1)2 (x + 3)

a fa f a f a
4x + 8 = A 2 x 1 x + 3 + B x + 3 + C 2 x 1 f 2

Put 2x 1 = 0
2x = 1
x = 1/2.

F 1 I + 8 = A a0f + B F 1 + 3I + C a0f
4
H 2K H2 K
10 = B F I B =
7 20
H 2K 7
Put x + 3 = 0
x = 3

a f af af ba f g
4 3 + 8 = A 0 + B 0 + C 2 3 1
2

12 + 8 = C a7f 2

4 = C a 49f C =
4
49
20 4
Substituting B = ,C= and x = 0
7 49
we get

a f a fa f
4 0 + 8 = A 1 3 +
20
7
af
3
4
49
1 2a f
60 4
32 = 3 A +
7 49
60 4
3 A = 32 +
7 49
1568 + 420 4
3A =
49
474 Basic Mathematics

1152
3A =
49
1152 384
A= =
49 3 49
Substituting in (1)

384 20 4
4x + 8 49 7 + 49
a2 x 1f a 2
x+3
=
2 xf 1
+
2x 1 a f 2
x+3

integrating with respect to x.

za z za z
384 20 4
4x + 8 49 dx + 7 49 dx
f a x + 3f =
2x 1 2
2x 1 f
2x 1 2
dx +
x+3

=
a +
f +
a
384 log 2 x 1 20 2 x 1 2 +1 1 4 f
log x + 3 + c a f
49 2 7 2 + 1 2 49

a
384 log 2 x 1 20 f 4
a f
=
49

2

14 2 x 1 49 a f
log x + 3 + c .

18.5 INTEGRATION BY PARTS:


From product rule in differentiation we have
d
dx
a f
uv = u
dv
dx
+v
du
dx
Integrating w.r.t. x. we get

z d
dx z z
a f
dv
dx
du
uv = u dx + v dx
dx

z zuv = u dv + v du.

z z FH IK
udv = uv v
du
dx
dx

By taking u as first function,


dv as second function
du as differential of 1st function
v as the integral of the 2nd function we get
Integration 475

za fa f a
I function II function dx = I function z
f aII functionf dx LMN zz II function
d
dx
a fOPQ
I function dx

This is known as integration by parts.


The success of the method of integration by parts depends on choice of 1st function and 2nd function.
Logarithmic, algebraic and exponential functions should be taken in the same order of priority. [LIATE:
log, Inverse trig., Algebraic, Trigonometric and Exponential]
For example, to integrate xex, x is first function and ex is 2nd function where as to integrate xlogx, logx
is first function and x is 2nd function.

WORKED EXAMPLES:

1. Evaluate: z x e x dx.

z z z FH
x e x dx = x e x
I II
ex
d
dx
I
K
x dx

z
= xe x e x 1 dx

xe x e x + c.

2. Evaluate: z x log x dx .

z z
x log x dx = log x x dx
I II

log x x dx z z LMN x2 d

2 dx
a fOP
Q
log x dx.

= log x z FGH
x2
2

x 1I
2
J dx
2 xK

log x zx2
2

x
2
dx

x2 1 x2
log x +c
2 2 2

x2 x2
log x + c.
2 4
476 Basic Mathematics

3. Evaluate: z log x dx.

z z
log x dx = log x 1 dx
I II

z z LMN
log x 1 dx x
d
dx
a fOPQ
log x dx

z
log x x x
1
x
dx

z
log x x 1 dx

log x x x + c.

4. Evaluate: z 2
x 3 e x dx.

Put x2 = t.
diff. w.r.t. x.
dt
2x =
dx
2x dx = dt

dt
x dx =
2
Substituting we get

z 2

z
x 3 e x dx = x 2 e x x dx
2

=
1
z
t e t dt
2 I II

1
2
LM
z z FH
t et
N et
d
dt
t dtI OP
K Q
1 t
2 z
te e t dt

1 t
te e t + c
2
2 2
x 2e x e x
= + c.
2
Integration 477

5. Evaluate: z x n log x dx where n 1

z log x x n dx
I II

z z
log x x n dx
LM x d alog xfOP dx
N n + 1 dx Q
n +1

= log x
x
n +1
n +1
M
Nn +1 xQ z
L x 1 OP dx n +1

log x
x n +1
n +1

xn
n +1
dx z
log x
x n +1

1 x n +1 LM
+ c if n 1
OP
n +1 n +1 n +1 N Q
x n +1 x n +1
= log x
n +1 n +1 2
+ c.
a f
18.6 INTEGRALS OF THE TYPE: z af af
e x f x + f x dx

Consider

z af af
e x f x + f x dx

= z x
I II
z
e f a x f dx + e f a x f dx x

afz z RST LMN a fOPQUVW z


f x e x dx ex
d
dx
f x af
dx + e x f x dx

af z af z af
f x e x e x f x dx + e x f x dx

af
= f x ex

z af af
e x f x + f x dx = e x f x + c. af
Examples:

1. z ex
LM 1 1 OP dx
MN x + 1 a1 + xf PQ
2
478 Basic Mathematics

1
ex +c
x +1

d F 1 I 1 1
Since H
dx x + 1
=
K a f = a1 + x f
x +1 2 2
.

2.
z ex
a1 + x log x f dx =
x z ex LM 1 + log x OP dx
Nx Q
= e x log x + c

Since
d
dx
a f 1
log x = .
x

3. z LM a f 1 OP dx = e log alog xf + c
e x log log x +
N x log x Q
x

log alog x f = alog x f


d 1 d
Since
dx log x dx

1 1 1
= = .
log x x x log x

4. z a f
e x x + 1 dx = e x x + c

3
d
dx
af
x = 1.

5. z ex
FG 2 x + 1IJ dx =
H2 xK z LMN
ex
2x
+
1
2 x 2 x
dx
OP
Q
z LMN
= ex x+
1
2 x
OP dx
Q
= ex x + c

Since
d
dx
d x i = 2 1x .
REMEMBER:

z x n dx =
x n +1
n +1
+c

Provided n 1
Integration 479

if n = 1

z z
x 1dx =
1
x
dx = log x + c.

z e x dx = e x + c.

z a x dx =
ax
log a
+c

z k dx = kx + c

z af af z af z af
k f x g x dx = k f x dx k g x dx

If z af af
f x dx = g x + c, then

za f a f
f ax + b dx =
g ax + b
a
+c

z af af af
To evaluate f x
n
f x dx , put f x = t and proceed to get the answer.

f xaf n +1
+ c for n 1
n +1
and log [f (x)] + c for n = 1
To evaluate ef(x) f (x) dx, put f (x) = t and proceed to get e f(x) + c as answer.
To evaluate integrals of the type

za px + q
fa
ax + b cx + d f
dx or za px + q
fa
ax + b 2 cx + d f
First resolve into partial fractions
px + q A B
aax + bfacx + d f = ax + b + cx + d
or
px + q A B C
a fa
ax + b cx + d
2 =
f a
ax + b
+
f a
ax + b f a
2 +
cx + d f
Find A, B, C, then finally integrate.

za fa
I function II function dx f
480 Basic Mathematics

= I function II function z z LMNz II function


d
dx
a
I function dx fOPQ
I function and II function are kept by making use of LIATE rule.

z af af
e x f x + f x dx = e x f x + c. af
EXERCISE
I. Ev alua
Evalua te:
aluate:

1. z FH x3 +
4
x
dxI
K 13. z 2x + 7
x + 7x 9
2
dx

2. z FH 4x 2
5 8
+
x2 x
dx I
K 14. za fd
2 x 9 x 2 9 x + 6 dx i 2

z FH ax 2 +
b c
+ + d dx I
K
15. z d i
x 3 + 6 x 1 x 2 + 2 dx

zd
3.
x2 x
+ bx + ci a2 ax + bf dx
zd x e + e x e x dx i
16. ax 2

z
4.

zd
1
i dx 17. dx

5. x + x +e e e x log x

6. ze 6 x 6 x 2 dx j 18. zd x2
2 7x3 i 4 dx

7. z FGH6 x 4 + 8x 9
x
dx
I
JK 19. z 2
2 x e x dx

8. z FH x+
1
x
I d x 8i dx
K
2 20. ze x + e x
e x e x
dx

9. za 6 x + 4f dx 3
21. z aa
log 7 x 6
7x 6
dx
f
f

10. z GHF4 x 9 x + 1I
x
2
JK dx 22. z1
x
6 log x dx

11. z LMMNa 4x + 7
1
f 2

1
8x + 7
dx
OP
PQ
23. z6 log x
x
dx

12. z 1
+
1
3 4 x 8 5x a f 3 dx 24. z x +6
x
dx
Integration 481

25. z 1
x + 7 x + 12
2
dx 31. za f a
x 1
x
2
x+2 f dx
26. z 1
x 2x 8
2
dx 32. z x 2 e x dx

27. z 2x + 1
2
x + 6x + 8
dx 33. z x 4 e 2 x dx

28. z 3x + 7
3x 7x 6
2
dx 34. za f
x x 1 20
dx

29. z a fa f
2x + 1
x +1 x + 2 2 35. z x 6 log x dx

30. ze4x
e2x
+ 4e 2 x + 3
dx 36. z e
log x + x 2 + a 2 dx j
37. zx 5 e x dx
2
Hint: x 5 = x 3 x 2 Put x 3 = t

38. zx e x
dx Hint: Put x = t 2

39. za f xe x
x +1 2

40. z MN a f PQ
ex
LM OP
x2 + 1
x +1 2
[Hint:
Hint: Add and Subtract 1 in the Numerator and simplify]

41. zlog x 1 dx

42. z1 x
1+ x
dx

43. z2x + 3
4x 7
dx

44. z LMN a f
e x log log x +
1
x log x
dx
OP
Q
45. z LMN a f
log log x
x
+ e log x dx
OP
Q
482 Basic Mathematics

ANSWERS

x4
d i
3
1. + 4 log x + c
4 2 x3 + 6x 1 2
15. +c
9
4x3 5
2. + + 8 log x + c
3 x 2
ax 2 + bx + c
16. +c
ax 3 b 2
3. + c log x + dx + k
3 x
17. log log x + c
x e+1
4. + ex ex + c
e +1 1
+c
x +1 x e+1
d
18. 63 2 7 x 3
i 3

5. + e x + e x + c
+1 e +1 2
19. e x + c
4
3 x
20. log e e + c
x
3x 3
6. 4x 2 +c
4

3x 4
21.
a
log 7 x 6 f 2
+c
7. + 8 x 9 log x + c 14
2
3
x 4 7x 2 2 6 log x
8. 8 log x + c 22.
2
+c
4 2 3

9.
a6 x + 4f 4
+c 6 log x
24 23. +c
log 6
5

10. 8x 2
6x
3
2
1
+ 2x 2 +c 24. d x +6 i 2
+c
5

a f F x + 3I + c
11.
1
a
4 4x + 7

log 8 x + 7
8 f +c
25. log H x + 4K
log F
x 2I
a
log 3 4 x 1 f 26.
1
H x + 4K
+c
12. 4
+
10 8 5x a f 2
+c 6

log a x + 4f log a x + 2f + c
7 3
d
13. log x + 7 x 9 + c
2
i 27.
2 2

14.
dx 2
9x + 6 i 2

+c
5 16
a
28. log 3x + 2 + log x 3 + c
3 11
f a f
2
Integration 483

LMx 3
OP
29.
1
3
a f 1
log x + 1 log x 2
3
3
x2
a
+c f 38. e
x

MN
2 + 3x + 6 x + 6 + c
PQ
F
e2 x + 1 I
1
30. 4 log GH
e2 x + 3
+c JK 39.
ex
1+ x
+c

1
a f 2
a f
2
a f LM x 1OP + c
31. x 1 + log x 1 log x + 2 + c
N x + 1Q
x
40. e
3 9 9

x log a x 1f x log a x 1f + c
1 1 1
32. x 2 e x 2 xe x + e x + c 41.
2 2 2

LM x x + 3x
4 2 OP
a f
2 x 3x 3
3
+c
33. e
N2 2 2 4 Q 42. 2 log 1 + x x + c

a x 1f ax 1f
21 22 1 LM
13 7 F I OP + c
34. x
21 462
+c 43.
2
x + log x
N4 4 H KQ
35.
x7
7
log x
x6
42
+c a f
44. e x log log x + c

e
36. x log x + x + a x + a + c
2 2 2 2
j a f
45. log x log log x log x +
x2
2
+ c.

3
ex
37. x3 1 + c
3
484 Basic Mathematics

19
Definite Integrals
19.1 INTRODUCTION:

z af af z af af af af
b b
If f x dx = g x + c. Then f x dx = g x =g b g a.
a a

z af z af
b
f x dx is called indefinite integral and f x dx [Read as integral from a to b f (x)dx] is called
a
definite integral of f (x) from a to b. Here a is called lower limit and b is called upper limit
low limit.
To evaluate definite integral, integrate the given function as usual. In the final answer substitute the
upper limit value for x lower limit value for x.

WORKED EXAMPLES:
I. Ev alua
Evalua te:
aluate:

z
2
x3 2
2 3 13 8 1 7
1. x 2 dx = = = = .
3 1 3 3 3 3 3
1

z
1 1
2. e x dx = e x = e1 e 0 = e 1.
0 0

z FH I dx = F x z I
1 2 1
1 1
x+
K H +
K
+ 2 dx
2
3. x x2
0 0

=
LM x + x + 2 xOP
3 2 +1 1

N 3 2 + 1 Q 0
Definite Integrals 485

Lx 1 O
= M + 2 xP
3 1

N3 x Q 0

LM1 1 + 2 a1fOP LM 0 1 + 2 a0fOP


3 3

N3 1 Q N 3 0 Q
1
1+ 2 0
3
1 4
+1= .
3 3

zd
2

4. i
1 + 2 x dx
1

a1 + 2 xf
za
1
+1

f
2 1 2
1 2
1 + 2x dx =
2 1 2
1
+1 1
2

a1 + 2 xf 3
2

1 2

3 2 1
2

a1 + 2 x f 3
2
2

3 1

b1 + 2 a2fg af
3 3
2 1+ 2 1 2

3 3

a1 + 4f 3
2

1+ 2
3
2
3 3
3 3
52 3 2 5 5 3 3 .
=
3 3

z
1

5. x e x dx
1

LM x
N z z ex ex
d
dx
x OP
Q
1

1
486 Basic Mathematics

1
xe x e x
1

1e1 e1 1e 1 e 1

e e e 1 e 1

0 2e 1

2
= 2 e 1 = .
e

z
e

6. x log x dx
II I
1

z z
LMlog x xdx L x d alog xfO dx OP e

MN 2 dx PQ PQ
2

MN 1

z
LMlog x x x 1 dxOP
2 2 e

N 2 2 x Q 1

z
LMlog x x x dx OP
2 e

N 2 2 Q 1

LMlog x x 1 x OP
2 2 e

N 2 2 2Q 1

e
x2 x2
log x
2 4 1

LMlog e e e OP LMlog1 1 1 OP
2 2 2 2

N 2 4Q N 2 4Q
LM1 e e OP L0 1 O
2 2

N 2 4 Q MN 4 PQ
e 2 e 2 1 2e 2 e 2 + 1 e 2 + 1
+ = = .
2 4 4 4 4
Definite Integrals 487

z
1
x+2
7. dx.
x + 4x + 8
2
0

Put x2 + 4x + 8 = t
diff. w.r.t. x.
dt
2x + 4 =
dx

a2 x + 4f dx = dt
2 a x + 2 f dx = dt

a x + 2f dx = dt2 .
when x = 0, x2 + 4x + 8 = t
af
0+4 0 +8=t
t=8
when x = 1, x2 + 4x + 8 = t
af
1+ 4 1 + 8 = t
t = 13

z z
1 13
x+2
13
dt 1
= = log t
x + 4x + 8
2
2t 2 8
0 8

1
log13 log 8
2

1 F I
13
2
log
H K
8
.

z
2

8. x 1 + x 2 dx
1

Put 1 + x2 = t
diff. w.r.t. x.
dt
2x =
dx
2x dx = dt
488 Basic Mathematics

dt
x dx =
2

when x = 1, 1 + x2 = t
1+1 = t
t=2
when x = 2, 1 + 22 = t
t=5
Substituting,

z z
2 5
t dt
x 1 + x dx = 2
2
1 2

5
1 t 1 2 +1
=
2 1 2 +1 2

5 5
2t 3 2 t3 2
=
32 2 3 2

1 32 1
5 23 2 = 5 5 2 2
3 3

5 52 2
= .
3

z
1
1 x
9. dx
1+ x
0

za f
1
x 1
dx
x +1
0

LM OP
z z z
1 1 1
x +111 x +1 2
=
0
x +1
dx =
MN 0
x +1
dx
0
x +1
dx
PQ
LM OP
MNz z
1 1
1

0
1 dx 2
0
x +1
dx
PQ
x 2 log x + 1 a f 1
0
Definite Integrals 489

m a f a fr
1 2 log 1 + 1 0 2 log 0 + 1

m 1 2 log 2 0 2 log 1 r

1 + 2 log 2 0

= 2 log 2 1.

z
3
2x + 3
10. dx
5x 7
2

F 3I
z z H 2 K dx
3 3 2 x+
2x + 3
5F x I
dx =
5x 7
H 5K
7
2 2

z
7 7 3
3 x + +
2 5 5 2 dx
=
5 7
2 x
5

LM OP
z z
7 7 3
3 x 3 +
2
MM 5 dx +
7
5 2 dx
7 PP
N Q
5 x x
2 2
5 5

LM OP
z z
29
3 3
2
MM 1 dx + 10 dx
7 PP
N Q
5 x
2 2
5

LM2 29 F I OP
7
3

N5
x+
10 H KQ
log x
5 2

2 RL 7 O L 7 OU
S M log F 3 I P M2 + log F 2 I P V
29 29
3+
5 T N 10 H K
5 Q N 10 H 5K QW

2R
S3 + 29 log FH 85IK 2 1029 log FH 35 IK UVW
5 T 10
490 Basic Mathematics

LM 8 F I OP
2
MM
29
1 + log 5 GG JJ P
5
MN
10 3
5
GH JK PPQ
2 LM
29 8 F I OP
5
1+
N
10
log
3 H KQ
19.2 PROPERTIES OF DEFINITE INTEGRALS:

z af z af z af
b b b

1. f x dx = f t dt = f z dz
a a a

z af z af
b a

2. f x dx = f x dx
a b

z af z z af af
b c b

3. f x dx = f x dx + f x dx where a < c < b.


a a c

z af za f
a a

4. f x dx = f a x dx
0 0

z af za f
b b

5. f x dx = f a + b x dx
a a

WORKED EXAMPLES:

za f
1
x x 1 dx
5
1. Evaluate:
0

za f
1
Let I = x x 1 5 dx
0

using property (4),

za fa f
1
I= 1 x 1 x 1 dx
5

0
Definite Integrals 491

za fa f
1
I= 1 x x
5
dx
0

zd
1
I= x 5 + x 6 dx i
0

1
x 5+1 x 6 +1
I= +
5 +1 6 +1 0

1
x 6 x 7
I= +
6 7 0

1 1
I= + 0
6 7
7 + 6
I=
42
1
I=
42

za f
1
1
x x 1 =
5
.
42
0

za f
a
xa x dx
4
2. Evaluate:
0

za f
a
I= xa x dx
4
Let
0

using property (4),

za f a f
a
I= axa a x dx
4

za f a f
a
I= x a x dx
4

za f
a
I= x 4 a x dx
0
492 Basic Mathematics

z
a
I= x 4 a x 5 dx
0

a
x5 x6
I= a
5 6 0

a 5 a a6
I= 0
5 6

a 6 a 6 6 a 6 5a 6 a 6
I= = =
5 6 30 30

z a x af
a
a6
x dx =
4
.
30
0

z
a
x
3. Evaluate: dx.
ax
0

z
a
x
Let I= dx
a x
0

using property (4)

z a f
a
ax
I= dx
a a x
0

z z
a a
a x a x
= dx =
aa+x x
0 0

z FH IK
a
a x
I= dx
x x
0

z z
a a
a
= dx 1 dx
x
0 0

a
= a log x x 0

a log a a 0
= a log a a.
Definite Integrals 493

Alieter:

z z
a a
x x
dx =
a x ax
0 0

z LMN OPQ
a
x a + a
= dx
ax
0

LM OP
MNz z
a a
ax a
=
0
ax
+
0
ax
dx
PQ
LM OP
MNz z
a a
a
1 dx
0 0
a x
dx
PQ
LM
x a log
aa xf OP a

N 1 Q 0

x + a log a a x f
a
0

a + a log a a af l0 + a log a a 0fq

a + a log 0 a log a

a + a log a
= a log a a.

z
4
x+4
4. Evaluate: dx
x4
0

z
4
x+4
Let I dx using property (4)
x4
0

z
4
4x+4
I=
4 x4
0

LM OP
z z z
4 4 4
8 x 8 x
I= dx = dx
0
x MN 0
x
0
x
dx
PQ
494 Basic Mathematics

4
8 log x x
0

a f
8 log 4 4 = 4 8 log 4.

Alieter:

z
4
x+4
dx
x4
0

z
4
x+44+4
= dx
x4
0

z z
4 4
x4 8
= dx + dx
x4 x4
0 0

z z
4 4
8
1 dx dx
4x
0 0

x 8
a
log 4 x f 4

1 0

a f
4 + 8 log 4 4 0 + 8 log 4 0 a f
4 0 0 8 log 4
4 8 log 4.

19.3 APPLICATION OF DEFINITE INTEGRALS TO FIND AREA:


One of the application of definite integrals is to find y

area.
y = f (x)
(i) Area enclosed by the curve y = f (x), the x-
axis and the lines x = a and x = b is given by

z z af
b b
A = y dx = f x dx.
a a
0 x=a x=b x
Fig. 19.1
WORKED EXAMPLES:
1. Find the area bounded by the curve y = x2 x, x-axis and the ordinates x = 1 and x = 2.
Definite Integrals 495

z
b
Solution: Area = y dx
a

zd
2
A= x 2 x dx i
1

2
x3 x2
A=
3 2 1

F 2 2 I F1 1 I
GH 3 2 JK GH 3 2 JK
3 2 3 2
A=

8 1 1 16 12 2 + 3
= 2 + =
3 3 2 6
5
Area = square units.
6
2. Find the area bounded by the curve y = 3x2 8 with x-axis and ordinates x = 0 and x = 3.

z
b
Solution: Area = y dx
a

zd
3
= 3 x 2 8 dx i
0

3
x3
=3 8x
3 0

3
= x 3 8x
0

af
33 8 3 0

27 24 = 3 Sq. units.
3. Find the area bounded by the parabola x2 = 4y, x-axis and x = 1 and x = 2.

z
b
Solution: A = y dx
a

z
2
x2 x2
A= dx Q x 2 = 4 y, 4 y = x 2 , y =
4 4
1
496 Basic Mathematics

2
1 x3
A=
4 3 1

LM
1 2 3 13 OP L O
1 8 1
Q MN PQ
7
A= = =
N
sq. units.
4 3 4 3 12

4. Find the area bounded by the x-axis and the curve y = x2 5x + 6


Solution. On X-axis, y = 0.
x 2 5x + 6 = 0

x 2 3x 2 x + 6 = 0
a f a f
x x3 2 x3 = 0

a x 2fa x 3f = 0
x = 2 or x = 3.

z
3
So Area bounded by the x-axis and the curve y = x 5 x + 6 = y dx
2

zd
3
= i
x 2 5 x + 6 dx
2

3
x3 x2
= 5 + 6x
3 2 2

33 2 3
3
5
32 2 2
2
+ 6 3 2 a f
27 8 94 F I af
3
5
2
+6 1
H K
19 25 38 75 + 36 1
+6= =
3 2 6 6
1
Q Area is +ve, A = sq. units.
6
5. Find the area bounded by the curve y = x2 x with x-axis.
Solution: On X-axis, y = 0.
x2 x = 0
Definite Integrals 497

a f
x x 1 = 0

x = 0 or x 1 = 0
x = 0 or x = 1
So Area bounded by the curve y = x2 x with x-axis

z zd
b 1
= y dx = i
x 2 x dx
a 0

1
x3 x2 1 1
= = 0
3 2 0
3 2

2 3 1
=
6 6
Q Area is non-negative,
1
Required Area = sq. units.
6
6. Find the area between the parabolas y2 = x and x2 = y.
Solution: y 2
x =y
Point of intersection: y 2 = x
(1, 1) 2
y =x
x2 = y
Squaring,

x 4 = y2 0
x
(0, 0)
x4 = x dQ y2 = x i
x4 x = 0

d i
x x3 1 = 0 Fig. 19.2

x = 0 or x = 1

z
b
Q y2 = x
Area bounded by the curve y2 = x, x-axis, x = 0 and x = 1 is y dx .
a
y= x

z
1
= x dx
0
498 Basic Mathematics

1
1
+1
x2
=
1
+1
2 0

1
x3 2
32 0

13 2 2
0=
32 3

z
b
Area bounded by the curve x2 = y, x-axis x = 0 and x = 1 is y dx . y = x2
a

z
1
= x 2 dx
0

1
x3 1
= = .
3 0
3

Area between the parabolas = Area bounded by the parabola y2 = x Area bounded by the parabola
x2 = y with X-axis, x = 0 and x = 1.
2 1 1
= sq units.
3 3 3
1
Reqd. area = sq. units.
3
7. Find the area between the parabola y2 = 4x and the line y = x.
Point of intersection: y
x
y2 = 4x and y = x y=

y2 = x 2
2
y = 4x
(4, 4)

4x = x 2

(0, 0) x
x2 4x = 0

a
x x4 =0f
x = 0 or x = 4.
Fig. 19.3
Definite Integrals 499

z
b
Area bounded by the parabola y2 = 4x, the lines x = 0, x = 4 and X-axis = y dx .
a

z y2 = 4x
4
= 2 x dx y = 4x = 2 x
0

4
1
+1
x2
2
1
+1
2 0

4
3
3 4
x2 22 2
2 = x
3 3
0
2 0

4
3
d i
43 2 0

4 32
8 = .
3 5

z
b
Area bounded by the line y = x, x = 0, x = 4 and x-axis = y dx
a

z
4 4
x2 4 2 16
= x dx = = = = 8.
2 0
2 2
0

Required area = Area bounded by the parabola y2 = x Area bounded by the line x =y with x-axis
and lines x = 0 and x = 4
32
= 8
3
32 24 8
= = sq. units.
3 3
8. Find the area between the curves x2 = 5y and y = 2x.
Solution: Required area = A1 ~ A2.
Now: x2 = 5y and y = 2x.

a f
x2 = 5 2x
500 Basic Mathematics

A x2 = 5y

2x
y=
A1 A2

0 B

Fig. 19.4

x 2 = 10 x
x 2 10 x = 0
a f
x x 10 = 0
x = 0 or x = 10.
af
y = 2 x y = 2 0 or y = 2 10 a f
y = 0 or y = 20.
Point of intersection: (0, 0) and (10, 20).
A1 = Area bounded by x2 = 5y, x-axis and lines x = 0 and x = 10

z z Q 5y = x 2
b 10
x2
y dx = dx x2
5 y=
a 0 5

10
1 x3
=
5 3 0

1 1000 200
10 3 0 3 = =
15 15 3
A2 = Area bounded by y = 2x, x-axis and lines x = 0 and x = 10.

z z
b 10 10
2x 2
= y dx = 2 x dx = = 10 2 = 100.
2 0
a 0

Required area = A1 A2.


200 200 ~ 300 100
= ~ 100 = =
3 3 3
Definite Integrals 501

100
Required area = sq. units.
3

REMEMBER:

If z af
f x dx = g x + c, thenaf z af
b

a
f x dx = g xaf b
a
af af
=g b g a

z af z af z af
b b b

f x dx = f t dt = f z dz and so on.
a a a

z af z af
b a

f x dx = f x dx
a b

z af z z af af
b c a

f x dx = f x dx + f x dx
a a c

z af za f
a a

f x dx = f a x dx
0 0

Area enclosed by the curve y = f (x), X-axis and the lines x = a and x = b is given by

z z af
b b
A = y dx = f x dx .
a a

EXERCISE
I. Evaluate:

z a x + 1f dx zd
b 2

1. 2. i
x 2 1 dx
a 1

zd za f
3 1

3. 3 x 2 x + 1 dx
2
i 4. 2 x 1 dx
8

1 0

z FGH IJ
zd
4 2

5. x+
1
x
dx
K 6. i
x 3 + 2 x 2 x + 1 dx
1 0
502 Basic Mathematics

zd z FH I
1 2

7. x + 4 x 2 dx
3
i 8. x2 +
1
x2 K
x dx
1 1

zd zd
2 2

9. x 3 + 3 x 2 + x dx i 10. i
6 x 5 + 2 x 4 + 4 dx
1 1

za fa f z a fa f
1 1

11. x x + 1 x 1 dx 12. x + 1 x 2 dx
0 0

z z
1 e
x+2 1
13. dx 14. dx
x2 + 4x + 3 x 1 + log x
0 0

za f
z
2 2 e
log x
15. dx 16. log x dx
x
1 1

zd za f
1 1

i dx
5
x
17. 1+ e 18. x2 1 x 2
0 0

z z
1 1
x
19. x 2 e x dx 20. dx
x +1
1 0

za z
0 1
dx x
21.
1
1 x x + 2fa f 22.
0
x +1
2 dx

z z
5 1
dx x3
23. 24. dx
x + 4x + 3
2
e2 x
2 0

za
e2

25. log x dx f 2

II. 1. Find the area bounded by the curve y = x3, the x-axis and the lines x = 1 and x = 2.
2. Find the area bounded by the curve y = x2 4x, x-axis and the lines x = 1 and x = 3.
x
3. Find the area enclosed by the curve y = x-axis and ordinates x = 0 and x = 1.
x +1
Definite Integrals 503

4. Find the area bounded by the x-axis and the curve y = x2 7x + 10.
5. Find the area bounded by the curve y = 4x x2 3 with x-axis.
6. Find the area bounded by the curve y = 4x x2 and x-axis.
7. Find the area between the curves y2 = 4x and y = 2x.
8. Find the area between the curves y2 = 2x 2 and the line y = x 5.
9. Find the area between the parabolas y2 = 4x and x2 = 4y.
10. Find the area between the parabola y2 = 4ax and the line y = 2x.
11. find the area between x2 = y and y2 = 8x.
12. Find the area between the curves y = 11x 2y x2 and y = x.

ANSWERS

1. ab af LMN b + 2a + 2 OPQ 2.
4
3
3. 20 4.
1
9
5.
20
3

28 4 57 365 1
6. 7. 4 8. 9. 10. 11.
3 3 4 32 4

12.
4
13. 8 3 14. 2 d 2 1i 15.
alog 2f 3
16. 1 17. 2
1
3 3 e

16 e2 5 2 1 5
18. 19. 20. 1 log 2 21. log 2 22. log 2 23. log
693 e 3 2 4

19 3
24. + 25. 2e e
2
8e 2 8

II. 1.
15
4
sq units 2.
22
3
sq units b g
3. 1 log e 2 sq units

9 4 64
4. sq units 5. sq units 6. sq units
2 3 3

1 2 16
7. a sq units 8. 18 sq units 9. sq units
3 3
1 2 8 4
10. a sq units 11. sq units 12. sq units
3 3 3
504 Basic Mathematics

20
Application of Calculus in
Business
20.1 TERMINOLOGY:
Cost Function: The outflows usually raw materials, rent, utilities, pay of salaries and so forth form the
total cost. It is sum total of all costs. Economists and accountants often define total cost as sum of 2
components. Total variable cost and total fixed cost.
Total variable cost varies with the level of output. (Eg.: raw materials) whereas total fixed cost
remains the same (for example rent). Hence Total cost = Variable cost + Fixed cost
era
Aver age Cost: Average Cost is the Cost per unit of the output. It is obtained by dividing total cost by
the total quantity produced. If TC is the total cost of producing x units or q units, then average cost AC
TC TC
is given by, AC = or .
x q
Mar
Margginal Cost: Marginal cost is the additional cost incurred as a result of producing and selling one
more unit of the product.
d d
If TC is the total cost of producing x units or q units then the derivative TC or TC repre-
dx dq
sents the instantaneous rate of change of total cost, given a change in number of units (x or q) produced.
d d
Marginal Cost, MC = TC or TC .
dx dq
Re
Revven ue function: The money which flows into an organisation from either giving service or selling
enue
products is called as revenue. The most fundamental way of computing total revenue from selling a
product is
Total revenue = Price per unit Quantity sold.
If TR is the total revenue p is the price per unit and q or x is the quantity sold then
TR = pq or TR = px.
Application of Calculus in Business 505

Mar
Marg ginal rreeven ue: It is the additional revenue derived from selling one more unit of a product. It is
enue:
obtained by differentiating total revenue with respect to quantity demanded. If MR is the marginal
revenue. TR is the total revenue and x or q is the No. of units produced. Then

MR =
d
dx
a f
TR or MR =
d
dq
TR .

Pr of
Prof it function: Profit for an organisation is the difference between total revenue and total cost. If TR
ofit
is the total revenue and TC is the total cost then
Profit = TR TC.
For many production situations, the marginal revenue exceeds the marginal cost, at lower level of
output. As the level of output increases, the amount by which marginal revenue exceeds marginal cost
becomes smaller. Eventually a level of output is reached at which marginal revenue = Marginal cost.
Beyond this point marginal revenue is less than marginal cost and the total profit begins to decreases
with added output. So from theoretical stand point, the profit maximization level of output can be
identified by the criterion.
Marginal revenue = Marginal cost.
i.e., MR = MC.

i.e.,
d
dx
a f
TR =
d
dx
TC

or
d
dq
a f
TR =
d
dq
TC

Note: We know

if
d
dx
af af
f x =g x

Then z af af
g x dx = f x + c where c is the constant of integration.

By applying this to,


d
TR = MR and
dx
d
TC = MC
dx
We get

za f
MR dx = TR and za f
MC dx = TC.

So if we know marginal cost/marginal revenue (MC/MR) we can calculate, total cost or Total
revenue (TC/TR) by using
506 Basic Mathematics

z
TC = MC dx or MR dq z
z
TR = MR dx or MR dq z
WORKED EXAMPLES:
1. If the total cost function C(x) of a firm is given by C(x) = x3 3x + 7. Then find the average cost
and marginal cost when x = 6 units.
Solution: Given c(x) = x3 3x + 7.
Total Cost
We have Average Cost = Average Cost =
x

x 3 3x + 7
=
x

x 3 3x 7 7
= + = x2 3 + .
x x x x

Marginal Cost =
d
dx
C xaf
d 3
= x 3x + 7
dx

= 3x 2 3 + 0

MC = 3 x 2 3
MC when x = 6 is 3(6)2 3
3(36) 3
108 3 = 105.
2. If the marginal cost function is given by x3 + 3x 7 and fixed cost is Rs 750 then find the total
cost function.
Solution: Given:
Marginal cost function = x3 + 3x 7.
MC = x 3 + 3 x 7

We know Total cost = z MC dx .

= zd i
x 3 + 3 x 7 dx
Application of Calculus in Business 507

z z z
= x 3 dx + 3x dx 7 dx .

x 4 3x 2
TC = + 7x + c
4 2
where c is the constant of integration.
Here it is fixed cost.
Given fixed cost = Rs. 750

x 4 3x 2
Total function cost = + 7 x + 750
4 2
3. The total revenue function is given by R = x + 3x2. Find the marginal revenue and demand
function.
Solution: Given TR = x + 3x2.
diff. w.r.t. x.
d d
TR = x + 3x 2
dx dx
d
dx
TR =
d
dx
x + 3
dx
af
d 2
x d i
Marginal revenue = 1 + 3.(2x)
MR = 1 + 6x
Demand function = Average revenue
Total revenue
=
x

x + 3x 2
=
x

x 3x 2
= + = 1 + 3x .
x x

af
4. If the total revenue function is given by R q = 5 +
96
q
+ 6q 2 where q is the number of units

manufactured. Find the maximum value of total revenue.

af
Solution: Given R q = 5 +
96
q
+ 6q 2

diff. w.r.t. x.

af
R q =
d LM
5+
96
+ 6q 2
OP
dq Nq Q
508 Basic Mathematics

af FG 1 IJ a f
R q = 0 + 96
H q2 K
+ 6 2q

R(q) attains maxima when R(q) = 0.


96
0= + 12q
q2

96
= 12q
q2

96 = 12q 3

96
q3 =
12

q3 = 8

q=2
Maximum value of the revenue = R(2)

af
R 2 =5+
96
2
+6 2 af 2
= 5 + 48 + 24 = 77.

5. The marginal revenue (in thousands of rupees) function for a particular, commodity is 4 + e0.03x,
where x is denotes the number of units sold. Determine the total revenue from the sale of 100 units
given that e3 = 0.05 (Approx).
Solution:

We know Total revenue = za Marginal revenue dx f


Total revenue from the sale of

za f
100
100 units = MR dx
0

zd
100
= i
4 + e 0.03 x dx
0

100
e 0.03 x
= 4x +
0.03 0

LM a f
= 4 100 +
OP
e 0.03100 LM a f
4 0 +
e0 OP
N 0.03 Q N 0.03 Q
Application of Calculus in Business 509

LM e 3OP L OP
Q MN
1
= 400
N 0.03

0.03 Q
0.05 1
400 + = 400 1.66 + 33.33 = 431.667
0.03 0.03
Total revenue = 431.667 thousands of rupees.
= Rs. 4,31,667
6. Find the total cost of producing 1000 electric bulbs if the marginal cost (in Rs. per unit) is given

af
by C x =
x
1000
+ 2.5 where x is the output.

Solution:
We know, Total cost = (Marginal cost)dx.

za f
1000
Total cost of producing 1000 electric bulbs = MC dx
0

z FH I
1000
x
=
0
1000
+ 2.5 dx
K

=
1LMx2
+ 2.5 x
OP 1000

N
1000 2 Q 0

1 L1000
M a fOP
2
= + 2.5 1000 0
1000 N 2 Q
1000
+ 2,500 = 500 + 2500
2
= 3000.
Total cost of producing 1000 bulbs = Rs 3000.
7. If the total revenue function and total cost function are given by TR = 40x x2 and TC = 2 + 4x
respectively. Then find at what level of output profit is maximised.
Solution: We know, profit is maximised when marginal cost = marginal revenue. Now
d
Marginal cost = [Total cost]
dx
d
MC = 2 + 4x = 0 + 4
dx
MC = 4
510 Basic Mathematics

d
Marginal revenue = [Total revenue]
dx
d
= 40 x x 2
dx
MR = 40 2x
Profit is maximised when, MR = MC.
40 2x = 4
40 4 = 2x
36 = 2x
x = 18
Profit is maximised when 18 units are produced.
8. Find the maximum profit that a company can make, if the profit function P(x) = 41 + 24x 18x2.
Solution: Consider

af
P x = 41 + 24 x 18 x 2
diff. w.r.t. x.
dP
dx
af a f
= 0 + 24 1 18 2 x

dP
= +24 36 x
dx
dP
P attains extrema when =0
dx
0 = +24 36 x
+24 2
x= =+ .
36 3
dP
Again consider = +24 36 x
dx
diff. w.r.t. x.

d2P
= 36 < 0
dx 2
P attains maxima at x = 2/3.
Maximum value of the profit

F 2 I 18 F 2 I 2
= 41 + 24
H 3K H 3K
Application of Calculus in Business 511

4
= 41 + 16 18
9
= 57 8 = 49.
9. A T.V. manufacturer produces x sets per week at a total cost of Rs x2 + 1560x + 50,000.
12000 P
He is a monopolist and the demand function for this product is x = where P is the price
179
per set. What is the monopoly price in order to maximise the profit?
Given: Total cost = x2 + 1560 x + 50,000
diff. w.r.t. x.
d
dx
a f
TC = 2 x + 1560

i.e., Marginal Cost = 2x + 1560 ...(1)


Now, also given
12000 P
Demand function = x =
179
P = 12000 179 x
Total revenue = Px.
TR = 12000 x 179 x 2
diff. w.r.t. x.
d
dx
a f
TR = 12000 179 2 x a f
i.e., Marginal revenue = 12000 358x
MR = 12000 358x ...(2)
Profit is maximixed when
Marginal cost = Marginal revenue.
i.e., Equation (1) = (2)
2 x + 1560 = 12000 358 x
2 x + 358 x = 12000 1560
360 x = 10440
10440
x=
360
x = 29.
Monopoly price P = 12000 179 x
512 Basic Mathematics

= 12000 179 29 a f
= Rs. 6809.

x3
10. The cost function of a company is given by C = 500 x 20 x 2 + , where x stand for output.
3
Calculate the output when marginal cost is equal to average cost.

x3
Given: C = 500 x 20 x 2 +
3
diff. w.r.t. x.

dC
dx
= 500 20 2 x +a f
3x 2
3
i.e., Marginal Cost = 500 40x + x2 ...(1)
Now,
Total cost
average cost =
x

x3
500 x 20 x 2 +
= 3
x

x2
= 500 20 x + ...(2)
3
Equating (1) & (2)

x2
500 40 x + x 2 = 500 20 x +
3
x2
40 x + x 2 + 20 x =0
3
x2
20 x + x 2 =0
3

60 x + 3x 2 x 2
=0
3
60 x + 2 x 2 = 0
a
2 x 30 + x = 0f
2 x = 0 or 30 + x = 0
x = 0 or x = 30.
Application of Calculus in Business 513

The output when the marginal cost = average cost is 30.

REMEMBER:
Total cost = Fixed cost + Variable cost.

Marginal cost =
d
dx
a f
Total cost or
d
dq
a
total cost f
Total cost = (Marginal cost) dx or (Marginal cost) dq.
d d
Marginal revenue = (Total revenue) or (Total revenue)
dx dq
Total revenue = (Marginal revenue) dx or (Marginal revenue) dq.
Total cost Total cost
Average cost = or
x q
Profit is maximised when marginal cost = Marginal revenue.
Total revenue Total revenue
Average revenue is nothing but demand function = or
x q

EXERCISE

1. If the marginal cost of a product is 3x2 4x where x is the output, find the total cost of producing
10 units.
2
2. If the marginal revenue function is given by , find the total revenue function.
2x + 1
3. A company has revenue function given by R = 100q q2. Find the marginal revenue function
(where q is the output).
4. Find the marginal revenue for the demand function 3x x2.
5. Find the average cost function for the marginal cost function x2 + 2x.
6. The demand function of a firm is given by P = 50 2x. Where P is the price per unit for x units.
Determine the marginal revenue.
48
7. The cost function C = 5 + + 3 x 2 where x is the number of articles produced. Find the mini-
x
mum value of C.
8. If the marginal cost is given by x2 + 7x + 6 and fixed cost is Rs 2500, determine the total cost of
producing 6 units.
9. If C is the total cost for producing x units of a product and the average cost function is given by
3000
0.003 x2 0.04 x + 6 + , find the marginal cost function.
x
514 Basic Mathematics

10. The total cost function of a firm is given by C(x) = 2x3 x2 + 5x. Find the average and marginal
cost function.

x3
11. If the marginal revenue is given by 30 , then find the total revenue function.
30

af
12. The cost function of a firm is given by C x = 300 x 10 x 2 +
1 3
3
x . Where C is the total cost for

x units. Calculate the output at which the average cost is minimum.


13. The unit demand function is x = 25 2p where x is the number of units and p is the price. Let the
average cost per unit be Rs 20. Find the price per unit that maximises the profit function.
14. The demand function of a company is given by P(q) = 800 0.4q and the total cost function is
given by C(q) = 80 q + 8000 where q is the level of output,P is the price per unit. Find the level
of output and the price charged which maximises the profit.
15. Find the total revenue, marginal revenue, marginal revenue and average revenue when the de-
mand function is given by Q = 30 4P + P2 where P is price and Q is the quantity demanded.
Also calculate the marginal revenue when P = 3.
16. A manufacturer has a total cost function C = 100 + 32 x . He would like to know at what level
of output his marginal cost will be Rs. 2.00.
17. If the total cost function is C = 8x3 12x2 + 20x where x is the level of output, find x at which
the average cost is minimised find the total cost for this output.
18. The marginal cost function of a firm is 150 10x + 0.2x2, where x is the output, find the total cost
function, if the fixed cost is Rs 750, what is the average cost?
19. If the total revenue (R) and total cost (C) functions are given by R = 30x x2 and C = 20 + 6x,
find x if marginal revenue = marginal cost. Also find average cost and average revenue.
q
20. If the marginal revenue is given by 15 + , find the total revenue obtained from an output of
15
30 units.

ANSWERS

x2 c
1. 1200 2. log(2x + 1) + c 3. 100 2q 4. 6x 3x2 5. +x+
3 x
6. 50 4x 7. 41 8. 2734 9. 0.009x2 0.08x + 6
3000
10. 0.006 x 0.04 11. 6x2 2x + 5 12. 15 13. Rs 12.34
x2
3
14. Rs 440 15. 30 4P + P2 16. 64 17.
4

0.2 x 2 750
18. AC = 150 5 x + + 19. x = 12 20. 480.
3 x
Examination Corner
516
Pa
Question P Print
aper Blue Print ffor Pre-Uni
or Second Year Pr e-Univ
e-Uni ersity
ver Hours]
sity [90 Hour s]

Basic Mathematics
Sl. Objectives No. of Hours Knowledge Understanding Application Skill

No. Content VSA SA ET VSA SA ET VSA SA ET VSA SA ET TOTAL

1. Mathematical Logic 8 1 1 1 7
2. Permutation Combination 15 1 1 1 1 1 1 15
and Probability
3. Binomial Theorem 4 1 1 4
4. Partial Fractions 4 1 4
5. Matrices and Determinants 15 1 1 1 1 15
6. Ratio, Proportions and Variations 10 1 1 1 1 7
7. Averages 4 1 4
8. Bill Discounting 4 1 1 5
9. Stocks and Shares 4 1 1 3
10. Learning Curve 4 1 4
11. L.P.P. 6 1 1 5
12. Circles, Parabola 10 1 1 1 9
13. Limits and Continuity 8 1 1 1 7
14. Differentiation 10 1 1 1 1 1 13
15. Applications of Derivatives 8 1 4
16. Integration 10 1 2 10
17. Applications of Integration 6 1 4
Total 130 7 6 8 2 16 20 1 6 24 2 28 120
Examination Corner 517

MODEL QUESTION PAPERS

MODEL QUESTION PAPER 1 (Given by P.U. Board)

I. Answer all questions 1 10 = 10


1. Negate if two triangles are similar, then their areas are equal.
2. If nc7 = nc3, find nc10.
2400 2401
3. Find the value of .
2402 2403
4. Find the fourth proportional of 2, 6, 5.
5. A bill drawn for 4 months was legally due on 10.10.2003. Find the date of drawing the bill.
6. Define yield.
7. What is feasible region in L.P.P.
8. Find the centre of the circle 4x2 + 4y2 15x 8y + 11 = 0

e5 x 1
9. Evaluate Lt .
x 0 x
10. Total cost of a commodity is given by C = 1/3 x3 x2 + 3x + 10. Find the marginal cost.
II. Answer any ten of the following: 2 10 = 20.
11. If p is 2 is prime, q is 3 is even, r is 2 + 3 = 5 , , find the truth value of (p q) r.
12. Find the number of words that can be formed using all the letters of the word BOOKS. How
many of them begin with B.
13. There are 10 points out of which 3 are collinear. Find the number of straight lines that can be
formed.
14. Two dice are thrown simultaneously. What is the probability of getting the sum 7?

F 1 I 5

H
15. Expand 2 x
x2 K using Binomial theorem.

F
16. Find the middle term or terms in the expansion of H x
3 I 13
+
K
3
.
x

LM 2 1OP
17. If A = M 1
N 3PQ
2 find AA . 1
1
518 Basic Mathematics

18. Find the interest earned on Rs. 2448.35 cash invested in 15% Stock at 81.5, given that brokerage
is 0.125%.
19. Find the equation of the parabola whose focus is (4, 0) and directrix is x + 4 = 0. Find the length
of latus rectum.

af
20. If f x =
2x 1
x
, is continuous at x = 0, find f (0).

e j
21. If y = log e x + 1 + x 2 , prove that
dy
dx
=
1
1 + x2
.

x + 3x 2 + 4
22. Integrate w.r.t. x, .
x

III. Answer any three of the following: 4 3 = 12


a f a
23. Prove that p q ~ p q is a tautology.f
24. Prove that ncr + ncr 1 = n + 1c ,
r and verify this for n = 5 and n = 2.
1
25. Resolve into partial fractions 2
x x +1
.
a f
26. Prove that if each element of a row (or column) constant multiples of corresponding elements of
other rows (or columns) are added, then the value of the determinant is unaltered.
IV. Answer any three of the following: 4 3 = 12
27. If two men and four women can do a work in 33 days and 3 men and 5 women can do the same
work in 24 days, how long shall 5 men and 2 women take to do the same work?
28. Calculate the total wages earned per week by 400 workers in a factory from the following data
Daily wages Number of Workers
12.50 17.50 51
17.50 22.50 38
22.50 27.50 42
27.50 32.50 59
32.50 37.50 80
37.50 42.50 60
42.50 47.50 50
47.50 52.50 20
29. A bill for Rs. 2725.75 was drawn on 03.06.1997 and made payable 3 months after due date. It was
discounted on 15.06.1997 at 16% p.a. What is the discounted value of the bill and how much has
the banker gained in this transaction.
30. A business men buys and sells chairs and tables. He has Rs. 3000/- to invest. A chair costs him
Examination Corner 519

Rs. 50/- and a table costs him Rs. 90/-. He has space which can accommodate at most 48 pieces.
If he sells each chair for Rs. 200 and each table for Rs. 400/-. Find the number of chairs and tables
he has to buy to obtain maximum profit.
V. 4
31. Find the equation of the circle passing through the origin and having centre at (3, 4).
OR
Derive the equation of a parabola in the form y2 = 4ax.
VI. Answer any three of the following: 4 3 = 12

x n an
32. Prove that Lt = na n 1 . for all rational values of n.
x a x a

dy 1
33. If x 1 + y + y 1 + x = 0 and x y, prove that
dx
=
a f
1+ x 2 .

d2y 1
34. If x = at2, y = 2 at prove = .
dx 2 2 at 3

35. Evaluate z 2
e x x 3 dx.

VII. Answer any two of the following: 10 2 = 20


36. (a) Two number are in the ratio 4:7. If 12 is added to each of them then the new ratio is 8:11. Find
the numbers.
(b) In how many ways 3 boys and 5 girls be arranged in a row so that
(i) no two boys are together.
(ii) all the girls are together.
(c) Solve by matrix method.
2x + y + z = 7.
x y + 2z = 5
3x 2y + 2z = 5.
37. (a) Find the equation of the circle with (3, 5) and (6, 1) as the extremities of a diameter.
(b) An engineering company has succeeded in winning a contract for supplying aircraft engines.
The prototype constructed to win the contract took 4000 labour hours and the company
experiences 80% learning effect. Find the total cost of 7 engines of new order if labour cost
is Rs. 30/- per hour.
(c) The radius of a circular blot of ink is increasing at the rate of 3 cm per minute. Find the rate
of increases of its area when its radius is 2 cms. What is the rate of increases of its circum-
ference?
38. (a) Find the range in which the function f (x) = x2 6x + 5 is (1) increasing, (2) decreasing.
(b) Find the area bounded by the curve y = x2 7x + 10 with x-axis.
520 Basic Mathematics

1 a a2
(c) Prove without expanding 1 b a fa fa f
b2 = a b b c c a .
1 c c2

***

MODEL QUESTION PAPER 2:


I. Answer all questions: 1 10 = 10
1. If the truth value of p and q are T and F respectively. Then what will be the truth value of
a p qf a ~ p ~ qf .
2. Find the number of permutation of the letters of the word BALLOON taken all together.
3. Find the mean proportional to 2:8.

LM2 7 OP
4. Find x if
Nx 14 Q
is singular.

5. A bill was drawn on 14.3.1997 for 3 months. When does the bill fall legally due?
6. Find the yield by investing Rs. 1140 on 15% stock quoted at Rs. 95.
7. Write any one step to formulate an LP problem.
8. Find the radius of the circle 4x2 + 4y2 + 8x + 9y 7 = 0.

9. Evaluate: lim
x 1
d 3
x 1 id 4 x 1 i
10. If the marginal revenue function is given by 3x2 8, then find the total revenue function.
II. Answer any ten of the following: 2 10 = 20
11. Construct the truth table for p ~ q ~ p . a f
12. How many four digits numbers greater than 2000 can be formed with the digits 0, 1, 2, 3, 4, 5.
13. Find the number of permutations of the letters of the word BANANA taking all letters. How
many of them begin with N.
14. Two cards are drawn at random from a pack of well shuffled 52 cards. What is the probability of
getting king and queen cards.

F 2 I 4
15. Expand using binomial theorem: 3 +
H x K .

F 1 + xI 10
16. Find the middle term or terms in the expansion of
H x 2K
2
.

2LM 0 1 OP
17. If A = 1
1 MN 6
3 2 PQ
2 , then find AA.
Examination Corner 521

18. Find the cash required to purchase Rs. 1600, 8 1/2 stock at 105, brokerage is 1/2%.
19. Find the equation of parabola whose vertex is (1, 1) and focus is (3, 1).

af
20. If f x =
e 3x 1
x
is continuous at x = 0. Then find f (0).

dy
21. If x2 + y2 = 16. Then find when x = 3 and y = 4.
dx
1
22. Integrate w.r.t. x: x 3 x +
3x + 2
III. Answer any three of the following: 4 3 = 12
a f a f
23. Prove that p ~ q ~ p q is a contradiction.
24. Prove that ncr = ncn r and hence find n if nc18 = nc10.
25. Resolve into partial fractions:

2 x 2 + 3x + 2
.
x2 x 2
26. If the elements of any row (or column) is multiplied by non zero constant K. Then prove that the
value of the determinant is multiplied by K.
IV. Answer any three of the following:
27. 16 men or 28 boys can fence a farm in 40 days. In how many days will 24 men and 14 boys
complete the same work.
28. 3 tests in English, 2 in Hindi, 4 in Kannada and 5 in Sociology are conducted. The average marks
scored by Rashmi in English is 60, that in Hindi is 56 and that in Kannada is 45. If the average
marks of all the subjects and all the tests taken together is 48. Then find the averages marks scored
by her in Sociology.
29. The difference between the bill discount and true discount on a certain sum of money due in 4
months in Rs. 10. Find the amount of the bill (or face value) if the rate of interest is 3% p.a.
30. Maximise Z = 7x + 4y subject to x + 3y 3, 6 x + 3 y 8 ; x 0, y 0. .
V.
31. Derive the equation of circle which is described on the line joining (x1 y1) and (x2 y2) as the ends
of diameter.
OR
Find the equation of parabola whose ends of latus rectum are (3, 2) and (3, 4).
VI. Answer any three of the following: 4 3 = 12

32. Evaluate: (i) lim


n
12 + 2 2 + 32 + ... n 2
n3 6 n
a
(ii) lim 1 + 3 x f 1
x
522 Basic Mathematics

e j d i
m
33. If y = x + 1 + x 2 , then prove that 1 + x 2 y2 + xy1 m 2 y = 0.

d2y 24
34. If xy + 4y = 3x, then prove that
dx 2
=
x+4 a f 3 .

35. Evaluate: (a) z x log x dx

( b) z Fx
GH x
e 1

e
+ e x 1
+ ex
I
JK
dx

VII. Answer any two of the following: 10 2 = 20


36. (a) Divide Rs. 118 among A, B and C so that A:B = 3:4, B:C = 5:6.
(b) From 8 lecturers and 4 students a committee of 6 is to be formed. In how many ways can this
be done so that the committee contains (i) Exactly 2 students (ii) atleast 2 students.
(c) Solve by matrix method:
5 x y 4 z = 4, x + 4 y + 2 z = 12, 3 x y z = 3.
37. (a) A circle with centre (2, 1) passes through (1, 3). Find the equation of the circle.
(b) The first sample batch is of 50 units of product. A took 80 hours to make. The company now
wishes to estimate the total time taken to make 100 additional units. Solve the problem using
80% learning effect.
(c) If the base of the triangle is 3 times the height and the height is decreasing at the rate of 4 cm/
sec. then find the rate of decrease of the area when height is 1 cm.
38. (a) Find 2 numbers whose sum is 20 and whose product is maximum.
(b) Find the area bounded by the curve y2 = 4x and the line y = x.

a2 ab ac
(c) Prove without expanding ab b 2 bc = 4 a 2 b 2 c 2 .
ac bc c2

***

MODEL QUESTION PAPER 3:


I. Answer all questions: 1 10 = 10
1. Write the contrapositive of If the triangle is not right angled then its two sides are not perpendi-
cular.
2. If nc2 = 105, then find n.

LM3 xOP + 3 LM y xOP = LM 0 1OP .


3. Find x and y if 2
N4 0Q N8 0Q N32 3Q
Examination Corner 523

4. If a : b = 2 : 3 and b : c = 5:7, then find a : c.


5. The present worth of a bill due sometime hence is Rs. 1100 and the true discount is Rs. 110. Find
the bankers gain.
6. Find the proceeds got by selling shares worth Rs. 3000 at 96 1/8 brokerage 1/8%.
7. What is an objective function in LPP.
8. Find the focus of the parabola x2 + 8y = 0.

9. Evaluate: lim
d
n n2 + 2 i
n 6n 8
3

10. If the marginal cost function is x3 + 2x 1 and fixed cost is Rs. 2500, then find total cost function.
II. Answer any ten questions: 2 10 = 20
a f a f
11. The compound proposition p q r s has truth value false. Find the truth values of p, q,
r and s.
12. In how many ways can 3 girls and 4 boys sit round the table so that no two girls sit next to each
other.
13. Find the number of triangles that can be formed from 12 points of which 4 are given to be
collinear.
14. 2 dice are thrown simultaneously what is the probability of getting the sum 8.

F a xI
15. Expand H + K
5
using binomial theorem.
x b

Fx 1 I 21
16. Find the term independent of x in the expansion of
H xK 2
.

17. Solve by Cramers rule: 3x + y = 8 and 4x 9y = 6.


18. Find the cash you would require to purchase a share worth Rs. 3000 of 4% at 102 8/9 Brokerage
1/9%.
19. Find the vertex focus, directrix and length of latus rectum of the parabola x2 8y + 24 = 0.

a f RST4kx + 23 ifif xx < 11 is continuous at x = 1, then find K.


20. If f x =

dy
21. If y = a x + y , then find .
dx

22. Evaluate: z x 2 e x dx.

III. Answer any three of the following: 4 3 = 12

23. Verify a p qf a~ pf q is a tautology or contradiction or neither.


524 Basic Mathematics

n
n
pr = 5
24. Prove that n r and hence find p2.

25. Resolve into partial fractions:


2x 3
a fa
x x+3 x2 f
26. A salesman has the following records of sales during 3 months for 3 items A, B, C which has
different rates of commission.

Months Sales of units Total Commission


in Rs.
A B C

Jan 90 100 20 800


Feb 130 50 40 900
Mar 60 100 30 850

Find out the rates of commission on items A, B and C.


IV. Answer any three of the following: 4 3 = 12
27. In a mixture of 35 litres, the ratio of milk and water is 6:1. If 2 litres of milk and 3 litres of water
are added to the mixture, what is the new ratio of milk and water?
28. The average age of A and B is 18 yrs., that of B and C is 17 yrs. and that of C and A is 20 yrs.
Find the ages of A, B and C.
29. The bankers gain on a certain bill due 6 months hence is Rs. 20. The rate of interest being 20%
per annum. Find the face value of the bill.
30. A company produces 2 products x and y. Each of the product require two operations one on
machine A and other on machine B. The machine hours required by these two products and the
total hours available are given as follows:

Machine hrs. Product Total machine


required hrs. available
x y

A 2 5 19
B 4 3 17

Each unit of the product x and y makes a profit of Rs. 3 and Rs. 4. Find the optimal solution of
the product to be produced to obtain the maximum profit.
V. (4)
31. Find the equation of circle passing through (4, 0) and (0, 4) and its centre lies on 3x + 4y = 7.
OR
Examination Corner 525

Find the equation of parabola whose vertex is (0, 0) axis: Y-axis and passing through (1, 4).
VI. Answer any 3 of the following: 4 3 = 12

x n an
32. Prove that lim = na n 1 for all rational values of n.
xa x a

dy
33. Find if x y = y x .
dx
mx
34. If y = ae + be , then prove that y2 m y = 0.
mx 2

35. Evaluate: z ex
LM x + 1 OP dx .
2

MN a x + 1f PQ
2

VII. Answer any two of the following: 10 2 = 20


36. (a) If a : b = 7 : 5, find 4a 2b : a + 3b. (2)
(b) In how many ways can 5 members forming a committee but of 10 be selected so that
(i) 2 particular members must be included.
(ii) 2 particular members must not be included. (4)
(c) Solve by matrix method:
x y z = 1
y + 3z = 3
z + 4z = 2 (4)

a
37. (a) Prove that x + y = 2 touches the circle x 2 + y 2 6 x + y + 10 = 0.f (2)
(b) What do you mean by 80% learning effect?
Find the index of learning for 80% learning effect. (4)
(c) A ladder 5 mts. long rests with its ends on a horizontal floor and against a smooth vertical
wall. If the upper end is coming downwards at the rate of 40 cm/sec. Find the rate at which
the lower end moves when the upper end is 3 mts. from the ground. (4)
2
38. (a) The area of square is increasing at the rate of 2 cm /sec. Find the rate at which the perimeter
is increasing when the side of the square is 16 cm. (2)
(b) Find the area between the parabolas.

x 2 = 4 y and y 2 = 4 x. (4)

1+ a b c
(c) Prove without expanding a 1 + b c = 1 + a + b + c. (4)
a b 1+ c

***
526 Basic Mathematics

CHAPTERWISE ARRANGED QUESTION BANK

1. MATHEMATICAL LOGIC:
I. One mark questions (VSA):
1. Define a proposition.
2. Give the truth value of 4 + 3 8.
3. If p : 6 is an even number, q : 4 is odd, then write the true value of p q.
4. Write in symbols: The necessary and sufficient condition for a triangle to be equilateral is all its
sides should be equal.
5. Write symbolically: 6 is even and 2 is not irrational.
6. Negate: Cow is not big or it is black.
7. If the truth values of p and q are T and F respectively then what will be the truth value of
a p qf a p qf .
8. If the truth values of p, q, r are T, T, F respectively, find the truth value of p (~r q).
9. If the truth value of p ~r is F, then find truth value of r.
10. Write the truth table for p ~p.
11. Write the truth table for p ~p.
12. Write the converse of If a + 2 = 3, then a = 1.
13. Write the converse of If cows can fly then birds cannot fly.
14. Write the inverse of If he is rich then he is happy.
15. Write the contrapositive of If e is not irrational then 2 is rational.
16. Construct the truth table for p ~p.
17. Negate: If 2 is even then 3 is odd.
18. Negate: p ~q.
19. Negate: If 6 is even and 7 is odd, then 2 is prime.
20. Prove that p ~(~p).

II. Two marks questions (SA):


1. Construct the truth table for ~p q.
2. Construct the truth table for p (~q p).
3. Negate: If a triangle is equilateral then its sides are equal and angles are equal.
4. Negate: If x + y 6 and x 5, then y 0 or y 7.
5. Write the inverse and converse of:
If a triangle is equilateral then its sides are equal and angles are equal.
6. Write the converse and contrapositive of If a number is real then it is either rational or irrational.
7. Prove that ~[p q] p ~q.
Examination Corner 527

8. If p, q, r and s are false propositions, then find the truth value of (p ~q) (~r s).
9. Define Tautology.
10. Define converse and inverse of a conditional.
11. Define logically equivalent propositions.
12. Define contradiction.
13. If p, q, r, s are propositions with truth values T, F, T, T respectively, then find the truth value of
a p q f a q ~ r f ~ s.
14. Negate: (p q) (p r).
15. Negate: If it rains today then principal does not declare a holiday and we are not happy.

III. Four marks questions (ET):


a f a f
1. Prove that ~ p q p ~ q is a tautology.

a f a f
2. Verify whether p ~ q ~ p q is a tautology or contradiction.

3. Prove that a p qf a p qf q is neither tautology nor contradiction.


4. Prove that the negation of disjunction of 2 proposition is logically equivalent to the conjunction
of their negation.

5. Prove that a p qf ~ p p is neither tautology nor contradiction.


6. Prove that ~ p a p ~ q f q is a contradiction.

7. Prove that ~ p q a p ~ qf aq ~ pf .
8. Define converse and contrapositive of a conditional and prove that a conditional and its
contrapositive are logically equivalent.
9. Prove that the converse and inverse of a conditional are logically equivalent.
10. Prove that contrapositive of a conditional is converse of inverse of a conditional.

2. PERMUTATION, COMBINATION AND PROBABILITY:


I. One mark questions (VSA):
1. If nc4 = nc6, then find nc7.
2. Find the number of permutations of the letters of the word BOOKS, taking all letters.
3. How many of the arrangements of the word ABACUS begin with A.
4. If nc2 = 105, find n.
5. If np2 = 30, find n.
6. If np5 = 24 nc4, then find n.
7. How many of the arrangements of the word CRICKET begin and end with C.
8. State addition rule of probability for mutually exclusive events.
528 Basic Mathematics

9. A ticket is drawn from a bag containing tickets bearing numbers 1 through 25. Find the probabil-
ity of its bearing a number which is a multiple of 3.
10. One card is drawn from a pack of 52 cards. What is the probability of getting a king card?

II. Two marks questions (SA):


1. In how many ways 4 boys and 3 girls may be arranged for a photograph so that all the 3 girls are
shown together?
2. In how many ways can 5 members forming a committee out of 10 be selected so that 2 particular
members must be included?
3. Six students have taken an examination. In how many ways can the first 3 positions be declared?
4. In how many ways can the letters of the word HEXAGON be arranged so that H appears in the
beginning.
5. There are 5 maths books, 4 accounts books and 3 economics books. In how many ways can the
books be arranged so that the books of the same subject are together.
6. How many words can be formed by using all the letters of the word ALLAHABAD?
7. 20 persons were invited to a party. In how many ways can they and host be seated at a circular
table?
8. There are 12 points in a plane. 4 points are collinear. Find how many straight lines can be drawn?
9. A box contains 7 red, 6 white and 4 blue balls. How many selections of 3 balls can be made so
that none is red.
10. How many diagonals are there in an octagon?
11. Three coins are tossed. What is the probability of getting.
1. Exactly 2 heads.
2. at least 2 heads.
12. A pair of dice is thrown. What is the probability that the sum of the numbers obtained is more than
10.
13. A card is drawn from a pack of 52 cards. What is the probability that it is either king card or a red
card.
14. A pair of dice is rolled. If the sum on the 2 dice is 9, find the probability that one of the dice is
showed 3.
15. Two dice are thrown, find the probability of a doublet.
16. Prove that 0 =1

III. Four marks questions (ET):


1. A student has to answer 8 out of 10 questions in an examination. How many choices has he? How
many choices has he if he must answer the first 3 questions.
n
2. Prove that
n
pr = .
nr
Examination Corner 529

n
3. Prove that cr =
n
.
nr r
4. Prove that ncr = ncn r and hence find n if nc8 = nc4.
5. Prove that ncr + ncr 1 = n + 1cr and verify this for n = 5 and r = 3.
6. A committee of 10 members is to be chosen from 9 teachers and 6 students. In how many ways
this can be done if
(1) the committee contains exactly 4 students.
(2) there are atmost 7 teachers.
7. How many 4 digits numbers greater than 3000 can be formed with the digits 1, 2, 3, 5, 7 (repeti-
tions of digits is not allowed). How many of them are even?
8. How many (i) Straight lines (ii) Triangles are determined by 12 points. No three of which lie on
the same straight line.
9. There are 5 questions in part A and 4 in part B of a question paper. In how many ways can student
answer 6 questions if he has to choose at least 2 from each part.
10. A man has 7 relatives. 4 of them are ladies and 3 are gentleman. His wife also has 7 relatives 3
of them are ladies and 4 are gentlemen. In how many ways can they invite 3 ladies and 3
gentleman to a dinner partly so that there are 3 of mans relatives and 3 of wives relatives.
11. State and prove addition rule of probability.
12. What is the probability of getting neither 8 nor 11 when a pair of dice is tossed?
13. State and prove multiplication rule of probability.
14. Out of the numbers 1 to 120, one number is selected at random. What is the probability that it is
divisible by 8 or 10.
15. A card is drawn from a pack of 52 cards. What is the probability that it is neither a red card nor
a jack card.

af 3
8
af
5
8
a
3
f a f a f
16. If P A = , P B = and P A B = , find P A B ; P B A . Are the events independent?
4

3. BINOMIAL THEOREM & PARTIAL FRACTIONS:


I. One mark questions (VSA):
1. The second term in the expansion of (x + 3)5 = 240. Find x.

F a + xI 4
2. Write the middle term in the expansion of H x aK .

F 1I . 8

H xK
3. Write the 5th term in the expansion of 2
530 Basic Mathematics

F x 1I 8
4. Find the middle term in the expansion of H xK .

5. State Binomial theorem.

II. Two marks questions (SA):

F 1I
1. Find the middle term in the expansion of H 3 x + K .
2
9

2. Expand: a a + bx f
4

3. Expand: a1 2 x f
5

4. Find the middle term in the expansion of H


F 1 xI . 10

x2
2K

F 1I . 5

H xK
5. Find the co-efficient of x in the expansion of ax +
5 2
3

F 1I 5

H xK
6. Find the term independent of x in the expansion of ax +
2
3 .

F 1I 10

H xK
7. Prove that there is no term independent of x in the expansion of 2 x + 2 .

8. Find the value of (1.01)3 correct to 3 decimal places.


9. Prove that the sum of Binomial co-efficient of order n is 2n.
10. Prove that the sum of odd Binomial co-efficient = sum of even Binomial coefficients of order
n = 2n 1.

III. Four marks questions (ET):


1. State and Prove Binomial theorem.

d
2. Simplify: 2 + 3 i + d2 3 i
5 5

F
3. Find the middle term in the expansion of G 2 x
1 I
J
11

H xK
2
.

F bI 9
6 3
H 3K
4. Find the coefficient of a b in the expansion 2 a .
Examination Corner 531

F 1 IJ
5. Find the constant term in the expansion of G 2 x
15

H xK .

F
6. Prove that there is no term independent of x in the expansion of G ay
b IJ 25

H +
K
2
.
y

7x 9
7. Resolve into partial fractions:
a fax 1 x 2 f
x +8
8. Resolve into partial fractions:
2x x 1
2

1
9. Resolve into partial fractions:
x + x2 2x
3

x 2 10 x + 13
a fd
10. Resolve into partial fractions: x 1 x 2 5 x + 6
i
9 x 27
a
11. Resolve into partial fractions: x 2 2 x + 1
fa f
x 1
a fd
12. Resolve into partial fractions: x + 1 x 2 + 1 i
x2 + 1
13. Resolve into partial fractions:
a fax +1 x + 2 f
2 x 3 + 3x 2 3x + 2
14. Resolve into partial fractions: a x 1f d x + 1i
2

5x 2 + 1
15. Resolve into partial fractions:
x3 1

4. MATRICES AND DETERMINANTS:


I. One mark questions (VSA):
1. If A = [1 2 3], then find AA.
2. If A is of order m n, B is of order n p, then does AB exists? If so what is its order?
532 Basic Mathematics

LM1OP
3. Find x if 1 2 3
MM7xPP = 3
NQ
L1 2OP and B = LM2OP , then find AB.
4. If A = M
N1 7Q N3 Q
L1 7OP .
5. Write the adjoint of M
N 0 6Q
L2 3OP is singular or non-singular.
6. State whether M
N 7 8Q
L2 0OP , then find A .
7. If A = M
N0 2 Q
2

99 100
8. Find the value of .
101 102

2 3 4
9. If D = 2 x 3x 4 x , then prove that D = 0.
9 8 7

10. If A =
LM 7 7OP , then find A .
N6 5Q 2

LMx OP LM xOP
11. If M y P = M y P , then find A.
A
MN z PQ MN z PQ
LM1 2 7OP
MM1 2 3PP
12. Find x if 1 x 8 is singular.
N Q
LM1 0 0OP LM xOP LM 2OP
MM0 0 1PP MM z PP MM4PP
13. If 0 1 0 y = 3 , then find x, y and z.
N QN Q N Q
14. The following matrix shows the belongings of 3 friends Raju, Rakshith and Rama.
Examination Corner 533

Shirts Trousers
Raju LM 5 3 OP
Rakshith MM 7 4 PP
Rama N 8 9 Q
Write the row matrix which represents the belongings of Rakshith.
15. A company sold 22 chairs, 15 tables in January and 16, 28 respectively in June. Represent the data
in matrix form.
16. A business organisation has 2 distribution outlets. At the beginning of the month, the stock on
hand of products A, B and C are given in matrix M and the quantities of A, B and C sold during
the month are given in matrix N. What are the stocks on hands (in matrix form) at the end of the
month?

LM20 O, LM17 OP
9PQ
30 5 12 2
M= N=
N6 7 N3 1 7 Q
17. If A =
LM1 2OP and B = LM2 0OP , then find AB .
N7 6 Q N6 7 Q
18. Find x and y if

LM x 7OP + LM7 1OP = LM 8 8OP


N4 0Q N9 yQ N13 8Q
L1 7OP .
19. Find the minor of 3 in M
N3 6 Q
L1 2 7OP , then prove that (A) = A.
20. If A = M
N1 5 6Q
II. Two marks questions (SA):

1. If A =
LM1 7OP and B = LM2 1OP , then prove that (AB) = BA.
N6 5 Q N 3 2Q
L7 11OP and A + 2 B = LM8 13OP , then find A.
2. If 2 A + B = M
N 7 4Q N14 5Q
LM1 7OP L2O
3. Find AB if A 2 6 and B = M P .
MM5 2PP N7Q
N Q
534 Basic Mathematics

4. If A =
LM1 0OP , then prove that A 4A + 3I = 0.
N 2 3Q
2

L7 0OP and A B = LM3 0OP .


5. Find A and B if A + B = M
N 2 5Q N3 3Q
xy yz zx
6. Prove that y z zx xy =0.
zx xy yz

100 101 102


7. Evaluate: 103 104 105 .
106 107 108

1 2 3
8. Without expanding the determinant prove that 2 3 4 =0.
3 4 5

9. If A =
LM2 2OP and B = LM7 0OP , then find A B .
N3 5 Q N1 6 Q 2
10. Solve by Cramers rule: 2x + y = 1, x 3y = 4.
11. Solve by Matrix method: 3x + 4y = 7, 7x y = 6.
12. Suppose the matrix A and B represent the number of items of different kinds produced by 2

LM1OP LM7OP
manufacturing units in one day
MM3PP MM5PP
A = 2 and B = 6 . Compute 2A + 3B. What does 2A + 3B
NQ NQ
represent?
13. Ajith buys 5 kgs of tomato, 2 kgs. of Beans and 8 kgs of carrot. If the cost of each is Rs. 5,
Rs. 7 and Rs. 9 respectively, then find the total cost by matrix method.
14. A company sold 40 metal chairs, 20 wooden chairs and 10 plastic chairs. The selling price of a
metal chair is Rs. 150 that of wooden chair is Rs. 500 and plastic chair is Rs. 300. Find the total
revenue using matrix method.

LM 1 2OP
and hence find A1.
15. Find the adjoint of
N7 0 Q
III. Four marks questions (ET):
1. Solve the following using Cramers rule:
4x + y = 7, 3y + 4z = 2 3z + 5x = 2.
Examination Corner 535

2. Solve by using Cramers rule: 3x y + 2 z = 18, 2 x + y z = 3, x + 3y 5z = 8.

LM 1 4 2 OP
3. Find the adjoint of M 2 5
PP
4 and hence find A1.
NM 1 2 1 Q
LM 1 1 2 OP
4. Find the inverse of 0
MM1 2 2.
3QP
P
N 1

LM1 2 2 OP
MM2 2
5. Prove that A = 2 1
PP
2 satisfies the equation A2 4A 5I = 0 .
N 1 Q
6. Solve by matrix method: x + y + z = 6, 3x + y z = 2, x + 2 z 5 = y.
7. Solve by matrix method: 4 x + y = 7, 3 y + 4 z = 5, 3 z + 5 x = 2.

1+ x 2 1
8. Prove that 1 2+ x 1 = x 2 x + 4 . a f
1 2 1+ x

x p q
9. Prove that p x a
q = x p xq x+ p+q .fa fa f
p q x

a + b + 2c a b
10. Prove that c b + c + 2a b a f
= 2 a + b + c 3.
c a c + a + 2b

5. RATIO, PROPORTIONS AND VARIATIONS:


I. One mark questions (VSA):
1. Find the fourth proportional of 2, 6, 5.
2. Express 13:169 in the simplest form.
3. Compare 2:5 and 3:2.
4. Find the ratio between 1 hr 10 min and 575 min.
5. Write the subtriplicate of 1:27.
6. If a:b = c:d, then prove that b:a = d:c.
536 Basic Mathematics

7. Find the missing term: x:4 = 27:12.


8. If a:b = 2:3, b:c = 4:5, then what is c:a?
9. If Ramu can do a piece of work in 8 days and Rithu can do the same work in 3 days, then in how
many days, both together can do the same work?
10. Divide Rs. 180 in the ratio 1:2:3.

II. Two marks questions (SA):


1. Two number are in the ratio 4:7. If 12 is added to each of them then the new ratio is 8:11. Find
the numbers.
2. The prices of a scooter and a T.V. are in the ratio 3:2. If the scooter costs Rs. 6000 more than the
T.V., then find the cost of TV.
3. A certain amount was divided between Kavitha and Reena in the ratio 4:3. If Reenas share was
Rs. 2400, then find the amount divided.
4. What number should be added to each one of 6, 14, 18, 38 to make it equally proportionate.
1 1 1
5. Three friends divide Rs. 624 among themselves in the ratio : : . Find the share of the third
2 3 4
friend.
6. Gold is 19 times as heavy as water and copper 9 times as heavy as water. Find the ratio in which
these two metals be mixed so that the mixture is 15 times as heavy as water.
7. The ages of Vivek and Sumit are in the ratio 2:3. After 12 years their ages will be in the ratio
11:15. Find the age of Sumit.
8. If 60 men can complete a job in 12 days, how many days will 36 men take to complete the same
job?
9. Find 3 numbers in the ratio 2:3:5, the sum of whose squares is 608.
3
10. Cycling of his usual speed, a student is 10 minute late to his class. Find his usual time to cover
4
the distance.

III. Four marks questions (ET):


1. If 2 men and 4 women can do a work in 33 days and 3 men and 5 women can do the same work
in 24 days. How long shall 5 men and 2 women take to do the same work.
2. A can do a piece of work in 6 days and B alone can do it in 8 days. A and B under took to do it
for Rs. 320, with the help of C, they finished it in 3 days. How much is paid to C?
3. 12 men or 18 women can reap a field in 14 days. In how many days 8 men and 16 women can
reap it?
4. 4 men and 6 women finish a job in 8 days while 3 men and 7 women finish it in 10 days. In how
many days 10 women working together will finish it?
5. 5 carpenters can earn Rs. 3600 in 6 days working 9 hours a day. How much will 8 carpenters earn
in 12 days working 6 hours a day?
Examination Corner 537

6. Two trains start at the same time from Aligarh and Delhi and proceed towards each other at 16
km/hr and 21 km/hr respectively, when they meet, it is found that one train has travelled 60 km
more than the other. Find the distance between 2 stations.
7. Kanthilal mixes 80 kgs of sugar worth Rs. 6.75 per kg with 120 kg worth of Rs. 8 per kg. At what
rate shall he sell the mixture to gain 20%?
8. A can contains a mixture of 2 liquids A and B in the ratio 7:5, when 9 litres of mixture is drawn
off and the can is filled with B, the ratio becomes 7:9. How many litres of the liquid A was
contained by the can initially?
9. Two men undertake to do a piece of work for Rs. 400. One alone can do it in 6 days and the other
in 8 days. With the help of a boy they finished it in 3 days. What is boys share?
10. Adarsh buys 2 horses for Rs. 1350 and sells one at 6% loss and other at 7.5% gain and on the
whole, he neither gains nor loses. What does each horse cost?

6. MATHEMATICS OF FINANCE:
(Averages, Bill Discounting, Stocks & Shares, Learning Curve, LPP)

I. One mark questions (VSA):


1. A bill drawn for 4 months was legally due on 10-10-2003. Find the date of drawing the bill.
2. Define yield.
3. What is feasible region in LPP?
4. The average age of 10 students is 6 years. The sum of the ages of 9 of them is 52 years. Find the
age of 10th student.
5. If 10 kgs of a commodity is purchased at Rs. 24 per kg and 5 kgs at Rs. 18 per kg, what is the
average price of the commodity per kg?
6. Find the cost of Rs. 9100, 8 34 % stock at 92.
7. The present worth of a certain sum due sometime hence is Rs. 1600 and the true discount is
Rs. 160. What is the bankers gain?
8. What is learning curve?
9. Find the index of learning for 80% learning effect.
10. What is the cost price of Rs. 100 share at 4 discount if the brokerage is 1/4%?

II. Two mark questions (SA):


1. Find the interest earned on Rs. 2448.35 cash invested in 15% stock at 81.5, given that brokerage
is 0.125%.
2. Find the market value of 5 and 1/4% stock, in which an income of Rs. 756 is derived by investing
Rs. 14976, brokerage being 1/4%.
3. The average age of 10 students is 14 years. Among them, the average age of 4 students is 12 years.
Find the average age of the remaining students.
4. Find the income derived from 44 shares of Rs. 25 each at 5 premium (brokerage 1/4 per share)
the rate of dividend being 5%. Also find the rate of interest on the investment.
538 Basic Mathematics

5. A worker takes 20 hours to produce the first unit of product. What is the cumulative average time
per unit taken by him for the production of first 2 units?

III. Four marks questions (ET):


1. A bill for Rs. 2725 was drawn on 3-6-1997 and made payable 3 months after due date. It was
discounted on 15-6-1997 at 16% p.a. What is the discounted value of the bill and how much has
the banker gained in this transaction.
2. A business man buys and sells chairs and tables. He has Rs. 3000 to invest. A chair costs him
Rs. 50 and a table costs him Rs. 90. He has space which can accommodate at most 48 pieces. If
he sells each chair for Rs. 200 and each table for Rs. 400, find the number of chairs and tables he
has to buy to obtain maximum profit.
3. An engineering company has succeeded in winning a contract for supplying aircraft engines. The
prototype constructed to win the contract took 4000 labour hours and the company experiences
80% learning effect. Find the total cost of 7 engines of new order if labour cost is Rs. 30 per hour.
4. A bill of Rs. 5000 drawn on 10-04-96 at 3 months was discounted on 1-5-96 at 7% per annum.
Find the discounted value of the bill and bankers gain.
5. A banker pays Rs. 2340 on a bill of Rs. 2500, 146 days before the legally due date. What is the
rate of discount charged by the banker?
6. A bill for Rs. 2920 drawn at 6 months was discounted on 10-4-97 for Rs. 2916. If the discount
rate is 5% per annum on what date was the bill drawn?
7. A batsman realises that by scoring a century in the 11th innings of his test matches he has bettered
his average of the previous 10 innings by 5 runs. What is his average after the 11 innings?
8. Ms. Vidya bought 17 books in a discount sale. The average price of the books being Rs. 53. The
average price of the eleven maths books is Rs. 71. If the prices of the remaining 6 accounts books
form an increasing arithmetic progression with last term Rs. 25, find the price of the cheapest
book.
9. Maximise Z = 10x + 30y subject to x + 2y 20. x + 5y 35 and x, y 0. Indicate the feasible
region on the graph sheet.
10. Minimise Z = 20x + 10y subject to x + 2y 40, 3x + y 30, 4x + 3y 60, x 0, y 0.
11. Mr. Harsha has invested a certain amount of money in 13% stock at 101. He hold the investment
when the market value went down to 96.5. He lost Rs. 3564 on this process. If he has paid the
brokerage of 1.5% for all the transactions, what was the amount of cash investment and what was
the stock value of the investment in the first instance.
12. A person has invested Rs. 4300 partly in 4.5% stock at 72 and partly in 5% stock at 95. If the total
interest is Rs. 250, find his investment in each type of stock.
13. The time required to produce the first unit of a product is 1000 hrs. If the manufacturer experi-
ences 80% learning effect, calculate the average time per unit and the time taken to produce
altogether 8 units. Also find the total labour charges for the production of 8 units at the rate of Rs.
10 per hour.
Examination Corner 539

7. CIRCLES, PARABOLA:
I. One mark questions (VSA):
1. Find the radius of the circle 2 x 2 + 2 y 2 3 y + 8 x 6 = 0 .

2. Find the centre of the circle 3x 2 + 3y 2 6 x + 7 y 8 = 0 .


3. Find the equation of the tangent to the circle x2 + y2 = 2 at (1, 1).
4. Find the length of the tangent from (1, 1) to the circle x 2 + y 2 + 2 x + 3y + 1 = 0. .

5. Find the distance of the centre of x 2 + y 2 + 4 x + 6 y + 9 = 0 from x-axis.


6. If the circle (x 6)2 + (y 6)2 = 36 touches both co-ordinate axes, find the distance of co-ordinate
axes from the centre.
7. Mention the formula to find the equation of tangent at (x1, y1) to the circle with centre origin and
radius r units.
8. Write the general equation of the circle.
9. If 3x 2 + ay 2 + bxy + 9 x 8 y + 7 = 0 represent a circle, then what is the value of a and b?

10. Where do the point (2, 3) lie with respect to the circle x 2 + y 2 + 16 x 3y 7 = 0.
11. Write the equation of the directrix of the parabola y2 = 12x.
12. Find the co-ordinates of ends of latus rectum of x2 = 4y.
13. Find the length of latus rectum of x2 + 8y 7 = 0.
14. What is the eccentricity of the parabola x2 = 8y.
15. Find the vertex of the parabola 3x2 2y = 0.

II. Two marks questions (SA):


1. Find the equation of the circle whose 2 diameters are 2x + y =3 and x y = 6 and radius 3 units.
2. Prove that x + y = 2 touches the circle x 2 + y 2 6 x 6 y + 10 = 0. .

3. If the circle x 2 + y 2 + 2gx + 2 fy + c = 0 touches x-axis then prove that g2 = c.

4. Find the length of the perpendicular from the centre of the circle x 2 + y 2 + 4 x y + 3 = 0 to the
chord 3x + 4y + 1 = 0.
5. A circle with centre (2, 1) passes through (1, 3). Find its equation.

6. If one end of diameter of the circle x 2 + y 2 x + y 2 = 0 is (1, 1), then find the other end.
7. Find the vertex, and axis of the parabola x2 + 2x y 4 = 0.
8. Find the focus and length of latus rectum of (y 2)2 = 8 (x + 2).
9. Find the ends of latus rectum for the parabola (y + 2)2 = 4 (x 1).
10. Find the equation of parabola given that vertex is (0, 0) and focus is (0, 4).
540 Basic Mathematics

11. Define a parabola: what is the eccentricity of the parabola.


12. Find the length of latus rectum for the conic 4 y 2 + 3y + 6 x = 1 .

III. Four marks questions (ET):


1. Derive the standard equation of the circle. Find its centre and radius.

b g
2. Derive the equation of tangent to the circle x 2 + y 2 + 2 gx + 2 fy + c = 0 at x1 , y1 on it.

3. Derive the length of the tangent to the circle x 2 + y 2 + 2gx + 2 fy + c = 0 from (x1, y1).
4. Find the condition for the line y = mx + c to be a tangent to the circle x2 + y2 = a2. Find the point
of contact also.
5. Derive the equation of circle which is described on the line joining (x1, y1) and (x2, y2) as diameter.
6. Derive the equation of parabola in the standard form.
7. Find the equation of the circle passing through (0, 2) having centre on x + y = 0 and having 6
as the length of the tangent from (1, 1).
8. Find the equation of the circle passing through the points (1, 1), (2, 2) and (6, 0).
9. Find the equation of the circle given that the length of the tangents from the points (1, 2),
(2, 3) and (2, 4) are respectively 2 3 , 30 and 17 .

10. Find the equations of the tangents to the circle x 2 + y 2 + 2 x + 2 y 7 = 0 which are parallel to 3x
4y + 8 = 0.
11. Find the equations of tangents to the circle 5 x 2 + 5 y 2 + 4 x + 2 y 60 = 0 which are r to 16x
7y + 7 = 0.
12. Find the equation of the parabola given that vertex (3, 4) and directrix y = 1.
13. Find the equation of the parabola given that Focus = (4, 0) and directrix x = 4.
14. Find the equation of the parabola given that vertex (0, 0) axis y-axis and passing through (1, 4).
15. Find the focus, directrix, axis, length of latus rectum of the parabola y 2 + y + 3x + 4 = 0.

16. Find the vertex, axis, ends of latus rectum of the parabola x 2 + 2 x y 4 = 0.
17. Find the equation of parabola given that ends of latus rectum (2, 3), (2, 6).
18. Prove that the circles x 2 + y 2 2 x 4 y 3 = 0 and x 2 + y 2 14 x 16 y + 81 = 0 touch each other.
Also find the co-ordinate point of contact.

8. LIMITS AND CONTINUITY:


I. One mark questions (VSA):

e3 x 1
1. Evaluate: lim
x 0 x
Examination Corner 541

x 4 34
2. Evaluate: lim
x 3 x 3 33

12 + 2 2 + ... n 2
3. Evaluate: lim
n n3

x 3 + 4 x 2 3x + 2
4. Evaluate: lim
x 2 7x 2 x + 2

83 x 1
5. Evaluate: lim
x 0 x

F 2 I 3n
6. Evaluate: lim 1 +
n H n K
II. Two marks questions (SA):

a f RST43xx 22 ifif xx <> 00 is continuous at x = 0.


1. Find f (0) if f x =

af
2. If f x =
x2 9
x2 4x + 3
for x 3 is continuous at x = 3, find f (3).

x
3. Prove that lim does not exist.
x 0 x

af a
4. If f x = 1 + 2 x f 1
x is continuous at x = 0 find f (0).

3+ x 3
5. Evaluate: lim
x 0 x

dx 3
3 3 i
dx i
6. Evaluate: lim 4 4
x 3 3

III. Four marks questions (ET):

x n an
1. Prove that lim = na n 1 for all rational values of n.
xa x a

2. Prove that the function f (x) = |x| is continuous at x = 0.


542 Basic Mathematics

R| 1 1
f a xf = S x
4
81 for x 3 is continuous at x = 3.
|T x 271
3. Prove that
1
3

4
for x = 3
9

R| loga1 + 6 xf for x 0
f a xf = S x
|T b
4. Verify for continuity of f (x) at x = 0, if .
for x = 0

5. Evaluate:

3n +1 + 1
(a) lim
n 3 n + 2 + 2

F1 + 2I 4n
(b) lim
n H nK
6. Define a continuous function and prove that f (x) = 3x + 2 is a continuous function.

9. DIFFERENTIATION AND APPLICATION OF DERIVATIVES:


I. One mark questions (VSA):
1. What is the derivative of 35 w.r.t. x?
2. Differentiate xe + ex + ee w.r.t. x.

3. If x 2 + y 2 = a 2 find
dy
dx
a f
at a, a .

dy
4. Find if y = 2e x 2 .
dx

5. If y = log x 2 + 1 , find y1.


x
6. If y = 2 , then find y.

af
7. If f x = x 2 +
1
x2
af
, then find f 1 .

1 3
8. If the total cost of a commodity is C = x + x 2 + 3 x + 10, then find marginal cost.
3
9. If the total cost function is C (x) = x3 7x + 8, then find average cost.
Examination Corner 543

1 1
10. If the average cost function is , then find marginal cost.
x x2
11. Differentiate 3ex w.r.t. logx.
12. If the total revenue function is given by R = 3q2 q + 8, then find marginal revenue function.

II. Two marks questions (SA):

e 2
j
1. If y = log x + 1 + x , then prove that
dy
dx
=
1
1 + x2
.

mx
2. If y = e + e , then prove that y2 m y = 0.
mx 2

1 d2y
3. If y = x + , then find .
x dx 2

a dy y
4. If x = a, y = , then prove that + = 0.
dx x
2
5. Differentiate e x with respect to ex.

e j
6. If y = log x + a 2 + x 2 , then prove that
dy
dx
=
1
a + x2
2
.

dy y
7. If yey = x, then prove that =
a f
dx x 1 + y
.

8. With usual notation, S = at + b, where a and b are constants. Prove that velocity is constant and
acceleration is zero.
9. Find 2 numbers whose sum is 10 and whose product is maximum.
10. If the total cost function c (x) of a firm is given by c (x) = x3 3x + 7, then find the average cost
and marginal cost where x = 6 units.

III. Four marks questions (ET):


dy 1
1. If x 1 + y + y 1 + x = 0 and x y, then prove that
dx
=
a f
1+ x 2 .

d2y 1
2. If x = at2, y = 2at, then prove that 2
= .
dx 2at 3
3. The radius of a circular blot of ink is increasing at the rate of 3 cm per minute. Find the rate of
increase of its area when its radius is 2 cm. What is the rate of increase of its circumference?
4. Prove that the largest rectangle that can be inscribed in a circle of given radius is a square.
5. Find the derivative of x3n from first principle.
6. State and prove quotient rule in differentiation.
544 Basic Mathematics

a
7. If x m y n = x + y f m +n
, then prove that
dy y
= .
dx x
8. Find the derivative of 5
x from first principles.
2 dy
9. If y = x x + e x , then find .
dx

10. Find the maximum and minimum value of the function 2 x 3 15 x 2 + 36 x + 10.
11. Prove that the area of the right angled triangle of given hypotenuse is maximum when the triangle
is isosceles.

12. If total revenue function is R q = 5 + af 96


q
+ 6q 2 where q is the number of units manufactured.

Find the maximum value of total revenue.


13. If total revenue function and total cost function are given by TR = 40x and TC = 2 + 4x respec-
tively, then find at what level of output profit is maximised?
14. A TV manufacturers produces x sets per week at a total cost of Rs. x2 + 1560x + 50,000. He is
12000 P
a monopolist and the demand function for this is x = where P is price per unit, what
179
is the monopoly price in order to maximise the profit?

10. INTEGRATION AND APPLICATION OF INTEGRATION:


I. One mark questions (VSA):

1. Evaluate: zd 2 x + e x x 2 dx i
2. Integrate e5x + 7 with respect to x.

3. Evaluate: z log x dx

zd
2

4. Evaluate: x 2 + 3 x 7 dx i
1

5. Find the area bounded by the curve y = x2, x-axis and the ordinates x = 0 and x = 1.
6. If the marginal cost function is given by x2 + 7x 8, then find total cost function.

II. Two marks questions (SA):

x + 3x 2 + 4
1. Integrate with respect to x: .
x
2. Integrate (x2 + 7) (2x3 7) with respect to x.
Examination Corner 545

3. Evaluate: z a x 1f
x3
2
dx

4. Evaluate: zd x2 + x 1 i a2 x + 1f dx
5

5. Evaluate: za 2x + 1
fa
x +1 x + 2
dx
f
6. Evaluate: z x 3 log x dx

z
1
1 x
7. Evaluate: dx
1+ x
0

8. Find the area bounded by the curve y = x2 x with x-axis.


9. The marginal cost function of a firm is 150 10x + 0.2x2 where x is the output. Find the total cost
function, if the fixed cost is Rs. 750. What is the average cost?
10. The marginal revenue function of a firm is 60x x2 where x is the output. Find the total revenue
function.

III. Four marks questions (ET):

1. Evaluate: z 2
e x x 3 dx

za f
a
xa
4
2. Evaluate: x dx
0

3. Find the area between the parabolas y2 = x and x2 = y.

4. Find the area bounded by the curve y = x 2 7 x + 10 with x-axis.

5. Evaluate: z ex
FG 2 x + 1IJ dx
H2 xK
z
2

6. Evaluate: x 1 + x 2 dx
1

z
3
2x + 3
7. Evaluate: dx
5x 7
2

4x + 8
a fa f
8. Evaluate: 2 x 1 2 x + 3 .
546 Basic Mathematics

GIST AND FORMULAE

1. MATHEMATICAL LOGIC:
p q is T only when both p and q are true otherwise it is false i.e., T T is T otherwise it is F.
p q is F only when both p and q are false. Otherwise it is True i.e., F F is F otherwise it is
T.
p q is F only when p is true and q is false i.e., T F is F otherwise it is T.
p q is T only when both p and q are together True or False, i.e., T T and F F is T
otherwise it is F.
~p is T when p is F and vice-versa.
Tautology is a compound proposition which is always true for all possible combinations of the
truth values of its components.
Contradiction is a compound proposition which is always False for all possible combinations of
the truth values of its components.
2 propositions X and Y are logically equivalent if and only if they have identical truth values. It
is denoted by X Y.
~ (~p) p
~ (p q ) ~ p ~ q
~ (p q) ~p ~q
~ (p q ) p ~ q
~ (p q) (p ~ q) (~p q)
Converse of the conditional p q is q p
Inverse of the conditional p q is ~p ~q
Contrapositive of conditional p q is ~q ~p.

2. PERMUTATION AND COMBINATION:


n

n
pr =
nr

n
p0 = 1, n
pn = n n p1 = n
Permutation of n objects of which p objects are of one kind, q are of another kind and so on is
n
.
p q ...

Number of circular arrangement of n persons round a table = n 1

n 1
Number of circular arrangement of n beads or n flowers to form a necklace or garland = .
2
Examination Corner 547

n

n
cr =
nr r

n
cn = 1

n
c1 = n

n
c0 = 1

n
cr = n cn r

n
cr + n cr 1 = n +1cr
Number of straight lines that can be drawn from n points of which p points are collinear =
nc pc + 1.
2 2
Number of triangles that can be drawn from n points of which p points are collinear = nc3 pc3.
Number of diagonals in a polygon of n sides = nc2 n.

3. PROBABILITY:

a f nnaaASff
P A = af
0 P A 1.

P d A i = 1 P a Af

P (AB) means P (A or B) and P (AB) means P (A and B)


Addition rule: P (AB) = P (A) + P (B) P (AB)
For mutually exclusive cases
P (AB) = P(A) + P(B)
a f af af af a f a f a f a
P A BC = P A + P B + P C P A B P BC P C A + P A BC f
P a A Bf = P a Af P a B Af

P a A Bf = P a Bf P a A Bf

P a A Bf = P a Af P a Bf if A and B are independent events.

P a A B C f = P a Af P a B Af P aC A Bf

If A , A , A , ... A are independent events P b A A ... A g = P b A g P b A g ... P b A g


1 2 3 n 1 2 n 1 2 n

4. BINOMIAL THEOREM:

a x + af n
= x n + nx n 1 a +
a f x a +
a fa f
n n 1 n 2 2 n n 1 n 2 n 3 3
x a + ... + a n .
2! 3!
548 Basic Mathematics

There are (n + 1) terms in the expansion of (x + a)n. The power indices of x go on decreasing
by 1 and those of a go on increasing by 1 at each stage so that the sum of power indices is n.

The general terms or (r + 1)th term is given by Tr +1 = n cr x n r a r .


If n is even the number of terms in the expansion of (x + a)n is (n + 1) which is odd.
There will be only one middle term i.e. Tn .
+1
2

If n is odd, the number of terms in the expansion of (x + a)n is (n + 1) which is even. there will
be 2 middle terms: Tn +1 and Tn +1 .
+1
2 2

To find the term containing xm in the expansion of (x + a)n i.e., to find the co-efficient of xm, write
Tr + 1. Simplify and equate power index of x to m. Get r and substitute the value of r is Tr + 1.
For getting the term independent of x or constant term, equate the power index of x to zero after
writing Tr + 1 simplify then get the value of r. If r is a positive integer greater than 0, substitute in
Tr + 1.
If r is negative or a fraction then conclude that there is no term independent of x in the expansion.

5. PARTIAL FRACTIONS:
Proper fraction Partial fraction
Nr A1 B C
ba x + b gb
1 1 gb
a2 x + b2 a2 x + b3 ... g + +
a1 x + b1 a2 x + b2 a3 x + b3
+ ...

Nr A B C
aax + bf acx + d f
2 +
ax + b ax + ba f a
2 +
cx + d f
Nr Ax + B C
+
dax 2
ia
+ bx + c dx + e f dax 2
+ bx + c i dx + e

Provided ax2 + bx + c is non-factorisable. If ax2 + bx + c is factorisable as (a1x + b1) (a2x + b2), then
replace ax2 + bx + c by (a1x + b1) (a2x + b2). Then resolve,
Nr A B C
bax + b gb1 ga
a2 x + b2 dx + e
as
a1 x + b1f b
+
a2 x + b2
+
g b
dx + e g a f
If an improper fraction is given to resolve, first by dividing the numerator by denominator write the
given fraction as the sum of the polynomial and the proper fraction, then the proper fraction is resolved
into partial fractions.

6. MATRICES & DETERMINANTS:


Matrix is an arrangement of numbers in horizontal rows and vertical columns.
Two matrices of the same order are said to be equal if and only if the corresponding elements are
equal.
Examination Corner 549

Two matrices can be added subtracted if they have same order. It is obtained by adding/subtract-
ing the corresponding elements.
Multiplication of a matrix by a scalar is obtained by multiplying each and every element by a
scalar.
If A is of order m n and B is of order n p then AB is of order m p, i.e., matrix multiplication
is possible only when number of column in 1st matrix is equal to number of rows in the 2nd
matrix.
Transpose of a matrix is obtained by interchanging rows and columns.
A unique value associated with every square matrix is called its determinant value.
If det A = 0, i.e., |A| = 0 for a matrix A, then A is called singular matrix. Otherwise it is called non-
singular matrix.
The value of the determinant is unaltered if its rows and columns are interchanged.
If 2 rows or columns are interchanged the value of the determinant changes its sign.
If in a determinant 2 rows or columns are identical then the value of the determinant is zero.
If the elements of any row (or column) is multiplied by k, the value of the determinant is multi-
plied by k.
If to the elements of any row (or column) of a determinant the same multiples of the correspond-
ing elements of other row (or column) of the determinant are added the value of the determinant
remain the same.
The determinant obtained by deleting the row and column containing the element is called minor
of that element. If the minors are multiplied with (1)i + j [where i = row number and j = column
number of the element] we get co-factors. The transpose of the co-factor matrix is called adjoint
of the matrix.
For a non-singular matrix A.
adj A
A 1 = .
A
If A is a square matrix and I is the identity matrix of the same order.
The characteristic equation: |A I| = 0. The values of obtained is called eigen values or
characteristic roots.
Every square matrix satisfies its characteristic equation, |A I| = 0. This is Cayley Hamilton
theorem.
The solution of system of equations
a1 x + b1 y + c1 z = d1

a2 x + b2 y + c2 z = d2
a3 x + b3 y + c3 z = d3
By Cramers rule:
550 Basic Mathematics

a1 b1 c1 d1 b1 c1
Let
= a2 b2 c2 , x = d 2 b2 c2
a3 b3 c3 d3 b3 c3

a1 d1 c1 a1 b1 d1
y = a2 d2 c2 , z = a2 b2 d2
a3 d3 c3 a3 b3 d3

x y
Then x= , y= and z = z .

By matrix method:

LM a 1 b1 c1 OP x LM OP d1 LM OP
Let MMa
A= a b2
PP MM PP
c2 , X = y and D = d2
MM PP
NQ
2

N 3 b3 c3 Q z d3 N Q
Then matrix equation
AX = D
X = A1D.
adj A
and A 1 = or A1 can be found by using Cayley Hamilton theorem.
A

7. RATIO AND PROPORTIONS, VARIATIONS:

For the given ratio a : b, duplicate ratio is a2 : b2 Subduplicate ratio is a : b , Triplicate ratio
is a3 : b3 and subtriplicate ratio is 3
a :3 b .
a : b : : c : d iff ad = bc.
If a : b = b : c, then b is called mean proportional and c is called 3rd proportional.
If a : b = c : d, then d is called fourth proportional.
If a : b = c : d, then
(i) b : a = d : c (Invertendo)
(ii) a : c = b : d (Alternendo)
(iii) a + b : b = c + d : d (Componendo)
(iv) a b : b = c d : d (Dividendo)
(v) a + b : a b = c + d : c d
(Componendo and dividendo)
If a : b = c : d represent a direct proportion, then a : b = d : c or b : a = c : d represent an inverse
proportion and vice-versa.
Examination Corner 551

Quantity of cheaper C.P. of Dearer Mean


Rule of alligation: Quantity of dearer = Mean C. P. of cheaper

ac f ad f
amf i.e., cheaper : dearer = d m : m c.

dm mc

8. AVERAGES:
x1 + x 2 + ... + x n
Mean X =
n
w1 x1 + w2 x 2 + ... + wn x n
Xw =
w1 + w2 + ... + wn

X1 N1 + X 2 N 2 + ... + X nN n
X123 =
N1 + N 2 + ... + N n

9. BILL DISCOUNTING:
Legally due date = Bill Drawing date + Bil period + 3 days (Grace period).
BD = Ftr [Simple interest on face value of the bill]
TD = Ptr [Simple interest on present worth of the bill]
BG = BD TD
F
Present worth =
1 + tr
BG = TD tr

BG =
aTDf 2
If r and t are not given
Pw

TD = B G Pw

10. STOCKS AND SHARES:


Nominal interest
Yield =
Amount invested
When stock is purchased, brokerage is added to cost price.
When stock is sold, brokerage is subtracted from selling price.
Interest or dividend is paid on the face value of the stock or share not the market value.
552 Basic Mathematics

4 % stock at 96 means a stock whose face value is Rs. 100 is available at Rs. 96. Interest earned
in 4 .
Shares need not be fully paid but stock must be fully paid.

11. LEARNING CURVE:

aAverage labour cost of first 2N unitsf


The learning curve ratio =
aAverage labour cost of first N unitsf
The learning curve equation is
y = axb
where y = cumulative average time/unit.
a = Time for first unit.
b = log of learning ratio to base 2.
x = Cumulative total number of units produced.
Index of learning, b = log of learning ratio to base 2.

12. LINEAR PROGRAMMING:


The objective function is a quantified statement in linear programming problem of what the best
result is aimed for Objective function will either to maximise or to minimise a value.
If the objective function is a function of 2 variables only, the linear programming problem can be
solved by graphical method.
Optimum solution to linear programming problem lies at one of the vertices. Hence find the co-
ordinates of each vertex and subject in the objective function. The values for which the objective
function is highest/least is the optimum solution for maximisation/minimisation.

13. CIRCLES:
Distance between 2 points (x1 y1) and (x2 y2)

bx 2 x1 g + by
2
2 g
y1 .
2

Slope of the line joining (x1 y1) and (x2 y2)


y2 y1
= .
x 2 x1
2 lines are r iff product of their slopes = 1.
Equation of circle with centre (0, 0) and radius r units: x2 + y2 = r2.

Equation of circle with centre (h, k) and radius r units: x h a f + a y kf


2 2
= r2.
Equation of circle which is described on line joining (x 1 y1 ) and (x2 y 2) as diameter =
b x x gb x x g + b y y gb y y g = 0.
1 2 1 2
Examination Corner 553

General equation of the circle x2 + y2 + 2gx + 2fy + c = 0

Centre = (g, f), Radius = g2 + f 2 c .


Centre is the mid point of diameter.

Fx +x y1 + y2 I
H 2 K
1 2
Co-ordinates of mid point of line joining (x1 y1) and (x2 y2) = ,
2
The point of intersection of 2 diameters of a circle = centre.
If a circle x2 + y2 + 2gx + 2fy + c = 0 touch x-axis, then, radius = f and g2 = c.

If a circle x2 + y2 + 2gx + 2fy + c = 0 touches y-axis, then Radius = g and f 2 = c.


Length of the perpendicular from (x1 y1) to the line ax + by + c = 0 is

ax1 + by1 + c
a 2 + b2
Equation of tangent at (x1 y1) on the circle x2 + y2 + 2gx + 2fy + c = 0 is

b g b
xx1 + yy1 + g x + x1 + f y + y1 + c = 0. g
Length of the tangent from (x1 y1) to the circle x2 + y2 + 2gx + 2fy + c = 0 is

x12 + y12 + 2 gx1 + 2 fy1 + c.


Any line is a tangent to the given circle if the length of the r from centre to the line = Radius of
the circle.

Condition for the line y = mx + c to be a tangent to the circle x2 + y2 = a2 is c = a m + 1 and


2 2 2
d i
F a I or F am I.
point of contact is GH am
m2 + 1
,
m2
J GH
+ 1K m2 + 1
,
a
m2
J.
+1K
Condition for the line lx + my + n = 0 to be a tangent to the circle x2 + y2 + 2gx + 2fy + c = 0 is
an lg mf f = dl
2 2
id
+ m2 g 2 + f 2 c . i
The point (x1 y1) lie inside the circle x2 + y2 + 2gx + 2fy + c = 0 if x12 + y12 + 2gx1 + 2 fy1 + c < 0.

Outside the circle if x12 + y12 + 2 gx1 + 2 fy1 + c > 0.

On the circle if x12 + y12 + 2 gx1 + 2 fy1 + c = 0.


Any line parallel to ax + by + c = 0 will have the equation ax + by + k = 0.
[Take x and y co-efficients same].
Any line perpendicular to ax + by + c = 0 will have the equation bx ay + k = 0 [interchange
x and y co-efficients and change the sign of anyone].
14. PARABOLA:

554
Basic Mathematics
Equation Figure Vertex Focus Equation Axis Ends of Equation Length
of of of of
directrix Latus rectum LR LR

y2 = 4ax (0, 0) (a, 0) x = a x-axis (a, 2a) x=a 4a


y=0

y2 = 4ax (0, 0) (a, 0) x=a x-axis (a, 2a) x = a 4a


y=0

x2 = 4ay (0, 0) (0, a) y = a y-axis (2a, a) y=a 4a


x=0

x2 = 4ay (0, 0) (0, a) y=a y-axis (2a, a) y = a 4a


x=0
(h, k)

(y k)2 = 4a (x h) (h, k) (a + h, k) x = a + h y=k (a + h, 2a + k) x=a+h 4a

(h, k)

(y k)2 = 4a (x h) (h, k) (a + h, k) x=a+h y=k (a + h, 2a + k) x = a + h 4a

(x h)2 = 4a (y h) (h, k) (h, k) (h, a + k) y = a + k x=h (2a + h, a + k) y=a+k 4a

(x k)2 = 4a (y h) (h, k) (h, k) (h, a + k) y=a+k x=h (2a + h, a + k) y = a + k 4a


Examination Corner 555

In any parabola, focus is inside the curve and directrix is away from the parabola.
Distance between vertex and focus = a.
For the given ends of latus rectum, there are 2 possible parabolas.
Focus is the mid point of latus rectum.
Axis is r to the directrix. Distance between directrix and vertex = a.

15. LIMITS AND CONTINUITY:

x n an
lim = na n 1
x a xa

ex 1
lim =1
x0 x

ax 1
lim = log e a
x 0 x

F 1 I n
lim 1 +
n H n K =e

lim 1 + n
n 0
a f 1
n =e

af af
A function y = f(x) is said to be continuous at x = a if lim f x = f a = lim+ f x
x a xa
af
i.e., LHL = f(a) = RHL.
Limit of a function exists at x = a if
af
lim f x = lim+ f x
x a xa
af
f(x) is continuous at x = a iff
af af
lim f x = f a
x a

16. DIFFERENTIAL CALCULUS:

dy
y
dx

xn nxn1
x 1
x2 2x
556 Basic Mathematics

x3 3x2
1
x 2 x

1 1

x x2
1 2
x2 x3
1 3
x3 x4
ex ex
ax ax loga
1
logx
x
du
Cu C
dx
du dv
uv
dx dx

I II I
d
dx
af
II + II
d
dx
af
I

Nr.* Dr.
d
dx
a f a f a f
Nr Nr
d
dx
Dr.
Dr.** a f
Dr. 2

Chain rule: If y = g(u) & u = f (x),


then y = g[f(x)] is differentiated by chain rule
dy dy du
=
dx du dx
Implicit dif
diffferentia
erentiation: Given function f (x, y) = 0.
entiation:
dy dy
diff. w.r.t. x., take common among the terms containing and shift the remaining terms to
dx dx
dy
RHS. Then find .
dx

*: Numerator **: Denominator


Examination Corner 557

Parametric differentiation:
Given x = f(t) and y = g(t).

dy
dy dy dt
Then to find , use dx = dx
dx
dt
Second order der
order deriivative:
tiv
dy
If y = f(x), then by differentiating we get or y1 or y1 or f1(x). This is a function of x. By
dx
d2y
differentiating this again with respect to x we get or y or f (x) or y2.
dx 2

17. APPLICATION OF DERIVATIVES:


dS
Velocity =
dt

dv d 2 S
Acceleration = =
dt dt 2
Rate means differentiation w.r.t. t.
dA
rate of change of area =
dt
Area of Square = S2
Area of circle = r2
Surface area of sphere = 4r2
4 3
Volume of Sphere = r
3
Volume of a cylinder = r2h
1 2
Volume of a cone = r h
3
dy dy
For an increasing function > 0 and for a decreasing function < 0.
dx dx
dy
To find maximum and/or minimum value of the function y = f(x), find , equate it to zero. Let
dx
d2 y d2y d2y
x = a, x = b, x = c be the points. Find , and .
dx 2 at x = a dx 2 at x = b dx 2 at x = c.
558 Basic Mathematics

d2y
if > 0 , then x = a is a point of minima. Minimum value of the function is y = f(a).
dx 2 at x = a

d2 y
if is less than zero, x = b is a point of maxima. Maximum value of the function is
dx 2 at x = b
y = f(b).

d2y
if is equal to zero, then x = c is called point of inflection. At x = c the function neither
dx 2 at x = c
attains maxima nor minima.

18. INTEGRATION:

z x n dx =
x n +1
n +1
+c

Provided n 1
if n = 1

z z
x 1dx =
1
x
dx = log x + c.

z e x dx = e x + c.

z a x dx =
ax
log a
+c

z k dx = kx + c

z af af z af z af
k f x g x dx = k f x dx k g x dx

If z af af
f x dx = g x + c, then

za f a f
f ax + b dx =
g ax + b
a
+c

z af af af
To evaluate f x
n
f x dx, put f x = t and proceed to get the answer.

af
f x
n +1
+ c for n 1
n +1
and log [f (x)] + c for n = 1
Examination Corner 559

To evaluate ef(x) f(x) dx, put f(x) = t and proceed to get ef(x) + c as answer.
To evaluate integrals of the type

za px + q
fa
ax + b cx + d
dx or
f za px + q
ax + b f acx + d f
2

First resolve into partial fractions


px + q A B
a fa
ax + b cx + d
=
f +
ax + b cx + d
or
px + q A B C
a fa
ax + b cx + d
2 =
ax + b f a
+
ax + b f a f a
2 +
cx + d f
Find A, B, C, then finally integrate.

za fa
I function II function dx f
z
= I function II function z LMNz II function
d
dx
a
I function dx fOPQ
I function and II function are kept by making use of LIATE rule.

z af af
e x f x + f x dx = e x f x + c. af
19. DEFINITE INTEGRALS:

z af af z af af af af
b
b
If f x dx = g x + c, then f x dx = g x a
=g b g a
a

z af z af z af
b b b

f x dx = f t dt = f z dz and so on.
a a a

z af z af
b a

f x dx = f x dx
a b

z af z af z af
b c a

f x dx = f x dx + f x dx
a a c

z af za f
a a

f x dx = f a x dx
0 0
560 Basic Mathematics

Area enclosed by the curve y = f(x), X-axis and the lines x = a and x = b is given by

z z af
b b
A= y dx = f x dx .
a a

20. APPLICATION OF CALCULUS IN BUSINESS:


Total cost = Fixed cost + Variable cost.

Marginal cost =
d
dx
a f
Total cost or
d
dq
a
total cost f
Total cost = (Marginal cost) dx or (Marginal cost) dq.
d d
Marginal revenue (Total revenue) or (Total revenue)
dx dq
Total revenue = (Marginal revenue) dx or (Marginal revenue) dq.
Total cost Total cost
Average cost = or
x q
Profit is maximised when marginal cost = Marginal revenue.
Total revenue Total revenue
Average revenue is nothing but demand function = or
x q
Examination Corner 561

Chapter One mark Two mark Four mark Total marks


questions questions questions
(VSA) (SA) (ET)

Mathematical Logic 1 1 1 7

Permutation, Combination 1 3 2 15
and Probability

Binomial Theorem and 2 1 8


Partial Fractions

Matrices and Determinants 1 1 3 15

Ratio, Proportion & Variation 1 1 1 7

Mathematics of Finance 3 1 4 21
(Averages, B.D., Stock and
Shares, L.C., LPP)

Circles, Parabola 1 2 1 9

Limits and Continuity 1 1 1 7

Differentiation and Its 1 2 3 17


Application

Integration and Its 1 3 14


Application

Total: 10 15 20 120

You might also like